333
PRINCIPAL MR. J. K. SHAH INTER CA Taxation Head Office Shraddha, 4th Floor, Old Nagardas Road, Near Chinai College, Andheri (E), Mumbai - 400 069. 022 - 2683 66 66 CAFCINTER CAFINAL CA THE RANKERS FACTORY

INTER CA Taxation - CA Study Web€¦ · INTER CA Taxation Head Office Shraddha, 4th Floor, ... 14. Advance Tax & Interest 268 - 271 15. Tax Deducted at Source (TDS) 272 - 294 16

  • Upload
    others

  • View
    2

  • Download
    0

Embed Size (px)

Citation preview

Page 1: INTER CA Taxation - CA Study Web€¦ · INTER CA Taxation Head Office Shraddha, 4th Floor, ... 14. Advance Tax & Interest 268 - 271 15. Tax Deducted at Source (TDS) 272 - 294 16

PRINCIPAL

MR. J. K. SHAH

INTER CA Taxation

Head Office Shraddha, 4th Floor, Old Nagardas Road,

Near Chinai College, Andheri (E), Mumbai - 400 069.

022 - 2683 66 66

CAFCINTER CAFINAL CA

THE RANKERS FACTORY

Page 2: INTER CA Taxation - CA Study Web€¦ · INTER CA Taxation Head Office Shraddha, 4th Floor, ... 14. Advance Tax & Interest 268 - 271 15. Tax Deducted at Source (TDS) 272 - 294 16

�������

Page 3: INTER CA Taxation - CA Study Web€¦ · INTER CA Taxation Head Office Shraddha, 4th Floor, ... 14. Advance Tax & Interest 268 - 271 15. Tax Deducted at Source (TDS) 272 - 294 16

SR. NO. CHAPTER PAGE NO.

1. Introduction 1 - 14

2. Residence of an Assessee 15 - 26

3. Income from Salaries 27 - 62

4. Income From House Property 63 - 75

5. Deductions From Gross Total Income - Chapter VIA (Part I) 76 - 91

6. Income From Business & Profession 92 - 150

7. Agriculture Income 151 - 156

8. Capital Gains 157 - 201

9. Income from Other Sources 202 - 218

10. Clubbing of Income 219 - 232

11. Set off & Carry Forward of losses 233 - 242

12. Deductions From Gross Total Income - Chapter VIA (Part II) 243 - 254

13. Assessment Procedure 255 - 267

14. Advance Tax & Interest 268 - 271

15. Tax Deducted at Source (TDS) 272 - 294

16. Non - Taxable Income 295 - 303

17. Home Work Problems 304 - 328

CONTENTS

THEORY, CLASSWORK & SOLUTIONS

Page 4: INTER CA Taxation - CA Study Web€¦ · INTER CA Taxation Head Office Shraddha, 4th Floor, ... 14. Advance Tax & Interest 268 - 271 15. Tax Deducted at Source (TDS) 272 - 294 16

: 1 :

J. K. SHAH CLASSES INTER C.A. - DIRECT TAXES

INTRODUCTION

What is a Tax?

Let us begin by understanding the meaning of tax. Tax is a fee charged by a government on a

product, income or activity. There are two types of taxes – direct taxes and indirect taxes (See

Chart below this paragraph). If tax is levied directly on the income or wealth of a person, then it is

a direct tax e.g. income-tax. If tax is levied on the price of a good or service, then it is called an

indirect tax . In the case of indirect taxes, the person paying the tax passes on the incidence to

another person.

TYPES OF TAXES

DIRECT TAXES INDIRECT TAXES

INCOME TAX CUSTOMS DUTY GST

Why are Taxes Levied?

The reason for levy of taxes is that they constitute the basic source of revenue to the government.

Revenue so raised is utilized for meeting the expenses of government like defence, provision of

education, health-care, infrastructure facilities like roads, dams etc.

Overview of Income-Tax Law in India

Income-tax is the most significant direct tax. In this material, we would be introducing the students

to the Income-tax law in India. The income-tax law in India consists of the following components–

INTRODUCTION

Page 5: INTER CA Taxation - CA Study Web€¦ · INTER CA Taxation Head Office Shraddha, 4th Floor, ... 14. Advance Tax & Interest 268 - 271 15. Tax Deducted at Source (TDS) 272 - 294 16

: 2 :

J. K. SHAH CLASSES INTER C.A. - DIRECT TAXES

INTRODUCTION

The various instruments of law containing the law relating to income-tax are explained below:

Income-tax Act, 1961: The levy of income-tax in India is governed by the Income-tax Act, 1961. In

this book we shall briefly refer to this as the Act. This Act came into force on 1st April, 1962. The

Act contains 298 sections and XIV schedules. These undergo change every year with additions

and deletions brought about by the annual Finance Act passed by Parliament. In pursuance of the

power given by the Income-tax Act, 1961 rules have been framed to facilitate proper administration

of the Income-tax Act, 1961.

The Finance Act: Every year, the Finance Minister of the Government of India introduces the

Finance Bill in the Parliament’s Budget Session. When the Finance Bill is passed by both the

houses of the Parliament and gets the assent of the President, it becomes the Finance Act.

Amendments are made every year to the Income-tax Act, 1961 and other tax laws by the Finance

Act.

The First Schedule to the Finance Act contains four parts which specify the rates of tax -

���� Part I of the First Schedule to the Finance Act specifies the rates of tax applicable for the

current Assessment Year.

���� Part II specifies the rates at which tax is deductible at source for the current Financial Year.

���� Part III gives the rates for calculating income-tax for deducting tax from income chargeable

under the head “Salaries” and computation of advance tax.

���� Part IV gives the rules for computing net agricultural income.

Income-tax Rules: The administration of direct taxes is looked after by the Central Board of

Direct Taxes (CBDT). The CBDT is empowered to make rules for carrying out the purposes of the

Act. For the proper administration of the Income-tax Act, 1961, the CBDT frames rules from time

to time. These rules are collectively called Income-tax Rules, 1962. It is important to keep in mind

that along with the Income-tax Act, 1961, these rules should also be studied.

Circulars and Notifications: Circulars are issued by the CBDT from time to time to deal with

certain specific problems and to clarify doubts regarding the scope and meaning of the provisions.

These circulars are issued for the guidance of the officers and/or assessees. The department is

bound by the circulars. While such circulars are not binding on the assessees, they can take

advantage of beneficial circulars. Notifications are issued by the Central Government to give effect

to the provisions of the Act.

For example, under section 10(15)(iv)(h), interest payable by any public sector company in respect

of such bonds or debentures and subject to such conditions as the Central Government may, by

notification in the Official Gazette, specify in this behalf would be exempt. Therefore, the bonds

and debentures, interest on which would qualify for exemption under this section are specified by

the Central Government through Notifications.

The CBDT is also empowered to make and amend rules for the purposes of the Act by issue of

notifications. For example, under section 35CCD, the CBDT is empowered to prescribe guidelines

for notification of skill development project. Accordingly, the CBDT has, vide Notification No.54/

2013 dated 15.7.2013, prescribed Rule 6AAF laying down the guidelines and conditions for

approval of skill development project under section 35CCD. .

Page 6: INTER CA Taxation - CA Study Web€¦ · INTER CA Taxation Head Office Shraddha, 4th Floor, ... 14. Advance Tax & Interest 268 - 271 15. Tax Deducted at Source (TDS) 272 - 294 16

: 3 :

J. K. SHAH CLASSES INTER C.A. - DIRECT TAXES

INTRODUCTION

Case Laws: The study of case laws is an important and unavoidable part of the study of income-

tax law. It is not possible for Parliament to conceive and provide for all possible issues that may

arise in the implementation of any Act. Hence the judiciary will hear the disputes between the

assessees and the department and give decisions on various issues. The Supreme Court is the

Apex Court of the country and the law laid down by the Supreme Court is the law of the land. The

decisions given by various High Courts will apply in the respective states in which such High

Courts have jurisdiction.

Income tax Act 1961

Introduction

In India, Constitution is the parent law. All other laws should be enacted without exceeding the

framework of the constitution and subject to the norms laid down therein. The Constitution of India

empowers Central Government to levy tax on income. By virtue of this power and to achieve this

objective the Income Tax Act 1961 was enacted in the place of the Income tax Act 1922.

According to Section 1 of the Income tax Act. The Act is to be called as “The Income Tax Act 1961”

and it extends to whole of India. It came into force on the 1st day of April 1962 i.e from AY 1962-

63 onwards.

Preliminary

a) Section 2 of the Income Tax Act gives definition of the various terms and expressions

used in the Act. Unless the context otherwise requires, these definitions should be applied.

The words “means” , “Includes” and “means and includes” are used in these definitions and

the significance of these terms needs to be understood.

b) When definition uses the word “means”, the definition is self explanatory, restrictive and in

a sense exhaustive. It implies that term or expression so defined means only as to what it is

defined as and nothing else. For example, the terms Agricultural Income, Assessment Year,

Capital Asset are exhaustively defined.

c) When the legislature wants to widen the scope of a term or expression and where an

exhaustive definition cannot be given, it uses the word includes in the definition. Hence, the

inclusive definition provides an illustrative meaning and not an exhaustive meaning. In

practical application the definition could include what is not specifically stated or mentioned

in the definition so long as the stipulated criteria are satisfied. To illustrate reference is

drawn to the definition of the terms Income, Person, Transfer.

d) When the legislature intends to define a term or expression to mean something and also

intends to specify certain items to be included both the words means and includes are

used. Such definition is not only exhaustive but also illustrative in specifying what is intended

to be included. Sometimes specific items are included in an exhaustive definition in order

to avoid ambiguity and with a view to provide clarity. One can find that these words are

used in the definition of the terms Assessee, Firm etc.

e) Apart from the definition under section 2 the Act defines various other terms under the

respective sections where they are used. For Example Section 17(2) defines perquisite,

Section 3 defines Previous year.

Page 7: INTER CA Taxation - CA Study Web€¦ · INTER CA Taxation Head Office Shraddha, 4th Floor, ... 14. Advance Tax & Interest 268 - 271 15. Tax Deducted at Source (TDS) 272 - 294 16

: 4 :

J. K. SHAH CLASSES INTER C.A. - DIRECT TAXES

INTRODUCTION

Section 4 :Charging Section

As per Section 4 of the Income tax Act, 1961 Income-tax is payable by every Person on his

total Income earned in the Previous Year at the Rates applicable for the relevant

Assessment Year.

DEFINITIONS

Some of the words used in the above statement require elaboration

1. PERSON : [Section 2 (31)]

Section 4 provides for charging tax on every person and person is defined under sec-

tion 2(31) as including :

1. An individual.

2. A Hindu Undivided Family (H.U.F.).

3. A Company.

4. A Firm; [Partnership Firm Assessed as Such (PFAS)].

5. An Association Of Persons (A.O.P.) or Body Of Individuals (B.O.I.) whether

incorporated or not.

6. Local Authority.

7. Every artificial juridical person, not falling under any one of the preceding

categories.

Thus, this section enumerates seven types of assessees who are covered under the Act.

The seventh category is residuary and includes all sorts of artificial juridical bodies not

covered under any of the first six categories.

(2) INCOME [Section 2(24)]

Income is defined under section 2(24) of the Act. The definition contained in that section

is inclusive and not exhaustive, which means that income includes not only those items

which are enumerated in the section but besides it may include various other items to

which the natural meaning of the word may apply.

Section 2(24) defines as "Income" includes the following;

1. Profits and gains which are covered by section 28 or section 41.

2. Dividend

3. In case of charitable or religious trust or institution or educational institution or uni-

versity or hospital, or an electoral trust, voluntary contribution received.

4. The value of any perquisite taxable u/s 17(2), or profit in lieu of salary taxable u/

s 17(3), under the head salaries.

5. Any special allowance granted by the employer to meet expenses wholly,

necessarily and exclusively for the performance of the duties of employment

& dearness allowance & city compensatory allowance.

6. The value of any benefit or perquisite received by :

director of a company,

person who has a substantial interest in the company,

relative of director or person who has substantial interest in the company,

from a company, which otherwise was the obligation payable by such persons.

Page 8: INTER CA Taxation - CA Study Web€¦ · INTER CA Taxation Head Office Shraddha, 4th Floor, ... 14. Advance Tax & Interest 268 - 271 15. Tax Deducted at Source (TDS) 272 - 294 16

: 5 :

J. K. SHAH CLASSES INTER C.A. - DIRECT TAXES

INTRODUCTION

7. The value of any benefit or perquisite obtained / paid by

by the representative assessee on behalf of the beneficiary -

which otherwise was the obligation payable by such beneficiary.

8. Any capital gains chargeable under section 45.

9. The profit and gain of any business or insurance carried on by a mutual

insurance company or by a co-operative society, computed in accordance

with section 44.

10. The profit and gains of any business of banking (including providing credit

facilities) carried on by a cooperative society with its members.

11. Any winnings from lotteries, crossword, puzzles, races including horse races,

card games and other games of any sort or from gambling or betting of any form

or nature whatsoever.

12. Any sum received by the assessee from his employees as contributions to any

provident fund or super annuation fund or any fund set up under the provisions of

the Employee's State Insurance Act, 1948, or any other fund for the welfare of

such employees.

13. Any sum received under a keyman insurance policy including the sum allocated

by way of bonus on such policy.

14. Any sum of money or any property (movable or immovable) received for no

consideration or inadequate consideration shall be considered as income, subject

to the provisions of section 56.

15. Any consideration received by a closely held company for issue of shares as

exceeds the fair market value of the shares referred to in section 56.

16. Any assistance in the form of subsidy, grant,cash incentive, duty drawback ,

concession or reimbursement provided by Central Government or State

Government or any authority or any agency, except for-

(a) Subsidy provided for acquiring fixed assets for business purposes; and

(b) Subsidy provided by Central Government for the corpus of a trust or

institution set up by Central/State Government.

Following are the broad principles to understand the concept of "Income".

1. Different forms of income - Income may be received in cash or in kind. When

it is received in kind, its valuation is to be made according to the rules prescribed

in the Income-tax Rules. If however there is no prescribed rule, valuation thereof

is made on the basis of market value.

2. Receipt Vs. Accrual - Income arises either on receipt basis or on accrual basis.

Income may accrue to a tax payer without its actual receipt.

Salaries : Due or receipt whichever is earlier

House - property : Due basis

Business & Profession : As per method of accounting regularly followed

Capital Gains : Due basis

Income from other sources : As per method of accounting regularly

followed.

3. Relief or reimbursement of expenses is not treated as income - Mere relief

or reimbursement of expenses is not treated as income; for instance,

reimbursement of conveyance expenses to an employee is not an income.

Page 9: INTER CA Taxation - CA Study Web€¦ · INTER CA Taxation Head Office Shraddha, 4th Floor, ... 14. Advance Tax & Interest 268 - 271 15. Tax Deducted at Source (TDS) 272 - 294 16

: 6 :

J. K. SHAH CLASSES INTER C.A. - DIRECT TAXES

INTRODUCTION

4. Temporary and permanent income - For the purpose of income tax there is

no distinction between temporary and permanent income. Even temporary

income is taxable such as casual winnings.

5. Income includes loss - While income and profit and gains represent "plus

income", losses represent "minus income".

6. Revenue receipt Vs. Capital receipts - A revenue receipt is taxable as income

unless it is expressly exempt under the provisions of the Act. On the other hand,

a capital receipt is generally exempt from tax, unless it is expressly taxable.

7. Illegal income - The income - tax law does not make any distinction between income

accrued or arisen from a legal source and income tainted with illegality.

8. Disputed title - Income-tax assessment cannot be held up or postponed merely

because of existence of a dispute regarding the title of income.

9 Surplus from mutual activity - A person cannot make taxable profit out of a

transaction with himself. Income must, therefore, come from outside.Income

should be real and not fictional. Transfer of goods between head office and branch

at invoice price does not result into income. If such goods are sold to outsiders

the profit on sale is regarded as income. Similarly income does not arise on

revaluation of assets.

A surplus arising to a mutual concern cannot be regarded as income chargeable

to tax. A body of individuals, raising contribution to a common fund for the mutual

benefit of members, cannot be said to have earned an income when it finds that

it has overcharged members and some portion of contribution raised may safely

be refunded to the members concerned.

13. Tax-free income- If a person receives tax-free income on which tax is paid by

the person making payment on behalf of the recipient, it has to be grossed up for

inclusion in his total income.

14. Same income cannot be taxed twice : It is a fundamental rule of the law

of taxation that, unless otherwise expressly provided , the same income

cannot be taxed twice.

15. Source of income need not exist in the assessment year : It is not necessary

that a source of income should exist in the assessment year.

Page 10: INTER CA Taxation - CA Study Web€¦ · INTER CA Taxation Head Office Shraddha, 4th Floor, ... 14. Advance Tax & Interest 268 - 271 15. Tax Deducted at Source (TDS) 272 - 294 16

: 7 :

J. K. SHAH CLASSES INTER C.A. - DIRECT TAXES

INTRODUCTION

Example : A and B are the joint authors of a book and are to share the remuneration

equally. The book is published in December, 2016 and in January 2017, as per the

Contract, A, the first author receives the entire remuneration of 2,50,000 and 50% of

the same is paid by A to B subsequently. This payment by A to B is "diversion of

income by overriding title", and his taxable income is only 1,25,000 and B's in-

come will also be 1,25,000. Now whatever A & B spend / invest out of their income of

1,25,000 each is an "application of income".

GROSS TOTAL INCOME (G.T.I.) [Section 80B(5)]

G.T.I. means the 'total income' computed under the five heads.

TOTAL INCOME [Section 2(45)]

It means the total amount of income referred to in section 5, computed in the

manner laid down in this Act.

* As per section 14, income of a person is computed under the following five heads

1. Income from salaries

2. Income from house property

3. Profits or gains of business or profession

4. Capital gains

5. Income from other sources

The aggregate of above is G.T.I. from which deductions under sections 80C to 80U are

deducted, and thus we arrive at the total income.

(3) PREVIOUS YEAR [Section 2(34)]

It means the previous year as defined in section 3. Section 3 has defined the pre-

vious year as the financial year immediately preceding the assessment year. In-

come tax is payable on the income earned during the previous year.It is brought to

tax in the immediately succeeding financial year, which is called as an Assess-

ment year. Therefore, the income earned during the previous year 2017-18 i.e. 1st

of April 2017 to 31st March, 2018, will be charged to tax in the assessment year

2018-19.

Provided that, in the case of a business or profession newly set up, or a source of

income newly coming into existence, in the financial year, the previous year shall

be the period beginning with the date of setting up of the business or profession

or, as the case may be, the date on which the source of income newly comes into

existence and ending with the said financial year i.e. 31st March.

For e.g. Mr. "S" sets up a new business on 10th July, 2017 then the period from

10.7.2017 to 31.3.2018 will be his first previous year for the assessment year 2018-

2019 Thereafter every year a period of 12 months of April-March will be his previous

year. Thus only in the first year previous year will be for a broken period.

(4) ASSESSMENT YEAR [Section 2(9)]

Assessment year means the period of twelve months commencing on the 1st day

of April every year.

Page 11: INTER CA Taxation - CA Study Web€¦ · INTER CA Taxation Head Office Shraddha, 4th Floor, ... 14. Advance Tax & Interest 268 - 271 15. Tax Deducted at Source (TDS) 272 - 294 16

: 8 :

J. K. SHAH CLASSES INTER C.A. - DIRECT TAXES

INTRODUCTION

This means that every financial year beginning on the first of April and ending on

the 31st of March, is the assessment year. For example, the period from the 1st

April, 2018 to 31st March 2019 is called assessment year 2018 - 2019 in respect

of previous year 2017 - 2018.

Thus Income-Tax is an annual tax, charged on the total income earned by a person.

For computation of Income, the law applicable for the relevant assessment year

should be referred. Income-tax law changes frequently.Income taxable in a year

may be exempt or taxable differently in another assessment year.

The rates of tax may change from one assessment year to another.Thus the amount

of tax may differ from one assessment year to another, although the income earned

is the same in both the years.

(5) TAX RATES FOR ASSESSMENT YEAR 2018-2019

Particulars Individual/H.U.F./A.O.P./B.O.I.A.J.P. Firm & Local Indian Company Foreign Company

Authority

Basic tax S l a b r a t e s Flat rate of 30% Flat rate of 30% Flat rate of 40%

Add: Surcharge I f To t a l I nc o me If Total Income If Total Income > `̀̀̀ 1 If Total Income > `̀̀̀ 1

> `̀̀̀50 lakhs, 10% > `̀̀̀ 1 Crore, 12% Crore but < = `̀̀̀ 10 Crore but < = `̀̀̀ 10

of Basic tax of Basic tax crores, 7% of Basic crores, 2% of Basic

tax. tax.

I f To t a l In co me If Total Income > If Total Income >

> `̀̀̀ 1 Crore, 15 % `̀̀̀10 crores, 12 % of `̀̀̀ 10 crores, 5% of

of Basic tax Basic tax. Basic tax

Add: Education 3% of (Basic tax – 3% of (Basic 3% of (Basic tax + 3% of (Basic tax +

Cess Rebate + Surcharge) tax + Surcharge) Surcharge) Surcharge)

TAX PAYABLE XXXX XXXX XXXX XXXX

(I) SLAB RATES

Resident Senior Citizens Resident Very Senior/ Others (Individuals aged < 60

(Age >= 60 years but < 80 Super Senior Citizens years at any time during the

years at any time during (Age >= 80 yearsat any P.Y., H.U.F, A.J.P.) and all not

the P.Y.) time during the P.Y.) resident individuals

Upto 3,00,000 NIL Upto 5,00,000 NIL Upto 2,50,000 NIL

3,00,001 - 5,00,000 5% 5,00,001 -10,00,000 20% 2,50,001 - 5,00,000 5%

5,00,001 -10,00,000 20% > 10,00,000 30% 5,00,001 -10,00,000 20%

> 10,00,000 30% > 10,00,000 30%

`̀̀̀ 3,00,000, `̀̀̀ 5,00,000 and `̀̀̀ 2,50,000 are called as " BASIC EXEMPTION LIMIT"

since income tax is payable by the person on the income exceeding this limit.

Note : If turnover or gross receipts for previous year 15-16 is upto 50 crores = basic tax is

25% (applicable only to companies).

Page 12: INTER CA Taxation - CA Study Web€¦ · INTER CA Taxation Head Office Shraddha, 4th Floor, ... 14. Advance Tax & Interest 268 - 271 15. Tax Deducted at Source (TDS) 272 - 294 16

: 9 :

J. K. SHAH CLASSES INTER C.A. - DIRECT TAXES

INTRODUCTION

(II)

An individual who is resident in India, and whose total income does not exceed

`̀̀̀ 3,50,000, shall be entitled to a deduction, of the amount of "Basic tax" (calculated as

per slab rates) or a maximum of `̀̀̀ 2,500, which ever is less.This rebate shall be

reduced from the basic tax.Please note that this rebate is availabe to "individuals"

only & not to the other six persons as per section 2(31).

(III) MARGINAL RELIEF

(ia) For persons other than companies

The total amount payable as income-tax and surcharge on total income

exceeding 50 lakhs rupees shall not exceed the total amount payable as

income-tax on a total income of 50 lakhs rupees by more than the amount of

income that exceeds one crore rupees.

If

[Basic tax + S.C. on Total Income] – [Basic tax on Total Income of `̀̀̀ 50 lakhs] >

Income in excess of `̀̀̀ 50 lakhs,

Then

Marginal Relief = [Basic tax + S.C. on Total Income] – [Basic tax on Total

Income of `̀̀̀ 50 lakhs] - Income in excess of `̀̀̀ 50 lakhs

(ib) For persons other than companies

The total amount payable as income-tax and surcharge on total income

exceeding one crore rupees shall not exceed the total amount payable as

income-tax on a total income of one crore rupees by more than the amount of

income that exceeds one crore rupees.

If

[Basic tax + S.C. on Total Income] – [Basic tax on Total Income of `̀̀̀ 1 crore]

> Income in excess of `̀̀̀ 1 crore,

Then

Marginal Relief = [Basic tax + S.C. on Total Income] – [Basic tax on Total

Income of `̀̀̀ 1 crore] - Income in excess of `̀̀̀ 1 crore.

(ii) For companies having income

(a) exceeding `̀̀̀ 1 crores but not exceeding `̀̀̀ 10 crores and

(b) exceeding `̀̀̀ 10 crores.

The total amount payable as income-tax and surcharge on total income

exceeding one crore rupees but not exceeding ten crore rupees, shall not

exceed the total amount payable as income-tax on a total income of one crore

Page 13: INTER CA Taxation - CA Study Web€¦ · INTER CA Taxation Head Office Shraddha, 4th Floor, ... 14. Advance Tax & Interest 268 - 271 15. Tax Deducted at Source (TDS) 272 - 294 16

: 10 :

J. K. SHAH CLASSES INTER C.A. - DIRECT TAXES

INTRODUCTION

rupees, by more than the amount of income that exceeds one crore rupees.

The total amount payable as income-tax and surcharge on total income

exceeding ten crore rupees, shall not exceed the total amount payable as

income-tax and surcharge on a total income of ten crore rupees, by more than

the amount of income that exceeds ten crore rupees.

If

[Basic tax + S.C. on Total Income] – [Basic tax + S.C. on Total Income of `̀̀̀ 10

crore] > Income in excess of `̀̀̀ 10 crore,

Then

Marginal Relief = [Basic tax + S.C. on Total Income] – [Basic tax on Total

Income of `̀̀̀ 10 crore] - Income in excess of `̀̀̀ 10 crore.

OTHER DEFINITIONS / TERMINOLOGIES/CONCEPTS

(a) Assessee [Section 2(7)]

Assessee means a person by whom any tax or penalty or interest is payable under

the Act. The term includes the following persons :

1. Every person in respect of whom any proceeding under the Act, has been

taken for (a) his income (or loss) or assessment of fringe benefits or (b) of

the income (or loss) of any other person in respect of which he is assessable

i.e. Legal Representative.

2. Every person in respect of whom any proceeding under the Act has been

taken for computation of refund due to him or to such person in respect of

whom he is assessable.

3. Every person who is deemed to be an assessee under any provision of the

Act (for instance, 'representative assessee' is treated as deemed assessee

in respect of income or loss of another person under Section 160 .i.e. say,

trustee in respect of a trust).

4. Every person who is deemed to be an assessee in default under any

provision of the Act (for instance, a person who is under duty to deduct tax at

source on payment of salary, dividend, interest, etc. does not deduct tax, or

after deducting tax fails to deposit the same to the Government's treasury, is

treated as assessee in default).

(b) Representative assessee : Section 160 gives the definition of

representative assessee as follows :

(i) in respect of the income of a non - resident, the agent of the non - resident including

a person who is treated as an agent under section 163.

(ii) in respect of the income of a minor, lunatic or idiot, the guardian or manager

who is entitled to receive or is in receipt of such income on behalf of the

minor,lunatic or idiot.

Page 14: INTER CA Taxation - CA Study Web€¦ · INTER CA Taxation Head Office Shraddha, 4th Floor, ... 14. Advance Tax & Interest 268 - 271 15. Tax Deducted at Source (TDS) 272 - 294 16

: 11 :

J. K. SHAH CLASSES INTER C.A. - DIRECT TAXES

INTRODUCTION

(iii) in respect of income which the Court of Wards, the Administrator

General, the Official Trustee or any receiver or manager appointed by or

under any order of the Court, receives or is entitled to receive, such Court of

Wards, Administrator General, Official Trustee, receiver or manager.

(iv) in respect of income which a trustee appointed under a trust declared by a duly

executed instrument in writing (including a Walk deed) receives or is entitled to

receive on behalf of or for the benefit of the person, such trustee or trustees.

(v) in respect of income which a trustee appointed under an oral trust

receives or is entitled to receive on behalf or for the benefit of the person, such

trustee or trustees.

Every representative assessee shall be deemed to be an assessee for the pur-

poses of the Act.

(c) Relative [Section 2(41)]

'Relative', in relation to an individual, means the husband, wife, brother or sister or any

lineal ascendant or descendant of that individual.

(d) Child [Section 2(15B)]

Child, in relation to an individual, includes a step-child and an adopted child of that

individual.

Rounding - off of Income / Tax [Section 288A / 288B]

The taxable income / tax shall be rounded off to the nearest multiple of ten rupees and

for this purpose any part of a rupee consisting of paise shall be ignored and thereafter

if such amount is not a multiple of ten, then, if the last figure in that amount is five or

more, the amount shall be increased to the next higher amount which is multiple of ten

and if the last figure is less than five, the amount shall be reduced to the next lower

amount which is a multiple of ten.

Example :

Income / tax before rounding off Income / Tax after rounding off as per sec. 288A /

288 B

1, 23, 454.90 1, 23, 450

1, 23, 455.00 1, 23, 460

1, 23, 458.90 1, 23, 460

1, 23, 464.80 1, 23, 460

Page 15: INTER CA Taxation - CA Study Web€¦ · INTER CA Taxation Head Office Shraddha, 4th Floor, ... 14. Advance Tax & Interest 268 - 271 15. Tax Deducted at Source (TDS) 272 - 294 16

: 12 :

J. K. SHAH CLASSES INTER C.A. - DIRECT TAXES

INTRODUCTION

Substantial Interest [section 2(32)]

"Person who has a substantial interest in the Company", in relation to a company, means

a person who is the beneficial owner of shares, [not being shares entitled to a fixed

rate of dividend whether with or without a right to participate in profits i.e. equity shares]

carrying not less than 20% of the voting power.

In the case of a non - corporate entity, a person can be said to have substantial interest

if he is entitled for 20% or more of share of profit.

Section 115O to 115Q : Dividend Distribution Tax (DDT) - shares

Dividends whether interim or final or deemed, declared and paid on or after

1.4.2003 by a domestic company (whether out of current or accumulated profits)

shall be charged to additional income-tax (tax on distributed profits) at the rate of

15% plus surcharge of 12% plus education cess of 3% i.e., effective tax rate of

17.304% over and above the normal income tax on the income of the domestic company.

The D.D T. basic rate ( excluding surcharge and education cess)will be

grossed up as under:-

Gross dividend say = 100

Then D. D . T. @15% = 15

Net dividend received = 85

If 15 is expressed as a % of dividend actually received, then :

15 x 100 =17.647% effective basic rate excluding Surcharge & Education Cess.

85

Now effective rate including surcharge and education cess will be :

Basic effective rate : 17.647

+ 12% surcharge : 2.118

Total 19.765

+ 3% education cess : 0.593

Effective rate : 20.358

Sec. 115R to 115T : Dividend Distribution Tax - Units of Mutual Fund

Any amount of income distributed by the specified company [as per sec.

2(h) of U.T.I. (Transfer of undertaking & Repeal) Act, of 2002] or a mutual

fund to its unit holders shall be chargeable to tax at the rate of :

1. 25% on income distributed to an individual or H.U.F.

2. 30% on income distributed to any person other than individual and

H.U.F.

The above rates shall be increased by surcharge of 12% plus education cess

of 3%.

Similar to section 115-0 grossing up :

���� For Individual & HUF recipient

* D.D.T. 25 33.33% effective basic rate

Net received 75 x 100 =

excluding surcharge & education cess

Page 16: INTER CA Taxation - CA Study Web€¦ · INTER CA Taxation Head Office Shraddha, 4th Floor, ... 14. Advance Tax & Interest 268 - 271 15. Tax Deducted at Source (TDS) 272 - 294 16

: 13 :

J. K. SHAH CLASSES INTER C.A. - DIRECT TAXES

INTRODUCTION

* Effective rate including surcharge & education cess.

Basic effective rate : 33.333 %

(+) 12% Surcharge : 3.996%

Total : 37.329%

(+) 3% education cess : 1.119%

38.449%

���� Other than Individual & HUF recipient

* (D.D.T. 30) 42.85% excluding surcharge &

(Net received 70)x 100 =

education cess

Basic rate : 42.85%

(+) 12 % Surcharge : 5.14%

Total : 47.99%

(+) 3 % education cess : 1.44%

Effective rate : 49.43 % including

surcharge & educational Cess

CASES WHERE INCOME OF PREVIOUS YEAR IS ASSESSED IN THE SAME YEAR

Normally, income earned during any previous year is assessed or charged to tax in the

immediately succeeding assessment year. In the undermentioned cases, the income is

assessed and brought to tax in the same previous year of earning the income. Thus the

previous year and the assessment year in these exceptional cases will be the same.

These exceptions have been incorporated in order to ensure smooth collection of income -

tax from such tax payers who may not be traceable if tax collection procedure is postponed till

the commencement of normal assessment year.

(a) Income of the non - resident from shipping business [Section 172]

A non - resident who is carrying on a shipping business of carrying passengers / goods etc.

from any port in India, will have to pay income - tax before the ship is allowed to leave the

Indian port. Thus, before the vessel leaves the Indian port, the master of the ship is duty

bound to submit the details of the revenue collected from India. 7.5% of the revenue

collected will be deemed to be the income (i.e. 92.5% of revenue is considered expenses)

and tax is levied at the rate applicable to a foreign company.

(b) Income of a person permanently leaving India [Sec. 174]

If the Assessing Officer (A.O.) has knowledge that any individual may leave India

permanently during the current assessment year or shortly after its expiry, the total

income of such individual upto the date of departure from India will be chargeable to

tax in the same assessment year.

Example : If a foreign national who is resident in India since long is leaving India permanently

on 4.5.18 and the assessing officer while completing the assessment of P.Y. 2016-17 A.Y.

2017-18 on 20.3.2018 knows this fact. The A.O. shall make the following three

assessments.

(i) For the P.Y. 2016-17 (1.4.16 to 31.3.17 income earning period) at the normal

rates of tax applicable for A.Y. 2017-2018.

(ii) For the P.Y. 2017-18 (1.4.17 to 31.3.18 income earning period) at the rates of

advance tax given in Part III of the first schedule to the FINANCE ACT, 2017,

applicable for A.Y. 2018-19.

(iii) For the income earning period 1.4.18 to 4.5.18(on estimated basis) i.e. P.Y. 18-

19 also assessed in advance at the rates applicable for A.Y. 18-19 as per (ii)

above, because as at 20.3.18 (date of assessment) the Finance Bill of 2018

would not have been passed and would not have become the Finance Act, 2018.

Page 17: INTER CA Taxation - CA Study Web€¦ · INTER CA Taxation Head Office Shraddha, 4th Floor, ... 14. Advance Tax & Interest 268 - 271 15. Tax Deducted at Source (TDS) 272 - 294 16

: 14 :

J. K. SHAH CLASSES INTER C.A. - DIRECT TAXES

INTRODUCTION

(c) Income of AOP / BOI / Artificial juridical person formed for a short duration [Sec. 174A]

If the A.O. is of the opinion that the above entities are formed for a short duration and

are likely to be dissolved in the assessment year or shortly after such assessment year,

the total income of such entity for the period from the commencement of that assessment

year, upto the date of its dissolution shall be chargeable to tax in that assessment year.

Example : If an A.O.P. consisting of one firm and one HUF is formed for

constructing & completing one fly over and the construction is likely to be completed

on 9.9.18. During the assessment proceedings on 8.8.18 of P.Y. 17-18 A.Y. 18-19,

the A.O. comes to know about the project completion and will make the following

two assessments for the Assessment Year 2018-19 :

Normal assessment for the previous year 17-19 (i.e. income earning period of

1.4.17 to 31.3.18) at the normal rates of tax applicable for A.Y. 18-19.

For the income period of 1.4.18 to 9.9.18 i.e. A.Y. 19-20 at the rates of tax

applicable for Advance tax payment in relation to A.Y. 19-20 i.e. rates of tax

given in Part III of the first schedule to Finance Act, 2018.

(d) Assessment of persons likely to transfer property to avoid tax [Sec. 175]

If it appears to the Assessing Officer during any current assessment year, that any person

is likely to sell, transfer etc. any of his assets with an intention to avoid his tax liability, then

the total income of such person from the commencement of A.Y. till the date of commencement

of assessment proceedings, shall be chargeable to tax in the same A.Y.

(e) Discontinued business [Sec. 176]

At the discretion of the assessing officer, if any business or profession is discontinued in

any assessment year, the income from the commencement of the assessment year till the

date of such discontinuation may be charged to tax in that assessment year.

Calculate Tax Payable

Q. 1. Person is Individual aged 55 years if income is

Case A = 10,00,000

Case B = 50,00,000

Case C = 1,05,00,000

Q. 2. Person is Indian Company and If income is

Case A = 80,00,000

Case B = 1,00,00,000

Case C = 5,00,00,000

Case D = 10,00,00,000

Case E = 15,00,00,000

Q. 3. Person is HUF and If Income is

Case A = 4,00,000

Case B = 15,00,000

Page 18: INTER CA Taxation - CA Study Web€¦ · INTER CA Taxation Head Office Shraddha, 4th Floor, ... 14. Advance Tax & Interest 268 - 271 15. Tax Deducted at Source (TDS) 272 - 294 16

: 15 :

J. K. SHAH CLASSES INTER C.A. - DIRECT TAXES

RESIDENCE OF AN ASSESSEE

WHAT IS RELEVANCE OF RESIDENTIAL STATUS

Tax incidence on a assessee depends on his residential status. For instance, whether an

income, accrued to an individual outside India, is taxable in India depends upon the residential

status of the individual in India. Similarly, whether an income earned by a foreign national in

India (or outside India) is taxable in India, depends on the residential status of the individual,

rather than on his citizenship. Therefore, the determination of the residential status of a person

is very significant in order to find out his tax liability.

WHAT ONE MUST KNOW FOR DECIDING RESIDENTIAL STATUS

���� Residential status is a term coined under Income Tax Act and has nothing to do with

nationality or domicile of a person. An Indian, who is a citizen of India can be non-

resident for Income-tax purposes, whereas an American who is a citizen of America

can be resident of India for Income-tax purposes. Residential status of a person depends

upon the territorial connections of the person with this country, i.e., for how many days

he has physically stayed in India.

���� The residential status of different types of persons is determined differently. Similarly,

the residential status of the assessee is to be determined each year with reference to

the “previous year”. The residential status of the assessee may change from year to

year. What is essential is the status during the previous year and not in the assessment year.

���� Duty of Assessee - It is assessee’s duty to place relevant facts, evidence and material

before the Income Tax Authorities supporting the determination of Residential status.

���� Dual Residential Status is possible - A person may be resident of one or more

countries in a relevant previous year e.g., Mr. X may be resident of India during previous

year 2013 14 and he may also be resident/non-resident in England in the same previous

year. The emergence of such a situation depends upon the following

(a) the existence of the Residential status in countries under considerations

(b) the different set of rules having laid down for determination of residential status.

���� Different residential status – An assessee is either (a) Resident in India, or (b) non

– resident in India. However, a resident Individual or a Hindu undivided family has to be

(a) resident and ordinarily resident, or (b) resident but not ordinarily resident. Therefore,

an individual and a Hindu undivided family can either be :

(a) Resident and ordinarily resident

(b) resident but not ordinarily resident

(c) Non resident in India.

All other assessees (viz, a firm, an association of person, a joint stock company and

every other person) can either be:

(a) Resident in India or

(b) Non resident in India

RESIDENCE OF AN ASSESSEE

Page 19: INTER CA Taxation - CA Study Web€¦ · INTER CA Taxation Head Office Shraddha, 4th Floor, ... 14. Advance Tax & Interest 268 - 271 15. Tax Deducted at Source (TDS) 272 - 294 16

The following chart highlights the same –

Person

Individual & HUF Others (Company, Firm, AOP, BOI, etc.)

Resident in India Non-resident Resident in India Non -resident

Ordinarily Resident in India Not Ordinarily in India

HOW TO DETERMINE RESIDENTIAL STAUS OF AN INDIVIDUAL – SECTION 6(1)

1. Basic Conditions:

(a) If the Individual stayed in India for a period of 182 DAYS OR MORE during the

Relevant Previous Year (RPY), he is Resident of India;

(OR)

(b) If he stayed in India for a period of 60 DAYS OR MORE during Relevant Previous

Year (RPY) and 365 DAYS OR MORE during the four preceding Previous Years,

he is Resident of India.

If the assessee fails to satisfy either of the above basic conditions, as applicable, then

the assessee is a Non-Resident for that Relevant Previous Year.

2. Additional Conditions: Sec. 6(6)(a)

(i) Resident in India for at least 2 years out of the preceding 10 Previous Years.

(Preceding to relevant PY)

(ii) Stay in India for at least 730 days during the 7 preceding Previous

Years(Preceding to relevant PY).

If the assessee satisfies above additional conditions, then assessee is Resident and

ordinarily otherwise Resident but not ordinarily resident.

Resident Basic Non - Resident ANY NONE

Resident

and

ordinarily

Resident

Additional

condition

(2 condition)

Resident but

not ordinarily

ResidentBOTH

ANY ONE OR

NONE

Residential Status in case of individual – section 6(1)

J. K. SHAH CLASSES INTER C.A. - DIRECT TAXES

: 16 : RESIDENCE OF AN ASSESSEE

Page 20: INTER CA Taxation - CA Study Web€¦ · INTER CA Taxation Head Office Shraddha, 4th Floor, ... 14. Advance Tax & Interest 268 - 271 15. Tax Deducted at Source (TDS) 272 - 294 16

: 17 :

J. K. SHAH CLASSES INTER C.A. - DIRECT TAXES

RESIDENCE OF AN ASSESSEE

Special exceptional situations:

For the following persons, condition mentioned in 1(a) above shall only apply to determine

their Residential Status:

(i) Indian citizen who leaves India during the previous year for the purpose of employment

outside India.

(ii) Indian Citizen who leaves India during the previous year as a member of the crew of an

Indian ship, or

(iii) Indian citizen or a person of Indian origin who comes on a visit to during the previous year.

A person is deemed to be of Indian origin if he, or either of his parents or any of his grand

parents was born in undivided India. Grand parents include both maternal and paternal.

Note:

1. The day on which he enters India as well as the day on which he leaves India

shall be taken into account as the stay of the Individual in India.

2. The stay in India need not be at the same place

3. It is also not essential that the stay should be continuous.

4. The place of stay and purpose of stay in India, is not material

HOW TO DETERMINE RESIDENTIAL STAUS OF AN HINDU UNDIVIDED FAMILY–

SECTION 6(2)

A Hindu Undivided Family is said to be R&OR/RNOR in India if control and management of its

affairs is, wholly or partly, situated in India. A Hindu Undivided Family is nonresident in India if

control and management of its affairs is wholly situated outside India.

1. Control and management means de- facto control and management and not merely the

right to control or manage. Control and management is situated at a place where the

head, the seat and the directing power are situated.

2. And ordinarily/ not ordinarily depends upon the individual residential status of the karta,

i.e. satisfaction of additional conditions by the Karta.

HOW TO DETERMINE RESIDENTIAL STAUS OF FIRM AND ASSOCIATION OF PER-

SON – SECTION 6(2)

A partnership firm and an association of person are said to be resident in India if control and

management of their affairs are wholly or partly situated within India during the relevant

previous year.

HOW TO DETERMINE RESIDENTIAL STAUS OF COMPANY [SECTION 6(3)]

An Indian company is always resident in India. A foreign company is resident in India

only if its "Place of effective management", in that year, is in India.

The term "Place of effective management" means a place where Key management

and commercial decisions that are necessary for the conduct of the business of an

entity as a whole are, in substance, made.

Usually control and management of a company's affairs is situated at the place where

meetings of board of directors are held. Moreover, control and management referred

to in section 6 is central control and management and not the carrying on of day to day

business by servants, employees or agents.

The term "control" does not mean shareholding control.

Page 21: INTER CA Taxation - CA Study Web€¦ · INTER CA Taxation Head Office Shraddha, 4th Floor, ... 14. Advance Tax & Interest 268 - 271 15. Tax Deducted at Source (TDS) 272 - 294 16

: 18 :

J. K. SHAH CLASSES INTER C.A. - DIRECT TAXES

RESIDENCE OF AN ASSESSEE

HOW TO DETERMINE RESIDENTIAL STAUS OF "EVERY OTHER PERSON" [SECTION 6(4)]

Every other person is resident in India if control and management of his affairs is wholly or

partly situated within India during the relevant previous year. On the other hand, every other

person is non-resident in India if control and management of its affairs is wholly situated

outside India.

SCOPE OF INCOME (SECTION 5)

Page 22: INTER CA Taxation - CA Study Web€¦ · INTER CA Taxation Head Office Shraddha, 4th Floor, ... 14. Advance Tax & Interest 268 - 271 15. Tax Deducted at Source (TDS) 272 - 294 16

: 19 :

J. K. SHAH CLASSES INTER C.A. - DIRECT TAXES

RESIDENCE OF AN ASSESSEE

Incomes deemed to accrue or arise in India [Section 9]

Certain Incomes are deemed to accrue or arise in India u/s 9 even though they may actually

arise outside India. The following is a list of such incomes in simple words.

1. Income from business connection – The following conditions should be satisfied –

Condition one – The Taxpayer has a “business connection” in India.

Condition Two – By virtue of business connection in India, income actually arises

outside India

If above conditions are satisfied, income which arises outside India because of business

connection in India is deemed to accrue or arise in India.

Some illustrative instances of a non-resident having business connection in India, are

given below:

(a) Maintaining a branch office in India for the purchase or sale of goods or

transacting other business.

(b) Appointing an agent in India for the systematic and regular purchase of raw

materials or other commodities, or for sale of the non-resident's goods, or for

other business purposes.

(c) Erecting a factory in India where the raw produce purchased locally is worked

into a form suitable for export abroad.

(d) Forming a local subsidiary company to sell the products of the non-resident

parent company.

(e) Having financial association between a resident and a non-resident company.

Explanation 1 to section 9(1)(i) lists out income which shall not be deemed to accrue or

arise in India. They are given below:

(1) In the case of a business, in respect of which all the operations are not carried

out in India [Explanation 1(a) to section 9(1)(i)]

In the case of a business of which all the operations are not carried out in India,

the income of the business deemed to accrue or arise in India shall be only such

part of income as is reasonably attributable to the operations carried out in India.

Therefore, it follows that such part of income which cannot be reasonably

attributed to the operations in India, is not deemed to accrue or arise in India.

(2) Purchase of goods in India for export [Explanation 1(b) to section 9(1)(i)]

In the case of a non-resident, no income shall be deemed to accrue or arise in

India to him through or from operations which are confined to the purchase of

goods in India for the purpose of export.

(3) Collection of news and views in India for transmission out of India [Explanation

1(c) to section 9(1)(i)]

In the case of a non-resident, being a person engaged in the business of running a

news agency or of publishing newspapers, magazines or journals, no income shall

be deemed to accrue or arise in India to him through or from activities which are

confined to the collection of news and views in India for transmission out of India.

(4) Shooting of cinematograph films in India [Explanation 1(d) to section 9(1)(i)] In

the case of a non-resident, no income shall be deemed to accrue or arise in

India through or from operations which are confined to the shooting of any

cinematograph film in India, if such non-resident is :

(i) an individual, who is not a citizen of India or

(ii) a firm which does not have any partner who is a citizen of India or who is

resident in India; or

(iii) a company which does not have any shareholder who is a citizen of India

or who is resident in India.

Page 23: INTER CA Taxation - CA Study Web€¦ · INTER CA Taxation Head Office Shraddha, 4th Floor, ... 14. Advance Tax & Interest 268 - 271 15. Tax Deducted at Source (TDS) 272 - 294 16

: 20 :

J. K. SHAH CLASSES INTER C.A. - DIRECT TAXES

RESIDENCE OF AN ASSESSEE

2. Income through or from any property, any asset or source of income in India.

3. Income through the transfer of Capital Asset situated in India.

4. Income which falls under the head "Salaries" shall be regarded as income earned in

India if the income is payable for -

service rendered in India, and

the rest period or leave period, which is preceded and succeeded by services,

rendered in India and forms part of the service contract of employment.

5. Salary paid by the Government to a citizen of India for rendering service outside India.

However the allowances and perquisites paid outside India by the Government to the

citizen of India, is exempt u/s 10(7).

6. Dividend paid by an Indian company outside India.

7. Interest payable by :

a) Government ; or

b) any resident person, unless the interest is payable on any debt incurred or

money borrowed and used for a business or profession carried on by him

outside India or for earning any income from any source outside India ; or

c) any non-resident person, when the interest is payable on any debt incurred or

money's borrowed and used for the purpose of a business or profession

carried on by him in India.

Page 24: INTER CA Taxation - CA Study Web€¦ · INTER CA Taxation Head Office Shraddha, 4th Floor, ... 14. Advance Tax & Interest 268 - 271 15. Tax Deducted at Source (TDS) 272 - 294 16

: 21 :

J. K. SHAH CLASSES INTER C.A. - DIRECT TAXES

RESIDENCE OF AN ASSESSEE

8. Royalty payable by :

(a) Government ; or

(b) any resident person, unless the royalty is payable in respect of any right, property

or information used or services utilised for the purposes of a business or

profession carried on by him outside India or for earning any income from

any source outside India ; or

(c) any non-resident person, when the royalty is payable in respect of any right,

property or information used or services utilised for the purposes of a

business or profession carried on by him in India or for earning any income

from any source in India.

9. Income by way of fees for technical services payable by :

a) Government ; or

b) any resident person, unless the technical fees is payable for a business or

profession carried on by him outside India or for earning any income from

any source outside India ; or

c) any non resident person, when the technical fees is payable for a business

or profession carried on by him in India or for earning any income from any

source in India.

The income of the non-resident shall be deemed to accrue or arise in India, whether

or not -

(i) the non-resident has a residence or place of business or business connection

in India

OR

(ii) the non-resident has rendered services in India.

OR

(iii) The possession or control of such right, property or information is with the

payer or used directly by the payer or the location is in India.

Page 25: INTER CA Taxation - CA Study Web€¦ · INTER CA Taxation Head Office Shraddha, 4th Floor, ... 14. Advance Tax & Interest 268 - 271 15. Tax Deducted at Source (TDS) 272 - 294 16

: 22 :

J. K. SHAH CLASSES INTER C.A. - DIRECT TAXES

RESIDENCE OF AN ASSESSEE

Page 26: INTER CA Taxation - CA Study Web€¦ · INTER CA Taxation Head Office Shraddha, 4th Floor, ... 14. Advance Tax & Interest 268 - 271 15. Tax Deducted at Source (TDS) 272 - 294 16

: 23 :

J. K. SHAH CLASSES INTER C.A. - DIRECT TAXES

RESIDENCE OF AN ASSESSEE

Q. 1. Dhaval, an Indian citizen serving in the U.S.A. since 1969, came to India for the first

time after 1969 on 30th June, 2011. He stayed here at a stretch for 3 years and left for

Japan on 1st July, 2014. He returned to India on 1st April, 2015 and remained in India

till 31st July 2016 when he went back to the U.S.A. He, permanently, again came to

India taking an employment with an American concern on 31st December, 2017.

Determine Residential status for assessment year 2018 - 19.

Q. 2. X & Co. is an Indian company carrying on business in India as well as in British

East Africa. The control and management of its affairs was wholly situated in India

during the year ended 31st March, 2018. Its income accruing or arising in British

East Africa in that year far exceeded its income accruing or arising in India.

Determine Residential status for assessment year 2018 - 19.

Q. 3. X, an Indian citizen, who is appointed as senior taxation officer by the Government

of Nigeria, leaves India, for the first time, on September 20, 2016 for joining his

duties in Nigeria. During the previous year 2017-2018 he comes to India on a visit

for 176 days. Determine the residential status of X for the assessment years

2017-18 and 2018-19.

Q. 4. 'F' was born in FRANCE in 1969 and his father was born in Australia in 1939, but

F's Grand father was born in Dhakka in 1915. Will 'F'be a resident in India if he

visits India for 181 days during the previous year 2017-18.

Determine Residential status for assessment year 2018 - 19

Q. 5. X, a citizen of India, left India for the 1st time on 6.6.2014 for employment abroad.

During 2015-16 and 2016-17 he visited India for 145 days and 165 days

respectively. In the previous year 2017-18 he came to India on 7.4.17 and left on

30.11.17. Determine the residential status for the assessment year 2018-19.

Q. 6. Mr. A is an Indian citizen and a member of the crew of a Singapore bound Indian ship

engaged in carriage of passengers in international traffic departing from Chennai port on

6th June, 2017. From the following details for the P.Y.2017-18, determine the residential

status of Mr. A for A.Y.2018-19, assuming that his stay in India in the last 4 previous years

(preceding P.Y.2017-18) is 400 days and last seven previous years (preceding P.Y.2017-

18) is 750 days:

Particulars Date

Date entered into the Continuous Discharge Certificate in respect

of joining the ship by Mr. A 6th June, 2017

Date entered into the Continuous Discharge Certificate in respect

of signing off the ship by Mr. A 9thDecember 2017

Page 27: INTER CA Taxation - CA Study Web€¦ · INTER CA Taxation Head Office Shraddha, 4th Floor, ... 14. Advance Tax & Interest 268 - 271 15. Tax Deducted at Source (TDS) 272 - 294 16

: 24 :

J. K. SHAH CLASSES INTER C.A. - DIRECT TAXES

RESIDENCE OF AN ASSESSEE

Q. 7. Determine the taxability of income of US based company Heli Ltd., in India on

entering following transactions during the financial year 2017-18.

(i) 5 lacs received from an Indian domestic company for providing technical

know how in India.

(ii) 6 lacs from an Indian firm for conducting the feasibility study for the new

project in Finland.

(iii) 4 lacs from a non-resident for use of patent for a business in India.

(iv) 8 lacs from a non-resident Indian for use of know how for a business in Singapore.

(v) 10 lacs for supply of manuals and designs for the business to be

established in Singapore.No payment for the same was made in India.

Q.8. Following are the particulars of taxable income of Mr R for the previous year ended

31.03.2018.

1. Royalty received from Government of India Rs 24,000

2. Income from business earned in Afganistan Rs 25,000 of which ̀̀̀̀ 15,000 were re-

ceived in India(Controlled from Afganistan).

3. Interest received from G a non- resident against a loan provided to him to run a

business in India `̀̀̀ 5000.

4. Royalty Received in India from S a resident for technical services provided to run a

business outside India `̀̀̀ 20,000.

5. Income from business in Jaipur `̀̀̀40,000. This business is controlled from France

` ` ` ` 20,000 were remitted to France.

6. Profit on sale of shares in Indian Company received in Germany Rs 15,000

7. Dividend

- From Japanese Company received in Japan – `̀̀̀10,000

- From RP Ltd an Indian Company – `̀̀̀ 5,000

8. Income from property in London deposited in a bank in London, later on remitted to

India ̀̀̀̀ 1,00,000.

9. Income from Business in Canada Controlled from Mumbai `̀̀̀ 50,000 of which

`̀̀̀ 27,000 is received in India.

Find out Total Income of Mr R, if he is –

a) Resident and Ordinarily Resident

b) Resident but not Ordinarily Resident

c) Non Resident.

Q . 9. State with reasons whether the following transcations attract income tax in India in the hands

of recipients .

(i) Salary paid by Central Government to Mr. John, a citizen of India `̀̀̀ 7,00,000 for the

services rendered outside India.

(ii) Interest on moneys borrowed from outside India `̀̀̀ 5,00,000 by a non-resident for

the purpose of business within India say in Mumbai.

(iii) Royalty paid by a resident to a non- resident in respect of a business carried on

outside India.

(iv) Legal charges of `̀̀̀ 5,00,000 paid to a lawyer of United Kingdom who visited India to

represent a case at the Delhi High court.

Page 28: INTER CA Taxation - CA Study Web€¦ · INTER CA Taxation Head Office Shraddha, 4th Floor, ... 14. Advance Tax & Interest 268 - 271 15. Tax Deducted at Source (TDS) 272 - 294 16

: 25 :

J. K. SHAH CLASSES INTER C.A. - DIRECT TAXES

RESIDENCE OF AN ASSESSEE

Ans.6. In this case, the voyage is undertaken by an Indian ship engaged in the carriage

of passengers in international traffic, originating from a port in India (i.e., the

Chennai port) and having its destination at a port outside India (i.e., the Singa-

pore port). Hence, the voyage is an eligible voyage for the purposes of section

6(1). Therefore, the period beginning from 6th June, 2017 and ending on 9th

December, 2017, being the dates entered into the Continuous Discharge Cer-

tificate in respect of joining the ship and signing off from the ship by Mr. A, an

Indian citizen who is a member of the crew of the ship, has to be excluded for

computing the period of his stay in India. Accordingly, 187 days

[25+31+31+30+31+30+9] have to be excluded from the period of his stay in

India. Consequently, Mr. A’s period of stay in India during the P.Y.2017-18 would

be 178 days [i.e., 365 days – 187 days]. Since his period of stay in India during

the P.Y.2017-18 is less than 182 days, he is a non-resident for A.Y.2018-19.

Note - Since the residential status of Mr. A is “non-resident” for A.Y.2018-19 con-

sequent to his number of days of stay in P.Y.2017-18 being less than 182 days,

his period of stay in the earlier previous years become irrelevant.

Ans. 7. A non- resident is chargeable to tax in India in respect of following incomes.

(i) Income received or deemed to be received in India, and

(ii) Income accruing or arising or deemed to accrue or arise in India.

In view of the above provisions taxability of income is determined in following manner :

Sr. no. Particulars `̀̀̀(in lacs)

(i) Amount received from an Indian domestic company for 5

providing technical knowhow in India is deemed to accrue

or arise in India and is, therefore, taxble in India.

(ii) Conducting the feasibility study for the new project in finland Nil

for the Indian firm is not taxable in India as the income

accrues outsde India since such study is done for a

business outside India

(iii) Income received from a non-resident for use of patent for a 4

business in India is taxable in India as it is deemed to

accruse or arise in India.

(iv) Income received from a non-resident Indian for use of knowhow Nil

for a business in Singapore.It is not taxable in India since

it does not accrue or arise in India nor it is deemed to

accrue or arise in India.

(v) Income received for supply of manuals and designs for the Nil

business to be established in Singapore is not taxable in India,

since it does not accrue or arise nor is it deemed to accrue

or arise in India.

Total Income 9

CLASS WORK SOLUTIONS

Page 29: INTER CA Taxation - CA Study Web€¦ · INTER CA Taxation Head Office Shraddha, 4th Floor, ... 14. Advance Tax & Interest 268 - 271 15. Tax Deducted at Source (TDS) 272 - 294 16

: 26 :

J. K. SHAH CLASSES INTER C.A. - DIRECT TAXES

RESIDENCE OF AN ASSESSEE

Ans. 9. Taxable / Amount Reason

Not liable

Taxable to tax( `̀̀̀)

(i) Taxable 7,00,000 As per section 9(1) (iii), salaries payble by the

Governemnt to a citizen of India for service rendered

outside India shall be deemed to accrue or arise in

Inida. Therefore salary paid by Central Government

to Mr. Jogn for services rendered outside India would

be deemed toa ccrue or arise in India since he is

citizen of India.

(ii) Taxable 5,00,000 As per section 9(1) (v)(c) interest payable by a

non- resident on moneys borrowed and used for the

purposes of business carried on by such person in

India shall be deemed to accrue or arise in India in

the hands of the recipient

(iii) Not ---- Royalty paid by a resident to a non - resident in

Taxable respect of a business carried outside India would

be taxable in the hands of the non- resident provided

the same is not received in India This has been

provided as on exception to deemed accrual

mentioned in section 9(1) (vi)(b)

(iv) Taxable 5,00,000 In case of a non-resident any income which accrues

or arises in India or which is deemed to accrue or

arise in India or which is received in India or is

deemed to be received in India is taxable in India.

Therefore, legal changes paid in India to a non-

resident lawyer of UK, who visited India to repre

sent a case at the Delhi High Court would be taxble in India.

Page 30: INTER CA Taxation - CA Study Web€¦ · INTER CA Taxation Head Office Shraddha, 4th Floor, ... 14. Advance Tax & Interest 268 - 271 15. Tax Deducted at Source (TDS) 272 - 294 16

: 27 :

J. K. SHAH CLASSES INTER C.A.- DIRECT TAXES

SALARIES

General Points :

In order to understand the meaning of expression “Salary” one has to keep in mind the

following :

(i) Relationship between payer and payee:

A payment cannot be taxed under the head “Salary” unless the relationship of em-

ployer and employee exists between the payer and payee.

(ii) A member of par liament or state legislature is not treated as an

employee of the Government.

Salary and allowances received by him, are therefore, chargeable to tax under

the head “Income from other sources”.

(iii) Salary and wages :

Conceptually there is no difference between salary and wages. Both are com-

pensation for work done or services rendered, though ordinarily salary is paid in

connection with service of non-manual type of work, while wages are paid in connec-

tion with manual services, therefore, remuneration received by an individual is taxable

under the head “Salaries” whether the remuneration is termed as salary or wages.

INCOME FROM SALARIES

Page 31: INTER CA Taxation - CA Study Web€¦ · INTER CA Taxation Head Office Shraddha, 4th Floor, ... 14. Advance Tax & Interest 268 - 271 15. Tax Deducted at Source (TDS) 272 - 294 16

: 28 :

J. K. SHAH CLASSES INTER C.A.- DIRECT TAXES

SALARIES

(iv) Salary from more than one source :

If an individual receives salary from more than one employer during the same previous

year, salary from each source is taxable under the head “Salaries”.

(v) Salary from former employer, present employer or prospective employer:

It is chargeable to tax under the head “Salaries”.

(vi) Foregoing of salary :

If an employee foregoes his salary, it does not mean that salary so foregone is not

taxable. Once salary has accrued to an employee, its subsequent waiver does not make

it exempt from tax liability. Such voluntary waiver or foregoing by an employee of salary due

to him, is merely an application of income and which is nonetheless chargeable to tax.

(vii) Surrender of salary :

If an employee opts to surrender his salary to the Central Government under section 2 of

the Voluntary Surrender of Salaries Act, the salary so surrendered would be excluded while

computing his taxable income. Thus, tax is not payable in respect of salary surrendered,

which can be basic salary as well as different allowances.Benefit of tax exemption in re-

spect of salary surrendered is available to all employees whether they are employed in

private sector or public sector.

(ix) Advance Salary :

Advance Salary is taxable when it is received by the employee, irrespective of the fact

whether it is due or not.

It may so happen that when advance salary is inckuded charged in a particular previous

year, the rate of tax at which the emplyee is assessed may be higher tyhan the normal rate

of tax to which he would have been assessed.Section 89(1) provides for relief in these

types of cases.

(x) Loan or Advance against salary :Loan is different from salary.When an employee takes

a loan from his employer , which is repayable in certain specified installments, the loan

amount cannot be brought to tax as salary of the employee.

Basis of charge (Section - 15) :

As per section 15, salary consists of:

a. any salary due from an employer (or a former employer) to an assesse in the previous

year, whether actually paid or not;

b. any salary paid or allowed to him in the previous year by or on behalf of an employer (or

a former employer), though not due or before it became due; and

c. any arrears of salary paid or allowed to him in the previous year by or on behalf of an

employer if not charged to income-tax for any earlier previous year.

Page 32: INTER CA Taxation - CA Study Web€¦ · INTER CA Taxation Head Office Shraddha, 4th Floor, ... 14. Advance Tax & Interest 268 - 271 15. Tax Deducted at Source (TDS) 272 - 294 16

: 29 :

J. K. SHAH CLASSES INTER C.A.- DIRECT TAXES

SALARIES

The same is explained in the table given below—

Nature of salary Is it taxable as income of the

previous year 2017-18

Salary becomes due during the previous year

2017-18(whether paid during the same year or not) Yes

Salary is received during the previous year

2017 – 18 (whether it becomes due in a subsequent year) Yes

Arrears of salary received during the previous year

2017-18 although it pertains to one of the earlier

years and the same were not taxed earlier on due basis Yes

Arrears of salary received during the previous year

2017-18although it pertains to one of the earlier years

but the same were taxed earlier on due basis No

���� Salary is taxable on "due” or "receipt" basis whichever is earlier Basis of charge in

respect of salary income is fixed by section 15. Salary is chargeable to tax either on

"due" basis or on "receipt" basis, whichever matures earlier.

���� Accounting method of the employee not relevant - It is worthwhile to mention that

salary is chargeable to tax on "due" or "receipt" basis (whichever matures earlier)

regardless of the fact whether books of account, in respect of salary income, are

maintained by the assessee on mercantile basis or cash basis. Method of

accounting cannot, therefore, vary the basis of charge fixed by section 15.

Format of computation of Income from Salaries

Page 33: INTER CA Taxation - CA Study Web€¦ · INTER CA Taxation Head Office Shraddha, 4th Floor, ... 14. Advance Tax & Interest 268 - 271 15. Tax Deducted at Source (TDS) 272 - 294 16

: 30 :

J. K. SHAH CLASSES INTER C.A.- DIRECT TAXES

SALARIES

RELIEF U/S. 89

Where an employee is in receipt of arrears of salary or advance salary in the P.Y., his

total income is assessed at a rate higher than that at which it would otherwise have been

assessed. the Assessing Officer shall, on an application made to him in this behalf, grant

relief from the tax payable by the employer for that year.

Under Rule 21A(2) following are the steps to give the relief :

1. Calculate the total tax payable on the total income, including the additional salary,

of the relevant previous year in which the same is received.

2. Calculate the total tax payable on the total income, excluding the additional salary,

of the relevant previous year in which the additional salary is received.

3. Find out the difference between the tax at (1) and (2) above.

4. Compute the total tax payable on the total income after including the additional

salary in the previous year to which such salary relates.

5. Compute the total tax payable on the total income after excluding the additional

salary in the previous year to which such salary relates.

6. Find out the difference between tax at (4) and (5) above.

7. The excess of tax computed at (3) over tax computed at (6) is the amount of relief

admissible under section 89. No relief is, however, admissible if tax computed

at (3) is less than or equal to tax computed at (6). In such a case, the employee

need not apply for relief.

Incomes deemed to be received in India [Section 7]

The following incomes shall be deemed (i.e. assumed) to be received in India and hence

taxable in India for the assessee, irrespective of his residential status.

1. Annual interest credited to the employee's account in the case of Recognised Provident

Fund in excess of 9.5% per annum rate of interest.

2. Contribution of employer in case of Recognised Provident Fund in excess of 12%

of salary.

3. The contribution made, by the Central Government or any other employer in the previous

year, to the account of an employee under a pension scheme referred to in section

80CCD.[eg. National Pension Scheme (NPS), Atal Pension Yojna (APY)]

Basic salary + Commission + Bonus

1. The first and foremost item of income from salary is basic salary ‘earned’ in the

computation of income.

If basic salary ‘received’ is given, then we have to find out ‘earned’ by adding back

whatever is deducted from salary. It is always fully taxable.

Page 34: INTER CA Taxation - CA Study Web€¦ · INTER CA Taxation Head Office Shraddha, 4th Floor, ... 14. Advance Tax & Interest 268 - 271 15. Tax Deducted at Source (TDS) 272 - 294 16

: 31 :

J. K. SHAH CLASSES INTER C.A.- DIRECT TAXES

SALARIES

2. The next item to be considered in the computation of income from salary is “Fixed %

commission on turnover achieved by employee”.

It is a mode of distribution of salary, where we get the best performance from an

employee because of the financial motivation. It forms part of ‘basic salary’

definition for several purposes.

If it is undecided % commission on turnover or simply commission or fixed %

commission on profits, it is taxable fully but does not form part of "salary" for various

purposes.

3. Bonus is a voluntary payment made by the employer to employee in relation to

employment. It is always taxable on receipt basis only (i.e. not on due basis).

ALLOWANCE :

Allowance is generally defined as a fixed quantity of money or other substance given

regularly in addition to salary for the purpose of meeting some particular requirement

connected with the services rendered by the employee or as compensation for unusual

conditions of that service. It is fixed, pro-determined and given irrespective of actual

expenditure. Under the Act, it is taxable under section 15 on "due" or "receipt" basis,

whichever comes earlier, irrespective of the fact that it is paid in addition to or in lieu of

salary. Tax treatment of different allowances is given below:

Foreign Allowance [Sec :10(7)]

Any allowance or perquisites paid or allowed outside India by the Government to an In-

dian citizen for rendering services outside India, is wholly exempted from tax.

House Rent Allowance [Sec 10(13A)] :

is exempt to the extent of least of the following :

(a) amount equal to 50% of the salary in case of Mumbai, Delhi, Calcutta & Chennai.

40% in case of any other city. [depending on location of rented premises]

(b) HRA actually received.

(c) Rent paid in excess of 10% of the salary.

���� Salary = Basic salary + D.A. (in terms) + Fix % commission on turnover, on due basis

for the period the rented accommodation is occupied by the employee only.[i.e. exclud-

ing Advance salary & Arrears of Salary]

���� Exemption is denied where an employee lives in his own house or in a house for which he does

not pay any rent or pays rent which does not exceed 10% of salary.

���� Mode of computation of exemption - The amount of exemption in respect of

house rent allowance received by an employee depends upon the following —

a. "salary" of the employee ;

b. house rent allowance ;

c. rent paid ; and

d. the place where house is taken on rent.

When these four are same throughout the previous year, the exemption should be

calculated on "annual" basis. When, however , there is a change in respect of any of

the aforesaid factors, then the exemption shall be worked out on "monthly" basis.

Page 35: INTER CA Taxation - CA Study Web€¦ · INTER CA Taxation Head Office Shraddha, 4th Floor, ... 14. Advance Tax & Interest 268 - 271 15. Tax Deducted at Source (TDS) 272 - 294 16

: 32 :

J. K. SHAH CLASSES INTER C.A.- DIRECT TAXES

SALARIES

Special allowance notified as Exempt - As per rule 2BB [Sec 10(14)(i) ] :

Allowance received which are exempt to the extent of amount spent for the purpose for

which they are given i.e. fully exempted if fully spent or partly exempt if partly spent

or fully taxable if nothing is spent.

���� Tour, Travel or transfer allowance, it also includes any sum paid in connection

with transfer, packing and transportation of personal effects on such transfer.

���� Any allowance, whether, granted on tour or for the period of journey in connection with

transfer, to meet the ordinary daily charges incurred by an employee on account of

absence from his normal place of duty, such as a daily allowance.

���� Conveyance allowance - This is exempt if spent to cover journey between office and

field and back to office.

If conveyance allowance is granted to cover journey between office and residence then

fully taxable. To cover expense of journey from residence to office & back, the

employee should get transport allowance which is discussed below.

���� Helper allowance - Any allowance granted by employer to employee to enable

the employee to meet the expenditure on a helper for the execution / performance

of his official duties.

���� Academic pursuit allowance - Any allowance granted for encouraging the

academic research and other professional pursuits.

���� Uniform allowance - Any allowance to meet the expenditure on the purchase or

maintenance of uniform for wearing during the performance of duties of an office.

Allowance notified as Exempt on the basis of limit - As per rule 2BB [Sec 10(14)(ii) ]

Allowances which are exempt not in relation to the amount spent, but are exempted to

the extent of least of :

Actual amount of allowance received

OR

Amount specified in Rule 2BB.

Page 36: INTER CA Taxation - CA Study Web€¦ · INTER CA Taxation Head Office Shraddha, 4th Floor, ... 14. Advance Tax & Interest 268 - 271 15. Tax Deducted at Source (TDS) 272 - 294 16

: 33 :

J. K. SHAH CLASSES INTER C.A.- DIRECT TAXES

SALARIES

Note on Dearness Allowance

The second item to be considered in the computation of income from salary is ‘Dearness

Allowance’ [DA].

It is a fixed amount paid by the employer to employee to enable him to meet the rise in

the cost of living. It is always fully taxable.

DA is of two types :-

���� In terms of employment/forming part of basic salary/considered for retirement benefits.

It forms part of basic salary for all definitions of salary except ‘salary’ definition for

entertainment allowance deduction u/s 16(ii).

���� Not in terms of employment / Does not form part of basic salary/Not considered for

retirement benefits.

It does not form part of any ‘salary’ definition except ‘salary’ definition of employee

covered by ‘Payment of Gratuity Act’.

Every year, the employee shall receive the total D.A. [ D.A. (in terms) and D.A. (not in

terms)] from the employer which shall be fully taxable for the employee. the break-

up of DA, as above, is relevant only for CALCULATION purposes.

RETIREMENT BENEFITS

Section Name of Retirement Benefits

10(10) Gratuity

10(10A) Pension

10(10AA) Leave Salary

10(10B) Retrenchment Compensation

10(10C) Voluntary Retirement Compensation

10(11) & 10(12) Statutory Provident Fund & Recognised Provident Fund

10(13) Superannuation Fund

1. Gratuity [Sec10(10)] :

Gratuity [Sec. 10(10)]- Gratuity is a retirement benefit. It is generally payable at

the time of cessation of /employment and on the basis of duration of service. Tax

treatment of gratuity is given below:

In case of Government Employees – Any death –cum retirement gratuity

received by Government employees is wholly exempt from tax u/s 10(10)(i).

In case of employees covered by thr payment of gratuity act, 1972[Sec.

10(10)(ii) - Any gratuity received by an employee, covered by the Payment of

Gratuity Act, 1972, is exempt from tax on the following basis

1. 15 days salary (7 days salary in the case of employees of a seasonal

establishment) based on salary last drawn for each year of service (i.e. 15

days salary x length of service)

15/26 x last drawn salary x Number of years of service.

2. 10,00,000

3. Gratuity actually received.

Page 37: INTER CA Taxation - CA Study Web€¦ · INTER CA Taxation Head Office Shraddha, 4th Floor, ... 14. Advance Tax & Interest 268 - 271 15. Tax Deducted at Source (TDS) 272 - 294 16

: 34 :

J. K. SHAH CLASSES INTER C.A.- DIRECT TAXES

SALARIES

What is exempt from tax – The least of the above three is exempt from tax.

How to find out length of service – If the period of service is 6 months, it shall

be ignored for this purpose. Conversely, if the period of the service is more than 6

months, it shall be taken as one full year.

Consider the following cases-

The difference between date of retirement Length of service for the purpose

and date of joining of section 10(10)(ii)

Case 1 26 years, 5 months and 29 days 26 years

Case 2 26 years and 6 months 26 years

Case 3 26 years, 6 months and 1 day 27 years

Case 4 26 years, 11 months and 29 days 27 years

What is salary – “Salary” for the purpose of the aforesaid limits means salary last

drawn by an employee and includes dearness allowance.

In the case of any other Employee – Any other gratuity received by an employee

on retirement, death, termination, resignation or on his becoming incapacitated

prior to the retirement, is exempt from tax on the following basis-

1. Rs. 10,00,000

2. Half month’s average salary for each completed year of service

½ x Average Salary of 10m x Number of years of service

3. Gratuity actually received

What is exempt from tax- The least of the above three is exempt from tax.

The following points merit consideration-

Completed years of service –How to determine – For calculating length of serv-

ice any fraction of the year shall be ignored. Consider the following cases-

The difference between date of retirement Length of service for the purpose

and date of joining of section 10(10)(iii)

Case 1 26 years, 5 months and 29 days 26 years

Case 2 26 years and 6 months 26 years

Case 3 26 years, 6 months and 1 day 26 years

Case 4 26 years, 11 months and 29 days

���� Average monthly salary - How to determine - Average monthly salary is

calculated on the basis of average salary for the ten months immediately

preceding tire month in which the employee has retired.

���� What is salary for this purpose - Salary for this purpose moans basic

salary. It includes dearness allowance if the terms of employment so provide

(or if dearness allowance/pay is taken into account for computing retirement

benefits). It also includes commission if commission is payable at a fixed

percentage of turnover achieved by an employee.

Page 38: INTER CA Taxation - CA Study Web€¦ · INTER CA Taxation Head Office Shraddha, 4th Floor, ... 14. Advance Tax & Interest 268 - 271 15. Tax Deducted at Source (TDS) 272 - 294 16

: 35 :

J. K. SHAH CLASSES INTER C.A.- DIRECT TAXES

SALARIES

���� When gratuity is received from two or more employers - Where gratuity

is received by a nongovernment employee (not covered by the Payment of

Gratuity Act) from two or more employers (maybe in the same year or differ-

ent years), the maximum amount of exemption under section 10(10)(ii) dur-

ing the lifetime of the concerned employee cannot exceed the notified amount,

i.e., Rs. 10,00,000. This provision is applicable only in the case of residual

category of employees.

���� Gratuity paid while in service - Any gratuity paid to an employee while he

continues to remain in service (whether or not after he has put in a minimum

specified period of service) is not exempt from tax.

2. Pension [Sec 10(10A)] :

Pension is chargeable tax as follows -

Pension Status of employee Is it chargeable to tax

Uncommuted pension Government / non- It is chargeable to tax

Government employee

Commuted pension Governemtn employee It is fully exempt tax under

section 10(10A) (i)

Commuted pension Non- Government It is fully or partly exempt from tax

employee under section 10(10A) (ii)

UNCOMMUTED PENSION - It is periodical payment of pension. For instance, X

gets monthly pension of Rs. 2,000. It is taxable as salary under section 15 in the

hands of a Government employee as well as non- Govenment employee.

COMMUTED PENSION - It is a lump sum payment in lieu of periodical payment.

For instance, after his retirement, X gets Rs. 2,000 per month as monthly pension.

As per service rules, he gets 25 per cent of his pension commuted for Rs. 60,000

(after commutation he will get the remaining 75 per cent, i.e., Rs. 1,500 by way of

monthly pension). In this case, Rs. 60,000 is commuted pension which X has re-

ceived in lieu of 25 per cent of his monthly pension.

Commuted pension is taxable as under—

Status of employee Gratuity received / Exemption in respect of commuted

not received pension under section 10(10A)

Governement employee Gratuity may (or may Entire commuted pension is

(i.e. an employee of central not) be received commuted from tax.

Governemnt, State

Government, local authority

and statutory corporation)

Non- Government employee Gratuity is received One-thi rd of the pension which he

is normally entitled to receive is

exempt from tax.

Non - Government employee Gratutity is not One-half of the pension which he is

received normally entitled to receive is

exempt from tax.

Page 39: INTER CA Taxation - CA Study Web€¦ · INTER CA Taxation Head Office Shraddha, 4th Floor, ... 14. Advance Tax & Interest 268 - 271 15. Tax Deducted at Source (TDS) 272 - 294 16

: 36 :

J. K. SHAH CLASSES INTER C.A.- DIRECT TAXES

SALARIES

3. Leave Salary [Sec 10(10AA)] :

The provisions regarding taxability of leave salary are given below—

WHAT IS LEAVE SALARY - As per service rules, an employee gets different leaves.

An employee has to earn leave in the first instance and only when he has leave to his

credit, he can apply for leave. If a leave (standing to his credit) is not taken within a

year, as per the service rules, it may lapse or il may be encashed or it may be accumu-

lated. The accumulated leaves standing to the credit of an employee may be availed by

the employee during his service time or, subject to service rules, such leaves may be

encashed at the time of retirement or leaving the job. Encashment of leave by surren-

dering leave standing to one's credit is known as "leave salary"

BROAD TAX TREATMENT - the broad tax treatment is given below -

Nature of leave enchasement Status of employee Whether it is taxable

Leave of enchasement during Government / It is chargeable to tax.

continuity of employment non-Government

employee

Leave enchasement at the time Government employee It is fully exempt from tax

of retirement / leaving job under section

Leave enchasement at the Non- Government It is fully partly exempt from

time of retirement. leaving job employee tax in some cases under section

GOVERNMENT EMPLOYEES GETTING LEAVE ENCASHMENT AT THE TIME OF

RETIREMENT (SEC. 10(10AA)(i)) - In the case of a Government employee, any amount

received as cash equivalent of leave salary in respect of period of earned leave at his

credit at the time of his retirement.

NON-GOVERNMENT EMPLOYEES GETTING LEAVE ENCASHMENT AT THE

TIME OF RETIREMENT [SEC 10(10AA)(ii)] - In the case of a non-Govemment

employee, leave salary' is exempt from tax on the basis of least of the following—

1. Period of earned leave (in number of months) to the credit of the employee at

the time of his retirement or leaving the job x Average monthly salary

2. 10 x Average monthly salary

3. The amount specified by the Government [i.e., `̀̀̀ 3,00,000]

4. Leave enchasement actually received at the time of retirement.

Page 40: INTER CA Taxation - CA Study Web€¦ · INTER CA Taxation Head Office Shraddha, 4th Floor, ... 14. Advance Tax & Interest 268 - 271 15. Tax Deducted at Source (TDS) 272 - 294 16

: 37 :

J. K. SHAH CLASSES INTER C.A.- DIRECT TAXES

SALARIES

Notes:

1. How to find out leave standing to the credit of an employee at the time of

retirement or leaving the job - It will be calculated as follows—

Step (a) - Find out duration of service in number of years (ignore any fraction of year).

Step (b) - Find out rate of earned leave entitlement from the service rules — how

many days leave is credited for each year of service (earned leave entitlements

cannot exceed 30 days for every year of actual sendee rendered for the em-

ployer from whose service he has retired). For instance, if earned leave is cred-

ited at the rate of 40 days leave for each year of sendee, for Step (b) calculation

shall be made at the rate of 30 days leave for each year of sendee. If, however,

earned leave is credited at the rate of 25 days leave for each year of sendee, for

Step (b) calculation shall be made at the rate of 25 days leave for each year of sendee.

Step (c) - Find out earned leave actually taken or encashed (in number of days)

during the service time.

The computation shall be made as follows —

[Step (a) x Step (b) minus Step (c) ��������30]

2. How to find out average monthly salary- Salary, for this purpose, means

basic salary and includes dearness allowance if terms of employment so

provide.It also includes commission based upon fixed percentage bof turno-

ver achieved by an employee.

"Average salary" for the aforesaid purpose is to be calculated on the basis

of average salary drawn during the period of 10 months immedaiately

preceeding the retirement.

When earned leave enchasement is received from two or more employers -

where leave salary or leave enchasement is received by a non- Governemtn

employee from tow or more employers (may be in the same or different years),

the maximum amount of exemption under section 10(10AA) (ii) during the

lifetime of the concerned employee cannot exceed ` ̀` ` 3,00,000.

4. Retrenchment Compensation [Sec.10(10B)] :

Retrenchment Compensation is exempt from tax to the extent of least of following :

(a) The amount calculated in accordance with provision of section 25F(b) of the

Industrial Dispute Act, 1947.

15/26

x Average salary of 3 months x no. of years of service

(b) `̀̀̀ 5,00,000/-.

(c) Actual amount received.

- Salary = Basic salary + dearness allowance (both)

- No. of years of service = Completed years or part thereof in excess of 6 months

Page 41: INTER CA Taxation - CA Study Web€¦ · INTER CA Taxation Head Office Shraddha, 4th Floor, ... 14. Advance Tax & Interest 268 - 271 15. Tax Deducted at Source (TDS) 272 - 294 16

: 38 :

J. K. SHAH CLASSES INTER C.A.- DIRECT TAXES

SALARIES

5. Payment on Voluntary Retirement [Sec.10(10C)]:

Exemption is least of " -

1. Higher of:

(a) "Salary"' x 3 months x Completed years of service.

(b) "Salary" x balance months to retire at superannuation

2. Actual compensation amount .

3. Maximum 5,00,000/-.

- Salary = Basic salary + dearness allowance (in terms) + commission

based on percentage of turnover

- No. of years of service = Completed years

6. Payment from provident fund along with accrued interest at the time of retirement

or termination from service [Sec. 10(11)&10(12)]:

A provident fund is a retirement benefit created for the employees so that they

receive a lumpsum amount on retirement or termination of service. There are 4

types of provident funds.

1. Statutory Provident Fund (SPF)

This is the provident fund maintained for employees working with Govern-

ment, semi-government organizations, rai lways, loca l author it ies,

universities and recognized educational institutions.

2. Recognized Provident Fund (RPF)

A provident fund to which the “Employees Provident Fund and Miscellane-

ous Provisions Act, 1952” applies is called as RPF. As per the Provident

Fund Act, an organization employing 20 or more employees shall be cov-

ered by the provisions of RPF. The Commissioner of Income Tax shall grant

approval or recognition to the PF if the rules imposed on the PF have been

complied with.

3. Unrecognized Provident Fund (URPF)

If the PF does not follow or comply with the rules imposed upon it, the Com-

missioner of Income tax shall not grant the recognition and the PF shall be

called as URPF.

Page 42: INTER CA Taxation - CA Study Web€¦ · INTER CA Taxation Head Office Shraddha, 4th Floor, ... 14. Advance Tax & Interest 268 - 271 15. Tax Deducted at Source (TDS) 272 - 294 16

: 39 :

J. K. SHAH CLASSES INTER C.A.- DIRECT TAXES

SALARIES

Page 43: INTER CA Taxation - CA Study Web€¦ · INTER CA Taxation Head Office Shraddha, 4th Floor, ... 14. Advance Tax & Interest 268 - 271 15. Tax Deducted at Source (TDS) 272 - 294 16

: 40 :

J. K. SHAH CLASSES INTER C.A.- DIRECT TAXES

SALARIES

7. Payment from an approved superannuation fund [Sec. 10(13):

���� Any payment from an approved superannuation fund (i.e. approved by the

Chief Commissioner or Commissioner of Income - tax) shall be fully exempted

if the payment is made to :

(1) the legal heirs on the death of the beneficiary.

(2) an employee in lieu or in commutation of an annuity on his retirement

at or after a specified age or on his becoming incapacitated prior to

such retirement.

(3) an employee on his leaving the service in connection with which the

fund is established otherwise than, in the circumstances mentioned

in point (2) above, i.e. leaving employment for better pay etc. then the

lump sum received will be fully taxable.

���� It is worthwhile to note that :

(1) during employment - employer's contribution to such a fund in excess

of 1,00,000 p.a. per employee is taxable in the hands of the em-

ployee u/s 17(2)(vii)

(2) Employee's contribution is entitled for deduction under Section 80C

from his Gross Total Income.

(3) Interest on accumulated balance is exempt from tax every year.

Perquisites :

Perquisite may be defined as any casual emolument or benefit attached to an office or

position in addition to salary or wages. It also denotes something that benefits a man by

going into his men pocket. Perquisites may be provided in cash or in kind. However,

perquisites are taxable under the head "Salaries" only if they are (a) allowed by an em-

ployer to his employee; (h) allowed during the continuance of employment; (c) directly

dependent upon service; (d) resulting in the nature of personal advantage to the em-

ployee; and (c) derived by virtue of employer's authority. It is not necessary that a recur-

ring and regular receipt alone is a perquisite. Even a casual and non-recurring receipt

can be perquisite if the aforesaid conditions are satisfied.

The following propositions should also be kept in view:

���� Perquisites are included in salary income only if they are received by an employee

from his employer (maybe former, present or prospective). Perquisites, received

from a person other than employer, are taxable under the head "Profits and gains

of business or profession" or "Income from other source's".

���� A benefit or advantage would be taxable as perquisites only if it has a legal origin.

As unauthorised advantage taken by an employee without his employer's authority

would create a legal obligation to restore such advantage, it would not amount to

"perquisite" taxable under the Act. On the other hand, if the benefit has been con-

ferred unilaterally without the aid of agreement between the parties, the employee

can be taxed on the perquisites. It is not necessary that the benefit should have

been received under an enforceable right.

Page 44: INTER CA Taxation - CA Study Web€¦ · INTER CA Taxation Head Office Shraddha, 4th Floor, ... 14. Advance Tax & Interest 268 - 271 15. Tax Deducted at Source (TDS) 272 - 294 16

: 41 :

J. K. SHAH CLASSES INTER C.A.- DIRECT TAXES

SALARIES

Perquisites" as defined in the Act- Under the Act, the term "perquisites" is defined by

section 17(2) as including the following items:

a. the value of rent-free accommodation provided to the assessee by his employer

[sec. 17(2)(i) ;

b. the value of any concession in the matter of rent respecting any accommodation

provided to the assessee by his employer [sec. 17(2)(ii)] ;

c. The value of any benefit or amenity granted or provided tree of cost or at

concessional rate in any of the following cases:

i. by a company to an employee who is a director thereof ;

ii. by a company to an employee, being a person who has substantial interest

in the company ;

ii i. by any employer (including a company) to an employee to whom provisions

of (i) and (ii) above do not apply and whose income under the head "Sala-

ries" exclusive of the value of all benefits or amenities not provided for by

way of monetary benefits, exceeds Rs. 50,000 [sec. 17(2)(iii)];

d. any sum paid by the employer in respect of any obligation which but for such pay-

ment would have been payable by the assessee [sec. 17(2)(iv)j;

e. any sum payable by the employer, whether directly or through a fund other than a

recognised provident fund or approved superannuation fund or a deposit-linked

insurance fund, to effect an assurance on the life of the assessee or to effect a

contract for an annuity [sec. 17(2)(v) ;

f. the value ol any specified security or sweat equity shares allotted or transferred,

directly or indirectly, by the employer, or former employer, free of cost or at

concessional rate to the assessee [sec. 17(2)(vi)];

g. the amount of any contribution to an approved superannuation fund by the em-

ployer in respect of the assessor, to the extent it exceeds Rs. 1,50,000 [sec.

17(2)(vii)]; and

h. the value of any other fringe benefit or amenity as may be prescribed [sec. 17(2)

(viii)]

Rules for valuation of perquisites :

(a) Valuation of Rent free Unfurnished / Furnished Accommodation

Rent - free Accommodation :

Taxable u/s 17(2)(i) Rent free accommodation

Taxable u/s 17(2)(ii) Concessional rent accommodation

For Central Government & State Government employees :

I Rent free unfurnished accommodation (to be valued at the licence fee which

would have been determined by the Central or State Government in

accordance with the rules framed by the Government for allotment of houses

to its officers).

II Add for furnishing, if any (10% per annum on original cost, if by employer or

actual hire charges, if hired by employer).

III Less rent recovered, if any, by employer, from employee.

Page 45: INTER CA Taxation - CA Study Web€¦ · INTER CA Taxation Head Office Shraddha, 4th Floor, ... 14. Advance Tax & Interest 268 - 271 15. Tax Deducted at Source (TDS) 272 - 294 16

: 42 :

J. K. SHAH CLASSES INTER C.A.- DIRECT TAXES

SALARIES

For other employees :

(a) Rent free unfurnished accommodation owned by employer :

I 15% of salary (in cities having population exceeding 25 lacs as per 2001

census) and 10% of salary (in cities having population less than 25 lacs

but greater than 10 lacs), and 7.5% of salary in any other place.

II Add for furnishing, if any (10% per annum on original cost or actual

hire charges).

III Less rent recovered, if any, by employer, from employee.

(b) Rent free unfurnished accommodation taken on lease or rent by the

employer :

I Actual amount of rent payable by employer or 15% of salary, which ever

is lower.

II Add for furnishing, if any (10% per annum on original cost or actual hire

charges).

III Less rent recovered, if any, by employer, from employee.

Other points :

(1) "Salary" here means all monetary payments for the period the accommodation is

provided, i.e. excluding any advance or arrears received but including remuneration

on accrual basis. Moreover, salary from all the employers in respect of the said period

shall be taken into consideration. However, salary does not include the following :

(i) D.A. (not in terms)

(ii) Employer's contribution to provident fund in excess of 12% of salary and Interest

accrued to RPF in excess of 9.5% rate of interest p.a.

(iii) Exempted allowances.

(iv) Deduction u/s 16 (ii) if applicable.

(v) Taxable Perquisites.

(2) Hotel accommodation provided to employees including family members is taxable @ 24%

of salary for the previous year or actual charges paid or payable to such hotel, whichever is

lower, for the period during which such accommodation is provided as reduced by the rent,

if any, actually paid or payable by the employee.

If hotel accommodation is provided because of employee's transfer & not exceeding in

aggregate 15 days, then nothing is taxable.

(3) Where on account of the transfer of an employee from one place to another, he is

provided, with accommodation at the new place of posting while retaining the

accommodation at the other place, the value of perquisite shall be determined with

reference to only one such accommodation which has the lower value for a period not

exceeding 90 days and thereafter the value of perquisite shall be charged for both

such accommodations.

Page 46: INTER CA Taxation - CA Study Web€¦ · INTER CA Taxation Head Office Shraddha, 4th Floor, ... 14. Advance Tax & Interest 268 - 271 15. Tax Deducted at Source (TDS) 272 - 294 16

: 43 :

J. K. SHAH CLASSES INTER C.A.- DIRECT TAXES

SALARIES

(b) Perquisite of sweeper / gardner / watchman / personal attendant :

(i) The value of benefit to the employee or any member of his household resulting

from the provision by the employer of the services of a sweeper, a gardener, a

watchman or a personal attendant, shall be the actual cost to the employer.

(ii) The actual cost in such a case shall be the total amount of salary paid or payable by

the employer or any other person on his behalf for such services as reduced by

any amount paid by the employee for such services.

(c) Perquisite of gas /electric energy/water steam , etc :

(i) The value of the benefit to the employee resulting from the supply of gas,

electric energy or water for his household consumption shall be determined

as the sum equal to the amount paid on that account by the employer to the

agency supplying the gas, electric energy or water.

(ii) Where such supply is made from resources owned by the employer, without

purchasing them from any other outside agency, the value of perquisite would

be the manufacturing cost per unit incurred by the employer.

(iii) Where the employee is paying any amount in respect of such services, the

amount so paid shall be deducted from the value so arrived at.

(d) Perquisite value of interest free loan / concessional loan to assessee and

family members :

(a) The value of the benefit to the assessee resulting from the provision of

interest-free or concessional loan for any purpose made available to the

employee or any member of his household during the relevant previous year

by the employer or any person on his behalf shall be determined as the sum

equal to the interest computed at the rate charged per annum by the State

Bank of India, as on the 1st day of the relevant previous year in respect of

loans for the same purpose advanced by it on the maximum outstanding

monthly balance as reduced by the interest, if any, actually paid by him or

any such member of his household. “Maximum outstanding monthly balance”

means the aggregate outstanding balance for each loan as on the last day

of each month.

(b) However, no value would be charged if such loans are made available for

medical treatment in respect of prescr ibed diseases ( like cancer,

tuberculosis, etc.) or where the amount of loans are petty not exceeding in

the aggregate `̀̀̀20,000.

Page 47: INTER CA Taxation - CA Study Web€¦ · INTER CA Taxation Head Office Shraddha, 4th Floor, ... 14. Advance Tax & Interest 268 - 271 15. Tax Deducted at Source (TDS) 272 - 294 16

: 44 :

J. K. SHAH CLASSES INTER C.A.- DIRECT TAXES

SALARIES

(e) Perquisite of free / concessional education facilities for any member of

employee's household :

The basis of valuation is briefly given below -

1. Expenditure relating to providing training to employees is not taxable.

2. If education facility is provided to the family members of employee,

expenditure incurred by the employer is the taxable value of perquisite.

Payment of tuition fees or reimbursement of tuition fees is chargeable to

tax.

3. If education facility is provided to the children of employee in an educational

institute owned or maintained by the employer, then reasonable cost of

education in a similar institute* in or near the locality is taxable. Up to Rs.

l,000t per month per child is not taxable, if the employer provides education

facility to the children of an employee in an educational institution owned or

maintained by the employer or where such educational facility is provided in

any institute (having an arrangement with the employer) by reason of

employee's employment with the employer. The benefit of exemption of Rs.

1,000 per month is not available, if such education facility is provided to

other family members (not being children of the employee).

4. Amount of scholarship given by an employer-company to children of its

employees (solely at its discretion without reference to terms of employment)

is not assessable as perquisite in the hands of employees)

(f) Perquisite of movable assets sold by an employer to its employees at a

nominal price :

Particulars Computer and Motor Car Any other Asset

other related electronics

Actual Cost XX XX XX

Less: Depreciation for

completed years (XX) (XX) (XX)

Estimated Cost XX XX XX

Less: Amount

recovered from Employee (XX) (XX) (XX)

Perquisite Value XX XX XX

1. Rate of depreciation and method of depreciation

Computer and other related electronics 50% WDV

Motor Car 20% WDV

Any other Asset 10% SLM

2. Completed year means actual completed year from the date of acquisition

of the asset to the date of transfer of such asset to the employees.

3. Goods manufactured by Employer sold to Employee at concessional price

is Exempt.

4. Any other asset sold to Employee after using it for 10 years or more is not

taxable.

Page 48: INTER CA Taxation - CA Study Web€¦ · INTER CA Taxation Head Office Shraddha, 4th Floor, ... 14. Advance Tax & Interest 268 - 271 15. Tax Deducted at Source (TDS) 272 - 294 16

: 45 :

J. K. SHAH CLASSES INTER C.A.- DIRECT TAXES

SALARIES

(g) Perquisite of Use of Movable Assets :

Particulars Perquisite value

Computer, laptop, cell phone, telephone, Exempt

subscription cost to journals and periodicals

Any Other Asset

Owned By Employer 10% pa of Original Cost

Less: Amount Recovered from Employee.

Hired by Employer Hire Charges Paid by Employer

Less: Amount Recovered from Employee.

(h) Valuation of medical facilities:

Valuation of medical facilities - Before discussing the broad provisions, one should

note down the following points -

1. Fixed medicla allowance is always chargeable to tax.

2. For the purpose of valuation of the perquisite in respect of medical facilities,

"family" means -

(a) the spouse and children of the individual; and

(b) the parents, brothers and sister of the individual or any of them, wholly or

mainly dependent on the individual.

MEDICAL FACILITIES IN INDIA - The provisions are given below -

1. Employer's hospital/Government hospital/approved hospital - The perquisite in

respect of medial facility provided by an employer in the following hospitals/clinic

is not chargeable to tax -

a. hospital owned/ maintained by the employer.

b. hospital of Central Government/ State Government/ local authority.

c. private hospital if it is also recommended by the Government for the treat-

ment of Government employees

d. specified medical facility (given in rule 3A) in a hospital approved* by the

Chief Commissioner.

2. Health insurance premium – Medical insurance premium paid or reimbursed by

the employer is not chargeable to tax.

3. Any other facility in India – Any other expenditure incurred or reimbursed by the

employer for providing medical facility in India is not chargeable to tax up to Rs.

15,000 in aggregate per assessment year (fied medical allowance is fully charge-

able to tax).

MEDICAL FACILITIES OUTSIDE INDIA – Any expenditure incurred by the employer

(or reimbursement of expenditure incurred by the employee) on medical treatment of

the employee or any member of the family of such employee outside India, is taxable

subject to the conditions given below-

Page 49: INTER CA Taxation - CA Study Web€¦ · INTER CA Taxation Head Office Shraddha, 4th Floor, ... 14. Advance Tax & Interest 268 - 271 15. Tax Deducted at Source (TDS) 272 - 294 16

: 46 :

J. K. SHAH CLASSES INTER C.A.- DIRECT TAXES

SALARIES

Perquisite not chargeable to tax Condition to be satisfied

Medical treatment of employee or any member Expenditure shall be excluded from perquisite

of family of such employee outside India only to the extent permitted by the Reserve

Bank of India.

Cost on travel of the employee/ any member Expenditure shall be excluded from perquisite

of his family and one attendant who only in the case of an employee whose

accompanies the patient in connection with gross total income, as computed before

treatment outside India. including therein the expenditure on travelling,

does not exceed Rs. 2,00,000.

Cost of stay abroad of the employee or any Expenditure shall be excluded from the per

member of the family for medical treatment quisite only to the extent permitted by the

and cost of stay of one attendant who Reserve Bank of India.

accompanies the patient in connection with

such treatment.

(i) Perquisite of free / subsidised meal :

(a) The value of free food and non-alcoholic beverages provided by the employer to an

employee shall be the amount of expenditure incurred by such employer. The amount

so determined shall be reduced by the amount, if any, paid or recovered from the

employee for such benefit or amenity:

(b) However, the following would not be treated as a perquisite -

(1) free food and non-alcoholic beverages provided by such employer during

working hours at office or business premises or through paid vouchers which

are not transferable and usable only at eating joints, to the extent the value

thereof either case does not exceed fifty rupees per meal or

(2) tea or snacks provided during working hours or

(3) free food and non-alcoholic beverages during working hours provided in a

remote area or an off-shore installation.

(j) Perquisite of gifts etc :

1. The value of any gift, or voucher, or token in lieu of which such gift may be

received by the employee or by any member of his household on ceremonial

occassions or otherwise shall be determined as the sum equal to the amount

of such gift. However, where the value of such gift, voucher or token, as the

case may be, is below `̀̀̀ 5,000/- in the aggregate during the previous year,

the value of perquisite shall be taken as nil. If the value of gift exceeds

`̀̀̀ 5,000/- only the value in excess of `̀̀̀ 5,000/- will be taxable.

2. The provisions are not applicable for cheques / cash gifts and they would be

fully taxable even if they are less than `̀̀̀ 5,000.

Page 50: INTER CA Taxation - CA Study Web€¦ · INTER CA Taxation Head Office Shraddha, 4th Floor, ... 14. Advance Tax & Interest 268 - 271 15. Tax Deducted at Source (TDS) 272 - 294 16

: 47 :

J. K. SHAH CLASSES INTER C.A.- DIRECT TAXES

SALARIES

(k) Perquisite of credit card expenses :

(a) The amount of expenses including membership fees and annual fees incurred

by the employee or any member of his household, which is charged to a

credit card (including any add-on-card) provided by the employer, or

otherwise, paid for or reimbursed by such employer shall be taken to be the

value of perquisite chargeable to tax as reduced by the amount, if any paid

or recovered from the employee for such benefit or amenity.

(b) However, such expenses incurred wholly and exclusively for official purposes

would not be treated as a perquisite if the following conditions are fulfilled.

(1) complete details in respect of such expenditure are maintained by

the employer which may, inter alia, include the date of expenditure

and the nature of expenditure;

(2) the employer gives a certificate for such expenditure to the effect that

the same was incurred wholly and exclusively for the performance of

official duties.

(l) Pequisite of Club Fees / Expenditure :

(a) The value of benefit to the employee resulting from the payment or reimbursement

by the employer of any expenditure incurred (including the amount of annual or

periodical fee) in a club by him or by a member of his household shall be determined

to be the actual amount of expenditure incurred or reimbursed by such employer on

that account. The amount so determined shall be reduced by the amount, if any, paid

or recovered from the employee for such benefit or amenity. However, where the

employer has obtained corporate membership of the club and the facility is enjoyed

by the employee or any member of his household, the value of perquisite shall not

include the initial fee paid for acquiring such corporate membership.

(b) Further, if such expenditure is incurred wholly and exclusively for business purposes,

it would not be treated as a perquisite provided the following conditions are fulfilled:-

(1) complete details in respect of such expenditure are maintained by the

employer which may, inter alia, include the date of expenditure, the nature of

expenditure and its business expediency;

(2) the employer gives a certificate for such expenditure to the effect that the

same was incurred wholly and exclusively for the performance of official duties.

(c) There would be no perquisite for use of health club, sports and similar facilities

provided uniformly to all employees by the employer.

Page 51: INTER CA Taxation - CA Study Web€¦ · INTER CA Taxation Head Office Shraddha, 4th Floor, ... 14. Advance Tax & Interest 268 - 271 15. Tax Deducted at Source (TDS) 272 - 294 16

: 48 :

J. K. SHAH CLASSES INTER C.A.- DIRECT TAXES

SALARIES

(m) Valuation of perquisite in respect of car :

Valuation of perquisite in request of car/ conveyance – Taxable value of perquisite in

different situations shall be calculated as follows -

���� Situation 1 Car is owned or hired by an employer, all expenses are incurred by

employer and provided to an employee only for official purposes - Technically it

is not a "perquisite", as the concerned employee dews not get any personal

benefit at the cost of employer. Consequently, nothing is chargeable to tax.

However, the Board has prescribed two conditions which are to be satisfied by

the employer and only then nothing would be chargeable to tax.

���� Situation 2-Car is owned or hired by employer, expenses are incurred by employer

and provided to an employee wholly for personal purposes - Entire expenditure

incurred by employer (including depreciation at the rate of 10 per cent per annum

of actual cost of the car), is taxable in tire hands of employee. Expenses

recovered from employee are deductible.

���� Situation 3 Car is owned or hired by employer, expenses are incurred by employer

and provided to an employee for partly official and partly personal purposes -

Rs. 1,800 per month (1600 cc or less ) /Rs. 2,400 per month (above 1600 cc) for

car is taxable. If driver is provided, an additional sum at the rate of Rs. 900 per

month shall be taxable. Expenditure recovered from the employee is not

deductible.

���� Situation 4 Car is owned or hired by employer, provided to an employee for

partly official and partly personal purposes, and expenses for private purposes

are incurred by employee - Rs. 600 per month (1600 cc or less)/Rs. 900 per

month (above 1600 cc) for car is taxable. If driver is provided, an additional sum

at the rate of Rs. 900 per month shall be taxable. Expenditure recovered from

the employee is not deductible.

���� Situation 5- Car is owned by employee, expenses are incurred by employer and

car is used for partly official and partly personal purposes - Actual expenditure

incurred by employer minus expenditure pertaining to official use minus anything

recovered from employee, is taxable in the hands of employee. Expenditure

pertaining to official use can be calculated as per logbook of the car . Alternatively,

expenditure pertaining to official use can be calculated at the rate of Rs. 1,800

per month (1600 cc or less)/ Rs. 2,400 per month (above 1600 cc) for car and

Rs. 900 per month for driver.

���� Situation 6 - Any automotive conveyance (other than car) owned by employee

and expenses are incurred by employer - If such conveyance is used only for

official purposes, nothing is chargeable to tax. However, the Board has prescribed

two conditions which are to be satisfied by the employer and only then nothing

would be chargeable to tax. Conversely, it such conveyance is used by the

employee partly for official and partly for private’ purpose, then actual expenditure

incurred by employer minus expenditure pertaining to official use minus anything

recovered from employee, is taxable in the hands of employee. Expenditure

pertaining to official use can be calculated as per logbook of the car.Alternatively,

expenditure pertaining to official use can be calculated at the rate of Rs. 900 per month.

Page 52: INTER CA Taxation - CA Study Web€¦ · INTER CA Taxation Head Office Shraddha, 4th Floor, ... 14. Advance Tax & Interest 268 - 271 15. Tax Deducted at Source (TDS) 272 - 294 16

: 49 :

J. K. SHAH CLASSES INTER C.A.- DIRECT TAXES

SALARIES

CONDITIONS TO BE SATISFIED IF CAR IS USED FOR OFFICIAL PURPOSES-

Where the employer or the employee claims that the motor car is used wholly and

exclusively in the performance of official duty (i.e., Situation l), the following two conditions

should be satisfied —

Condition 1 The employer has maintained complete details of journey undertaken for

official purpose which may include date of journey, destination, mileage, and the amount

of expenditure incurred thereon.

Condition 2 - The employer gives a certificate to the effect that the expenditure was

incurred wholly and exclusively for the performance of official duties.

���� The above conditions should also be satisfied in Situation 5 if the employee

and/or employer claim that the expenses for official purposes is more than Rs.

1,800 per month (or Rs. 2,400 per month if rating of car exceeds 1,600cc)

Profits in lieu of salary :

Profits in lieu of salary “ is defined by section 17(3).These payments are made to an employee

in lieu of or in addition to salary. These are the following –

���� Compensation for loss of employment or modification of the employment terms -

Compensation for loss of employment or modification of terms of employment is

generally treated as a capital receipt. But by virtue of section 17(3)(i), any compensation

due to or received by an assessee from his employer or former employer at or in

connection with the termination of his employment or modification of terms of

employment is taxable as profit in lieu of salary'. It is taxable on "due" or "receipt"

basis, whichever comes earlier. I he recipient may claim exemption under section

10(10B) or 10(10C)

The following are the salient features —

a. compensation is received by an assessee from his employer or former employer;

b. it is received at or in connection with termination of his employment or

modification of terms and conditions relating thereto.

���� Payment from unrecognised provident or unapproved superannuation fund - At any given

time, accumulated balance in any provident fund/superannuation fund consists of the

following —

a. employer's contribution;

b. interest on employer's contribution;

c. employee’s contribution; and

d. interest on employee's contribution.

The first two are taxable as "profits in lieu of salary" subject to the following propositions —

1. The provident fund/superannuation fund is an unapproved fund.

2. These are taxable at the time of payment to the assessee. In other words,

employer's contribution to unrecognised provident/superannuation fund is not

taxable in the year in which contribution is made but is taxable when payment

becomes due or payment is actually made to an employee. Similarly, interest on

employer's contribution is not taxable in the year in which the amount is credited

to the provident fund account but in the year in which payment becomes due or

the payment is made. It may be noted that interest on employee's contribution is

taxable under the head "Income from other sources".

Page 53: INTER CA Taxation - CA Study Web€¦ · INTER CA Taxation Head Office Shraddha, 4th Floor, ... 14. Advance Tax & Interest 268 - 271 15. Tax Deducted at Source (TDS) 272 - 294 16

: 50 :

J. K. SHAH CLASSES INTER C.A.- DIRECT TAXES

SALARIES

���� Payment under Keyman insurance policy - Any sum received under Keyman insurance

policy (including the sum allocated by way of bonus on such policy) is taxable as "profits in

lieu of salary".

���� Any payment t before or after employment - Any sum due or received (whether in lump sum

or otherwise) by an assessee before his joining any employment or alter cessation of Iris

employment will be taxable as "profits in lieu of salary".

���� Any other payment - Any other payment (other than what is exempt under different clauses

of section 10) due to or received by an assessee from his employer (or former employer) is

treated as "profits in lieu of salary". For instance, medical allowance is taxable as "profits

in lieu of solar,-".

Exemption is, however, available in respect of following—

c. payment of gratuity exempted under section 10(10)

b. payment of house rent allowance exempted under section 10(13A)

c payment of commuted pension exempted under section 10(10A)

d payment of retrenchment compensation exempted under section 10(10B);

e payment from an approved Superannuation Fund under section 10(13);

f payment from statutory provident fund or public provident fund;

g. payment from recognised provident fund to the extent it is exempt under

section10(12)

Leave Travel Concession

Valuation of leave travel concession in India [Sec. 10(5)] - Leave travel assistance extended by an

employer to an employee for going anywhere in India along with his family is exempt on the basis

of provisions given in the table below —

Different situations Amount of exemption (exemption is

available only in respect of fare for going

anywhere, in India along with family twice in

a block of four years)

Where journey is performed by air Amount of economy class air fare of the

national carrier by the shortest route or the

amount spent, whichever is less.

Where journey is performed by rail Amount of air-conditioned first class rail

fare by the shortest route or amount spent,

whichever is less.

Where the places of origin of journey and Amount of air-conditioned

destination are connected by rail and journey first class rail fare by the shortest route or

is performed by any other mode of transport the amount spent, whichever is less.

Where the places of origin of journey and

destination (or part thereof) are not connected by rail

Where a recognised public transport system exists First class or deluxe class fare by the

shortest route or the amount spent,

whichever is less.

Air-conditioned first class rail fare by the

shortest route (as if the journey had been

performed by rail) or the amount actually

spent, whichever is less.

Page 54: INTER CA Taxation - CA Study Web€¦ · INTER CA Taxation Head Office Shraddha, 4th Floor, ... 14. Advance Tax & Interest 268 - 271 15. Tax Deducted at Source (TDS) 272 - 294 16

: 51 :

J. K. SHAH CLASSES INTER C.A.- DIRECT TAXES

SALARIES

���� Meaning of “family" - The aforesaid exemption is available in respect of fare for going

anywhere in India along with "family". For this purpose, "family" includes spouse and

children** of the employee. It also includes parents, brothers and sisters of the employee,

who are wholly or mainly dependent upon the employee. However, family does not include

more than two surviving children of an individual born on or after October 1, 1998 (in

reckoning this limit of two children, children born out of multiple births after the first child will

be treated as 'one child only').

���� Only two journeys in a block of 4 years is exempt - Exemption on the aforesaid basis is

available in respect respect of two journeys performed in a block of four calendar years.

The differ blocks are –

(a) 2010-2013 (i.e., January 1, 2010 to December 31,2013);

(b) 2014-2017 (i.e., January 1,2014 to December 31, 2017).

(c) 2018-2021 (1) Jan 18 to 31/12/21)

���� “Carry-over" concession - If an assessee has not availed travel concession or assistance

during any of the specified four-year block periods on one of the two permitted occasions

(or on both occasions), exemption can be claimed in the first calendar year of the next

block (but in respect of only one journey). This is known as "carry over" concession. In such

case, the exemption so availed will not be counted for the purposes of claiming the future

exemptions allowable in respect of two journeys in the subsequent block

���� Exemption is available in respect of shortest route - Where the journey is performed by a

circuitous route, the exemption is limited to what is admissible by the shortest route.

Likewise, where the journey is performed in a circular form touching different places, the

exemption will be limited to what is admissible for the journey from the place of origin to the

farthest point reached, by the shortest route.

What are permissible deductions from salary (Section 16)

1. Standard deduction – Now standard deduction is not available.

2. Entertainment allowance - Entertainment allowance received is fully taxable and is first to

be included in the salary and thereafter the following deduction is to be made: However,

deduction in respect of entertainment allowance is available in case of Government

employees. The amount of deduction will be lower of:

(i) One-fifth of his basic salary or

(ii) ` ` ` ` 5,000 or

(iii) Entertainment allowance received.

Amount actually spent by the employee towards entertainment out of the entertainment

allowance received by him is not a relevant consideration at all.

3. Professional tax or Tax on employment - Professional tax or taxes on employment levied by

a State under Article 276 of the Constitution is allowed as deduction only when it is actually

paid by the employee during the previous year.

If professional tax is reimbursed or directly paid by the employer on behalf of the employee,

the amount so paid is first included as salary income and then allowed as a deduction u/s 16.

Page 55: INTER CA Taxation - CA Study Web€¦ · INTER CA Taxation Head Office Shraddha, 4th Floor, ... 14. Advance Tax & Interest 268 - 271 15. Tax Deducted at Source (TDS) 272 - 294 16

: 52 :

J. K. SHAH CLASSES INTER C.A.- DIRECT TAXES

SALARIES

Meaning of 'salary' for various purposes :

Page 56: INTER CA Taxation - CA Study Web€¦ · INTER CA Taxation Head Office Shraddha, 4th Floor, ... 14. Advance Tax & Interest 268 - 271 15. Tax Deducted at Source (TDS) 272 - 294 16

: 53 :

J. K. SHAH CLASSES INTER C.A.- DIRECT TAXES

SALARIES

Q. 1. Mr Narendra Modi of Mumbai is employed in XYZ Ltd since 2009. For the Previous

year 2017-2018 he had following salary particulars

Basic Salary – `̀̀̀ 45,000pm

Dearness allowance (entering into retirement benefits) – `̀̀̀ 3,000pm

Dearness allowance (entering into retirement benefits) - `̀̀̀ 2,000pm

Bonus due – 85,000.

Commission based on percentage of turnover - `̀̀̀ 40,000

Commission based on percentage of Net Profit - `̀̀̀ 50,000

Advance Salary – `̀̀̀ 60,000

Arrears of Salary – `̀̀̀40,000

Uniform allowance - `̀̀̀ 2,500pm (Actual expenses on purchase of uniform amounts to

`̀̀̀ 22,500)

Children Education Allowance – `̀̀̀ 240pm for 3 children (Actual Expenses on Educa-

tion Amounts to `̀̀̀ 40,000)

Transport Allowance – `̀̀̀ 1,450pm upto December 2017. From January 2018 it was

increased to `̀̀̀ 2,100pm. (Actual expenses on commutation amounts to `̀̀̀ 10,000)

House Rent Allowance received – `̀̀̀ 20,000pm(Rent paid ? 23,000pm)

Project Allowance `̀̀̀ 2,000pm (Actual Expenses amounts to ? 15,000)

Entertainment Allowance `̀̀̀ 700pm (Actual Expenses on entertaining office visitors

`̀̀̀ 12,000)

Professional Tax paid during PY 2017 - 2018

For PY 2016 – 2017 – `̀̀̀ 3,000

For PY 2017 – 2018 - `̀̀̀ 2,000

Compute income from salary for PY 2017 – 2018.

Q. 2. Mr. Karan, an employee of XYZ Co. Ltd. at Mumbai and covered by Payment of Gratu-

ity Act, retires at the age of 60 years (age fixed by company for retirement) on 31-12-

2017 after completing 30 years and 7 months of service. At the time of retirement, his

employer pays `̀̀̀ 18,00,000 as Gratuity and `̀̀̀ 6,00,000 as accumulated balance of

Recognised Provident fund. He is also entitled for monthly pension of `̀̀̀ 8,000. He gets

75% of pension commuted for `̀̀̀ 4,50,000 on 1st February, 2018. Leave encashment of

`̀̀̀ 3,30,000 for 330 days leave balance in his account. He was credited 30 days leave

for each completed year of service. Amount received from superannuation Fund –

`̀̀̀ 4,00,000

Determine the salary chargeable to tax for Mr. Karan for the Assessment Year

2018-19 with the help of following information:

Basic Salary - `̀̀̀ 80,000pm

Bonus received – 36,000

Employers Contribution towards RPF – 1,10,000.

House rent allowance – `̀̀̀ 15,000 pm

Rent paid by Mr Karan – 10,000 pm

Professional Tax paid – 2,700.

CLASS WORK PROBLEMS

Page 57: INTER CA Taxation - CA Study Web€¦ · INTER CA Taxation Head Office Shraddha, 4th Floor, ... 14. Advance Tax & Interest 268 - 271 15. Tax Deducted at Source (TDS) 272 - 294 16

: 54 :

J. K. SHAH CLASSES INTER C.A.- DIRECT TAXES

SALARIES

Q. 3. Mr. Albert Pinto is working as officer in multinational organisation since 1st Aug 2013

on a salary scale of `̀̀̀18,000-2,000-24,000-3,000-45,000.In the previous year

2017-18 his other salary particulars as below :

D.A. upto July 2017 (not entering into retirement benefit) : 80% of basic salary

D.A .since Aug. 2017 (entering into retirement benefit as per modified agreement):

40% of basic salary.

Compensation received for modification in terms of employment `̀̀̀ 60,000 (TDS

`̀̀̀ 20,000)

H R A `̀̀̀12,000 per month in a non-metro(Rent paid to house owner being his

father, `̀̀̀14,000 p.m). Gratuity received `̀̀̀2,00,000/-.

Equity shares having fair market price of `̀̀̀ 1,00,000 were allotted to him by the

company at a concessional price of `̀̀̀ 20,000 on 30.5.2016.

You are required to calculate his income from salary for A.Y 2018 -19

Tax on employment paid by employee `̀̀̀2,500.

Q. 4. Mr Vinay aged 42 years, is the sales manager at PQR Ltd (in Mumbai). From the

following details compute his income from Salary for Assessment Year 2018-2019

���� Basic Salary – `̀̀̀ 30,000pm

���� Dearness allowance (entering into retirement benefits) – `̀̀̀ 2,000pm

���� Dearness allowance (entering into retirement benefits) - `̀̀̀ 1,000pm

���� Bonus due – 30,000.

���� Commission – 0.1% of the turnover of the company. The turnover for the F.Y.

2017-18 was `̀̀̀ 10.00 crores

���� Advance Salary – `̀̀̀ 60,000

���� Arrears of Salary – `̀̀̀ 40,000

���� Conveyance allowance - `̀̀̀ 2,500pm (Actual expenses on amounts to ? 22,500)

���� Motor car running and maintenance charges fully paid by employer (The motor

car is owned by the company and driven by the employee. The engine cubic

capacity is above 1.60 litres. The motor car is used for both official and personal

purpose by the employee.) – `̀̀̀ 60,000

���� Expenditure on accommodation in hotels while touring on official duties met by

the employer – `̀̀̀ 80,000.

���� Lunch provided by the employer during office hours. Cost of lunch provided

`̀̀̀ 27,000.

���� Free education was provided to his two children Vidhi and Vinaya in a school

maintained and owned by the company. The cost of such education for Vidhi is

computed at `̀̀̀ 800 per month and for Vinaya at `̀̀̀ 1,300 per month. No amount

was recovered by the company for such education facility from Vinay.

���� Leave travel concession given to employee, his wife and three children (one

daughter aged 7 and twin sons aged 3). Cost of air tickets (economy class)

reimbursed by the employer `̀̀̀30,000 for adults and` ` ` ` 45,000 for three children.

Vinay is eligible for availing exemption this year to the extent it is permissible in

law.

���� He is provided free housing facility which has purchased by company 2 years

ago. He is also provided with refrigerator (Cost of refrigerator Rs 80,000).

���� Company also gifted him iPhone costing `̀̀̀ 82,000.

���� Telephone provided at the residence of Shri Bala and the bill aggregating to

`̀̀̀ 25,000 paid by the employer.

���� Profession tax paid `̀̀̀ 3,000 of which `̀̀̀ 2,000 was paid by the employer.

Page 58: INTER CA Taxation - CA Study Web€¦ · INTER CA Taxation Head Office Shraddha, 4th Floor, ... 14. Advance Tax & Interest 268 - 271 15. Tax Deducted at Source (TDS) 272 - 294 16

: 55 :

J. K. SHAH CLASSES INTER C.A.- DIRECT TAXES

SALARIES

Q. 5. Ascertain taxable perquisite value of motor car in the 3 independent situations.

He is provided car facility with cubic capacity of 1200 cc / 1.2 Lt cc :

Details regarding Car facility :

Petrol & maintenance expenses incurred by employer 45,000,

Cost of car 3,25,000,

Driver salary 48,000 incurred by employer

1) Car(owned by ER) is used for full private use & employer recovers 10,000 from EE

2) Car(owned by ER) is used for part private use & employer recovers

10,000 from EE.

3) Car is owned by employee & used by employee for office as well as personal use.

Q. 6. Mr. R, an employee of DB Real Estate Ltd., Mumbai, a private sector company, re-

ceived the following for P.Y. 2017-2018.

Basic pay `̀̀̀ 3,00,000

Bonus `̀̀̀ 50,000

Project Allowance `̀̀̀ 30,000

D.A. (not in terms) `̀̀̀ 2,00,000

D.A. (in terms) `̀̀̀ 2,50,000

Employer's Contribution to R.P.F. `̀̀̀ 6,000 p.m.

Advance Salary `̀̀̀ 20,000

The company gave him an option either to take a rent free unfurnished

accommodation at Mumbai for which the company would directly bear the rent of

`̀̀̀10,000 p.m. or to accept a house rent allowance of `̀̀̀12,000 p.m. and find out his

own accommodation. If he opts for house rent allowance, he will have to pay

`̀̀̀ 9,000 p.m. for an unfurnished house in Pune. Which of the two options should be

selected in order to minimise his tax liability?

Q. 7. Mr. A retires on 15th October 2017, after serving 30 years and 7 months. He gets

`̀̀̀ 3,80,000 as gratuity. His salary details are given below:

FY 2017 -18 Salary `̀̀̀16,000 pm D.A. 50% of salary. 40% forms part of retirement

benefits.

FY 2016 -17 Salary `̀̀̀ 15,000pm D.A. 50% of salary. 40% forms part of retirement

benefits.

Determine the taxable value of gratuity in the following cases:

(i) He retires from Government service

(ii) He retires from private sector, covered by Payment of Gratuity Act, 1972

(iii) He retires from private sector, not covered by payment of Gratuity Act, 1972.

Page 59: INTER CA Taxation - CA Study Web€¦ · INTER CA Taxation Head Office Shraddha, 4th Floor, ... 14. Advance Tax & Interest 268 - 271 15. Tax Deducted at Source (TDS) 272 - 294 16

: 56 :

J. K. SHAH CLASSES INTER C.A.- DIRECT TAXES

SALARIES

Q. 8. X, an employee of HMT Ltd., retires on March 31, 2018 after 29 years and 10 months

of continuous service. He receives `̀̀̀85,000 as cash portion of his earned leave, from

the following particulars, compute the amount of exemption u/s. 10(10AA).

(i) Average salary received during the past 10 months :

Basic 4,800 pm

DA (forming part for retirement benefits) 1,200 pm

DA (not forming part for retirement benefit) 1,000 pm

CCA (City compensatory Allowance) 500 pm

Fixed % Commission on turnover 150 pm

(ii) Leave entitlement for each year of service completed 1-1/2 month.

(iii) Leave availed while in service 18 months.

Would your answer be different if X were an I.A.S. Officer of the Central Government.

Q. 9. John is employed in a private company and is paid a sum of `̀̀̀6,00,000 on voluntary

retirement from service. The normal age of retirement in the company is 60 and John,

who was 45 at the time of retirement had completed 20 years of service.

His monthly salary at the time of retirement was as follows :

Basic Pay `̀̀̀ 10,000

Dearness Allowance (50% includible for retirement benefits) `̀̀̀ 6,000

H.R.A. `̀̀̀ 3,000

Conveyance Allowance `̀̀̀ 800

What is the amount of compensation taxable under the Act?

Q. 10. Mr. X retired in P.Y. 2008-09 and had received gratuity of Rs. 45,000 at the time of

retirement. For P.Y. 2017-2018, he was receiving pension of Rs. 10,000 per month. On

31st December 2017, he commuted 40% of his pension for Rs. 6,00,000/-. Calculate

taxable pension for P.Y. 2017-2018 if

(i) Mr. X is a non –government employee

(ii) Mr. X is a government employee.

Q. 11. From the following information submitted by Sunil, in respect of monthly salary and

allowances, find out the house rent allowance chargeable to tax for the assess-

ment year 2018-2019 :

Page 60: INTER CA Taxation - CA Study Web€¦ · INTER CA Taxation Head Office Shraddha, 4th Floor, ... 14. Advance Tax & Interest 268 - 271 15. Tax Deducted at Source (TDS) 272 - 294 16

: 57 :

J. K. SHAH CLASSES INTER C.A.- DIRECT TAXES

SALARIES

Q. 12. Mr. Sai, an employee furnished the following particulars for the previous year ending

31.3.2018 :

`̀̀̀

(a) Salary income as computed (after all deduction) for the year 6,70,000

(b) During the year arrears of salary were received (not included

in the above) which relate to financial year 2011-12 90,000

(c) Assessed income of financial year 2011-12 4,85,000

You are requested to compute relief u/s 89 in terms of tax payable.

The rates for the Assessment Year 2012-13 are :

0 - 1,80,000 - NIL

1,80,001 to 5,00,000 - 10%

5,00,001 to 8,00,000 - 20%

Above Rs. 8,00,000 - 30%

Surcharge NIL, Education cess 3%.

Q. 13.Mr. A is employed with X Ltd. (salary before the medical perquisite for the year is

`̀̀̀ 1,80,000). Mr. A has to undergo heart surgery outside India and X Ltd. bears the

following expenses of Mr. A and one attendant.

Cost of medical treatment of Mr. A outside India `̀̀̀ 4,50,000 (out of which permitted

by R.B.I. `̀̀̀ 4,38,000).

Cost of stay abroad of Mr. A and one attendant `̀̀̀ 1,50,000 (out of which permitted

by R.B.I. `̀̀̀ 1,42,000).

Cost of travel of Mr. A and one attendant `̀̀̀ 2,75,000 (out of which permitted by

R.B.I. `̀̀̀ 2,50,000).

Ascertain the taxable perquisite value if :

���� Mr. A does not have any other income.

���� Mr. A has interest on Government securities of `̀̀̀ 6,000.

Q. 14.Mr Akshay a government employee retired on 31.10.2017 after rendering service

for 15 years 10 months . He provides you the following information for the year

ended 31.03.2018.

Basic Salary – `̀̀̀ 25,000pm.

Dearness allowance – `̀̀̀ 10,000pm. (30% of dearness allowance enters into retire-

ment benefit)

Commission - `̀̀̀ 2,000pm

Advance Salary – `̀̀̀ 30,000

Arrears of Salary - `̀̀̀ 70,000

Transport Allowance - `̀̀̀ 2,000pm

Gratuity received – `̀̀̀ 2,00,000

He is entitle for monthly pension of `̀̀̀10,000pm. He gets 70% of his pension

commuted for `̀̀̀4,00,000 on 1st February 2018.

Leave encashment received – 3,00,000.

Amount received from Statutory Provident Fund – 20,00,000 ( `̀̀̀ 7,50,000 is

employer contribution, 7,50,000 is employee contribution and balance 5,00,000 is

interest)

Professional Tax paid – `̀̀̀ 2,500.

Compute Income from salary for Assessment year 2018 – 2019.

Page 61: INTER CA Taxation - CA Study Web€¦ · INTER CA Taxation Head Office Shraddha, 4th Floor, ... 14. Advance Tax & Interest 268 - 271 15. Tax Deducted at Source (TDS) 272 - 294 16

: 58 :

J. K. SHAH CLASSES INTER C.A.- DIRECT TAXES

SALARIES

Q. 15. Mr Anupam a government employee provides the following detail of his salary

income for previous year 2017-2018

Basic Salary – 40,000pm

Dearness Allowance - 10,000pm. (80% of dearness allowance does not enter into

retirement benefit)

Entertainment allowance – 5,000pm (Actual Expenses on entertaining office visitors -

20,000)

Transport Allowance - 1350pm (Actual Expenses on commutation – 10,000)

Advance salary – 40,000.

Arrears of Salary - 90,000.

Gratuity received – 1,00,000.

Leave encashment - 70,000.

Employers Contribution to Statutory Provident Fund - 7,000 pm. Employees

Contribution to Statutory Provident Fund - 7,000 pm. Interest credited @ 10% is

1,00,000.

The government has provided him with rent-free accommodation in Mumbai along with

television (WDV 10,000; Cost 25,000) and two air conditioners. The rent paid by

the company for the air conditioners is 400 p.m. each. The television was provided

on 1.7.2017.The licence fees determined by the Government for this accommodation

was 2,000 p.m.

Professional Tax paid – 2,000.

Compute Income from Salary for Previous year 2017 – 2018.

Q .16. Mr. Salman received retrenchment compensation of 10,00,000 after 30 years 4

months of service. At the time of retrenchment, he was drawing basic salary 20,000

p.m.; dearness allowance (entering into retirement benefit) 3,000 p.m. Dearness

allowance (not entering into retirement benefit) 2,000 p.m. Compute his taxable

retrenchment compensation.

Page 62: INTER CA Taxation - CA Study Web€¦ · INTER CA Taxation Head Office Shraddha, 4th Floor, ... 14. Advance Tax & Interest 268 - 271 15. Tax Deducted at Source (TDS) 272 - 294 16

: 59 :

J. K. SHAH CLASSES INTER C.A.- DIRECT TAXES

SALARIES

Ans. 3. Mr. Albert Pinto

Computation of income from salaries for A.Y. 2018-2019

Basic Salary 3,12,000

D.A. (in terms) 86,400

D.A. (not in terms) 76,800

Compensation for modification in terms of employment

(60,000+20,000) 80,000

H.R.A. received (12,000 p.m. x 12 months) 1,44,000 23,840

Less: Exempt u/s. 10(13A) (1,20,160)

Gratuity received during service period, FULLY TAXABLE 2,00,000

Perquisite of equity shares allotted at a concessional price,

taxable u/s. 17(2)(vi) (1,00,000 – 20,000) 80,000

GROSS SALARY 8,59,040

Less: Deduction u/s. 16(iii): Professional tax paid (2,500)

NET TAXABLE SALARY 8,56,540

W.N. 1Basic Salary

Date of increase Basic salary per month

1-8-2013 18,000

1-8-2014 20,000

1-8-2015 22,000

1-8-2016 24,000

1-8-2017 27,000

1-8-2018 30,000

Basic salary for P.Y. 2017-2018

From 1-4-2017 to 31-7-2017 :24,000 x 4 months = 96,000

From 1-8-2017 to 31-3-2018: 27,000 x 8 months = 2,16,000

3,12,000

W.N. 2Exemption u/s. 10(13A)

April- July August - March

1. Basic Salary 96,000 2,16,000

2. D.A. (in terms) Nil 86,400

3. Fixed % Commission on turnover Nil Nil

SALARY 96,000 3,02,400

Exemption :- LEAST of

1. 40% of SALARY 38,400 1,20,960

2. Actual HRA received 48,000 96,000

(12,000 x4) (12,000 x 8)

3. Rent paid – 10% of SALARY 46,400 81,760

[(14,000 x 4) – [(14,000 x 8) –

(10% x 96,000)] (10% x3,02,400)]

Exemption = 38,400 + 81,760 = 1,20,160/-

Note:- The TDS of Rs. 20,000 shall be reduced from the tax payable by Mr. Albert Pinto

on his total income since this amount must have been already deposited by the employer

on his behalf with the Central Government.

CLASS WORK SOLUTIONS

Page 63: INTER CA Taxation - CA Study Web€¦ · INTER CA Taxation Head Office Shraddha, 4th Floor, ... 14. Advance Tax & Interest 268 - 271 15. Tax Deducted at Source (TDS) 272 - 294 16

: 60 :

J. K. SHAH CLASSES INTER C.A.- DIRECT TAXES

SALARIES

Ans.10. Mr. X

Computation of taxable pension for A.Y. 2018-19

(i) Mr. X is a non –government employee

Uncommuted pension

- From April to December = 10,000 p.m. x 9 months = 90,000

- From January to March = (10,000 p.m. x 60%) x 3 months = 18,000

Taxable uncommuted pension 1,08,000

Commuted pension

Pension received 6,00,000

Less: Exempt u/s. 10(10A)

T.C.V. = 6,00,000 / 40% = 15,00,000

Exempt = 1/3rd of TCV of 15,00,000 (5,00,000)

Taxable Commuted pension 1,00,000

Total pension taxable (A+B) 2, 08,000

(ii) Mr. X is a government employee

Uncommuted pension

- From April to December = 10,000 p.m. x 9 months = 90,000

- From January to March = (10,000 p.m. x 60%) x 3 months = 18,000

Taxable uncommuted pension 1,08,000

Commuted pension

Pension received 6,00,000

Less: Exempt u/s. 10(10A) (6,00,000)

Taxable Commuted pension NIL

Total pension taxable (A+B) 1,08,000

Ans. 12. Mr Sai

Computation of relief under section 89 for A.Y 2018-19

Particulars Amount( `̀̀̀)

Step 1: Actual income for PY 17-18 6,70,000

Add: Arrears of salary for PY 11-12 90,000

TOTAL INCOME 7,60,000

Page 64: INTER CA Taxation - CA Study Web€¦ · INTER CA Taxation Head Office Shraddha, 4th Floor, ... 14. Advance Tax & Interest 268 - 271 15. Tax Deducted at Source (TDS) 272 - 294 16

: 61 :

J. K. SHAH CLASSES INTER C.A.- DIRECT TAXES

SALARIES

Basic tax (as per slab rates for AY 18-19) 64,500

Add: Education cess @ 3% 1,935

Tax at (A) 66,435

Step 2: Actual income for PY 2017-18 6,70,000

Basic tax (as per slab rates for AY 18-19) 46,500

Add: Education cess @ 3% 1,395

Tax at (B) 47,895

Step 3: Tax at (A) - Tax at (B) (C) 18,540

Step 4: Actual income for PY 11-12 4,85,000

Add: Arrears of salary for PY 11-12 90,000

TOTAL INCOME 5,75,000

Basic tax (as per slab rates for AY 12-13) 47,000

Add: Education cess @ 3% 1,410

Tax at (D) 48,410

Step 5: Actual income for PY 11-12 4,85,000

Basic tax (as per slab rates for AY 12-13) 30,500

Add: Education cess @ 3% 915

Tax at (E) 31,415

Step 6: Tax at (D) - Tax at (E) (F) 16,995

Step 7: Relief u/s. 89 1,545

If C > F, Relief = C - F [ 18,540 (-) 15,300]

If C < = F, Relief = NIL

COMPUTATION OF TAX PAYABLE FOR A.Y. 2018-19

Total income(including arrears) `̀̀̀ 7,60,000

Basic Tax `̀̀̀ 64,500

Add: Education cess @ 3% `̀̀̀ 1,935

`̀̀̀ 66,435

Less: Relief u/s 89 (`̀̀̀ 1,545)

TAX PAYABLE `̀̀̀64,890

Page 65: INTER CA Taxation - CA Study Web€¦ · INTER CA Taxation Head Office Shraddha, 4th Floor, ... 14. Advance Tax & Interest 268 - 271 15. Tax Deducted at Source (TDS) 272 - 294 16

: 62 :

J. K. SHAH CLASSES INTER C.A.- DIRECT TAXES

SALARIES

Ans.13. Computation of taxable perquisite value for A.Y. 2018-2019

Particulars Case I Case II

Perquisite of medical treatment abroad (4,50,000 – 4,38,000) 12,000 12,000

Perquisite of cost of stay abroad (1,50,000 – 1,42,000) 8,000 8,000

Perquisite of cost of travel (W.N.) NIL 2,75,000

Taxable Perquisite Value 20,000 2,95,000

W.N. G.T.I. (excluding perquisite of travel cost)

Particulars Case I Case II

Salary before perquisite of medical treatment abroad 1,80,000 1,80,000

Perquisite of medical treatment abroad 12,000 12,000

Perquisite of cost of stay abroad 8,000 8,000

Any other income NIL 6,000

G.T.I 2,00,000 2,06,000

In the second case, the GTI (excluding perquisite of travel cost) exceeds ̀̀̀̀ 2,00,000

and hence the total travel cost borne by the employer shall be taxable for the

employee and RBI limit for this purpose is irrelevant in deciding taxability.

Page 66: INTER CA Taxation - CA Study Web€¦ · INTER CA Taxation Head Office Shraddha, 4th Floor, ... 14. Advance Tax & Interest 268 - 271 15. Tax Deducted at Source (TDS) 272 - 294 16

������

���������������� ��������� ������������

���������������� � ���

��������������

������������� �������������������� ����������������������������������������� ������������������������������������������������������������������������������������� �������������������������������������� ����!��" ���������������������������������#�����������������������������������#� ������������������

������������������������ ��!�����""#��������""��$���������������������������%%��������������������&������������������'��$��$���'������������������(�����$��$���'������������������������$%& ��������������������������� ��������� �����������������$�& ������������������������������������������$�������� �������

����&$ & ���������������������� ���������������������������������������������

��������������������� ����������#

���%'���������������""$�& '�� �������� ��������� ��(������ �'�� ��������� ���)������'�� ������*������

+������,���������������-������� �.���������������� �������� ������������

$�& /����"������������� ������������������� ������ �0�������������������� ��������� ���� ��� ����� ��� �� ������� � ��� ��������� �� ��� �� ����� ����������� ������ �� ���� ����� ��� ��� ����� ����������� ����������� ��������� ��������������������������������������������������1����� ���������������������� ���������������������

$�& �������������2������ ��������������� ������������������������ ������������������������������������ ���������#�� ��3����������3��45���������������*��+������

$�& )��%��� �*�������+�������� �������� ���3�� ������� ����������������������������������������������������1���������������������������������������3������������������������������������������������ ���������������������������������������������������������#��������������

$& �������������� ��������������������������������������������������������������� ��������������������3���!�

$�& �� ����������������������� ����������������������� ���������������������������������������������� �������������������������������0�������������#�������������������������� �0���������������������6��������������������

$�& ,���� ��$��$���'������������������������������������6����������� ������������������������������������#�����������������������������(�6�*(�$(����������������������������������������������� �6�17)&������������ ������ �� ������������������������������#�"����������������� ������������������������8(������������ ��������� �������������������������$����� ������������������&���

��������������������������������

���������������� � ���

Page 67: INTER CA Taxation - CA Study Web€¦ · INTER CA Taxation Head Office Shraddha, 4th Floor, ... 14. Advance Tax & Interest 268 - 271 15. Tax Deducted at Source (TDS) 272 - 294 16

���-��

���������������� ��������� ������������

���������������� � ���

���� �������� � ��� ������ ������ ����� ����������� �������� � ��� ������ ������ ����� ����������� �������� � ��� ������ ������ ����� ����������� �������� � ��� ������ ������ ����� ���������� ��� � ����������� ��� ����� �������� ������ ����� ��� � ����������� ��� ����� �������� ������ ����� ��� � ����������� ��� ����� �������� ������ ����� ��� � ����������� ��� ����� �������� ������ ������� �������� �������� ������ ��� ������� �� ������� �������� �������� ������ ��� ������� �� ������� �������� �������� ������ ��� ������� �� ������� �������� �������� ������ ��� ������� �� ����� �� �������� ��������� �� �������� ��������� �� �������� ��������� �� �������� ������

� ������� !��"��������#���$��%�����&����������

� ������� !� ��������#���$��%�����&�����������'

���������������$%&��������������$��

� ������� !��!����������������������������������������������

�����'���������������������������������������������������������

���� '� ��� $%&���� ��� ��������� �������� � �� ����������� ��

������� !�"�(

� ������� !��)��������������#��������������������������������

�#�������������� ����������������������������������� ��������

������������������'����$%&������������������ ���������� !�� �

��(��*����$%&��'��������� �������������������������������

��(�(�(�(��'������$%&������������������ ���������� !��"�

+,��� � %$$-%�� &%�-.� �+(%(&(� � � ������� !�+,��� � %$$-%�� &%�-.� �+(%(&(� � � ������� !�+,��� � %$$-%�� &%�-.� �+(%(&(� � � ������� !�+,��� � %$$-%�� &%�-.� �+(%(&(� � � ������� !�

��� %��������� �������%(�(&(��/�,�������������� �������,(�(&(�

���������������������#����������� �(�(��� ������� ������������

�����������������0�������������������������,���������

%�����������������(

��� %���� ���� ��������� /� ����������� ���� ��� ������� � ���

��������������������������������������������������(

��� %���� �� ���� ������� ���� ���� ���� ���� ��� ������� ��

��������#��������(

1������2�������������+(%(&(

1������3�4�������������+(%(&(

������� !� ��� ���� %�������� !� ��� ���� %�������� !� ��� ���� %�������� !� ��� ���� %�

������������������������� ������������������������������

���������������������5

��� ��� ��� ���� #����� ��� �������� �� �� ����������� ��

�6���������������������������7���

��� #������������������������������������������ ��������

��� ����� ���� ��������� ��� ����������� ��� ��� �#���� ��

����������������������6���������������������������������

��������������������������������������7���

��� #������������������������������������������ ��������

#������������� � ���#������������������� ����������

����������#�� ����������������#�� ����������������#�� ����������������#�� ���������������� ��������������������

��� �������������� ����#���� ���������� ������� ��� ����� ���

�����������������������������������������������������

����������(

����������������6���������������������������������������������

�����������������������������������������������������������#����

�����������������������6���#������������������#������������ ��

������������������� ��� ����������������������������������5

�� ���������������������������������������������������#����

�����6�������������������������(

.6���������5������������������������������������������������

��5���������������������������������������� �������������

����#�����������������������0����������������������������

������������������������������#���������#�����������������(

�� � �� � � �� � � � � � ������ � ��� � ���� � � � � � �

�������������������������������������������������� �

����������������������������������������������������� ��������������������� ���� ��� �������

��������������������������

���� ���������������������

���� �� ���������� ������� ��

����������������� ����������������� ������ ��� ��� ������� � �

��������

���� ������������������������������������������������ �����������

���� ���������� ����

���� �� ��������� ��� ������ �������������������������������������������������� �������������

� ��������� ������������������ � �

���������� !������� � ��� ���

���������������������������

Page 68: INTER CA Taxation - CA Study Web€¦ · INTER CA Taxation Head Office Shraddha, 4th Floor, ... 14. Advance Tax & Interest 268 - 271 15. Tax Deducted at Source (TDS) 272 - 294 16

���.��

���������������� ��������� ������������

���������������� � ���

��%�� /���$���� � 0��$���'� !�/0#� ��� 0��$���'� ����1��� ���� ��%�� /���$���'�

��������"��!"#

���/0��������$�&�+ ��*�������1����.����������$�&�+ ��*�������.������0�

1������ *� �����1������ ��"�������� �� �� ����� ��� ��������� ���'������� ��

.��������������������� �������������������� ���������������������������

������������ ���������

�����/0�&����������'��� ����������������9��

$�& ������������������%������������������������������������

!�# ����������������������������������������

��2���������/0�����%3�'����4����������������+*.������ ������������ ���9

� :������� ��� �

� )����(�

� +���������

� :������� ��#�

���&���������%�������$��$���' ��������"��!-#

����������������6�������������������������������� �������������������

������� $�&���������������������������������������� ��6��������������������

��������������� ����$;/*.&��

"���� ��� ����������;/*.����� ������ �� ����3��� $%&$�&�

5�2�����4/0��:������� �2� ������6�)����������������������������������������

<#�������

<#����������+�������(������������� ������,"2�����;/*.�

����"�!.#����������������������������������� �������� �������������������

� ��������0�������������������������������������������������� ���������

��� ������������������������������������� ��� ����������������������������

������������������������������������������������������� ���������������

��������������� ����������������������������������������������������

������������ ����0�������� ���

���6��/����2��4�)4��,/7��������6�4�2�46��!����2�#�8�����"-9��

"������������8�"���� �2� �����������������������������73���=������

����������������� ���9

!:# �������������������

�������;<�������������%�1�%����������������������������������#���������

��������������������#� ��������������������������� �

��������������������������������������#����� �0�������������

������ ������������ ���������������� ������������������������ �#����

��������������� ������������ �����

Page 69: INTER CA Taxation - CA Study Web€¦ · INTER CA Taxation Head Office Shraddha, 4th Floor, ... 14. Advance Tax & Interest 268 - 271 15. Tax Deducted at Source (TDS) 272 - 294 16

������

���������������� ��������� ������������

���������������� � ���

!"# ���������/��)����3����$���%�

$�& ��������������������� ����� ���� ��������������������� ���������

����� ���������������������������������������������������� �������

�������������>��������������������������$ - ( ( ( " &

$��& "������������������� �� ����������� � �������������� ����� �������

�������������� �������������������������������� ���������������

����

$���& ��������������������3�������������������������� �

$��& ��������������� �����0������������������ � ���������������������

������������������ ���� ������������

$�& .����������������������������� ������%34���������������4������

$��& �/����6��/�������������� ����������������3����������������

���������#�������������������+������4�

$���& �/����6��/������� ��������0����������������������������������������

���� ��������������������>�������������������������$-((("&�

����� ��� �� ���� ��� �0�� ���� ������ � �������� �0� ���� ��� ���?����������

���������������������������������������������� ��������

$����& '��0����3�-�������������������������� ����������������� �

������������%��������4�/�0��*���4�/�0��*�4�/�0��3������/�0

������� �������������� ��� �����������������������������=$�&����������

��#����� ����

������������%����������/�0�

�����#����������������������� � @�@@@���� ���@@�@@@

������/0����������&�=�&�&���� �">;;>;;;?

����� ������=������������������ ����� �����������������������������������

��#����� ���@@�@@@�

%�� ��� �������� �����0������������%�=�%AAA�

��� /��������������-�������������"�>�����������+*.�

������������$/�������� �����������(������������(��������(�������&

�� ������������� �������������������������������>������������ ���������

4���������������������)������� �B�������������� ��������0��

<���9 /�����0���� �@�"����@@C

)B��� %�:����@@A

"���4����� %�:����@%=

Page 70: INTER CA Taxation - CA Study Web€¦ · INTER CA Taxation Head Office Shraddha, 4th Floor, ... 14. Advance Tax & Interest 268 - 271 15. Tax Deducted at Source (TDS) 272 - 294 16

���@��

���������������� ��������� ������������

���������������� � ���

���������������������>��������������� �������� %�:������@%=��� ��#����

����������� ���@@�@@@��*������������ � @�@@@��� ��

=�� -�������������������� ��������������

�����������������������������������������#���������������������+*.���

�� �� � @�@@@�

0�� ����������������������*�0�����0����������������

������������������������������������ ��� �� ������ ���.�������������.����������

.��-����������.����������������������������������������������� ���������������

���������>��������������������������������� �� �������� ����9

.��-����������.����������D������������

#�����������

�#�.��-���������

������ ����������������������������.����

.�������������.����

)����9��;������0���� ���

�� �����������/"�<(

��;����������������������

���9 "���������������.�B������������������������3���>��������������� ��

������������������������������� ���� �����������>�� ������ ����� ����$����%34

�������&���������������.�B���������������������������>��������������� ��

�����3��������������1��'������

"��� ������D����������.�������B���Г%34�#�.-.�

��������������1��'�����

"��� ������D�����������.�������B��

��=������&����/�����1��'���������������������������1��'����6����%����������6*��".�

7����� ���"���������(�������������������� ������������ ����������� ���������

���������������������������������������������������������

��� �� ���� ���� ��� ����� ��� ���� ���� ���� ������� �� ��� ���� ��� ���� ���� ���

+���>�� ����������������������� �������������� �������� ������� ���������F�

�������������������������

��� ����%��'���(�������"���������(�������������������� ��������� ����������

���#�����������������������1����.�������� ��

A���������=���%��'��������#�� ����������������������� �������������������������

���� ����������������

/3������$���������������������� ����#�� ��������������������������������������

���������������������������������

������������"�������������������� @G����������������������������������� ���

����������� ���� ����������������"�������#�������������������

Page 71: INTER CA Taxation - CA Study Web€¦ · INTER CA Taxation Head Office Shraddha, 4th Floor, ... 14. Advance Tax & Interest 268 - 271 15. Tax Deducted at Source (TDS) 272 - 294 16

���B��

���������������� ��������� ������������

���������������� � ���

/ /����(�0���������"��

�����������������������������������������H��� �������������������������������

������������� ����������������������������%%�������������������������$���������

���������������$�������!��/�0#>����������������������������������������������� ��

��� �������������������� �������

������������������������� �����������������������������������������������������

����������������� ����������� ��������������������������� ��� ���� ���8�/

�������� ���#���������#���������������������� ������������������������

��#����� ������� @�@@@�3���@@�@@@����������������������������3�������3���=$�&��

��������������������������������������� ����������������������� ���������

�������� ����� �� ���������/�*�.�����������������������������������������������

������������������ ������������ ����������������� ������ ������������������

���������������������������������������

���+�*�.��8�/�3�/����� ���������/�*�.������������������������������������������

��������

���7���/�����8��������"@9

"�����+������!��� ��������������� ���������������������������������

������������ ���������������#��������������9

$%& "����������� ���������������������������������>�����������������������

���������$�������������������������������������������� �������&���������

��������� ��$�����������������������&�

$�& "�����������������������������������������������������������������������

����� ���������������������������� ���������������������������

$ & "����������������� ������������������������������� �����������������4 "���

���("8+)<(�*)�.(*.<(�B�"-���%CC��

+���4 "�������������������� ��������� ���������������������������9

$�& �������� ��������������������������������������������� ������������� ��

����������������������� ��������

$�& �������������� �������0��������������������������

$�& ������������������������������ ���������������������������������

������������� ��������������������������������������&������

������������������ ��������

$=& ����� ���������������� �������������������������� ���������������

��������������

�� ����:���"���������������������������5�� ���������������1�����������

5�� ��������������5�������*����������������������������������

���������� �������

1���������������4��������������������������H��������������������

������ ���������� ������������ ���������������� ��������������

$4& "�������������>��������������$#� ���������������������������� ��������������

��������������������#������������&������������������������� �����������

����������������������������������� ������������������ �������%�������

$�������#��������� ������������������������#������������������������

������������ �������%������&�

Page 72: INTER CA Taxation - CA Study Web€¦ · INTER CA Taxation Head Office Shraddha, 4th Floor, ... 14. Advance Tax & Interest 268 - 271 15. Tax Deducted at Source (TDS) 272 - 294 16

���C��

���������������� ��������� ������������

���������������� � ���

D��:� I����������������������; ����5@G���������������������+*.�����=@G���

/*.���������� �������� %4�@@@�����/�*�.������� ���+*.������%�%@��@%!��

%�%���@%!��1�������<�)��%�%��@%C���������������� �������� ����4@@

������������������������������������������� ����� %�%���@%4���"� ���

�����0������ %@�@@�@@@������������������������������������%�%��@%4�������

������������ ��(�������������%@G���������������������������������

�����������������������������������%��@�@@@���������� �����

������� ���������������������������������������@�@@@�

����� ������#���������������������������������������������������

��������� %� ��@%C�

:������� ��#���������.�B��%!�%C9

)���.�B���@%4��@%5 %4�@@@

)���.�B���@%5��@%! ���������@�@@@

)���.�B���@%!���%C�:������� ��#��� @�@@@

�������������������� % �@@@

-� �� ����������������������������I�������"�B���@%C��@%A�

D��"� :���+���H� ������������������� ��������������� ������ �������7���%� ���+�*�.�

������������������������/�*�.������4%3���������� @�@@@����������+�*�.������5

��������������������%3��������*������F�������� ��������� ������������ ��

)������������������� ��� ������������������������!�@@@�����������5�@@@�����

������ ���+������������������������� �"������ @�@@@������:������� ��#�

����������������������� ������������� ��������@@@���������� ���

���$�0�����4�4�AC&���������������������������� �54�@@@�

-����������������������������

D���� :���-���0��J������������� ���� ����������������� ����������������� ����������������

������������.�B���@%!�%C�

Page 73: INTER CA Taxation - CA Study Web€¦ · INTER CA Taxation Head Office Shraddha, 4th Floor, ... 14. Advance Tax & Interest 268 - 271 15. Tax Deducted at Source (TDS) 272 - 294 16

��@;��

���������������� ��������� ������������

���������������� � ���

���������%������&�������

$%& -���0��0����������������� ������ ������� ����������#������ �������'�����

$�& �������� �������������"��������� �������������+����"�������������������

�>������#���������

$ & ������� ��������������������� ���9

"������'�� ����� �%�@@�@@@���$<#��� ��4�@@@&

'����� ��'�� ����� �!C�@@@��$�������� ������@%%�%�&

$=& ���� ��� � ��� �������� ��� ������� ��� ��� $��� ������� ��� ��� ���

��������������&

:�������� �����$��� ����%AA@&� �%�5@�@@@��6�����#������������������� ��� ���

/������'�� ����� ��@@�@@@�$����������%AC!����@%������� ������@% &

A���������+�����������%��%������������&�����&�������$��$���'�������A��";:B :C�

D��-� B�������>����� ���� �� �� ���������������������������� ��� �������

�� �������������9

D��.� 0���64��

:�2� @�@@@ @�4@@ @�@@@ =@�@@@ !@�@@@

)�(� ��4@@ �@@@ ���4@ A�@@@ !4�@@@

+((-" �A�!4@ �A�4@@ %�4@@ =��4@@ 5@�@@@

"��� ��������� �@@@ �=�A C �%�@@@ �!�@@@ C�@@@

.������2������������� % %%3�

4 %@

2��������� �@@@ �45� %4�@@@ A�@@@ =@�@@@

Page 74: INTER CA Taxation - CA Study Web€¦ · INTER CA Taxation Head Office Shraddha, 4th Floor, ... 14. Advance Tax & Interest 268 - 271 15. Tax Deducted at Source (TDS) 272 - 294 16

��@:��

���������������� ��������� ������������

���������������� � ���

D���� �����������"��������'��� ����������������������������������>�� ������

���������������������������������������� �����%AAC�%AAA�����������

��������������������� ���������������������-������

;���������������� ������@%!�%C���������������������������������������

�������� ���������#��������� ��������������������%��@@@�������������������

�������� ��� ���������������������������A�@@�@@@��������������� ��#����

�@G������������ ��� ��������������������������������������#�������

����� ���9

����

$�& (����� =@�@@@

$��& ���������������$����& %4�@@@

$���& ���������� ���� �����0������������������������ �@@�@@@

*������� ��������������������������������������������������

-������������������������K�����������1����.������������������

�������������������@%C�%A�

D��@� :���'������������������������������� ����������������������8��; ���

������ ����������������������%�@@@��>������������ �����������>�� �������

�� ����� � ����� '� ����� �� ����������� ��� %�@=��@%5� ���� ���� �� �

�������������� @�@5��@%!��'������������������� �������@%�@!��@%!�����

����������� ���������������������%4�@@@����������1�������������������

����� ��� %�%���@%!� ���� '� �������� �� ���� ����� ������� �� �����

@%�@%��@%C��� %�@ ��@%C�

)� �������������������������

$�& )������� ��� ���������� ��� �����%�@@�@@@���������

$��& :������� ��� ���������� ��� �����!��@@@���������

$���& :������� ��#������ �����C�@@@

$��& (����������������������������� ����� @�@@@

'� ���� ���� �� �� �������� ���� ��� ����� %�� �0���L� %@�G� ����� ��� @%�@!��@%5�� 8�

������������������ � %�@ ��@%C��-�������������������������������

"�B���@%C��@%A�

D��B� :��"������������������������������������������������������ ������

�� ���������������������@%!��@%C��������� ���9

0������%��� (������ (�������

:������� ��� � �=�@@@ =��@@@

)����(� =�@@@ 5�@@@

+�������(� @�@@@ =4�@@@

:������� ���#������ 5�@@@ C�@@@

(����� %��@@@ =�@@@

���������.����� %�4@@ %�C@@

��������������������� �$����������� %�@ ��@%%& %��@@@ 5@�@@@

8��9����������������������� ����� @�@A��@%5

B�������>����������������������������������������������������� �

������������������������� ���������������������� �������

Page 75: INTER CA Taxation - CA Study Web€¦ · INTER CA Taxation Head Office Shraddha, 4th Floor, ... 14. Advance Tax & Interest 268 - 271 15. Tax Deducted at Source (TDS) 272 - 294 16

��@"��

���������������� ��������� ������������

���������������� � ���

D��C� :��������������������������� �����������������7�+�"��+�����������L�M

��@@@����������+��������������� ��#�����M�%�4@@������������������ ����

�@%!�%C��+��� ���������������������������:��������������� ��������������

�������������������� ������� ������������ �������������%@�@@@��+���������

��������� ������������%%�@@@�����������������������������%@�@@@����������:�������

�#��������������������������������!�4@@��:����������������������������

����0������ ����������������� �������0�����@�5��@%%��+���������� ������

�����4=�@@@���� ��������������������=�@@@������� ���������� �����������0����

�����5@��-������� ���������������������������:���������

Page 76: INTER CA Taxation - CA Study Web€¦ · INTER CA Taxation Head Office Shraddha, 4th Floor, ... 14. Advance Tax & Interest 268 - 271 15. Tax Deducted at Source (TDS) 272 - 294 16

��@���

���������������� ��������� ������������

���������������� � ���

������ �����7������E���F�&��

�&$���������������&�����&�(�����0��$���'�������A��";:B ";:C

0������%��� 0��%� ������� 7���&

"�/�2�3�(�/�2���������������������������������������������������������$�& %�%@�@@@ ��%@�@@@

"��� ���������� �N�7��� �������$�& %�54�@@@ !@�@@@

.�� ��9�%4�@@@�#�%��������N������%4�@@@

"������9�%@�@@@�#�!���������8�/

"�������$�&��2����������$�& %�54�@@@ %�@@�@@@

"������9�!@�@@@��$%@�@@@�#� ������&

,�"�2 %�54�@@@ ��%@�@@@

/���9�:������� ��#�������������� $ @�@@@& $�4�@@@&

8�"�2� %� 4�@@@ %�C4�@@@ 8�/

/���9�;���������3����=

+���=�$�&9� @G����8"2 $=@�4@@& $44�@@@& 8�/

+���=$�&9���������'�������-���� $C@�@@@& 8�/ $ @�@@@&

�"�����9�8���;+�D�8����������$+���4&

�:����9�+������ �������������������%�=�%AAA

:->.;; :>"C>.;; !�;>;;;#

��� ��������������������� ������������� ����%�%=�@@@

$.�� ���"�������:����&

"��9�(������������� �O�����6��������������73���4"

"����� %�@@�@@@

'����� � ��!C�@@@

:���� %�5@�@@@

/����� ��@@�@@@

4� C�@@@

$�&�+����������������L� @G $�%�5%�=@@& ������>@�>�;;

����)4����/7��,/7�(/6���0/0��A �����->C;>�;;

�����������%���������D�.��

%� :���-���0��J����������0����'���������������� �������%@���������

���������� ��1������������������ ��������#�� ����������������*��

+�������

�� ������������;������������������������������������������������

���������1����.����������������������

� (������������� �O������������������������� ����#�� �������������

�������������������� ����������������������������������������

�������

4�����/F� �/46��/��

Page 77: INTER CA Taxation - CA Study Web€¦ · INTER CA Taxation Head Office Shraddha, 4th Floor, ... 14. Advance Tax & Interest 268 - 271 15. Tax Deducted at Source (TDS) 272 - 294 16

��@-��

���������������� ��������� ������������

���������������� � ���

�����.�

�0������%��� : " � - .

:������� ��� �

)����(�

<#����(��$1��������:2�����)(&

+�������(�

���$�:��"���� � �� �2� �

���$�"�"��� �(��������������������

���� ����N�7��� ����(��$������&

���$����(��������������������

,�����"���� ��� �

�������� �&$���������������&�����&���/�0������������A���";:B :C

0������%��� �����! # )�&�%�! #

+ �����������������$�4G&

8�"���� ��� � 8�/ 8�/

/���9�;�������������������=�$�&

�������� �����0����������������� � !�4@@

$������4G���� �%�4� �0�&������������#����

��� � @�@@@������������������������������

������������������%�@=�%AAA @�@@@ @�@@@

/���������� ���������������� $ @�@@@& $ @�@@@&

�&$���������������&�����&�4�/�0��������A��";:B :C

0������%���

/��������������$!4G&

,�����"���� �2� �

$�& :������� ��� ��$!4G���� �A� �0�& 5�!4�@@@

$�& "��� ����P$ �%��@@@�#�5�#%�&���$ �%��@@@�#%#=&Q C�%5�@@@

D� �C5=�@@@��� �=C�@@@

������������������ C�%5�@@@

/���9�:������� ��#��!4G����%�C@�@@@�$�@G���� �A� �0�& %� 4�@@@

8�"���� �2� ��$8"2& 5�C%�@@@

/���9��;�������������������=

$�& @�G����8"2 ��@=� @@

$�& �������� �����0�������������P!4G���� � � �0�Q ���4�@@@ =��A� @@

����������� ������������������������� ��4%�!@@

+�������������������$4@G& %��4�C4@

Page 78: INTER CA Taxation - CA Study Web€¦ · INTER CA Taxation Head Office Shraddha, 4th Floor, ... 14. Advance Tax & Interest 268 - 271 15. Tax Deducted at Source (TDS) 272 - 294 16

��@.��

���������������� ��������� ������������

���������������� � ���

D��@����������&$���������������&�����&�������$��$���'����7���)�������A��";:B :C

0������%��� 5�������%��� ,������%���

!��%������$���#

,���������� ��� ��$+����� ��& 8� A@�@@@

/���9�:������� ��#��$���������� ���& =�@@@

8������ ��� � $"& 8� �C5�@@@

/��9;�������������������=

$�& @G����������� ��� � �4�C@@

$�& ���������������������

-�������������

�%��@@�@@@�#�%@G�D� �%��@�@@@ �5@�@@@ 5@�@@@

.�����������������

�%��@@�@@@�#�%@G�#�A3%��D� �A@�@@@

�A@�@@@�� �������4�>�� ����� ���

�A@�@@@34�D� �%C�@@@�������� A�@@@ A�@@@

��� ���������������������= $'& 5A�@@@ �A=�C@@

�����������������������$"&��$'& $5A�@@@& $C�C@@&

/���������������R�����������������������R����:���' $!!�C@@&

$������������ �������������� ���&

���<#����(������������������������������������������������ ��� �������

���� ���������������A������������������������������������������� ���� �

��� @�5��@%!

<#������ D� �!4�@@@����������������

)�������D�%�@@�@@@�#�A3%��D� �!4�@@@

:������� ��� ��D�!��@@@#�A3%��D� �4=�@@@

"��� ����D �A@�@@@$ %4�@@@����������5������������S� ����;�������@%!&

,�����"���� �2� ��D� �A@�@@@$��������������<#����(����� �!4�@@@����������

������ �A@�@@@&

Page 79: INTER CA Taxation - CA Study Web€¦ · INTER CA Taxation Head Office Shraddha, 4th Floor, ... 14. Advance Tax & Interest 268 - 271 15. Tax Deducted at Source (TDS) 272 - 294 16

: 76 :

J. K. SHAH CLASSES INTER C.A. - DIRECT TAXES

General Points :

1. As the name suggests these deductions will reduce the total income of an assessee which

in turn will result in the lower amount of tax liability.

2. The purpose of these deductions is to encourage industrialisation, foreign exchange

earning and to assist the tax payer in meeting their essential expenditure.

3. The various deductions allowed under this chapter can be broadly divided into two

groups.

(i) Deduction in respect of certain payment

(ii) Deduction in respect of certain income

4. All these deductions are allowed out of Gross Total Income (G.T.I.)

(i) If G.T.I. is negative no deduction u/s. 80 can be claimed.

(ii) Total deduction u/s. 80 cannot exceed G.T.I.

(iii) By claiming these deduction you can't make your income negative.

5. These deductions are allowed from G.T.I. and not from particular head of income.

6. The assessee claiming such deductions has to produce proof relating to such de-

ductions.

7. The assessee should claim these deductions in his return of income.

8. These deductions are not available in respect of short - term capital gains u/s

111A, long term capital gains, casual winnings and against profit of any speci-

fied business referred to in section 35 AD.

9. Deduction from G.T.I. (sec.80) is different from income exempt from tax (sec.10). In

case of the latter (sec.10), the income is not at all included in the computation of the

income i.e. they are tax free income, where as in the case of the former, the income is

first added/taken in the computation of the income and thereafter the deduction u/s. 80

will be allowed.

DEDUCTION UNDER SECTION 80C

Section 80C provides deduction in respect of specified qualifying amounts paid paid or deposited

by the assessee in the previous year.

Salient features of section 80C- The following are the main provisions of section 80C —

���� Under section 80C, deduction would be available from gross total income.

���� Deduction under section 80C is available only to an individual or a Hindu undivided family.

���� Deduction is available on the basis of specified qualifying investments/contributions/

deposits/payments (hereinafter referred to as "gross qualifying amount") made by the

taxpayer during the previous year.

���� The maximum amount deductible under sections 80C, 80CCC and 80CCD(1) cannot

exceed Rs. 1,50,000.

DEDUCTIONS FROM GROSS TOTALINCOME - CHAPTER - VIA (PART I)

Page 80: INTER CA Taxation - CA Study Web€¦ · INTER CA Taxation Head Office Shraddha, 4th Floor, ... 14. Advance Tax & Interest 268 - 271 15. Tax Deducted at Source (TDS) 272 - 294 16

: 77 :

J. K. SHAH CLASSES INTER C.A. - DIRECT TAXES

Computation of deduction under section 80C- The deduction is calculated as per the

following steps—

Step 1 - Gross qualifying amount

Step 2 - Net qualifying amount

Step 3 - Amount of deduction

GROSS QUALIFYING AMOUNT - Gross qualifying amount is the aggregate of the following :

Nature of payment

1. Life insurance premium (including payment made by Government employees to the

Central Government Employees' insurance scheme and payment made by a person

under children's deferred endowment assurance policy) [sec Note 1]

2. Payment in respect of non-commutable deferred annuity [see Note 2]

3. Any sum deducted from salary payable to a Government employee for the purpose of

securing him a deferred annuity (subject to a maximum of 20% of salary) [see Note 3]

4. Contribution (not being repayment of loan) towards statutory provident fund and

recognized provident fund

5. Contribution (not being repayment of loan) towards 15-year public provident fund [sec

Notes 4,6 and 9]

6. Contribution towards an approved superannuation fund

7. Subscription to National Savings Certificates, VIII Issue or IX Issue [sec Note 7] and

deposit in Sukanya Samriddhi Account [see Note 10]

8. Contribution for participating in the unit-linked insurance plan (ULIP) of Unit Trust of

India [see Note 5]

9. Contribution for participating in the unit-linked insurance plan (ULIP) of LIC Mutual Lund

(i.e., formally known as Dhanraksha plan of LIC Mutual Fund) [see Note 5]

10. Payment for notified annuity plan of LIC (i.e., Jeevan Dhara, Jeevan Akshav) or any

other insurer

11. Subscription towards notified units of Mutual Fund or UTI

12. Contribution to notified pension fund set up by Mutual Fund or UTI

13. Any sum paid (including accrued interest) as subscription to notified Home Loan Account

Scheme of the National Housing Bank or contribution to any notified pension fund set

up by the National Housing Bank

14. Any sum paid as subscription to any scheme of—

a. public sector company engaged in providing long-term finance tor purchase/

construction of residential houses in India (i.e., public deposit scheme of HUDCO

b. housing board constituted in India for the purpose of planning,

development or improvement of cities/towns

Page 81: INTER CA Taxation - CA Study Web€¦ · INTER CA Taxation Head Office Shraddha, 4th Floor, ... 14. Advance Tax & Interest 268 - 271 15. Tax Deducted at Source (TDS) 272 - 294 16

: 78 :

J. K. SHAH CLASSES INTER C.A. - DIRECT TAXES

15. Any sum paid as tuition fees (not including any payment towards development fees/donation/

payment of similar nature) whether at the time of admission or otherwise to any university/

college/educational institution in India for full time education of any two children of an

individual [Note 8]

16. Any instalment or part payment towards the cost of purchase/construction of a residential

property to a housing board or co-operative society (or repayment of housing loan taken

from Government, bank, cooperative bank, LIC, National Housing Bank, assessor's

employer where such employer is public company/public sector company/ university/

co-operative society)

17. Amount invested in approved debentures of, and equity shares in, a public company engaged

in infrastructure including power sector or units of a mutual fund proceeds of which are

utilised for the developing, maintaining, etc., of a new infrastructure facility

18. Amount deposited as term deposit for a period of 5 years or more in accordance with a

scheme framed by the Government

19. Subscription to any notified bonds of National Bank for Agriculture and Rural Development

(NABARD)

20. Amount deposited under Senior Citizens Saving Scheme [Note 9]

21. Amount deposited in five year time deposit scheme in post office.

Notes:

1. In the case of an individual, policy should be taken on his own life, life of the spouse or any

child (child may be dependent/ independent, male/female, minor/major or married/

unmarried). In the case of a Hindu undivided family, policy may be taken on the life of any

member of the family.

Insurance premium cannot exceed the maximum ceiling given below -

Policy on the life of a person Policy on the life of

with disability or severe disability or any other person

on the life of a person suffering

from disease or ailment as given

in section 80DDB

- If policy is issued before April 1, 2012 20% of sum assured 20% of sum assured*

- If policy is issued during 2012-13 10% of sum assured** 10% of sum assured**

- If policy is issued on or after April 1,2013 15% of sum assured** 10% of sum assured**

*Sum assured does not include any premium agreed to be returned and/or any benefit

by way of bonus.

'"Sum assured means minimum amount assured under the policy without including any

premium agreed to be returned and/or any benefit by way of bonus.

2. Annuity plan should be taken in the name of the individual, his wife/her husband or any

child of such individual.

3. It should be for the benefit of the individual, his wife or children.

Page 82: INTER CA Taxation - CA Study Web€¦ · INTER CA Taxation Head Office Shraddha, 4th Floor, ... 14. Advance Tax & Interest 268 - 271 15. Tax Deducted at Source (TDS) 272 - 294 16

: 79 :

J. K. SHAH CLASSES INTER C.A. - DIRECT TAXES

4. According to the Public Provident Fund Scheme, an individual can open public provident

fund account in his own name or in the name of minor of whom he is guardian. However,

according to the Income-tax Act, to get the benefit of the deduction under section 80C,

amount deposited by an individual in his own account or in the account of his/her spouse

or in the account of any child (in the case of HUF in the account of any member of the

family) is eligible for deduction.

5. In the case of an individual, ULIP should be taken on his own life, life of the spouse or

any child (child may be dependent/ independent, male/female, minor/major or married/

unmarried). In the case of a Hindu undivided family, ULIP may be taken on the life of

any member of the family.

6. There is no maximum ceiling under the Income-tax Act. However, under the public

provident fund scheme, the maximum contribution is Rs. 1,50,000.

7. Accrued interest (which is deemed as reinvested) is also qualified for deduction

(applicable for all years except last year).

8. Full-time education includes any educational course offered by any university, college,

school or other educational institution to a student who is enrolled full-time for the said

course. Full-time education includes even play-school activities, pre-nursery and nursery

classes. The amount allowable as tuition fees shall include any payment of fee to any

university, college, school or other educational institution in India except the amount

representing payment in the nature of development fees or donation or capitation fees

or payment of similar nature Circular No. 8/2012, dated October 5,2012.

9. Date of encashment of cheque/draft is taken as date of deposit in the case of public

provident fund and Senior Citizens Savings Scheme.

10. In the case of an individual, deposit in Sukanya Samriddhi Account can be made in the

name of individual, or any girl child of that individual or any girl child for whom such

person is the legal guardian, if the scheme so specifies.

NET QUALIFYING AMOUNT - Gross qualifying amount is the total of all investments specified

above. However, deduction under section 80C is available on the basis of net qualifying amount

which is determined as under

- Gross qualifying amount; or

- `̀̀̀1,50,000, whichever is lower.

AMOUNT OF DEDUCTION - Net qualifying amount is the deductible.

The maximum amount deductible under section 80C is `̀̀̀ 1,50,000. Moreover, the aggregate

amount of deduction under sections 80C, 80CCC and 80CCD(1) [i.e., contribution, by employee

(or any other individual) towards National Pension Scheme (NFS)] cannot exceed `̀̀̀ 1,50,000

DEDUCTION IN RESPECT OF PENSION FUND (SECTION 80CCC)

Who can claim deduction under section 80CCC – Deduction under section 80CCC is

available only to an individual.

What is the qualifying payment to avail deduction – Amount should be paid or deposited

any amount out of his income chargeable to tax to effect or keep in force a contract for any

annuity plan of LIC of India or any other insurer for receiving pension from the fund referred to

in section 10(23AAB), he shall be allowed a deduction in the computation of his total income.

Page 83: INTER CA Taxation - CA Study Web€¦ · INTER CA Taxation Head Office Shraddha, 4th Floor, ... 14. Advance Tax & Interest 268 - 271 15. Tax Deducted at Source (TDS) 272 - 294 16

: 80 :

J. K. SHAH CLASSES INTER C.A. - DIRECT TAXES

How much deduction available under section 80CCC - The maximum permissible

deduction is `̀̀̀ 1,50,000 (Further, the overall limit of `̀̀̀ 1,50,000 prescribed in section 80CCE

will continue to be applicable i.e. the maximum permissible deduction under sections 80C,

80CCC and 80CCD(1) put together is `̀̀̀1,50,000).

What is tax treatment of pension - Where any amount standing to the credit of the assessee

in a fund referred to in clause (23AAB) of section 10 in respect of which a deduction has been

allowed, together with interest or bonus accrued or credited to the assessee’s account is

received by the assessee or his nominee on account of the surrender of the annuity plan in

any previous year or as pension received from the annuity plan, such amount will be deemed

to be the income of the assessee or the nominee in that previous year in which such withdrawal

is made or pension is received. It will be chargeable to tax as income of that previous year.

Note: Where any amount paid or deposited by the assessee has been taken into account for

the purposes of this section, a deduction under section 80C shall not be allowed with reference

to such amount.

DEDUCTION IN RESPECT OF CONTRIBUTION TO A NATIONAL PENSION

SCHEME (SECTION 80CCD)

What is NPS and Who can join NPS: As per the “New Restructured Defined Contribution

Pension System” applicable to new entrants to Government service, it is mandatory for persons

entering the service of the Central Government on or after 1st January, 2004, to contribute ten

per cent their of salary every month towards their pension account. A matching contribution is

required to be made by the Government to the said account. The benefit of this scheme is

also available to individuals employed by any other employer as well as to self-employed

individuals. Section 80CCD provides deduction in respect of contribution made to the new

pension scheme of the Central Government.

Employer’s contribution to NPS – Is it Income – Employers contribution to NPS is taxable

as salary income in the year of contribution.

Deduction available under section 80CCD(2) in respect of employer’s contribution to

NPS - Under section 80CCD(2), contribution made by the Central Government or any other

employer in the previous year to the said account of an employee, is allowed as a deduction

in computation of the total income of the assessee. However, deduction under section

80CCD(2) would be restricted to 10% of salary.

Deduction available under section 80CCD(1) in respect of employee’s contribution to

NPS - Section 80CCD(1) provides a deduction for the amount paid or deposited by an

employee in his pension account subject to a maximum of 10% of his salary. The deduction in

the case of a self-employed individual would be restricted to 20% of his gross total income in

the previous year.

Page 84: INTER CA Taxation - CA Study Web€¦ · INTER CA Taxation Head Office Shraddha, 4th Floor, ... 14. Advance Tax & Interest 268 - 271 15. Tax Deducted at Source (TDS) 272 - 294 16

: 81 :

J. K. SHAH CLASSES INTER C.A. - DIRECT TAXES

Is there any combined maximum ceiling – The aggregate amount of deduction under

section 80C, 80CCC and 80CCD(1) cannot exceed `̀̀̀1,50,000. However employer ’s

contribution towards NPS shall not be considered for the ceiling of ̀̀̀̀ 1,50,00

Additional deduction of `̀̀̀ 50,000 under section 80CCD(1B) - Section 80CCD(1B) provides

for an additional deduction of up to ̀̀̀̀ 50,000 in respect of the whole of the amount paid or deposited

by an individual assessee under NPS in the previous year, whether or not any deduction is allowed

under section 80CCD(1).

What is tax treatment of pension – The amounts standing to the credit of an assessee in NPS

for which a deduction has already been claimed by the assessee and accretions to such account

shall be taxed as follows –

1. Amount received by the assessee on closure of account or on his opting out of the NPS

Scheme – taxable.

2. If amount is received by a nominee on the death of the assessee – Exempt.

What is Salary – Salary includes basic salary and dearness allowance entering into retirement

benefit.

Limit on deductions under sections 80C, 80CCC & 80CCD(1) [Section 80CCE]

This section restricts the aggregate amount of deduction under section 80C, 80CCC and 80CCD(1)

to ‘ 1,50,000. It may be noted that the deduction of upto ̀̀̀̀ 50,000 under section 80CCD(1B) and

employer’s contribution to pension scheme, allowable as deduction under section 80CCD(2) in

the hands of the employee, would be outside the overall limit of ̀̀̀̀ 1,50,000 stipulated under section

80CCE.

The following table summarizes the ceiling limit under these sections w.e.f. A.Y.2016-17 –

Section Particulars Ceiling limit

( `̀̀̀)

80C Investment in specified instruments 1,50,000

80CCC Contribution to certain pension funds 1,50,000

80CCD(1) Contribution to NPs of Government 10% salary or

20% of GTI,

as the case may be

80CCE Aggregate deduction under sections 80C, 1,50,000

80CCC & 80CCD(1)

80CCD(1B) Contribution to NPS notified by the Central 50,000

Government (Outside the limit of `̀̀̀ 1,50,000

under section 80CCE)

Page 85: INTER CA Taxation - CA Study Web€¦ · INTER CA Taxation Head Office Shraddha, 4th Floor, ... 14. Advance Tax & Interest 268 - 271 15. Tax Deducted at Source (TDS) 272 - 294 16

: 82 :

J. K. SHAH CLASSES INTER C.A. - DIRECT TAXES

DEDUCTION IN RESPECT OF MEDICAL INSURANCE PREMIUM (SECTION 80D)

Deduction under section 80D is available if the following conditions are satisfied –

1. The taxpayer is an Individual or a HUF

2. Payment should be made out of income chargeable to tax.

3. Payment should be made by any mode other than cash. However, payment on account of

preventive health check-up be made by any mode (including cash)

Maximum deductible amount – The maximum deductible amount and other relevant points are

given below –

Deduction in the case Deduction in the case of

of individual HUF

For whose benefit payment can be made Family Parents Any member of HUF

A a. Medi-claim Insurance premium Eligible Eligible Elig ible

b. Contribution to CGHS Eligible - -

c. Preventive health check up payment Eligible Eligible -

Maximum deduction

- General deduction (applicable in respect

of a,b and c) 25,000 25,000 25,000

- Additional deduction (applicable

only in case of (a) when mediclaim policy is

taken on the life of a senior citizen) 5,000 5,000 5,000

B Medical expenditure on the health of a person

who is a super senior citizen if mediclaim

insurance is not paid on the health of

such person Eligible Eligible Eligible

Maximum deduction in respect of (B) 30,000 30,000 30,000

C Maximum deduction in respect of (A) and (B) 30,000 30,000 30,000

Notes-

1. Family includes individual, spouse of the individual and dependent children of the individual

2. Parents include father and mother (dependent or not).

3. The aggregate payment on account of preventive health check-up of self, spouse, dependent

children, father and mother cannot exceed ` ` ` ` 5,000.

DEDUCTION IN RESPECT OF MAINTENANCE INCLUDING MEDICAL TREATMENT

OF A DEPENDENT BEING A PERSON WITH DISABILITY – SECTION 80DD

Who can claim deduction - A resident being an individual or resident Hindu undivided family.

What is the qualifying expenditure - Any amount incurred for the medical treatment (including

nursing), training and rehabilitation of a dependant, being a person with disability, or any amount

paid or deposited under a scheme framed in this behalf by the Life Insurance Corporation or any

other insurer or the Administrator or the Specified Company as referred to in section 2(h) of the

Unit Trust of India (Transfer of Undertaking and Repeal) Act, 2002, for the maintenance of a

dependant, being a person with disability, qualifies for deduction.

Page 86: INTER CA Taxation - CA Study Web€¦ · INTER CA Taxation Head Office Shraddha, 4th Floor, ... 14. Advance Tax & Interest 268 - 271 15. Tax Deducted at Source (TDS) 272 - 294 16

: 83 :

J. K. SHAH CLASSES INTER C.A. - DIRECT TAXES

Who is dependent relative suffering from disability - The term ‘dependent’ has been defined

to include in the case of an individual, the spouse, children, parents, brothers and sisters of the

individual and in the case of a Hindu Undivided Family (HUF), a member thereof, who is wholly or

mainly dependent on the assessee and has not claimed any deduction under section 80U in the

computation of his income.

How much deduction is allowed under section 80D - The quantum of deduction is 75,000

and in case of severe disability (i.e. person with 80% or more disability) the deduction shall be

1,25,000.

Certificate if any required - For claiming the deduction, the assessee shall have to furnish a

copy of the certificate issued by the medical authority under the Persons with Disability (Equal

Opportunities, Protection of Rights and Full Participation) Act, 1995 along with the return of income

under section 139. Where the condition of disability requires reassessment, a fresh certificate

from the medical authority shall have to be obtained after the expiry of the period mentioned in the

original certificate in order to continue to claim the deduction.

DEDUCTION IN RESPECT OF MEDICAL TREATMENT, ETC. [SECTION 80DDB]

Who can claim deduction - A resident being an individual or resident Hindu undivided family.

What is the qualifying expenditure - Any amount actually paid for the medical treatment of such

disease or ailment as may be specified in the rules made in this behalf by the Board for himself or

a dependent, in case the assessee is an individual or for any member of a HUF, in case the

assessee is a HUF will qualify for deduction.

Person suffering from disease : The above deduction is available to an individual for medical

expenditure incurred on himself or a dependant. It is also available to a Hindu undivided family

(HUF) for such expenditure incurred on any of its members. The term ‘dependent’ has been

defined to include in the case of an individual, the spouse, children, parents, brothers and

sisters of the individual and in the case of a Hindu Undivided Family (HUF), a member thereof,

who is wholly or mainly dependent on the assessee and has not claimed any deduction under

section 80U in the computation of his income.

How much is deductible under section 80DDB - The amount of deduction under this section

shall be equal to the amount actually paid or `̀̀̀ 40,000, whichever is less, in respect of that

previous year in which such amount was actually paid. In case the amount is paid in respect of

a senior citizen, the deduction would be the amount actually paid or `̀̀̀ 60,000, whichever is

less. Further, a higher limit of deduction of upto `̀̀̀ 80,000 is allowable to the assessee, for the

expenditure incurred in respect of the medical treatment of himself or a dependent, being a

very senior citizen.

Certificate if any required – The assessee is required to obtain a prescription from a

specialist doctor for the purpose of availing this deduction.

Page 87: INTER CA Taxation - CA Study Web€¦ · INTER CA Taxation Head Office Shraddha, 4th Floor, ... 14. Advance Tax & Interest 268 - 271 15. Tax Deducted at Source (TDS) 272 - 294 16

: 84 :

J. K. SHAH CLASSES INTER C.A. - DIRECT TAXES

DEDUCTION IN RESPECT OF PAYMENT OF INTEREST ON LOAN TAKEN

FOR HIGHER EDUCATION [SECTON – 80E]

Who can claim deduction – Only an individual can claim deduction under section 80E.

What is the qualifying expenditure – If loan is taken by an individual for any study in India or

outside India (i.e any study after passing senior secondary examination or its equivalent) from

a bank, financial institution or an approved charitable institution, interest is deductible in the

year in which interest is paid.

For whose education loan should be taken - The loan must have been taken for the purpose

of pursuing his higher education or for the purpose of higher education of his or her relative

i.e. spouse or children of the individual other student for whom the individual is the legal

guardian.

Maximum monetary ceiling – Entire interest is deductible in the year in which the assessee

starts paying interest on loan and subsequent 7 years or until interest is paid in full. However,

interest should be paid out of income chargeable to tax.

Deduction for interest on loan borrowed for acquisition of self- occupied

house property by an individual [Section 80EE]

(i) As a step towards achieving the Government’s aim of providing ‘housing for all’, first

home buyers availing home loans are encouraged, by providing additional deduction

under section 80EE from A.Y.2017-18 in respect of interest on loan taken by an individual

for acquisition of residential house property from any financial institution. The maximum

deduction allowable is ` ` ` ` 50,000.

(ii) The conditions to be satisfied for availing this deduction are as follows –

(iii) The benefit of deduction under this section would be available till the repayment of loan

continues.

(iv) The deduction of upto ` ` ` ` 50,000 under section 80EE is over and above the deduction

of upto `̀̀̀ 2,00,000 available under section 24 for interest paid in respect of loan

borrowed for acquisition of a self-occupied property.

The assessee

should not own any

residential house

on the date of sanc-

tion of loan

Loan should be

sanctioned during

P.Y. 2016-17Conditions

Value of house <

` ` ` ` 50 lakhs

Loan sacntioned

< `̀̀̀ 35 lakhs

Page 88: INTER CA Taxation - CA Study Web€¦ · INTER CA Taxation Head Office Shraddha, 4th Floor, ... 14. Advance Tax & Interest 268 - 271 15. Tax Deducted at Source (TDS) 272 - 294 16

: 85 :

J. K. SHAH CLASSES INTER C.A. - DIRECT TAXES

DEDUCTION IN RESPECT OF INVESTMENT MADE UNDER EQUITY SAVING

SCHEME (SECTION – 80CCG)

The provision of section 80CCG are given below –

Conditions – Deduction under this section is available if the following conditions are satisfied –

1. The assessee is a resident individual.

2. His GTI does not exceed Rs 12 Lakh.

3. He has acquired listed shares or listed units in accordance with a notified scheme.

4. The assessee is a new retail investor as specified in the above notified scheme.

5. The investment is locked-in for a period of 3 years from the date of acquisition in

accordance with the scheme.

Quantum of deduction – 50% of amount invested or 25,000 whichever is less.

Period of deduction – Deduction is permissible for three consecutive assessment years,

beginning with the assessment year relevant to the previous year in which the listed equity

shares or listed units of equity oriented funds are first acquired.

Note : No deduction for new investments allowed from AY 2018-2019. But deduction upto

FY 2018-2019 can be claimed for investment made in FY 2016-2017 since deduction was

allowed for 3 years.

DEDUTION IN RESPECG OF INTERETS ON SEPOSITS IN SAVING ACCOUNTS -

WHEN AVAILABLE [SEC.80TTA]

Section 80TTA provides a deduction up to ` ` ` ` 10,000 in aggregate to an assessee (being an

individual or a Hindu undivided family( in respect of any income by way of interest on depends

(not being time deposits) in a savings account with-

(a) A banking company ;

(b) a co-operative society engaged in carrying on the business of banking (including a

co-operative land mortgage bank or a co-operative land development bank) ; or

(c) post office

However, where the aforesaid income is derived from any deposit in a savings account held

by, or on behalf of a firm, an association of persons or a body of individuals, no deduction be

allowed in respect of duch income in computing the total income of any partner of the firm or

any member of the association or body.

���� Post office savings bank interest exemption under section 10(15)(i) - Post office savings

bank interets is exempt up to ` ` ` ` 3,500 (in an individual account) and ` ` ` ` 7,000 (in a joint

account) under section 10(15)(i)

Page 89: INTER CA Taxation - CA Study Web€¦ · INTER CA Taxation Head Office Shraddha, 4th Floor, ... 14. Advance Tax & Interest 268 - 271 15. Tax Deducted at Source (TDS) 272 - 294 16

: 86 :

J. K. SHAH CLASSES INTER C.A. - DIRECT TAXES

DECUTION IN THE CASE OF A PERSON WITH DISABILITY - TO WHAT EXTENT

AVAILABLE [SEC.80U]

The provisions ofn section 80U are given in brief -

���� Conditions -The following conditiond should be satisfied -

1. The taxpayer is an individual and resident in India

2. He suffers 40 percent or more than 40 percent of any disability (i.e. blindness,

low vision, leprosy - cured hearing impairment, locomotor disability, mental

retardation, mental illness).

3. The taxpayer shall have a certificate issued by the medical authority. Where

the condition of disability requires reassessment, a fresh certificate from

the medical authority shall have to be obtained after the expiry of the period

mentioned on the original certificate in order to continue to calim the

deduction.

���� Amount of deduction - Fixed deduction of ` ` ` ` 75,000 is available. A higher

deduction of `̀̀̀ 1,25,000 is allowed i respect of a person with severe disability

(i.e., having of 80 percent or above).

Page 90: INTER CA Taxation - CA Study Web€¦ · INTER CA Taxation Head Office Shraddha, 4th Floor, ... 14. Advance Tax & Interest 268 - 271 15. Tax Deducted at Source (TDS) 272 - 294 16

: 87 :

J. K. SHAH CLASSES INTER C.A. - DIRECT TAXES

CLASS WORK PROBLEMS

Q. 1. Mr. X is employed by a multinational company in India on a basic salary of `̀̀̀ 45,000 p.m.

Dearness allowance of `̀̀̀ 25,000 p.m. of which 40% is entering into retirement benefits.

The employer and employee contribute `̀̀̀ 3,500 p.m. (each) to Recognised Provident

Fund. Interest credited to this RPF during the year is `̀̀̀ 65,000 @ 10.5% p.a. The

employer and employee also contribute `̀̀̀ 8,000 p.m.and `̀̀̀ 6,000 p.m.respectively to

the notified pension scheme.

He has income from house property of `̀̀̀ 1,20,000/-. However, he has b/f loss of house

property of `̀̀̀ 70,000 of A.Y. 2012-13. His other incomes taxable under other sources is

`̀̀̀ 6,00,000.

During the year 2017-18, he made the following payments :

(1) Contribution to P.P.F. `̀̀̀ 15,000.

(2) Life insurance premium on policy purchased for self in the year 2007 `̀̀̀ 16,000

(sum assured `̀̀̀ 1,80,000)

(3) Life Insurance Premium on dependent mother `̀̀̀ 6,000 (sum assured ̀̀̀̀ 50,000)

(4) On 21.7.2014 he purchased a life Insurance policy in the name of his wife for which

he paid a premium of `̀̀̀ 14,500. [Sum Assured `̀̀̀ 1,20,000]

(5) Fees paid to school for education of his three children `̀̀̀ 6,000 each.

(6) Fees paid to a coaching class for tution of his three children `̀̀̀ 3,000 each.

You are required to calculate his total income for A.Y. 2018-19.

Q. 2. X is a resident individual who contributes annually a sum of `̀̀̀ 15,000 with LIC for

the maintenance of his handicapped dependant grand father.

What is the deduction u/s 80DD? Will it make any difference if the dependant is a

disabled brother (40% disabled)?

Q. 3. Calculate the deduction u/s 80DDB for the following persons :

Page 91: INTER CA Taxation - CA Study Web€¦ · INTER CA Taxation Head Office Shraddha, 4th Floor, ... 14. Advance Tax & Interest 268 - 271 15. Tax Deducted at Source (TDS) 272 - 294 16

: 88 :

J. K. SHAH CLASSES INTER C.A. - DIRECT TAXES

Q. 4. Mrs. Deepali (aged 40 years), working with M/s Good Company Ltd., a manufacturer

of tyres based at Mumbai, has received the following payments during the financial

year 2017-18 from her employer:

Basic salary `̀̀̀ 60,000 per month.

Dearness allowance 40% of basic salary.

Her employer has taken on rent her own house on a monthly rent of `̀̀̀15,000 and the

same has been provided for residence of Mrs. Deepali. Company is recovering

`̀̀̀ 2,000 per month as rent of house.

Mrs. Deepali has further furnished the following details:

(i) S he has paid professional tax of `̀̀̀ 6,000 during financial year 2017-18.

(ii) She is owning only one house and payment of interest of `̀̀̀1,75,000 and

principal of `̀̀̀ 1,00,000 was made for housing loan taken for purchase of house.

(iii) She has also taken a loan of `̀̀̀ 2,00,000 from her employer for study of her son.

SBI rate for such loan is 10%. Her employer has recovered `̀̀̀10,000 as interest

from her salary for such loan during the year.

Compute taxable income and tax liability for assessment year 2018-19.

Q. 5. Mr. Raghu, Marketing Manager of KL Ltd., based at Mumbai furnishes you the following

information for the year ended 31.03.2018:

Basic salary - `̀̀̀ 1,00,000 per month

Dearness allowance (Forming part of salary for retirement benefits) - `̀̀̀ 50,000 per

month

Bonus - 2 months basic salary

Contribution of employer to Recognized Provident Fund - 15% of basic salary plus

dearness allowance Rent free unfurnished accommodation was provided by the

company at Mumbai (accommodation owned by the company). Other details are as

follows

(i) Recognised Provident Fund contribution made by Raghu 1,50,000

(ii) Health insurance premium for insurance of his wife’s health 30,000

(iii) Health insurance premium in respect of parents (senior citizens) 33,000

(iv) Medical expenses of dependent brother with ‘severe disability’ (covered by

Section 2(o) of National Trust for Welfare of Persons with Austism, Cerbral Palsy,

Mental Retardation and Multiple Disabilities Act, 1999) - 6,000

(v) Interest on loan taken for education of his son studying B.Com (full-time) in a

recognized college - 24,000

(vi) Interest on loan taken for education of a student for whom Mr. Raghu is the legal

guardian for pursuing B.Sc.(Physics) ( fu ll - t ime) in a recognized

university - 20,000

Compute the total income of Mr. Raghu for the assessment year 2018-19.

Page 92: INTER CA Taxation - CA Study Web€¦ · INTER CA Taxation Head Office Shraddha, 4th Floor, ... 14. Advance Tax & Interest 268 - 271 15. Tax Deducted at Source (TDS) 272 - 294 16

: 89 :

J. K. SHAH CLASSES INTER C.A. - DIRECT TAXES

Q. 6. Mr. A purchased a residential house property for self-occupation at a cost of `̀̀̀ 45 lakh

on 1.6.2016, in respect of which he took a housing loan of `̀̀̀ 35 lakh from Bank of

India@11% p.a. on the same date. Compute the eligible deduction in respect of inter-

est on housing loan for A.Y.2018-19 under the provisions of the Income-tax Act, 1961,

assuming that the entire loan was outstanding as on 31.3.2018 and he does not own

any other house property.

Q. 7. The gross total income of Mr. Nepal for the Assessment Year 2018-19, was `̀̀̀ 12,00,000.

He has made the following investment/payments during the previous year 2017-18.

Particulars `̀̀̀

1 L.I.C. premium paid (Policy value `̀̀̀ 1,00,000) (taken on 1.03.2012) 25,000

2 Contribution to Public Provident Fund (PPF) 70,000

3 Repayment of housing loan to Indian Bank 50,000

4 Payment made to L.I.C. pension fund 20,000

5 Medical insurance premium for self, wife and dependent children. 28,000

6 Mediclaim premium for parents (aged over 80 years) 32,000

Compute eligible deduction under Chapter VI-A for the Assessment Year 2018-19.

Page 93: INTER CA Taxation - CA Study Web€¦ · INTER CA Taxation Head Office Shraddha, 4th Floor, ... 14. Advance Tax & Interest 268 - 271 15. Tax Deducted at Source (TDS) 272 - 294 16

: 90 :

J. K. SHAH CLASSES INTER C.A. - DIRECT TAXES

Ans. 1. Mr. X

Computation of total income for A.Y. 2018-2019

Particulars `̀̀̀

Income from Salary (W.N. 1) 9,42,190

Income from House Property (W.N. 2) 50,000

Income from Other Sources 6,00,000

GROSS TOTAL INCOME (G.T.I.) 15,92,190

Less deductions u/s 80C to 80U (2,35,000)

NET TAXABLE TOTAL INCOME (N.T.T.I.) 13,57,190

W.N. 1 Income from Salary

Basic Salary (45,000 p.m. x 12 months) 5,40,000

D. A. (in terms) (25,000 p.m. x 12 months x 40%) 1,20,000

D. A. (not in terms) (25,000 p.m. x 12 months x 60%) 1,80,000

Employer’s Contribution to R.P.F. (3,500 p.m. x 12 months) 42,000

Less: Exempt upto 12% of SALARY [12%X(5,40,000+ (42,000) NIL

1,20,000)= 79,200 maximum]

Interest credited to R.P.F. @ 10.5% p.a. 65,000

Less: Exempt upto 9.5% p.a. [65,000 / 10.5% * 9.5% ] (58,810) 6,190

Employer’s contribution to Pension fund

(8,000 p.m. x 12 months) 96,000

GROSS SALARY 9,42,190

Less: Deductions u/s. 16 NIL

NET TAXABLE SALARY 9,42,190

W.N. 2 Income from House Property

Income (as given) 1,20,000

Less: Set off of b/f House Property loss of A.Y. 2012-2013 u/s. 71B (70,000)

50,000

CLASSWORK SOLUTION

Page 94: INTER CA Taxation - CA Study Web€¦ · INTER CA Taxation Head Office Shraddha, 4th Floor, ... 14. Advance Tax & Interest 268 - 271 15. Tax Deducted at Source (TDS) 272 - 294 16

: 91 :

J. K. SHAH CLASSES INTER C.A. - DIRECT TAXES

W.N. 3 Deductions u/s. 80C to 80U

u/s. 80C: Employee contribution to RPF 42,000

Contribution to PPF 15,000

Life Insurance premium:

(a) Self 16,000

(b) Mother, NOT ALLOWED NIL

(c) Wife (Max 10% of Sum Assured) 12,000

School Fees (6,000 x 2 children max) 12,000

TOTAL 97,000

Maximum allowable u/s. 80C 1,50,000

Deduction allowed 97,000

u/s. 80CCD(1):

Employee’s contribution to pension fund, in excess 22,000

of limit of ` ` ` ` 50,000 as per Sec 80CCD(1B)

[72,000 - 50,000]

10% of SALARY of Rs. 6,60,000 66,000

Whichever is less 22,000

Total deductions u/s. 80C, 80CCC and 80CCD(1) 1,19,000

Deduction allowable [in accordance with Section 80CCE] (i) 1,19,000

Employee’s Contribution to Pension Fund (6,000pm x12) 72,000

Maximum allowable u/s. 80CCD(1B) 50,000

Deduction allowed u/s. 80CCD (1B) (ii) 50,000

u/s. 80CCD(2):

Employer’s contribution to pension fund

(8,000 p.m. x 12 months)

10% of SALARY of Rs. 6,60,000 96,000

Whichever is less 66,000

(iii) 66,000

TOTAL DEDUCTIONS u/s. 80C to 80U (i+ii+iii) 2,35,000

Page 95: INTER CA Taxation - CA Study Web€¦ · INTER CA Taxation Head Office Shraddha, 4th Floor, ... 14. Advance Tax & Interest 268 - 271 15. Tax Deducted at Source (TDS) 272 - 294 16

J.K.SHAH CLASSES INTER C.A. – DIRECT TAXES

: 92 :

Meaning of Business and Profession

Sec.2(13) :- Business

• Any trade, commerce or manufacture; or

• Any *adventure or concern in the nature of trade, commerce or manufacture.

Normally business means recurring economic activity, but for income tax

purposes an isolated activity may be termed as business depending upon facts

and circumstance. Following element shall be considered to judge a transaction

as business transaction.

� Nature of commodity

� Nature of transaction (Whether incidental to a business or not)

� Intention of the related party

� The periodicity or duration of transaction

� Effort applied in transaction

*adventure :- doing activity for the first time without knowing the outcome.

Sec. 2(36) :- Profession

Profession includes vocation.

Profession requires purely intellectual skill or manual skill on the basis of some special

learning and qualification gathered through past training or experience e.g. Chartered

Accountants, Doctors, Lawyer etc. Profession skill can be required only after patient

study and application.

Vocation

Vocation implies natural ability of a person to do some particular work e.g. singing,

dancing etc. Vocation must have the earning feature. Unlike profession, vocation does

not required a degree or special learning.

Notes

2. profit motive – if the motive of an activity is pleasure only, it shall not be treated as

business activity.

3. business Vs Profession - an income arising out of trade, commerce, manufacture,

profession or vocation shall have the same treatment in income tax act. However, a little

segregation is required to be made between business and profession while applying

Sec. 44AA, Sec.44AB, Sec.40(b) etc.

Sec. 28 :- Basis of charge

The following income shall be chargeable to income-tax under the head “PGBP”,-

1. Profit and Gains of any business or Profession [Sec. 28 (i)]:-the profits and

gains of any business or profession which was carried on by the assessee at any

time during the previous year.

CHAPTER NO. 6 PROFITS AND GAINS OF BUSINESS AND PROFESSION

Page 96: INTER CA Taxation - CA Study Web€¦ · INTER CA Taxation Head Office Shraddha, 4th Floor, ... 14. Advance Tax & Interest 268 - 271 15. Tax Deducted at Source (TDS) 272 - 294 16

J.K.SHAH CLASSES INTER C.A. – DIRECT TAXES

: 93 :

2. Compensation or other payment to management agency [Sec. 28(ii) :- any

compensation or other payment due to or received by an agent, managing the

whole or substantially the whole of the affairs of any person, at the termination of

his management or the modification of the terms and conditions relating thereto;

3. Income of trade or professional association’s [Sec.28(iii)]: income derived by a

trade, professional or similar association from specific services performed for its

members;

4. Export Incentive Sec.28 (iiia) to (iiie) :-

i) Profit on sale of import entitlement license; (iiia)

ii) Cash assistance received by any person against exports under any scheme

of the GOI; (iiib)

iii) Any duty of customs or excise re-paid or re-payable as duty drawback; (iiic)

iv) Any profit on the transfer of the Duty Entitlement Pass Book Scheme; (iiid)

v) Any profit on the transfer of Duty Free Replenishment Certificate; (iiie)

5. Perquisite from business or profession [Sec. 28(iv)] :-the value of any benefit or

perquisite, whether convertible into money or not, arising from business or the

exercise of a profession;

6. Remuneration to Partner [Sec.28 (v)] : any interest, salary, bonus, commission

or remuneration, by whatever name called, due to, or received by, a partner of a

firm from such firm :

provided that where any such sum or any part thereof has been disallowed to the

firm u/s 40(b), it shall not be taxable in the hands of partner to that extent;

[for example, if firm pays remuneration or interest of Rs.50,000 to the partner, but

Rs.23,000 is disallowed u/s 40(b), and only Rs.27,000 is allowed as deduction to the

firm, then only Rs.27,000 shall be taxable in the hands of the partner.]

7. Amount received or receivable for certain agreement [Sec.28(va)]

1. Not carrying out any activity in relation to any business; or

2. Not sharing any know-how, patent, copyright, trade-mark, license, franchise

or any other business or commercial right of similar nature or information or

technique likely to assist in the manufacture or processing of goods or provisions

for services :

Provided that sub-clause (a) shall not apply to –

I. Any sum, whether received or receivable, in cash or kind, on account of

transfer of the right, which is chargeable under the head “Capital gains”

II. Any sum received as compensation, from the multilateral fund of the

Montreal Protocol under the United Nations Environment Program, in

accordance with the terms of agreement entered into with the Government of

India.

8. Keyman insurance policy :-any sum received under a keyman insurance policy

including the sum allocated by way of bonus on such policy.

9. Recovery against certain capital assets covered u/s.35AD [Sec.28(vii)]anysum

received or receivable, in cash or kind, on account of any capital asset being

Page 97: INTER CA Taxation - CA Study Web€¦ · INTER CA Taxation Head Office Shraddha, 4th Floor, ... 14. Advance Tax & Interest 268 - 271 15. Tax Deducted at Source (TDS) 272 - 294 16

J.K.SHAH CLASSES INTER C.A. – DIRECT TAXES

: 94 :

demolished, destroyed, discarded or transferred, if such capital expenditure has

been allowed as a deduction u/s 35AD.

Explanation 2 – Where speculative transactions carried on by an assessee are of

such a nature as to constitute a business, the business (hereinafter referred to as

“speculation business”) shall be deemed to be distinct and separate from any other

business.

“Speculative Transaction”

It means a transaction in which a contract for the purchase or sale of any commodity,

including stocks and shares, is periodically or ultimately settled otherwise than by the

actual delivery or transfer of the commodity or scrip : [Sec. 43(5)]

However the following shall not be deemed to be a speculative transaction. –

• Hedging contract in respect of raw materials or merchandise entered into by a

person in the course of his manufacturing or merchanting business to guard

against loss through future price fluctuations in respect of his contracts for

actual delivery of goods manufactured by him or merchandise sold by him; or

• a contract in respect of stocks and shares entered into by a dealer or investor

therein to guard against loss in his holdings of stock and shares through price

fluctuations; or

• a contract entered into by a member of a forward market or a stock exchange in

the course of any transaction in the nature of jobbing or arbitrage to guard

against loss which may arise in the ordinary course of his business as such

member; or

• an eligible transaction in respect of trading in derivatives carried out in a

recognized stock exchange, or

• an eligible transaction in respect of trading in commodity derivatives carried out

in a recognized association which is chargeable to commodities transaction tax.

Section 29 : Computation of Taxable Profits

According to section 29, the profits and gains of any business or profession are to be

computed in accordance with the provisions contained in sections 30 to 43D. It must,

however, be remembered that in addition to the specific allowances and deductions

stated in sections 30 to 36, the Act further permits allowance of items of expenses

under the residuary section 37(1), which extends the allowance to items of business

expenditure not covered by sections 30 to 36, where these are allowable according to

accepted commercial practices.

Page 98: INTER CA Taxation - CA Study Web€¦ · INTER CA Taxation Head Office Shraddha, 4th Floor, ... 14. Advance Tax & Interest 268 - 271 15. Tax Deducted at Source (TDS) 272 - 294 16

J.K.SHAH CLASSES INTER C.A. – DIRECT TAXES

: 95 :

Format for calculation of taxable profits

Name of Assessee

Computation of Income from Business and Profession

Particulars Rs Rs

Net profit as per Profit and loss account XX

Add: 1. Expenses debited to profit and loss account but not allowed as deduction

2. Incomes not credited to profit and loss account but taxable

XX

XX

XX

Less: 1. Expenses not debited to profit and loss account but allowed as deduction

2. Incomes credited to profit and loss account but taxable under other heads or Exempt

(XX)

(XX)

(XX)

Taxable profits XX

Page 99: INTER CA Taxation - CA Study Web€¦ · INTER CA Taxation Head Office Shraddha, 4th Floor, ... 14. Advance Tax & Interest 268 - 271 15. Tax Deducted at Source (TDS) 272 - 294 16

J.K.SHAH CLASSES INTER C.A. – DIRECT TAXES

: 96 :

What are specific deductions under the act

Section 30 to 37 covers expenses which are expressly allowed as deduction while computing

business income, Sections 40,40A,43B cover expenses which are not deductible. The following

expenses are expressly allowed as deductions against profits and gains of business or

profession:

Section 30 allows deduction in respect of the rent, rates, taxes, repairs and insurance of

buildings used by the assessee for the purpose of his business or profession.

a. the rent for premises, the amount of repairs, if he has undertaken to bear the cost of

repairs(this is applicable if the assessee has occupied the property as a tenant)

b. the amount of current repairs (if the assessee has occupied the premises otherwise than as a

tenant)

c. any sum on account of municipal taxes

d. premium in respect of insurance against risk of damage or destruction of property.

Points to be considered:

1. Capital expenditure is not allowed under sec 30. Assessee is entitle to depreciation

2. Notional rent is not allowed as deduction.

3. Municipal taxes are deductible subject to the conditions given under section 43B

Section 31 allows deduction in respect of the expenses on current repairs and insurance of

machinery, plant and furniture in computing the income from business or profession.

Usage of the asset: In order to claim this deduction the assets must have been used for

purposes of the assessee’s own business the profits of which are being taxed.

Insurance premium: The deduction allowable in respect of premia paid for insuring the

machinery, plant or furniture.

Cost of repairs and current repairs of capital nature not to be allowed : As per section 31,

the amount paid on account of current repairs of machinery, plant or furniture is allowed as

deduction in the computation of income under the head “profits and gains of business or

profession”

Rent, rates, taxes repairs and insurance for building: [Section 30]

Repairs and insurance of machinery, plant and furniture [Section 31]

Page 100: INTER CA Taxation - CA Study Web€¦ · INTER CA Taxation Head Office Shraddha, 4th Floor, ... 14. Advance Tax & Interest 268 - 271 15. Tax Deducted at Source (TDS) 272 - 294 16

J.K.SHAH CLASSES INTER C.A. – DIRECT TAXES

: 97 :

(1) Section 32 allows a deduction in respect of depreciation resulting from the

diminution or exhaustion in the value of certain capital assets. The Explanation to this

section provides that deduction on account of depreciation shall be made compulsorily,

whether or not the assessee has claimed the deduction in computing his total income.

(2) Conditions to be satisfied for allowance of depreciation:

The allowance of depreciation which is regulated by Rule 5 of the Income-tax Rules,

1962, is subject to the following conditions which are cumulative in their application.

(a) The assets in respect of which depreciation is claimed must belong to either of the

following categories, namely:

(i) buildings, machinery, plant or furniture, being tangible assets;

(ii) know-how, patents, copyrights, trademarks, licences, franchises or any other

business or commercial rights of similar nature.

(b) The assets should be actually used by the assessee for purposes of his business

during the previous year - The asset must be put to use at any time during the previous

year. The amount of depreciation allowance is not proportionate to the period of use

during the previous year.

Asset used for less than 180 days - However, it has been provided that where any asset

is acquired by the assessee during the previous year and is put to use for the purposes

of business or profession for a period of less than 180 days, depreciation shall be

allowed at 50 per cent of the allowable depreciation according to the percentage

prescribed in respect of the block of assets comprising such asset. It is significant to note

that this restriction applies only to the year of acquisition and not for subsequent years.

The balance depreciation can be claimed in the immediately succeeding previous year. If

the assets are not used exclusively for the business of the assessee but for other

purposes as well, the depreciation allowable would be a proportionate part of the

depreciation allowance to which the assessee would be otherwise entitled. This is

provided in section 38.

Use includes passive use in certain circumstances: One of the conditions for claim of

depreciation is that the asset must be “used for the purpose of business or profession”.

Section 32 Depreciation

Page 101: INTER CA Taxation - CA Study Web€¦ · INTER CA Taxation Head Office Shraddha, 4th Floor, ... 14. Advance Tax & Interest 268 - 271 15. Tax Deducted at Source (TDS) 272 - 294 16

J.K.SHAH CLASSES INTER C.A. – DIRECT TAXES

: 98 :

Courts have held that, in certain circumstances, an asset can be said to be in use even

when it is “kept ready for use”.

For example, stand by equipment and fire extinguishers can be capitalized if they are

‘ready for use’’.

(c) The assessee must own the assets, wholly or partly - Depreciation is allowable not

only in respect of assets “wholly” owned by the assessee but also in respect of assets

“partly” owned by him and used for the purposes of his business or profession.

(3) Computation of Depreciation Allowance - Depreciation allowance will be calculated

on the following basis:

(i) Power generation undertakings: In the case of assets of an undertaking engaged in

generation or generation and distribution of power, such percentage on the actual cost

to the assessee as prescribed by Rule 5(1A).

Rule 5(1A) - As per this rule, the depreciation on the abovementioned assets shall be

calculated at the percentage of the actual cost at rates specified in Appendix IA of these

rules. However, the aggregate depreciation allowed in respect of any asset for different

assessment years shall not exceed the actual cost of the asset. It is further provided that

such an undertaking as mentioned above has the option of being allowed depreciation

on the written down value of such block of assets as are used for its business at rates

specified in Appendix I to these rules.

However, such option must be exercised before the due date for furnishing return

under section 139(1) for the assessment year relevant to the previous year in which it

begins to generate power. It is further provided that any such option once exercised

shall be final and shall apply to all subsequent assessment years.

(ii) Block of assets: In the case of any block of assets, at such percentage of the written

down value of the block, as may be prescribed by Rule 5(1).

Block of Assets: A “block of assets” is defined in clause (11) of section 2, as a group of

assets falling within a class of assets comprising—

(a) tangible assets, being buildings, machinery, plant or furniture;

Page 102: INTER CA Taxation - CA Study Web€¦ · INTER CA Taxation Head Office Shraddha, 4th Floor, ... 14. Advance Tax & Interest 268 - 271 15. Tax Deducted at Source (TDS) 272 - 294 16

J.K.SHAH CLASSES INTER C.A. – DIRECT TAXES

: 99 :

(b) intangible assets, being know-how, patents, copyrights, trademarks, licenses,

franchises or any other business or commercial rights of similar nature, in respect of

which the same percentage of depreciation is prescribed.

(iii) Additional depreciation on Plant & Machinery acquired by an Industrial

Undertaking: Additional depreciation is allowed on any new machinery or plant (other

than ships and aircraft) acquired and installed after 31.3.2005 by an assessee engaged in

the business of manufacture or production of any article or thing or in the business of

generation or transmission or distribution of power at the rate of 20% of the actual cost

of such machinery or plant. Businesses eligible for claim of additional depreciation

under section 32(1)(iia)

Such additional depreciation will not be available in respect of:

(i) any machinery or plant which, before its installation by the assessee, was used within

or outside India by any other person; or

(ii) any machinery or plant installed in office premises, residential accommodation, or in

any guest house; or

(iii) office appliances or road transport vehicles; or

(iv) any machinery or plant, the whole or part of the actual cost of which is allowed as a

deduction (whether by way of depreciation or otherwise) in computing the income

chargeable under the head “Profits and Gains of Business or Profession” of any one

previous year.

Asset put to use for less than 180 days: Balance 50% of additional depreciation to be

allowed in the subsequent year, where the plant and machinery is put to use for less

than 180 days during the previous year of acquisition and installation

[Sections 32(1)]

To remove the discrimination in the matter of allowing additional depreciation on plant

or machinery used for less than 180 days vis-a-vis used for 180 days or more, third

proviso to section 32(1)(ii) provides that the balance 50% of the additional depreciation

on new plant or machinery acquired and used for less than 180 days which has not been

allowed in the year of acquisition and installation of such plant or machinery, shall be

allowed in the immediately succeeding previous year.

Page 103: INTER CA Taxation - CA Study Web€¦ · INTER CA Taxation Head Office Shraddha, 4th Floor, ... 14. Advance Tax & Interest 268 - 271 15. Tax Deducted at Source (TDS) 272 - 294 16

J.K.SHAH CLASSES INTER C.A. – DIRECT TAXES

: 100 :

(iv) Terminal depreciation: In case of a power concern as covered under clause (i)

above, if any asset is sold, discarded, demolished or otherwise destroyed in the previous

year, the depreciation amount will be the amount by which the monies payable in

respect of such building, machinery, plant or furniture, together with the amount of

scrap value, if any, falls short of the written down value thereof. The depreciation will

be available only if the deficiency is actually written off in the books of the assessee.

4) Written down value [Section 43(6)]

(i) Assets acquired by the assessee during the previous year: In the case of assets

acquired by the assessee during the previous year, the written down value means the

actual cost to the assessee.

(ii) Assets acquired before the previous year: In the case of assets acquired before the

previous year, the written down value would be the actual cost to the assessee less the

aggregate of all deductions actually allowed in respect of depreciation. For this purpose,

any depreciation carried forward is deemed to be depreciation actually allowed [Section

43(6)(c)(i) read with Explanation 3].

The written down value of any asset shall be worked out as under in accordance with

section 43(6)(c):

Page 104: INTER CA Taxation - CA Study Web€¦ · INTER CA Taxation Head Office Shraddha, 4th Floor, ... 14. Advance Tax & Interest 268 - 271 15. Tax Deducted at Source (TDS) 272 - 294 16

J.K.SHAH CLASSES INTER C.A. – DIRECT TAXES

: 101 :

Meaning of “Moneys payable” and “Sold”

Term Meaning

Moneys payable In respect of any building, machinery, plant or furniture

includes—

(a) any insurance, salvage or compensation moneys

payable in respect thereof;

(b) where the building, machinery, plant or furniture is

sold, the price for which it is sold, so, however, that

where the actual cost of a motor-car is, in accordance

with the proviso to section 43(1), taken to be ` 25,000,

the moneys payable in respect of such motor-car shall

be taken to be a sum which bears to the amount for

which the motor-car is sold or, as the case may be, the

amount of any insurance, salvage or compensation

moneys payable in respect thereof (including the

amount of scrap value, if any) the same proportion as

the amount of ` 25,000 bears to the actual cost of the

motor-car to the assessee as it would have been

computed before applying the said proviso;

Sold Includes a transfer by way of exchange or a compulsory

acquisition under any law for the time being in force

but does not include a transfer, in a scheme of

amalgamation, of any asset by the amalgamating

company to the amalgamated company where the

amalgamated company is an Indian company or a

transfer of any asset by a banking company to a banking

institution in a scheme of amalgamation of such

banking company with the banking institution,

sanctioned and brought into force by the Central

Government.

Page 105: INTER CA Taxation - CA Study Web€¦ · INTER CA Taxation Head Office Shraddha, 4th Floor, ... 14. Advance Tax & Interest 268 - 271 15. Tax Deducted at Source (TDS) 272 - 294 16

J.K.SHAH CLASSES INTER C.A. – DIRECT TAXES

: 102 :

An illustrative example of few blocks of assets.

Block

Number

Nature of Assets Rate of

Dep.

Block 1

BUILDINGS Residential buildings other than hotels 5% and boarding houses

5%

Block 2

BUILDINGS - office, factory, godowns, hotels, boarding houses or buildings which are not mainly used for residential purpose i.e. other

10

Block 3

than block 1 and block 3 below. Buildings : 100% * Building acquired after 1.9.02 for installing machinery and plant forming part of water supply prozect or water treatment system and which is put to use for the purpose of business of providing

infrastructure facilitates u/s 80 IA (4) (i) * Temporary erections such as wooden structures

100

Block 4 Furniture - Any furniture / fittings including electrical fittings. 10

Block 5

PLANT & MACHINERY : * Any plant and machinery which is not covered by B lock

6,7,8,9,10,11 or 12 * Motor cars other than those used in a business of

running them on hire, acquired or put to use on or after 1.4.90

15

Block 6 Plant & Machinery of any type of ship 20

Block 7 Plant and Machinery : * Buses, lorries, and taxies used in the business o f

running them on hire * Machinery used in semi conductor industry * Moulds used in rubber and plastic factories and l ife

saving medical equipment

30

Block 8 * Plant & Machinery of Aeroplans Aeroengines * Specified life saving medical equipment

40

50

Block 9 Plant and Machinery Containers made of glass or plastic used as refi lls

50

Block 10

Plant & Machinery - Computers including computer software. 'Books' other than annual subscription

60

Block 11

Plant & Machinery – Energy saving devices, renewal energry devices, rollers in flour mills, sugar work s and steel industry

80

Block 12

Plant & Machinery - * AIR / water pollution control equipments * Solid waste control equipments * Rectifying and resource recovery systems * Machinery aquired and installed on or after 1.9.02 in a

water supply prozect or water treatment system or f or the purpose of providing instrastructure facility

* Wooden parts used in artificial silk manufacturing machinery

* Cinematograph films * Bulbs of studio lights * Wooden match frames

100

Page 106: INTER CA Taxation - CA Study Web€¦ · INTER CA Taxation Head Office Shraddha, 4th Floor, ... 14. Advance Tax & Interest 268 - 271 15. Tax Deducted at Source (TDS) 272 - 294 16

J.K.SHAH CLASSES INTER C.A. – DIRECT TAXES

: 103 :

* Some plants used in mines, quarries and salt works * Professional reference books * Books in the business of running lending libraries

Block 13

Intangible assets acquired on or after 1.4.98 such as know - how patents, copyrights, trade marks,licences, franchises and any other business or commercial rights of simi lar nature.

25

6) Other points

1) Balancing Charge

Section 41(2) provides for the manner of calculation of the amount which shall be

chargeable to income-tax as income of the business of the previous year in which the

monies payable for the building, machinery, plant or furniture on which depreciation has

been claimed under section 32(1)(i), i.e. in the case of power undertakings, is sold,

discarded, demolished or destroyed. The balancing charge will be the amount by which

the moneys payable in respect of such building, machinery, plant or furniture, together

with the amount of scrap value, if any, exceeds the written down value. However, the

amount of balancing charge should not exceed the difference between the actual cost

and the WDV.

The tax shall be levied in the year in which the moneys payable become due. The

Explanation below section 41(2) makes it clear that where the moneys payable in

respect of the building, machinery, plant or furniture referred to in section 41(2) become

due in a previous year in which the business, for the purpose of which the building,

machinery, plant or furniture was being used, is no longer in existence, these provisions

will apply as if the business is in existence in that previous year.

2) Composite Income: Explanation 7 provides that in cases of ‘composite income’, for

the purpose of computing written down value of assets acquired before the previous

year, the total amount of depreciation shall be computed as if the entire composite

income of the assessee is chargeable under the head “Profits and Gains of business or

profession”. The depreciation so computed shall be deemed to have been “actually

allowed” to the assessee.

For instance, Rule 8 prescribes the taxability of income from the manufacture of tea.

Under the said rule, income derived from the sale of tea grown and manufactured by

seller shall be computed as if it were income derived from business, and 40% of such

income shall be deemed to be income liable to tax. If the turnover is, say, ` ` ` ` 20 lakh, the

depreciation ` ` ` ` 1 lakh and other expenses ` ` ` ` 4 lakh, then the income would be ` ` ` ` 15

lakh. Business income would be ` ` ` ` 6 lakh (being 40% of ` ` ` ` 15 lakh). As per earlier Court

decisions, only the depreciation “actually allowed” i.e., ` ` ` ` 40,000, being 40% of ` 1 lakh,

has to be deducted to arrive at the written down value.

Page 107: INTER CA Taxation - CA Study Web€¦ · INTER CA Taxation Head Office Shraddha, 4th Floor, ... 14. Advance Tax & Interest 268 - 271 15. Tax Deducted at Source (TDS) 272 - 294 16

J.K.SHAH CLASSES INTER C.A. – DIRECT TAXES

: 104 :

The ambiguity in this case has arisen on account of the interpretation of the meaning of

the phrase “actually allowed” used in section 43(6)(b). However, the correct legislative

intent is that the WDV is required to be computed by deducting the full depreciation

attributable to composite income i.e. ` ` ` ` 1 lakh in this case. Explanation 7 clarifies this

legislative intent.

3. Building previously the property of the assessee: Where a building which was

previously the property of the assessee is brought into use for the purposes of the

business or profession, its actual cost to the assessee shall be the actual cost of the

building to the assessee, as reduced by an amount equal to the depreciation calculated

at the rates in force on that date that would have been allowable had the building been

used for the purposes of the business or profession since the date of its acquisition by

the assessee [Explanation 5].

4. In case of succession of firm/sole proprietary concern by a company or

amalgamation or demerger of companies

In order to restrict the double allowance under the proviso to section 32, in the cases of

succession or amalgamation or demerger, the aggregate deduction in respect of

depreciation allowable in the hands of the predecessor and the successor or in the case

of amalgamating company and the amalgamated company or in the case of the

demerged company and the resulting company, as the case may be, shall not exceed

the amount of depreciation calculated at the prescribed rates as if the

succession/amalgamation, demerger had not taken place.

It is also provided that such deduction shall be apportioned between the two entities in

the ratio of the number of days for which the assets were used by them.

Page 108: INTER CA Taxation - CA Study Web€¦ · INTER CA Taxation Head Office Shraddha, 4th Floor, ... 14. Advance Tax & Interest 268 - 271 15. Tax Deducted at Source (TDS) 272 - 294 16

J.K.SHAH CLASSES INTER C.A. – DIRECT TAXES

: 105 :

A numeric illustration of block method of accounting for depreciation u/s 32,

spread into several years in relation to its varied features.

Year 1 PLANT & MACHINERY - 15%

Opening W.D.V. 5,000 58,00,000

(+) Purchase during the year

and

used for < 180 days during such

a year of purchase 8,000 12,00,000

TOTAL 13,000 70,00,000

(-) Sold during the year (8,000) (12,00,000)

Qualifying amount for depreciation 5,000 58,00,000

(-) depreciation u/s 32

12,00,000 × ��

� × 50 % = 90,000

46,00,000 × ��

� × 100 % = 6,90,000

(BAL.) 100

Closing W.D.V. 5,000 50,20,000

Remarks :

In this situation the 8,000 machines which were purchased during the year and used for

less than 180 days during such a year of purchase, are sold, yet depreciation is

reduced as above and the reason is that the identity, personality and individuality of the

asset gets lost in the common hotch pot of the block and value based calculations are

made. Thus, whatever is sold, is not recognised under the system, and the fact remains

that such purchase value of `̀̀̀12,00,000 is retained in the block for less than 180 days

and thus is entitled to depreciation @ 50% of the block's rate of depreciation.

In order to attract the provisions of restriction of depreciation to 50% of the block's rate,

two conditions are to be satisfied together i.e.

� asset should be purchased during the year.

AND

� used for less than 180 days during such year of purchase. If an asset is purchased

in say year 1 but not used in year 1 at all, no depreciation would be allowed in

year 1. If such an asset is installed and used in year 2 for < 180 days, yet

depreciation would be allowed @ 100% of the block's rate and not 50%, since the

asset is not purchased in year 2.

� In case such 8,000 machines would have been sold for say `̀̀̀ 60,00,000, then

qualifying amount for depreciation would be `̀̀̀10,00,000. Such `̀̀̀ 10,00,000 would

be subject to restriction of 50% of the block's rate since it is less than purchase

value of `̀̀̀ 12,00,000. In other words the restriction is applicable on purchase

value of less than 180 days or qualifying amount for depreciation, whichever is

less.

Page 109: INTER CA Taxation - CA Study Web€¦ · INTER CA Taxation Head Office Shraddha, 4th Floor, ... 14. Advance Tax & Interest 268 - 271 15. Tax Deducted at Source (TDS) 272 - 294 16

J.K.SHAH CLASSES INTER C.A. – DIRECT TAXES

: 106 :

Year 2 PLANT & MACHINERY - 15%

Opening WDV 5,000 50,20,000

(+) Purchase during the year ---- ----

The year TOTAL 5,000 50,20,000

(-) Sold during the year (100) (20,00,000)

Q. A. for depreciation 4,900 30,20,000

(-) Depreciation u/s 32 (4,53,000)

Closing WDV 4,900 25,67,000

Remarks :

In this situation apparently there is a huge capital gain on the sale of 100 machines, which under the block method is not calculated separately if there is positive assets and positive value as qualifying amount for depreciation, such gain gets absorbed in current depreciation and closing WDV and the effect is that depreciation is reduced by Gain* block's rates of depreciation and the balance of the gain reduces the closing WDV.

Year 3 PLANT & MACHINERY - 15%

Opening WDV 4,900 25,67,000

(+) Purchased during the year --- ----

Total 4,900 25,67,000

(-) Sold during the year (4,000) (500)

Q. A. for depreciation 900 25,66,500

(-) Depreciation (3,84,975)

Closing WDV 900 21,81,525

Remarks :

In this situation apparently, there is a huge capital loss on the sale of 4,000 machines, which under the block method of accounting for depreciation is not calculated separately if there are positive assets and positive value as qualifying amount for depreciation, but such loss gets adjusted in current depreciation and closing WDV and the effect is that current depreciation is increased by Loss* block's rate of depreciation and the balance of loss increases the closing WDV.

Year 4 PLANT & MACHINERY - 15%

Opening WDV 900 21,81,525

(+) Purchased during the year ---- ----

Total 900 21,81,525

(-) Sold during the year (300) (50,00,000) 21,81,525

(28,18,475)

Q. A. for depreciation 600 NIL

(-) Depreciation NIL

Closing WDV 600 NIL

Page 110: INTER CA Taxation - CA Study Web€¦ · INTER CA Taxation Head Office Shraddha, 4th Floor, ... 14. Advance Tax & Interest 268 - 271 15. Tax Deducted at Source (TDS) 272 - 294 16

J.K.SHAH CLASSES INTER C.A. – DIRECT TAXES

: 107 :

Capital Gains

Sale consideration 50,00,000

(-) Cost of acquisition of depreciable asset u/s 50(1)

(All assets in the block are not sold)

Opening WDV 21,81,525

(+) Purchased during the year NIL (21,81,525)

S.T.C.G. taxable u/s 45 28,18,475 Remarks : In this situation i.e. when all assets are not sold and the sale consideration of whatever is sold, exceeds the opening WDV. (+) purchased during the year (+) transfer expenses, such excess is always short - term capital gain as above. � Always short - term because the identity, individuality gets lost in the common hotch pot of the block,

thus what is sold is not recognised under the system and hence even if the asset is held for more than 3 years prior to its sale, it would be short - term. Further also because value based calculations are made and such value keeps on changing every year.

� Always capital gains In this situation when all assets are not sold, there can never be a possibility of short - term capital loss (if there is actually a loss, it will be covered by year 3 situation) i.e. there will always be short - term capital gains only, or the block continues.

Year 5 PLANT & MACHINERY - 15%

Opening WDV 600 NIL (+) Purchase during the year ---- ---- Total 600 NIL (-) Sold during the year (300) (3,00,000)

NIL Q. A. for depreciation 300 (3,00,000) NIL (-) Depreciation u/s 32 NIL Closing WDV 300 NIL Capital Gains Sale consideration 3,00,000 (-) Cost of acquisition of depreciable asset u/s 50(1) (All assets in the block are not sold) Opening WDV NIL (+) Purchased during the year NIL NIL S.T.C.G. taxable u/s 45 3,00,000

Page 111: INTER CA Taxation - CA Study Web€¦ · INTER CA Taxation Head Office Shraddha, 4th Floor, ... 14. Advance Tax & Interest 268 - 271 15. Tax Deducted at Source (TDS) 272 - 294 16

J.K.SHAH CLASSES INTER C.A. – DIRECT TAXES

: 108 :

Remarks : Same as year 4 situation. Year 6 PLANT & MACHINERY - 15%

Opening WDV 300 NIL (+) Purchased during the year & used for > 180 days 200 2,00,000 TOTAL 500 2,00,000 (-) Sold during the year (500) (1,20,000) 2,00,000 Q. A. for depreciation NIL 80,000

NIL Block ceases to exist Capital Gains

Sales consideration 1,20,000 (-) Cost of acquisition of depreciable asset u/s 50(2) (All assets in the block are sold) Opening WDV NIL (+) Purchased during the year 2,00,000 (2,00,000) Short - term Capital Loss (80,000)

Remarks : In this situation all assets in the block are sold and here there are two possibilities, i.e. short - term capital loss as above or short - term capital gains / NIL. If the 500 machines would have been sold for say `̀̀̀ 3,00,000, then `̀̀̀ 1,00,000 would be short - term capital gains taxable u/s 45.

Sec. 32AD:- Investment in new Plant or Machinery [Inserted by FA 2015 w-e-f PY 15-16]

Assessee Any Assessee, who sets up an undertaking or enterprise for manufacture or production of any article or thing, on or after the 1st April, 2015 in any backward area notified by the Central Government in this behalf, in the State of Andhra Pradesh, Bihar, Telangana or West Bengal

Investment Acquires and installs new plant and machinery during the period beginning on 1st April, 2015 and ending on 31st March, 2020 in the said backward area. New plant or machinery shall not include – a) Ship or aircraft b) Any plant or machinery which before its installation by the assessee was used either within or outside India by any other person;

c) Any plant or machinery installed in any office premises or any residential accommodation, including accommodation in the nature of a guest house;

d) Any office appliances including computers or computer software;

Page 112: INTER CA Taxation - CA Study Web€¦ · INTER CA Taxation Head Office Shraddha, 4th Floor, ... 14. Advance Tax & Interest 268 - 271 15. Tax Deducted at Source (TDS) 272 - 294 16

J.K.SHAH CLASSES INTER C.A. – DIRECT TAXES

: 109 :

e) Any vehicle; or f) Any plant or machinery, the whole of the actual cost of which is allowed as deduction (whether by way of depreciation or otherwise) u/h PGBP

Deduction Deduction = 15% of Actual Cost of new P&M acquired and installed during the PY,

Lock in period 5 years from the date of installation

Withdrawal of deduction

If sold or otherwise transferred within 5 years, the deduction allowed under this section shall be treated as income u/h PGBP. Exception : amalgamation or demerger of companies or business reorganization u/s 47 (xiii), (xiiib) or (xiv). However the lock in will be applicable for remaining period to the successor

Sec. 35:- Expenditure on Scientific Research

“Scientific Research” means any activity for the extension of knowledge in the fields of

natural or applied science including agriculture, animal husbandry or fisheries. Besides

scientific research, research in social sciences like human behavior and marketing

research work are also covered under this section.

Contribution made by the assessee to outside agencies

Sec 32(2AA) Contribution to National Laboratory or university

or an IIT, to carry out scientific research

Provided that such university, college or other institution –

(A) Is for the time being approved by prescribed authority; and (B) Is notified in the Official Gazette, by the Central Government; Provided that the prescribed authority shall, before granting approval, satisfy itself about the feasibility of carrying out the scientific research and shall submit its report to the Principal Chief Commissioner or Chief Commissioner or Principal Director General or Director General in such form as may be prescribed.

Deduction=150%

of sum paid

Sec. 35(1)(ii) : contribution to university, college or association

to carry out scientific research

Provided that such university, college or other institution –

(A) Is for the time being approved; and (B) Is notified in the Official Gazette, by the Central Government;

Deduction=150%

of sum paid

Sec. 35(1)(iia) : Contribution to a company to be used by it for

scientific research :

Provided that such company –

(i) Is registered in India, (ii) Has as its main object the scientific research and

development, (iii) Is, for the purposes of this clause, for the time being

approved by the prescribed authority in the prescribed manner, and

Deduction=100%

of sum paid

Page 113: INTER CA Taxation - CA Study Web€¦ · INTER CA Taxation Head Office Shraddha, 4th Floor, ... 14. Advance Tax & Interest 268 - 271 15. Tax Deducted at Source (TDS) 272 - 294 16

J.K.SHAH CLASSES INTER C.A. – DIRECT TAXES

: 110 :

(iv) Fulfills such other conditions as may be prescribed; [Sec.35(1)(iia)(C)]

Sec. 35(1)(iii): Contribution to research association, university, college or other institutions, to carry out research in social sciences or statistical research. Provided that such association, university, college or other institution – (A) Is for the time being approved; and (B) Is notified in the Official Gazette, by the Central Government;

Deduction=100% of sum paid

Explanation : - The deduction in respect of any sum paid to a scientific research association, university, college or other institution, or national laboratory university, Indian Institute of Technology shall not be denied merely on the ground that, subsequent to the payment of such sum by the assessee, the approval granted to the association, university, college or other institution has been withdrawn.

Scientific Research carried on by the assessee himself (Research work shall relate to the business of the assessee)

REVENUE EXPENDITURE [Sec. 35(1) (i)]

Pre-commencement period (3 years preceding the commencement of business)

a) Payment of salary to research personnel; b) Purchase of materialized in scientific research (as certified by the prescribed authority)

Post – commencement period All expenses laid out or expended on scientific research

CAPITAL EXPENDITURE [Sec.35(1) (iv) and 35(2)]

Pre-commencement period (3 years preceding the commencement of business)

Any capital expenditure (Except cost of land)

Post – commencement period Any capital expenditure (except cost of land)

EXPENDITURE ON IN-HOUSE R&D BY COMPANIES [Sec.35(2AB)]:

ASSESSEE Company only

ELIGIBLE BUSINESS

Bio-technology or any business of manufacture or production of any article or thing, not being an article or thing specified in the list of the Eleventh Schedule

EXPENDITURE Capital or revenues expenditure excluding cost of any land and building.

TIME Expenditure or revenue expenditure excluding cost of any land and building. No deduction shall be allowed to a company accepting donations u/s.35(1)(iia)(C)

CONDITIONS • The R&D facility is approved by the prescribed authority

• The co. has entered into an agreement with the prescribed authority for co-operation in such R&D facility and fulfils such conditions with regard to maintenance of accounts and audit thereof and furnishing of reports in such manner as may be prescribed.

Page 114: INTER CA Taxation - CA Study Web€¦ · INTER CA Taxation Head Office Shraddha, 4th Floor, ... 14. Advance Tax & Interest 268 - 271 15. Tax Deducted at Source (TDS) 272 - 294 16

J.K.SHAH CLASSES INTER C.A. – DIRECT TAXES

: 111 :

• The prescribed authority shall submit its report in relation to the approval of the said facility to the 34 [Principal Chief Commissioner or Chief Commissioner or] 35 [Principal Director General or] Director General in such form and within such time as may be prescribed.

DEDUCTION 150% of revenue and capital expenditure except cost of land and building Note :- pre-commencement expenses and cost of building is not allowed under section 35(2AB). Hence they shall be entitled for 100% deduction u/s 35(1) and 35(2)

Other issues :

1. Sale of the asset : Sec.41(3) : the surplus arising on sale of the asset is chargeable

as business income to the extent of deduction allowed. Balance, if any, shall be

treated as capital gains.

2. Consequences in case of amalgamation [Section 35(5):- if the amalgamating

company transfers capital asset to the amalgamated company (being an Indian

company), the above provisions would apply to the amalgamated company.

3. Unabsorbed Capital Expenditure : if on account of inadequacy or absence of

profits of the business, deduction on account of capital expenditure on scientific

research cannot be allowed, fully or partly, the deficiency so arising is to be C/F as if

it is unabsorbed depreciation.

Sec. 35ABB:- Expenditure for obtaining license to operate telecommunication services

EXPENDITURE Capital expenditure for acquiring right to operate telecommunication services

DEDUCTION :

1) Fee paid before commencement of business

Amortized over period starting from the PY in which business is commenced and ending with the PY in which the license expires.

2) Fee paid after commencement of business

Amortized over period starting from the PY in which the license fee is paid and ending with the PY in which the license expires.

SALE OF LICENSE

(i) Entire license is transferred

a) Where sale proceeds < un-amortized amount, such deficiency shall be allowed as deduction in the PY in which the license is transferred.

b) Where sale proceeds > un-amortized amount, such excess, to the extent of deductions already allowed in the past, shall be taxable as business income of the year in which the license is transferred.

(ii) Part of the license is transferred

a) Where sale proceeds < un-amortized amount of the total license, the balance shall be deductible over the remaining life of the license.

Page 115: INTER CA Taxation - CA Study Web€¦ · INTER CA Taxation Head Office Shraddha, 4th Floor, ... 14. Advance Tax & Interest 268 - 271 15. Tax Deducted at Source (TDS) 272 - 294 16

J.K.SHAH CLASSES INTER C.A. – DIRECT TAXES

: 112 :

b) Where sale proceeds > un-amortized amount, such excess, to the extent of deductions already allowed in the past, shall e taxable as business income of the year in which the license is transferred.

AMALGAMATION OR DEMERGER

The amalgamated or resulting company is entitled to claim deduction to the same extent as it would have been allowable to the amalgamating company or demerged company.

Sec. 35ABA:- Spectrum charges (Same as 35ABB)

Sec. 35AD:- Deduction in respect of expenditure on specified business

Capital exp. [Other than land / goodwill / financial Instrument

Nature of business

Deduction

100%

Business commenced on or after

Operating a cold chain facility 1-4-09

Warehousing facility for storage of agricultural produce 1-4-09

Affordable housing project 1-4-11

Production of fertilizer 1-4-11

Hospital (100 beds) 1-4-10

Cross country pipeline for oil and as 1-4-07

Hotel (2 star +) 1-4-10

Slum re-development housing 1-4-10

inland container depot or a container freight station notified or approved under the Customs Act, 1952

1-4-12

Bee-keeping and production honey and bees wax 1-4-12

Warehousing facility for storage of sugar 1-4-12

Laying and operating a slurry pipeline for the transportation of iron ore

1-4-14

Setting up and operating a semi-conductor wafer fabrication manufacturing unit, which is notified by the CBDT

1-4-14

NATURE OF EXPENDITURE

Prior period expenses

Capital expenditure incurred prior to the commencement and capitalized in the books,

shall be allowed as deduction during the PY in which the operations commence

Post commencement expenses

Any capital expenditure incurred during the PY

Page 116: INTER CA Taxation - CA Study Web€¦ · INTER CA Taxation Head Office Shraddha, 4th Floor, ... 14. Advance Tax & Interest 268 - 271 15. Tax Deducted at Source (TDS) 272 - 294 16

J.K.SHAH CLASSES INTER C.A. – DIRECT TAXES

: 113 :

Other conditions

1. It is not set up by splitting up, or the reconstruction, of a business already in

existence:

2. It is not set up by the transfer to the specified business of machinery or plant

previously used for any purpose;

Exceptions :

i) Imported plant and machinery, and

ii) 20% of total value

3. Where the assessee builds a hotel of 2 star or above category as classified by the

Central Government and subsequently, while continuing to own the hotel, transfers

the operation thereof to another person, he shall be deemed to be carrying on the

specified business

4. Where the business is of cross-country natural gas or crude or petroleum oil pipeline

network, such business, -

1. Is owned by an indian company or by a consortium of such companies or by an

authority or a board or a corporation established or constituted under any Central

or State Act;

2. Has been approved by the Petroleum and Natural Gas Regulatory Board and

notified by the Central Government in the Official Gazette in this behalf;

3. Has made not less than prescribed percentage of its total pipeline capacity

available for use on common carrier basis by any person other than the

assessee or an associated person;

No Deduction

Deduction not allowed u/s 10AA and 80IA to 80RRB in relation to specified business for

the same or any other AY

Double deduction

No deduction in respect of such expenditure shall be allowed under any other section in

any previous year or under this section in any other previous year.

Use of Asset [inserted by FA 14 w-e-f PY 14-15]

Any asset in respect of which a deduction is claimed and allowed under this section

shall be used only for the specified business, for a period of 8 years beginning with

the previous year in which such asset is required or constructed.

Where such asset, is used for a purpose other than the specified business during such

period, the following amount shall be deemed to be the income of the assessee

chargeable under the head “Profits and gains of business or profession” of the previous

year in which the asset is so used

Total amount of deduction so claimed and allowed in one or more previous years

xxx

Less : the amount of depreciation allowable u/s 32, as if no deduction under this section was allowed

xxx

Page 117: INTER CA Taxation - CA Study Web€¦ · INTER CA Taxation Head Office Shraddha, 4th Floor, ... 14. Advance Tax & Interest 268 - 271 15. Tax Deducted at Source (TDS) 272 - 294 16

J.K.SHAH CLASSES INTER C.A. – DIRECT TAXES

: 114 :

Exception : if the company becomes a sick industrial company during the period

specified.

Note : Capital Expense exceeding `̀̀̀ 10,000/- not allowed as deduction if made through

other than A/c payee Cheque/Draft/ ECS in a single day to a single person.

Sec. 35D: Amortization of certain Preliminary Expenses

Assessee Indian company or any other person who is resident in India.

Eligible expenses

(I) Before commencement of business

Expenses for setting up any undertaking or business

(II) After commencement of business

Expenses in connection with expansion of an undertaking or in connection with setting up a new unit

Qualifying Expenditure

a) Expenditure in connection with preparation of a feasibility report, preparation of a project report, conducting market survey or any other survey necessary for the business of the assessee, engineering services relating to the business of the assessee.

b) Legal charges for drafting any agreement relating to the setting up or conduct of the business.

c) Where the assessee is company, following expenditure – i) Legal charges of drafting the MOA and AOA of the

company; ii) Printing charges of the MOA and AOA iii) Registration fees of a company iv) Expenses in connection with public issue of

shares or debentures, underwriting commission, brokerage and chares for drafting, typing, printing and advertisement of the prospectus.

Maximum Ceiling Indian company : 5% of the cost of the project, or 5% of the capital employed, w-e-more Any other assessee : 5% of the cost of the project

Deduction 1/5thof the qualifying expenditure in 5 successive PY’s

Audit of Books Required for the years in which the expenditure was incurred.

“Cost of the Project”

Means the actual cost of fixed assets namely, land, buildings, leaseholds, plant,

machinery, furniture, fittings and railway sidings, etc. which are shown in the books of

the assessee as on the last day of the previous year in which the business is

commenced.

Page 118: INTER CA Taxation - CA Study Web€¦ · INTER CA Taxation Head Office Shraddha, 4th Floor, ... 14. Advance Tax & Interest 268 - 271 15. Tax Deducted at Source (TDS) 272 - 294 16

J.K.SHAH CLASSES INTER C.A. – DIRECT TAXES

: 115 :

“Capital Employed”

Means the capital employed in the business of the company and includes the aggregate

of the issued share capital, debentures and long term borrowings, as on the last day of

the previous year in which the business is commenced.

Sec. 35DD :- Amortization of expenditure in case of amalgamation or demerger

Where an assessee, being an Indian Company, incurs any expenditure, wholly and

exclusively for the purposes of amalgamation or demerger of an undertaking, the

assessee shall be allowed a deduction of an amount equal to 1/5th of such

expenditure for each of the 5 successive PY’s beginning with the previous year in

which the amalgamation or demerger takes place.

Sec. 35DDA :- Amortization of compensation paid under VRs

Where an assessee incurs any expenditure in any PY by way of payment of any sum to

an employee in connection with his voluntary retirement, 1/5th of the amount so paid

shall be deductible in 5 succeeding PY’s.

TREATMENT IN CASE OF AMALGAMATION OR DEMERGER : where the

undertaking is transferred, before the expiry of 5 years, in a scheme of amalgamation,

or demerger, the deduction under this section shall, be allowed to the amalgamated or

resulting company for the balance PY’s.

Similar treatment in case of transfer of undertaking by a firm to a company u/s 47(xiii) or

a private company or unlisted public company to the LLP u/s 47(xiiib), or by a

proprietary concern to a company u/s 47(xiv).

Sec. 35CCC:- Expenditure on agricultural extension project

1. Where an assessee incurs any expenditure on agricultural extension project

notified by the Board, deduction = 100% of such expenditure.

2. Where a deduction under this section is claimed and allowed for any assessment

year in respect of such expenditure, deduction shall not be allowed in respect of

such expenditure under any other provisions of this Act for the same or any other

assessment year.

Sec. 35CCD :- Expenditure on skill development project

1. Where a company incurs any expenditure (not being expenditure in the nature of

cost of any land or building) on any skill development project notified by the

Board, deduction = 150% of such expenditure.

2. Where a deduction under this section is claimed and allowed for any assessment

year in respect of such expenditure, deduction shall not be allowed in respect of

such expenditure under any other provisions of this Act for the same or any other

assessment year.

Page 119: INTER CA Taxation - CA Study Web€¦ · INTER CA Taxation Head Office Shraddha, 4th Floor, ... 14. Advance Tax & Interest 268 - 271 15. Tax Deducted at Source (TDS) 272 - 294 16

: 116 :

J. K. SHAH CLASSES INTER C.A.- DIRECT TAXES

INCOME FROM BUSINESS & PROFESSION

OTHER DEDUCTIONS

36(1)(i) Insurance premium on stock in The amount of any premium paid in respect of

trade insurance against risk of damage or destruction of

stocks or stores, used for the purposes of the

business or profession, is allowable as deduction.

36(1)(ib) Premium paid by employer Group insurance scheme of General Insurance Cor

poration and approved by the Central Goverment

or any other insurer and approved by the Insurance

Regulatory and Development Authority, and premium

paid on mediclaim insurance policy which is paid by

any mode of payment other than cash, is allowed

as a deduction.

36(1)(ii) Bonus & commission to employees Allowed on payment basis in view of sec.43B. Should

be in relation to employment and not in relation to

share holding i.e. payment of dividend as understanding.

36(1)(iii) Interest on borrowed capital it is The following points merit consideration.

allowed if : (a) Brokerage & commission for securing a loan will

(a) The loan is borrowed by the not be allowed here in this section i.e. allowed

assessee u/s 37(1).

(b) Loan used for the purpose of (b) Interest paid during construction period is to be

his business. capitalised (absolutely new organisation).

(c) Interest is paid / payable by Interest paid on loan towards acquisition of

the assessee on such a loan. an asset for the period prior to the date on which

the asset is put to use will not be allowed

under this section, i.e. it has to be capitalised

and depreciation has to be claimed.

36(1)(iii a) Pro-rata deduction of discount on Zero coupon bonds are issued by any infrastructure

a zero coupon bond. capital company / fund or a public sector company

or scheduled bank on or after 1.6.05 and the Life of

the bond should be 10 years or more but not

exceeding 20 years.

The difference between the amount payable on the

redemption or maturity of such bond, and the amount

received on the issue of such zero coupon bond is

known as discount, and such discount will be allowed

as a deduction from the year in which the services

commence on pro-rata basis.

Page 120: INTER CA Taxation - CA Study Web€¦ · INTER CA Taxation Head Office Shraddha, 4th Floor, ... 14. Advance Tax & Interest 268 - 271 15. Tax Deducted at Source (TDS) 272 - 294 16

: 117 :

J. K. SHAH CLASSES INTER C.A.- DIRECT TAXES

INCOME FROM BUSINESS & PROFESSION

Pro-rata basis means that the deduction is available

u/s 36(1)(iiia) having regard to the period of life of

such bond i.e. the period commencing from the date

of issue of the bond and ending on the date of maturity

or redemption of such bond.

36(1)(iv) Em p loye r ' s c on t r ibu t i on t o It is allowed on payment basis u/s 43 B.

recognised provident fund or

approved superannuation fund.

36(1)(iva) Employer’s contribution towards It is allowed as a deduction provided it does not exceed

pension scheme as referred to in 10% of the salary of the employee in the previous year

section 80CCD [Salary means Basic + DA (in terms ) only].

36(1)(v) Contribution to approved gratuity Allowed, i f it is for the exclusive benefit of the

fund. employees & under an irrevocable trust, regulated by

the provisions of 43 B.

36(1)(va) Deduction made from staff salary Contributions received from employees is considered

for s taf f welfare scheme i.e. income and as and when the assessee (employer)

employee's contribution credits the employees account in the relevant fund

on or before the fund's due date, it is allowed as a

deduction under this section.

36(1)(vii) Bad debts Condition for allowance :

(1) It should be related to business of assessee.

(2) It should have been taken into income consideration

initially (in mercantile system of accounting for

credit sales).

(3) Bad debts in the course of money lending business

carried on by the assessee is also allowed.

(4) Bad debts of a business discountinued prior to

the commencement of the previous year is not

deductible. Even such bad debt cannot be deducted

from the profits of a separate existing business.

(5) In case one of the partners takes over the entire

business after dissolution or a firm is converted

into a private limited company, the successor of a

business is entitled to claim deduction in respect

of debt created by the predecessor.

(6) Subsequent recovery of bad debts after its allowance,

will be taxable in the year of receipt u/s. 41(4).

Page 121: INTER CA Taxation - CA Study Web€¦ · INTER CA Taxation Head Office Shraddha, 4th Floor, ... 14. Advance Tax & Interest 268 - 271 15. Tax Deducted at Source (TDS) 272 - 294 16

: 118 :

J. K. SHAH CLASSES INTER C.A.- DIRECT TAXES

INCOME FROM BUSINESS & PROFESSION

36(1)(viia) Provision for bad and doubtful debts made by banks and financial institutions

1. This Section applies to :

(a) Scheduled Bank (incorporated in India) or Non scheduled Bank

(b) Co-operative Bank(incorporated in India)

(c) Public Financial Institutions

(d) State Financial Corporation

(e) State Industrial Investment Corporation

(f) A foreign bank

(g) Non-Banking Financial Institutions

2. For point (a) & (b) above :

Deduction allowed : 8.5% of G.T.I.(excluding deduction under this section)

+

10% of Aggregate average advances made by RURAL branches of such banks.

3. For point (c), (d), (e), (f) & (g) above :

Deduction allowed = 5% of GTI (excluding deduction under this section)

4. If the actual bad debts written off in a particular year exceed the provision made

u/s 36(1)(viia) upto that year, the excess shall be allowed as a deduction u/s

36(1)(vii) in the year of write off.

36(1)(ix) Family planning expenses Any expenditure incurred by a company for family

planning programs is allowed. An expenditure of capital

nature will be allowed over a period of 5 years in 5

equal annual instalments.

36(1) (xv) Securities transaction tax (S.T.T.) When a share broker sells equity shares / units of an

paid by the assessee equity oriented mutual fund held as stock in trade

through recognized stock exchange and S.T.T.(Security

Transaction Tax) is paid on sales value, such S.T.T.

paid is allowed as a deduction under this section.

36(1)(xvi) Commodities transaction tax An amount equal to the commodities transaction

(CTT) paid by the assessee tax paid by the assessee in respect of the taxable

commodities transactions entered into in the course

of his business during the previous year, if the income

arising from such taxable commodities transactions

is included in the income computed under the head

“Profits and gains of business or profession”.

Page 122: INTER CA Taxation - CA Study Web€¦ · INTER CA Taxation Head Office Shraddha, 4th Floor, ... 14. Advance Tax & Interest 268 - 271 15. Tax Deducted at Source (TDS) 272 - 294 16

: 119 :

J. K. SHAH CLASSES INTER C.A.- DIRECT TAXES

INCOME FROM BUSINESS & PROFESSION

37(1) Any Expenditure which is not covered u/s. 30 to 36, which is not capital in

nature, and if incurred wholly and exclusively/necessarily for the purpose

of the assessee's business or profession will be allowed as a deduction.

New explanation is inserted with retrospective effect from 1.4.62

by the Fin Act 1998,

"For the removal of doubts, i t is here by declared that any

expenditure, incurred by an assessee for any purpose which is an

offence or which is prohibited by law shall not be deemed to have

been incurred for the purpose of business or profession and no

deduction or allowance shall be made in respect of such expenditure".

Thus payment on account of protection money, extortion, hafta, bribes

etc. will not be allowed as a business expenditure in the computation

of legal as well as illegal business.

The following points emerge :

1. Expenditure incurred by the assessee in the previous year.

2. Expenditure related to assessee's business.

3. Expenditure should not be capital in nature.

4. Expenditure should not be personal in nature.

5. Expenditure should not be covered by specified deductions.

6. Should not be prohibited by law.

Then allowed as a deduction u/s 37(1).

Note : W.e.f. P.Y. 2014-15, any expenditure incurred by a company on activities

related to Corporate Social Responsibility (CSR) as required by the Companies

Act 2013 shall not be allowed as a deduction.

EXPRESS DISALLOWANCE OF EXPENDITURE

37(2B) Advertisement expenses in No allowance shall be made in respect of expenditure

relation to a political party. incurred by an assessee on advertisement in any

souvenir, brochure, tract, pamphlet, magazine or the

like published by a political party.

This is because assesse is entitled to claim deduction

u/s 80GGB & u/s 80 GGC from his G.T.I. towards such

expenditure.

38 W h en b u i l d i ng p l a n t & Proportionate expenditure in relation to non - business

machinery or furniture not used for rent, rates, taxes, insurance, repairs and

wholly used for business depreciation will be disallowed.

Page 123: INTER CA Taxation - CA Study Web€¦ · INTER CA Taxation Head Office Shraddha, 4th Floor, ... 14. Advance Tax & Interest 268 - 271 15. Tax Deducted at Source (TDS) 272 - 294 16

: 120 :

J. K. SHAH CLASSES INTER C.A.- DIRECT TAXES

INCOME FROM BUSINESS & PROFESSION

If the following three conditions are satisfied, the assessce (i.e., the

payer) is supposed to deduc t tax source (TDS) under section 195—

1. The amount paid is interest, royalty, fees for technical sendees

or any other sum (not being salary).

2. The aforesaid amount is chargeable to tax in India in the hands

of the recipient.

3. The aforesaid amount is paid/payable to a non-resident.

If the above three* conditions are satisfied, theassessee (the

paver) is supposed to deduct tax at source and deposit :he same

with the Government.

Disallowance of expenditure under section 40(a)(1) - If TDS default is

committed in respect of payment/credit given j to a foreign company/

non-resident, the expenditure is disallowed in the hands of payer un-

der section 40(a)(i). These provisions are given below -

TDS default Is such expenditure Is such expenditure deductible

deductible in the current year in any susbequent previous year

Case - 1 Tax is deductible 100 per cent of such expen- If tax is deducted in any subsequent

but not deducted diture is disallowed in the year, theexpenditure (which is

current year disallowed in thre current year)

will be deducted in the year in

which TDS will be deposited by

the assessee with the Government.

Case 2 - Tax is deductible 100 per cent of such ependiture If tax is deposited with the

(and is so deducted) is disallowed in the current year Government after the due date of

during the current financial submission of return of income,

year but it is not deposited the expenditure (which is disal-

on or before the due date lowed in the current year) will be

of submission of return deductible in that year in which

of income under tax will be deposited)

Section 1399(1)

Note - If recipient is non -resident or foreign company and TDS default is committed

by the prayewr, 100 per cent of the expenditure is disallowed in the hands of payer, as

given above. Similar rule is applicable if receipient is a resident and TDS default is

committed (however, not 100 percent but 30 per cent expenditure is disallowed). The

rule pertaining to TDS default (in case receipient is resident) is given by section 40(a) (ia)

Page 124: INTER CA Taxation - CA Study Web€¦ · INTER CA Taxation Head Office Shraddha, 4th Floor, ... 14. Advance Tax & Interest 268 - 271 15. Tax Deducted at Source (TDS) 272 - 294 16

: 121 :

J. K. SHAH CLASSES INTER C.A.- DIRECT TAXES

INCOME FROM BUSINESS & PROFESSION

Any sum payable to a resident [Sec. 40 (a) (ia) ] - In respect of the following paymenta /

credit to a resident, tax is deductible under te Income tax Act (i.e. ,sections 192 to

206AA) -

1. Salary

2. Interest

3. Dividends

4. Winnings from lottery or crossword puzzles

5. Winnings from horse races

6. Payments to contractors

7. Commission or brokerage (including insurnace commission)

8. Payment in respect of life insurance policy

9. Payment in respect of deposits under NSS

10. Payment on account of certain units

11. Rent

12. Payment on purchase of immovable property

13. Technical / professional fees, royalty , fees to a part time director

14. Payment of compensation on acquisition of immovable property

Dissallowance of above expenditure under sectuion 40(a) (ia) - If TDS default

is commited in respect of the above payment / credit given to a resident, 30

percent of such expenditure is disallowed in the hands of payer under section

40(a) (ia) .These provisions aregiven below -

TDS default Is such expenditure Is such expenditure deductible

deductible in the current year in any susbequent previous year

Case 1 - Tax is deductible 30 per cent of such expenditure Is tax is deducted in any ssubsequent

deducted is disallowed in the current year year, the expenduiture (which is

disallowed in the current year) will

be deducted in the year in which TDS

will be deposited by the assrssee with

the Government

Case 2 - Tax is deductoible 30 per cent of such expenditure If tax is deposited with the Govern-

(and is so deducted) during is disallowed in the current year ment after the due date of submission

the current financial year but of return of income, the expenditure

it is not deposited on or before (which is disallowed in the current

the due date of submission of year) will be deductible in that year in

return of income under which tax will be deposited

section 139(1)

When Receipient has paid tax -

Under the provisions, the relief is given in Case 1 (and not in cASe 2). Thie relief is

available if the following conditions conditions are satisfied -

1. Tax is deuductible on the aforesaid payments but it is not deducted by the prayer

(i.e. Case 1)

Page 125: INTER CA Taxation - CA Study Web€¦ · INTER CA Taxation Head Office Shraddha, 4th Floor, ... 14. Advance Tax & Interest 268 - 271 15. Tax Deducted at Source (TDS) 272 - 294 16

: 122 :

J. K. SHAH CLASSES INTER C.A.- DIRECT TAXES

INCOME FROM BUSINESS & PROFESSION

2. The payer is not deemed to be an assessee - in - defalut under the first proviso

to section 201(1). Under the first proviso to section 201(1), the payer is not

deemed to be an assessee -in -deafult if -

(a) The resident recipient has furnished his return of income under section

139;

(b) The resident recipient has taken into account the above income in such

return of income

(c) The resident recipient has paid the tax due on the income declared in

such return income, and

(d) The payer furnishes a certificate [in Form No. 26] to this effect from a

chartered accountant.

If the above conditions are satisfied, then for the pupose of section 40(a)(ia) shall be

deemed that the payer has deducted and paid the tax on such amount on the date of

the furnishing of return of income by the resident recipient.

Tax of non - monetary perquisite paid by the employer [sec. 40(a) (v) ] - the provisions

of section 40(a) (v) are given below-

1. The employer provides non- monetary perquisites to employees

2. Tax on non -monetary perquisites is paid by the employer.

3. The tax so piad by the employer is no taxable in the hands of employees by

virtue of section 10(10CC).

4. While calculating income of the employer, the tax paid by the employer on non-

monetary perquisites is not deductible under section 40(a) (v).

Provisions illustrated

During the previous year 2015-16, Ltd. pays Rs. 40,000 per month as salary to X

(age : 3 years) and provides a rent - free unfurnished house (lease rent being Rs.

10,000 per month). The tax on perquisite is paid by A Ltd. as follows -

Rs.

Salary 4,80,000

Value of perquisite (5 percent of salary) 72,000

Gross : Salary 5,52,000

Les Deduction ---

Net income 5,52,000

Tax on net income (including education cess and

secondary and higher education cess) 36,462

Average rate of tax (Rs. 36,462/Rs. 5,52,000 x 100) : 6.605%

Tax on prequisite (6.605% of Rs. 72,000) 4,756

Total expenditure incurred by A Ltd. in respect of employee X is as follows:

Rs.

Salary to X 4,80,000

Rent - free house to X (Rs. 10,00 x 12) 1,20,000

Tax on perquisite borne by A Ltd. 4,756

Total 6,04,756

Page 126: INTER CA Taxation - CA Study Web€¦ · INTER CA Taxation Head Office Shraddha, 4th Floor, ... 14. Advance Tax & Interest 268 - 271 15. Tax Deducted at Source (TDS) 272 - 294 16

: 123 :

J. K. SHAH CLASSES INTER C.A.- DIRECT TAXES

INCOME FROM BUSINESS & PROFESSION

While calculating business income of A Ltd. Rs. 4,756 is not deductible by virtue of

section 40 (a (v) [amount deductible being Rs. 4,80,000 + Rs. 1,20,000]. In the

hands of X, Rs. 4,756 is not chargeable to tax. The same rule is applicable if tax on

perquisite paid by A Ltd. is lower than Rs. 4,756.If however, tax paid by A Ltd. is

more than Rs. 4,756 then the "excess" amount is deductible in the handas of A Ltd.

and the same is chargeable to tax in the hands of X.

40(a) Income tax is not deductible. Income tax also includes interest paid on delayed

(ii) & (iia) payment of Income tax, Interest paid on delayed filing of income tax and TDS

returns.

40(b) Interest on capital at a rate exceeding 12% p.a. simple interest. (for all

partners)

Salary, bonus, commission or other remuneration to non working partner

(sleeping partner).

Salary, bonus, commission or other remuneration to working partners if

not authorised or covered in partnership deed.

Salary, bonus, commission or other remuneration in relation to a period

prior to the date of partnership deed.

For working partners, excess of remuneration vis a vis the limits prescribed

is disallowed.

LIMIT AGGREGATE REMUNERATION

ALLOWABLE

* On the first 3,00,000 of the book `̀̀̀1,50,000/- or at the rate of 90% of the

profits or in the case of a loss book profit, whichever is more.

* On the balance of book profits 60% of the Balance

* Remuneration allowable u/s 37(1) along with the provisions of

sec. 40(b) would be least of :

Actual aggregate remuneration paid to working partners.

OR

���� Maximum aggregate remuneration allowable as per limits

discussed above.

"Working partner" means a partner who is actively engaged in conducting the

affairs of the business or profession of the firm.

"Book profits" means the net profit as shown in the profit and loss account

for the relevant previous year, computed as per the provisions of Income

from business or profession, and such profits should be increased by the

remuneration paid or payable to all the partners of the firm, if the same

has been deducted while computing the above net profits.

Taxable Income of Firm

Book profit / (loss)

Less : Remuneration allowed

Income / (loss) as per Taxation

Page 127: INTER CA Taxation - CA Study Web€¦ · INTER CA Taxation Head Office Shraddha, 4th Floor, ... 14. Advance Tax & Interest 268 - 271 15. Tax Deducted at Source (TDS) 272 - 294 16

: 124 :

J. K. SHAH CLASSES INTER C.A.- DIRECT TAXES

INCOME FROM BUSINESS & PROFESSION

40 A(2) Amounts not deductible in respect of payment to relatives [Sec. 40A (2)]-

Section 40 A(2) is aj if expenditure is incurred for goods, services or facilities,

payment is made to the persons given below am payment is considered as

excessive or unreasonable having regard to -

(a) the fair market value of the commodity or service or facility; or

(b) Legitimate business needs of the assessee; or

(c) benefit derived by or accruing to assessee as a result of the expediture.

To the extent payment is considered as excessive or unreasonable, it will be

disallowed.

To whom the payment is amde - Section 40A(2) is applicable in the

following cases (list is bot complete only important cases are given) -

1. Payment made by an individau to his or her relative

2. Payment made by a company to a director of the company or any

relative of the director

3. Payment made by a firm/ AOP/HUF to a partner/ member or a

relative of partner / member.

4. Payment made to an individaul who has a substantial interest in

the business of the payer or a relative of such individual

5. Payment made to a company who has a substantial interest in

the business of the payer, any director of such company or

relative of such director.

6. Payment made to a firm /AOP/ HUF who has a substantial inter-

est in the business of the payer or partner/ member of such

person or relative of partner /member.

7. Payment made to a person in whose business the payer/ any

relative of payer ahs s substantial interest.

Relative - "Realtive in relation to an individual, means the husband ,

wife, brother or sister or any lineal ascendantor descendant of that

individual.

Susbstantial intrest -A person is deemed to have susbtantial interets in

the business or profession, if such person is the benefie ownre of at

least 20 percent of equity capital (in any other case) at any time during

the previous year.

40A(3) Amount not deductible in respect of expendture exceeding Rs. 10,000

[Sec. 40A(3)] - The provisions of section 40(A) (3) are given below -

Rule of a disalllowance - Disallowance is attrated under section 40A(3)

if the following conditions are satisfied -

1. The assessee incurs any expenditurte which is otherwise

deductible under the other provisions of the Act for computing business/

profession income (e.g., expenditure for purchase of raw material, trad-

ing goods, expenditure on salary, etc). The amount of expenditure

exceeds ` ` ` ` 10,000.

Page 128: INTER CA Taxation - CA Study Web€¦ · INTER CA Taxation Head Office Shraddha, 4th Floor, ... 14. Advance Tax & Interest 268 - 271 15. Tax Deducted at Source (TDS) 272 - 294 16

: 125 :

J. K. SHAH CLASSES INTER C.A.- DIRECT TAXES

INCOME FROM BUSINESS & PROFESSION

2. A Payment (or aggregate of payments made to a person in a day) in

respect of the above expenditure exceeds `̀̀̀ 10,000.

3. The above payment is made otherwise than by an account payee cheque

or an account payee demand draft.

If all the above conditions are satisfied, then 100 per cent of such

payment will be disallowed.However, the monetary limit of `̀̀̀ 10,000 has

been raised to `̀̀̀ 35,000 in case of payment made for plying, hiring or

leasing goods carriages.

Exceptions - The above rule is not applicable to a afew cases given

below -

1. Payment m,ade to a bank (including private sector banks, co-operative

bank, credit societies), LIC, etc.

2. Payment made of Government.

3. Payment through banking system.

4. Payment made by book adjustment by an assessee in the account of the

payee against money due to the assesee for any goods supplied or

services rendered by him to the payee.

5. Payment made to a cultivator, grower or producer in respect of the pur-

chase of agricultural or forest produce or product of animal husbandry

(including licestock, meat, hides and skins) or diary or poultry farming or

fish or fish products of horticulture or apiculture (even if these products

have been subjected to some processing provided the processing has

been done by the cultivator, grower or the producer of the product).

6. Payment made to a producer in respect of the purchase of the products

manufactured or processed without the aid of power in a cottage

industry.

7. Payment made to a person who ordinarily resides or carries on

business in a village not served by any bank.

8. Payment of terminal benefits, such as gratuity, retrenchment

compensation, etc. not exceeding `̀̀̀ 50,000.

9. Payment made by an assessee by way of salary to his employee after

deducting tax and when such employee is temporarily posted for a con-

tinuous period of 15 days or more in a place other than his normal

place of duty or on a ship and does not maintain any account in any bank

at such place or ship.

10. Payment required to be made on a day on which the banks were closed

either on account of holiday or strike.

11. Payment made by any person to his agent who is required to make pay-

ment in cash for goods or services on behalf of such person.

12. Payment made by an authorized dealer or a money changer against

purchase of foreign currency or travellers cheques in the normal course

of his business.

Page 129: INTER CA Taxation - CA Study Web€¦ · INTER CA Taxation Head Office Shraddha, 4th Floor, ... 14. Advance Tax & Interest 268 - 271 15. Tax Deducted at Source (TDS) 272 - 294 16

: 126 :

J. K. SHAH CLASSES INTER C.A.- DIRECT TAXES

INCOME FROM BUSINESS & PROFESSION

40A(3A) (WHEN OUTSTANDING LIABILITY OF EARLIER YEARS IS PAID IN THE CUR

RENT YEAR [SEC.40A(3A)]- Section 40A (3A) is applicable if an outstanding

liability was allowed as deduction in any of the earlier years and during the

current year payment in respect of such liability is made otherwise than by an

account payee cheque or draft. If such payment to a person in a day exceeds

`̀̀̀ 10,000 ( `̀̀̀35,000 in the case of payment for plying, hiring or leasing goods

carriages), the payment so made shall be chargeable to tax as business

income in the year of payment. This rule will, however, be not applicable in the

exceptions given above.

40A(7) (Amount not deductible is respect of provision for unapproved gratuity fund

[Sec.40A(7)]- Provision for gratuity fund (for meeting future liability) is

deductible only if such gratuity fund is an approved gratuity fund. In other words,

any provision for unapproved gratuity fund (for meeting future liability) is not

deductible.

The following points should be noted –

1. An employee retires during the current year. The employer does not

maintain any gratuity fund. Gratuity is paid to him during the current year.

it is deductible during the current year.

2. An employee retires during the current year. Gratuity is payable to him. A

part of the amount is paid during the current year and the balance will be

paid in the next year. A provision is made towards gratuity in the books

of account of the current year for makingpayment in the next year. The

entire amount is deductible during the current year (if no deduction was

claimed earlier). In this case, deduction is available during the current

year even if provision is made for gratuity fund, which is unapproved.

3. A company has 50 employees. To meet future liability to pay them gratu-

ity at the time of retirement, a gratuity fund is created and the employer

makes contribution every year.Employers’ contribution to this fund id de-

ductible only if the fund is an approved gratuity fund.

40A (Amount not deductible is respect of contributions to non-statutory funds

(9,10,11) [Sec. 40A(9)]- Any sum paid by the assessee as an employer by way of

contribution towards recognised provident fund, or approved superannuation fund

or an approved gratuity fund, is deductible to the extent it is required by any law.

- What is not deductible – If the following conditions are satisfied, then contri-

bution or payment is not deductible by section 40A(9)-

1. The contribution/payment is made by an assessee as an employer.

2. It is paid toward setting up (or formation of) any trust, company, association

of person, body of individuals, society or it is paid by way of contribution to

any fund.

3. The contribution or payment is not required by any law.

Page 130: INTER CA Taxation - CA Study Web€¦ · INTER CA Taxation Head Office Shraddha, 4th Floor, ... 14. Advance Tax & Interest 268 - 271 15. Tax Deducted at Source (TDS) 272 - 294 16

: 127 :

J. K. SHAH CLASSES INTER C.A.- DIRECT TAXES

INCOME FROM BUSINESS & PROFESSION

41 Deemed Profit and their Treatment

The following receipts are chargeable to tax as business income :

Nature of deemed profit Taxable in which PY

1. Recovery against deduction of any loss, PY in which the amount is re

expenditure or trading liability, or remission or covered or liability is remitted

cessation of tradingliabilities. (including or the liability is written back as

“remission or cessation” of any liability arising income upon its cessation

out of a unilateral act of the assessee).

(chargeable in the hands of successor in

business also)

2. Balancing charge on assets on which PY in which the asset is sold

depreciation is charged on SLM basis, or transferred

in case of an undertaking engaged in

generation or generation and distribution

of power.

3. Surplus, (i.e., sale proceeds + deduction PY in which the sale took place

allowed – cost of the asset) arising on transfer

of capital asset used in scientific research or

family planning, or the amount of deduction

allowed, whichever is less

4. Recovery of bad debt allowed as deduction PY in which it is recovered

under section 36(1)(vii)

43CA Full value of consideration for transfer of land or building held as stock in trade

1. Where the consideration received or accruing as a result of the transfer by

an assessee of land or building or both, is less than the stamp value [adopted

or assessed or assessable by any authority of a State Government for the

purpose of payment of stamp duty], such stamp value shall be deemed to

be the full value of the consideration received or accruing as a result of

such transfer.

2. The provisions of section 50C(2) and (3) shall, so far as may be, apply.

3. Where the date of agreement fixing the value of consideration and the date

of registration of such transfer are not the same, and the amount of consid-

eration or a part thereof has been received by any mode other than cash on

or before the date of agreement, the stamp value may be taken as the

value assessable on the date of the agreement.

43B Certain deduction to be allowed only on actual payment :-

(a) Any amount payable to Government for any tax, duty, cess, fee, or any

other payment.

(b) Employer's contribution to provident fund or superannuation fund or

gratuity fund.

(c) Any sum payable as bonus or commission to employees for services rendered.

(d) Interest payable on loan or borrowing taken from Public Financial Institution

or State Financial Corporation or a State Industrial Investment Corporation,

in accordance with the terms and conditions of the agreement governing

such loan or borrowing.

Page 131: INTER CA Taxation - CA Study Web€¦ · INTER CA Taxation Head Office Shraddha, 4th Floor, ... 14. Advance Tax & Interest 268 - 271 15. Tax Deducted at Source (TDS) 272 - 294 16

: 128 :

J. K. SHAH CLASSES INTER C.A.- DIRECT TAXES

INCOME FROM BUSINESS & PROFESSION

(e) Interest on any loan or advance from a scheduled bank in accordance with

the terms and conditions of the agreement governing such loan or

advance.

If such outstanding interest is converted into a loan or advance, it shall

not be deemed to have been actually paid.

(f) Any sum payable by the asessee as an employer in lieu of any leave at

the credit of his employee.

(g) Any payment made to Indian Railways for use of Railway assets.

(h) Interest on loans from Co-opearative banks also allowed on Payment

basis. Co-operative banks will not include 'Primary Agricultural Credit

Society' and 'Priamry Co-operative Agricultural and Rural Develpoment

Bank'

Conditions :

(i) The above items will be allowed only in the year of actual payment.

(ii) This is irrespective of regular accounting method followed by

assessee and irrespective of the year, in which the liability accrues.

(iii) However, if payments are made for the above after the close of

Previous Year, but before due date of filing the return u/s. sec. 139(1), will

be allowed in the previous year in which the liability to pay accrues,

provided the assessee attaches or encloses proof of such payment along

with the return.

44 AA Maintenance of books of account

a) ‘specified profession – specified profession include persons carrying

on legal, medical, engineering or architectural profession or the

profession of accountancy or technical consultancy or interior decoration

or any other profession as is notified by the Board in the Official Gazette.

Authorized representatives, film artists and company secretaries have

been notified for this purpose.

b) ‘non-specified profession’:- a non-specified profession is a profession

other than a ‘specified profession’ mentioned above.

Who shall maintain compulsory books of account? – in order to determine who

shall maintain compulsory books of account, different taxpayers are grouped

into different categories -

Page 132: INTER CA Taxation - CA Study Web€¦ · INTER CA Taxation Head Office Shraddha, 4th Floor, ... 14. Advance Tax & Interest 268 - 271 15. Tax Deducted at Source (TDS) 272 - 294 16

: 129 :

J. K. SHAH CLASSES INTER C.A.- DIRECT TAXES

INCOME FROM BUSINESS & PROFESSION

Rule 6F : the books of account and other documents shall be the following,

namely –

i) A cash book

ii) A journal, if mercantile system of accounting

iii) A ledger

iv) Carbon copies of bills and receipts, whether machine numbered or

otherwise serially numbered, wherever such bills or receipts are issued

by the person, for sums exceeding Rs.25/-

v) Original bills and receipts wherever issued to the person in respect of

expenditure incurred or, where such bills and receipts are not issued

and the expenditure incurred is = < Rs.50/- payment vouchers prepared

and signed by the person unless the cashbook contains adequate

particulars in respect of such expenditure

vi) A person carrying on medical profession shall, in addition to the books

of account and other documents specified above, keep and maintain

the following, namely :-

a) A daily case register in Form No.3C;

b) An inventory under broad heads, as on the first and the last day of

the PY, of the stock of drugs, medicines and other consumable

accessories.

The books of account and other documents specified above shall be kept and

maintained for a period of 6 years from the end of the relevant AY.

44 AB Audit of Books of Accounts

Who has to get his accounts audited on compulsory basis?

Business / Conditions Audit required or not

profession

In case of If Assessee covered by section 44AD/ADA /AE Audit is not required.

business a. If assessee declares minimum income prescribed

in relevant sections

b. If assessee declares less than minimum Audit is required

income prescribed in relevant sections (irrespective of

turnover)

If assessee does not covered by above sections

a. If total sales / turnover / gross receipts exceeds Audit is required

Rs. 1 crore

b. If total sales / turnover / gross receipts does not

exceeds Rs 1 crore Lacs Audit is not required

Page 133: INTER CA Taxation - CA Study Web€¦ · INTER CA Taxation Head Office Shraddha, 4th Floor, ... 14. Advance Tax & Interest 268 - 271 15. Tax Deducted at Source (TDS) 272 - 294 16

: 130 :

J. K. SHAH CLASSES INTER C.A.- DIRECT TAXES

INCOME FROM BUSINESS & PROFESSION

In case of If Assessee covered by section 44ADA

Profession a. If assessee declares minimum income prescribed

in relevant section Audit is not required.

c. If assessee declares less than minimum income

prescribed in relevant section Audit is required

(irrespective of turnover)

If assessee does not covered by above sections

c. If total gross receipts exceeds Rs. 50lacs Audit is required

d. If total sales / turnover / gross receipts does not

exceeds Rs 50 Lacs Audit is not required

Points to be noted

Year-to-year basis

Applicability / non-applicability of section 44AB is to be decided on year-to-year

basis.

More than one business

If assessee carrying on more than one business, aggregate of sales / turnover /

gross receipts of all the business should be considered (i.e. section is applicable

assessee wise not business wise.

Due date for filling of audit report

• Due date = 30th September of relevant assessment year.

• However, if the audit report is submitted on or before due date but return is

filed subsequently, then the following should be submitted along with return -

(i) copy of the tax audit report; and

(ii) the proof of filling the tax audit report.

44AD Special provisions for comput ing profits and gains of business on

presumptive basis

(i) Eligible business:

The presumptive taxation scheme under section 44AD covers all small

businesses with total turnover/gross receipts of up to ̀ 200 lakh (except

the business of plying, hiring and leasing goods carriages covered un-

der section 44AE).

(ii) Eligible assessee: Resident individuals, HUFs and partnership firms (but

not LLPs) and who has not claimed deduction under any of the section

10AA or deduction under any provisions of Chapter VIA under the head-

ing “C.—Deductions in respect of certain incomes” in the relevant as-

sessment year would be covered under this scheme.

(iii) Presumptive rate of tax: The presumptive rate of tax would be 8% of

total turnover or gross receipts. However, the assessee has the option

to declare in his return of income, an amount higher than the presumptive

income so calculated, claimed to have been actually earned by him.

Page 134: INTER CA Taxation - CA Study Web€¦ · INTER CA Taxation Head Office Shraddha, 4th Floor, ... 14. Advance Tax & Interest 268 - 271 15. Tax Deducted at Source (TDS) 272 - 294 16

: 131 :

J. K. SHAH CLASSES INTER C.A.- DIRECT TAXES

INCOME FROM BUSINESS & PROFESSION

Amendment : Persumptive Income reduced to 6% from 8% for turnover

or gross receipts which is received by A/c Payee Cheque /Draft/ ECS

upto due date of thing ROI.

(v) No further deduction would be allowed: All deductions allowable under

sections 30 to 38 shall be deemed to have been allowed in full and no

further deduction shall be allowed. Moreover, expenditure in the nature

of salary, remuneration, interest etc. paid to the partner as per section

40(b) shall not be deductible while computing the income under section

44AD since section 40 does not mandate for allowance of any expendi-

ture; it merely places a restriction on deduction of amounts, otherwise

allowable under section 30 to 38.

(vi) Written down value of the asset: The WDV of any asset of such business

shall be deemed to have been calculated as if the assessee had claimed

and had been actually allowed the deduction in respect of depreciation

for each of the relevant assessment years.

(vii) Relief from maintenance of books of accounts and audit: The intention

of widening the scope of this scheme is to reduce the compliance and

administrative burden on small businessmen and relieve them from the

requirement of maintaining books of account.

Such assessees opting for the presumptive scheme are not required to

maintain books of account under section 44AA or get them audited

under section 44AB.

(viii) Higher threshold for non -audit of accounts for assessees opting for

presumptive taxation under section 44AD: Section 44AB makes it

obligatory for every person carrying on business to get his accounts of

any previous year audited if his total sales, turnover or gross receipts

exceed ` 1 crore. However, if an eligible person opts for presumptive

taxation scheme as per section 44AD(1), he shall not be required to get

his accounts audited if the total turnover or gross receipts of the relevant

previous year does not exceed ̀ 2 crore. The CBDT, has vide its Press

Release dated 20 th June, 2016, clarified that the higher threshold for

non-audit of accounts has been given only to assessees opting for pre-

sumptive taxation scheme under section 44AD.

(xi) Advance tax: Further, since the threshold limit of presumptive taxation

scheme has been enhanced to Rs.2 crore, the eligible assessee is now

required to pay advance tax by 15th March of the financial year.

(x) Persons not eligible for presumptive taxation scheme: The following

persons are specifically excluded from the applicability of the

presumptive provisions of section 44AD -(a) a person carrying on

profession as referred to in section 44AA(1) i.e., legal, medical,

engineering or architectural profession or the profession of accountancy

or technical consultancy or interior decoration or any other profession

as is notified by the Board (namely, authorized representatives, film

artists, company secretaries and profession of information technology

Page 135: INTER CA Taxation - CA Study Web€¦ · INTER CA Taxation Head Office Shraddha, 4th Floor, ... 14. Advance Tax & Interest 268 - 271 15. Tax Deducted at Source (TDS) 272 - 294 16

: 132 :

J. K. SHAH CLASSES INTER C.A.- DIRECT TAXES

INCOME FROM BUSINESS & PROFESSION

have been notified by the Board for this purpose); (b) a person earning

income in the nature of commission or brokerage; or (c) a person

carrying on any agency business.

(xi) Where an eligible assessee declares profit for any previous year in

accordance with the provisions of this section and he declares profit for

any of the five consecutive assessment years relevant to the previous

year succeeding such previous year not in accordance with the

provisions of sub-section (1), he shall not be eligible to claim the benefit

of the provisions of this section for five assessment years subsequent to

the assessment year relevant to the previous year in which the profit has

not been declared in accordance with the provisions of sub-section (1).

This is provided in new sub-section (4).

Example: Let us consider the following particulars relating to a resident

individual, Mr. A, being an eligible assessee whose gross receipts do

not exceed Rs. 2 crore in any of the assessment years between A.Y.2017-

18 to A.Y.2019-20 -

Particulars A.Y.2017-18 A.Y.2018-19 A.Y.2019-20

Gross receipts (Rs.) 1,80,00,000 1,90,00,000 2,00,00,000

Income offered for taxation (Rs.) 14,40,000 15,20,000 12,00,000

% of gross receipts 8% 8% 6%

Offered income as per presumptive

taxation scheme u/s 44AD Yes Yes No

In the above case, Mr.A, an eligible assessee, opts for presumptive

taxation under section 44AD for A.Y.2017-18 and A.Y.2018-19 and of-

fers income of Rs.14.40 lakh and Rs.15.20 lakh on gross receipts of

Rs.1.80 crore and Rs.1.90 crore, respectively. However, for A.Y.2019-

20, he offers income of only Rs.12 lakh on turnover of Rs.2 crore, which

amounts to 6% of his gross receipts. He maintains books of account

under section 44AA and gets the same audited under section 44AB.

Since he has not offered income in accordance with the provisions of

section 44AD(1) for five consecutive assessment years, after A.Y. 2017-

18, he will not be eligible to claim the benefit of section 44AD for next five

assessment years succeeding A.Y.2019-20 i.e., from A.Y.2020-21 to 2024-25.

(xii) An eligible assessee to whom the provisions of sub-section (4) are ap-

plicable and whose total income exceeds the basic exemption limit has

to maintain books of account under section 44AA and get them audited

and furnish a report of such audit under section 44AB. This is provided

in new sub-section (5) of section 44AD.

(xiii) Summary of amendments in section 44AD

- Increase in treshold limit of eligible business from Rs.1 crore to

Rs.2 crore

- Salary, interest, remuneration paid to partner as per section 40(b)

not deductible

- Advance tax to be paid on or before 15th March of the financial year

- In case of non-offering of income as per section 44AD for five

continuous years, eligible assessee cannot opt for section 44AD

for the next five AYs after the assessment year of first non-option

Page 136: INTER CA Taxation - CA Study Web€¦ · INTER CA Taxation Head Office Shraddha, 4th Floor, ... 14. Advance Tax & Interest 268 - 271 15. Tax Deducted at Source (TDS) 272 - 294 16

: 133 :

J. K. SHAH CLASSES INTER C.A.- DIRECT TAXES

INCOME FROM BUSINESS & PROFESSION

44ADA Presumptive Taxation Scheme for assessees engaged in eligible profession

(i) Section 44AD provides for a presumptive taxation scheme for eligible

persons engaged in eligible business in order to reduce compliance

burden of small tax payers.

(ii) For reducing the compliance burden of small tax payers having income

from profession, the Finance Act, 2016 has introduced a presumptive

taxation regime for professionals.

(iii) In this regard, new section 44ADA has been inserted in the Income-tax

Act, 1961 providing a presumptive taxation scheme for estimating the

income of an assessee: · who is engaged in any profession referred to

in section 44AA(1) such as legal, medical, engineering or architectural

profession or the profession of accountancy or technical consultancy or

interior decoration or any other profession as is notified by the Board in

the Official Gazette; and · whose total gross receipts does not exceed

fifty lakh rupees in a previous year, at a sum equal to 50% of the total

gross receipts, or, as the case may be , a sum higher than the aforesaid

sum claimed to have been earned by the assessee. (iv) Eligible Assessee

(iv) Eligible Assessee

Eligible Assessees

Resident assessee engaged in notified Total gross receipts

profession u/s 44AA(1) = Rs.50 lakhs

(v) Under the scheme, the assessee will be deemed to have been allowed

the deductions under section 30 to 38. Accordingly, no further deduction

under those sections shall be allowed. (vi) Further, the written down value

of any asset used for the purpose of the profession of the assessee will

be deemed to have been calculated as if the assessee had claimed and

had actually been allowed the deduction in respect of depreciation for

the relevant assessment years. (vii) The eligible assessee opting for

presumptive taxation scheme will not be required to maintain books of

account under section 44AA(1) and get the accounts audited under

section 44AB in respect of such income unless the assessee claims that:

(a) the profits and gains from the aforesaid profession are lower than

the profits and gains deemed to be his income under section

44ADA(1); and

(b) his income exceeds the maximum amount which is not charge-

able to income-tax.

(viii) Consequential amendment has been made in section 44AB requiring

every person carrying on profession to have his accounts audited by an

accountant before the specified date and furnish audit report by that date

if such persons have claimed lower profits and gains than the deemed

profits under section 44ADA and his total income exceeds the basic

exemption limit.

Page 137: INTER CA Taxation - CA Study Web€¦ · INTER CA Taxation Head Office Shraddha, 4th Floor, ... 14. Advance Tax & Interest 268 - 271 15. Tax Deducted at Source (TDS) 272 - 294 16

: 134 :

J. K. SHAH CLASSES INTER C.A.- DIRECT TAXES

INCOME FROM BUSINESS & PROFESSION

44 AE Special provisions for computing profits and gains of business of

plying, hiring or leasing goods carriages:

(i) Eligible business: This section provides for estimating business

income of an owner of goods carriages from the plying, hire or

leasing of such goods carriages ;

(ii) Eligible assessee: The scheme applies to persons owning not more

than 10 goods vehicles at any time during the previous year ;

(iii) Presumptive Income: The estimated income from each goods

vehicle , whether heavy goods vehicle or other than heavy goods

vehicle, will be deemed to be `̀̀̀ 7,500 for every month or part of a

month during which such vehicle is owned by the assessee for the

previous year. The assessee can also declare a higher amount in his

return of income. In such case, the latter will be considered to be his income;

(iv) All other deduction deemed to be allowed: The assessee will be

deemed to have been allowed the deductions under sections 30 to

38. Accordingly, the written down value of any asset used for the

purpose of the business of the assessee will be deemed to have

been calculated as if the assessee had claimed and had actually

been allowed the deduction in respect of depreciation for each of

the relevant assessment years.

(v) Not requirement to maintain books of accounts and get the accounts

audited: The assessee joining the scheme will not be required to

maintain books of account under section 44AA and get the accounts

audited under section 44AB in respect of such income.

(vi) Option to claim lower profits: An assessee may claim lower profits

and gains than the deemed prof i ts and gains specif i ed in

sub- section (1) of that section subject to the condition that the books

of account and other documents are kept and maintained as required

under sub-section (2) of section 44AA and the assessee gets his

accounts audited and furnishes a report of such audit as required

under section 44AB.

Page 138: INTER CA Taxation - CA Study Web€¦ · INTER CA Taxation Head Office Shraddha, 4th Floor, ... 14. Advance Tax & Interest 268 - 271 15. Tax Deducted at Source (TDS) 272 - 294 16

: 135 :

J. K. SHAH CLASSES INTER C.A. - DIRECT TAXES

INCOME FROM BUSINESS & PROFESSION

Q. 1. Kishore Industries owned six machines which were in use in its business of

manufacturing of plastic goods in March, 2017. Depreciation on those machines

was available as "plant". The written down value of these machines at the end of

previous year relevant to assessment year 2017-18 was `̀̀̀ 6,50,000.

A new plant was bought for `̀̀̀ 6,50,000 on 30th November, 2017. Three of the old

machines were sold on 10th June, 2017 for `̀̀̀ 9,00,000. Rate of depreciation is 15%.

Required :

(i) compute the claim of depreciation for assessment year 2018-19 ;

(ii) capital gains liable to tax for the same assessment year ;

(iii) if Kishore Industries had sold the three machines in June, 2016 for `̀̀̀14,00,000,

will there be any difference in your above working? Explain.

Q. 1.(b) (as modified) May 1999 - (7 Marks)

Q. 2. Determine the previous year in which the expenditure is allowable in the following

cases (TDS is supposed to be deducted with regard to all the payments) :

(i) PQR Ltd., paid interest of `̀̀̀ 2,10,000 to Mr. A, a non - resident, on 16.2.2018

and deducted tax at source on the same date. However the tax was deposited

on 27.6.2018.

Will it make any difference if Mr. A was a resident?

(ii) The company has paid technical fees on 1.1.2018 and no tax has been

deducted at source.

(iii) XYZ Ltd. paid professional fees of 1,00,000 to Mr. X on 12.01.2018 and

deducted tax at source on the same date. Tax was deposited on 27.12.2018.

Q. 3. Dr. Krishna furnishes you the following information :

Income and Expenditure Account for the year ended 31st March, 2018

` `` `` `` `

(i) Rent paid includes rent for his residential accommodation of `̀̀̀ 30,000 (paid

by cheque).

(ii) Hospital equipments (eligible for depreciation @ 15%)

01.04.2017 Opening WDV `̀̀̀ 5,00,000

07.12.2017 Acquired (Cost) `̀̀̀ 2,00,000

(iii) Medic ines consumed include medicines (cost) `̀̀̀10,000 used fo r

Dr. Krishna'a family.

(iv) Rent received relates to a property situated at Mysore (Gross Annual Value).

The municipal tax of `̀̀̀ 2,000 paid in December, 2017 has been included in the

"administrative expenses".

(v) He received `̀̀̀ 5,000 per month as salary from Full Cure Hospital. This has not

been included in the "fee receipts" credited to income and expenditure account.

Compute Dr. Krishna's professional income for the year ended 31.3.2018.

CLASS WORK PROBLEMS

Page 139: INTER CA Taxation - CA Study Web€¦ · INTER CA Taxation Head Office Shraddha, 4th Floor, ... 14. Advance Tax & Interest 268 - 271 15. Tax Deducted at Source (TDS) 272 - 294 16

: 136 :

J. K. SHAH CLASSES INTER C.A. - DIRECT TAXES

INCOME FROM BUSINESS & PROFESSION

Q. 4. X is a business man, following is the P & L for the previous year 2017-18.

Profit & Loss A/c

Particulars `̀̀̀ Particulars `̀̀̀

To Purchase 21,00,000 Sales 75,00,000

To Salaries and Wages 3,00,000 Closing Stock 5,50,000

To Rent and taxes 30,000 I.T. refund

To Commission 20,000 (including interest of `̀̀̀ 2,000) 10,500

To Sundry expenses 50,000

To Income tax 25,000

To Gifts to clients 30,000

To Contribution to approved authority

for approved prog. set by

National Committee 50,000

To Interest outstanding 40,000

To Reserve for bad debts 17,000

To Net Profit 53,98,500

Total 80,60,500 Total 80,60,500

The following additional particulars are furnished.

(1) Salary and wages include :

(a) Bonus to staff `̀̀̀ 50,000.

Paid allowable bonus on or before due date of filing the return `̀̀̀ 27,000.

(b) Contribution to unrecognised provident fund 75,000.

(c) Contribution to approved gratuity fund prior to due date of filing return

but after due date of the fund 80,000.

(d) Entertainment allowance of `̀̀̀15,000 paid to employees.

(2) Outstanding interest includes outstanding bank interest of `̀̀̀ 25,000 on term

loan of a scheduled commercial bank, which is unpaid till the date of filing

the return.

(3) Commission includes `̀̀̀ 15,000 paid to a resident Indian on which tax has

not been deducted at source.

(4) Purchases include goods purchased from a relative for `̀̀̀ 2,00,000 whose

market value is `̀̀̀ 1,50,000/-. Mr. X had paid `̀̀̀ 1,00,000 by account payee

cheque and `̀̀̀ 1,00,000 in cash.

(5) Sundry expenses of `̀̀̀ 50,000 represent cost of computer purchased on

30.9.2017 put to use on 7.10.2017

Compute income from business for the year ended 31.3.2018.

Page 140: INTER CA Taxation - CA Study Web€¦ · INTER CA Taxation Head Office Shraddha, 4th Floor, ... 14. Advance Tax & Interest 268 - 271 15. Tax Deducted at Source (TDS) 272 - 294 16

: 137 :

J. K. SHAH CLASSES INTER C.A. - DIRECT TAXES

INCOME FROM BUSINESS & PROFESSION

Q. 5. Mr. Sivam, a retail trader of Cochin gives the following Trading and Profit and Loss

Account for the year ended 31st March, 2018.

Trading and Profit and Loss Account for the year ended 31.3.2018

` `` `` `` `

To Opening stock 90,000 By Sales 12,11,500

Purchases 10,04,000 Income from UTI 2,400

Gross Profit 3,06,000 Other business receipts 6,100

Closing Stock 1,80,000

14,00,000 14,00,000

To Salary 60,000 Gross Profit b/d 3,06,000

Rent and rates 36,000

Interest on loan 15,000

Depreciation 1,05,000

Printing and Stationery 23,200

Postage and Stationery 1,640

Loss on sale of shares

(Short - term) 8,100

Other general expense 7,060

Net Profit 50,000

3,06,000 3,06,000

Additional Information :

(i) It was found that some stocks were omitted to be included in both the Opening

and Closing Stock, the values of which were :

Opening stock `̀̀̀ 9,000

Closing stock `̀̀̀ 18,000

(ii) Salary includes `̀̀̀ 10,000 paid to his brother, which is unreasonable to the

extent of `̀̀̀ 2,000.

(iii) The whole amount of printing and stationery was paid in cash.

(iv) The depreciation provided in the Profit and Loss Account `̀̀̀1,05,000 was

based on the following information :

The written down value of plant and machinery is `̀̀̀ 4,20,000. A new plant

falling under the same Block of depreciation of 15% was bought on 1.7.2016

for `̀̀̀ 70,000. Two old plants were sold on 1.10.2016 for `̀̀̀ 50,000.

(v) Rent and rates includes sales tax liability of `̀̀̀ 3,400 paid on 7.4.2017.

(vi) Other general expenses include `̀̀̀ 2,000 paid as donation to a Public Charitable

Trust and penalty of `̀̀̀ 3,000 for contravention of sales tax.

You are required to advise Mr. Sivam whether he can offer his business income

under section 44AD i.e. presumptive taxation.

Page 141: INTER CA Taxation - CA Study Web€¦ · INTER CA Taxation Head Office Shraddha, 4th Floor, ... 14. Advance Tax & Interest 268 - 271 15. Tax Deducted at Source (TDS) 272 - 294 16

: 138 :

J. K. SHAH CLASSES INTER C.A. - DIRECT TAXES

INCOME FROM BUSINESS & PROFESSION

Q. 6. Sun Manufacturing Company Limited sets up a factory in the notified backward area in

the state of Telangana on 1-6-2017. It purchased new plant and machinery costing

`̀̀̀ 30 crores on 16-7-2017. This new machinery was put to use on 21-10-2017.

Compute the deductions allowable to the assessee in respect of the purchase of this

machinery under various provisions of the Income tax Act, 1961.

Describe the changes in the following independent cases if :

(a) The assessee was an individual; or

(b) The factory was set up in Maharashtra; or

(c) The cost of the machinery was `̀̀̀ 10 crores.

Q. 7. An assessee acquired a telecom license from T.R.A.I. [Telecom Regulatory

Authority of India] in year 1 for `̀̀̀ 10,00,00,000 (life 10 years) in three independent

situations as under:

- Situations 1 : Pays entire amount in year 1 and starts services in year 1 itself.

- Situation 2 : Acquired license in year 1 but commenced services in year 3

(construction and setting equipment in year 1 & 2).

- Situation3: Acquired license in year 1, starts services in year 1 but paid in

instalments as under :

Year 1 : Paid `̀̀̀ 5,00,00,000

Year 2 : Paid `̀̀̀ 3,00,00,000

Year 3 : Paid `̀̀̀ 2,00,00,000

Compute deduction u/s 35ABB

Q. 8. X & Co., a partnership firm as such, furnishes the following profit and loss

account for the previous year ending 31st March, 2018 :

To Cost of Goods 5,00,000 By Sales 12,60,000

To Other expenses 1,10,000

To Interest to partners on capital 1,50,000

@ 15% P.A.

To Remuneration to working partners 4,00,000

To Net Profit 1,00,000

12,60,000 12,60,000

The other expenses debited include `̀̀̀ 20,000 not allowable under section 37(1) of

the Act.

You are required to compute for the assessment year 2018-19 :

(i) Book profits of the firm

(ii) Permissible remuneration to partners under section 40(b)

(iii) The income of the firm

What difference would it make if the sales would be `̀̀̀ 8,50,000 and all other things

remain unchanged.

Page 142: INTER CA Taxation - CA Study Web€¦ · INTER CA Taxation Head Office Shraddha, 4th Floor, ... 14. Advance Tax & Interest 268 - 271 15. Tax Deducted at Source (TDS) 272 - 294 16

: 139 :

J. K. SHAH CLASSES INTER C.A. - DIRECT TAXES

INCOME FROM BUSINESS & PROFESSION

Q. 9. On April 1, 2017, X Ltd. commences the operation of a cold storage facility in

Andhra Pradesh. The following information is available from the records of

company.

Expenses incurred prior to April 1, 2017

Purchase of land for cold storage facility 50,00,000

Construction cost of cold storage facility 3,00,000

Purchase of know-how for cold storage facility 6,00,000

These expenses are capitalized on March 31, 2017

Expenses incurred during 2017-18

Construction cost of cold storage facility 100,00,000

Purchase of old plant and machinery (from domestic market) 2,00,000

Purchase of new plant and machinery 9,00,000

Profit and loss account for the year 2017-18

Find out the taxable income of X Ltd. for the assessment year 2018-19 on the

assumption that X Ltd. has income from the business of commission agency :

20,15,000 (computed under the provisions of the Income-tax Act).

Q. 10. Mr. Raju, a manufacturer at Chennai, gives the following Manufacturing, Trading

and Profit & Loss Account for the year ended 31.03.2018:

Manufacturing, Trading and Profit & Loss A/c for the year ended 31.03.2018

Particulars Particulars

To Opening Stock 71,000 By Sales 32,00,000

To Purchase of Raw Materials 16,99,000 By Closing stock 2,00,000

To Manufacturing Wages & Expenses 5,70,000

To Gross Profit 10.60.000

34,00,000 34,00,000

To Administrative charges 3,26,000 By Gross Profit 10,60,000

To State VAT penalty 5,000 By Dividend from domestic 15,000

companies

To State VAT paid 1,10,000 By Income from agriculture (net) 1,80,000

To General Expenses 54,000

To Interest to Bank 60,000

To Depreciation 2,00,000

To Net Profit 5,00,000

12,55,000 12,55,000

Page 143: INTER CA Taxation - CA Study Web€¦ · INTER CA Taxation Head Office Shraddha, 4th Floor, ... 14. Advance Tax & Interest 268 - 271 15. Tax Deducted at Source (TDS) 272 - 294 16

: 140 :

J. K. SHAH CLASSES INTER C.A. - DIRECT TAXES

INCOME FROM BUSINESS & PROFESSION

Following are the further information relating to the financial year 2017-18.

(1) Administrative charges include 46,000 paid as commission to brother of

the assessee. The commission amount at the market rate is 36,000.

(2) The assessee pa id 33 ,000 in cash to a t ranspor t ca r r ier on

29.12.2017.This amount is included in manufacturing expenses (Assume that

the provisions relating to TDS are not applicable to this payment.)

(3) Bank term loan interest actually paid upto 31.03.2018 was 20,000 and

the balance was paid in Dec. 2018.

(4) Depreciation allowable under the act is to be computed on the basis of the

following information.

Plant & Machinery (depreciation rate @ 15%)

Opening WDV as on 01.4.2017 12,00,000

Additions during the year (used for more than 180 days) 2,00,000

Total additions during the year 4,00,000

Compute the business income of Mr. Raju for the assessment year 2018-19.

Q. 11.Sai Ltd. has a block of assets carrying 15% rate of depreciation, whose written down

value on 01.04.2017 was 40 lacs. It purchased another asset (second-hand plant

and machinery) of the same block on 01.11.2017 for 14.40 lacs and put to use on the

same day. Sai Ltd. was amalgamated with Shirdi Ltd with effect from 01.01. 2018

You are required to compute the depreciation allowable to Sai Ltd. & Shirdi Ltd. for the

previous year ended on 31.03.2018 assuming that the assets were transferred to Shirdi

Ltd. at 60 lacs.

Q. 12. Mr. Y carries on his own business. An analysis of his trading and profit & loss for the year

ended 31-3-2017 revealed the following information :

(1) The net profit was ` ` ` ` 11,20,000.

(2) The following incomes were credited in the profit and loss account:

(a) Dividend from UTI ` ` ` ` 22,000.

(b) Interest on debentures ` ` ` ` 17,500.

(c) Dividend From Indian Companies ` ` ` ` 15,000.

(3) It was found that some stocks were omitted to be included in both the opening and

closing stocks, the value of which were:

Opening stock ` ` ` ` 8,000.

Closing stock ` ` ` ` 12,000.

(4) ` ` ` ` 1,00,000 was debited in the profit and loss account, being contribution to a

University approved and notified under section 35(1)(ii).

(5) Salary includes ‘ 20,000 paid to his brother which is unreasonable to the extent of

` ` ` ` 2,500.

(6) Advertisement expenses include 15 gift packets of dry fruits costing ` ` ` ` 1,000 per

packet presented to important customers.

Page 144: INTER CA Taxation - CA Study Web€¦ · INTER CA Taxation Head Office Shraddha, 4th Floor, ... 14. Advance Tax & Interest 268 - 271 15. Tax Deducted at Source (TDS) 272 - 294 16

: 141 :

J. K. SHAH CLASSES INTER C.A. - DIRECT TAXES

INCOME FROM BUSINESS & PROFESSION

(7) Total expenses on car was ` ` ` ` 78,000. The car was used both for business and

personal purposes. ¾th is for business purposes.

(8) Miscellaneous expenses included `̀̀̀ 30,000 paid to A &Co., a goods transport

operator in cash on 31-1-2017for distribution of the company’s product to the

warehouses.

(9) Depreciation debited in the books was ` ` ` ` 55,000. Depreciation allowed as per

Income-tax Rules, 1962 was ` ` ` ` 50,000.

(10) Drawings ` ` ` ` 10,000.

(11) Investment in NSC ̀̀̀̀ 15,000.

(12) Contribution to PPF – 10,000

Compute the total income of Mr. Y for the assessment year 2017-18.

Q. 13. Mr. Vidyasagar, a resident individual aged 64, is a partner in Oscar Musicals & Co., a

partnership firm. He also runs a wholesale business in medical products. The following

details are made available for the year ended 31.3.2018:

Sr No. Particulars ` `` `` `` `

1. Interest on capital received from Oscar Musicals & Co.,at 15% 1,50,000

2. Interest from bank on fixed deposit (Net of TDS `̀̀̀ 1,500) 13,500

3. Income-tax refund received relating to assessment year

2016-17 including interest of `̀̀̀2,300 34,500

4. Net profit from wholesale business 5,60,000

Amounts debited include the following:

Depreciation as per books 34,000

Motor car expenses 40,000

Municipal taxes for the shop

(For two half years; payment for one half year made on

12.6.2018 and for the other on 14.11.2018) 7,000

Salary to manager by way of a single cash payment 21,000

5. The WDV of the assets (as on 1.4.2017) used in above wholesale

business is as under:

Computers 1,20,000

Motor car (20% used for personal use) 3,20,000

6. LIP paid for major son 60,000 60,000

PPF of his wife 70,000

Compute the total income of the assessee for the assessment year 2018-19. The

computation should show the proper heads of income. Also compute the WDV of the different

blocks of assets as on 31.3.2018.

Page 145: INTER CA Taxation - CA Study Web€¦ · INTER CA Taxation Head Office Shraddha, 4th Floor, ... 14. Advance Tax & Interest 268 - 271 15. Tax Deducted at Source (TDS) 272 - 294 16

: 142 :

J. K. SHAH CLASSES INTER C.A. - DIRECT TAXES

INCOME FROM BUSINESS & PROFESSION

Q. 14. Ms Deepika, a resident individual aged 50, provides the following information for the

FY 2017 – 18

(1) She is a partner in SK & Co and received the following amounts from the firm:

Share of profit from the firm Rs 35,000

Interest on capital @ 15% p.a Rs 3,00,000

Salary as working partner Rs 1,00,000

(Fully allowed in the hands of the firm)

(2) She is running a rice mills as proprietor. The Net profit as per Profit and Loss

account is Rs 4,50,000. The following items are debited to Profit and Loss

account:

- Income tax Paid Rs 1,00,000

- Personal Drawings Rs 50,000

The following items are credited to profit and loss account:

- Interest on savings bank account with SBI Rs 12,000

- Interest on savings account with post office Rs 5,000

- Dividend from Indian Company Rs 80,000

(3) She owned a house property in Mumbai which was sold in January 2016. She

received Rs 90,000 by a way of arrear rent in respect of the said property in

October 2017

(4) She made the following investments

Life Insurance Premium on a policy in the name of her married daughter Rs

60,000 (The policy was taken on 1.10.2015 and sum assured being Rs 5,00,000)

Health Insurance premium on a policy covering her mother aged 75, She is not

dependant on Ms Deepika. Premium paid by cheque Rs 35,000

Compute the total Income for AY 2018-2019.

Q. 15. Mr. Rajiv, aged 50 years, a resident individual and practicing Chartered Accountant,

furnishes you the receipts and payments account for the financial year 2017-18.

Receipts and Payments Account

Receipts `̀̀̀ Payment `̀̀̀

Opening balance (1.4.2017) Staff salary, bonus and stipend to 21,50,000

Cash on hand and at Bank 12,000 articled clerks

Fee from professional Services 59,38,000 Other administrative expenses 11,48,000

Rent 50,000 Office rent 30,000

Motor car loan from Canara Bank 2,50,000 Housing loan repaid to SBI 1,88,000

(@ 9% p.a.) (includes interest of `̀̀̀ 88,000)

Life insurance premium 24,000

Motor car (acquired in Jan. 2018) 4,25,000

Medical insurance premium

(for self and wife) 18,000

Books bought (annualpublications) 20,000

Computer acquired on1.11.2017

(for professional use) 30,000

Domestic drawings 2,72,000

Public provident fund subscription 20,000

Motor car maintenance 10,000

Closing balance (31.3.2018)

Cash in Hand and Bank 19,15,000

Total 62,50,000 Total 62,50,000

Page 146: INTER CA Taxation - CA Study Web€¦ · INTER CA Taxation Head Office Shraddha, 4th Floor, ... 14. Advance Tax & Interest 268 - 271 15. Tax Deducted at Source (TDS) 272 - 294 16

: 143 :

J. K. SHAH CLASSES INTER C.A. - DIRECT TAXES

INCOME FROM BUSINESS & PROFESSION

Following further information is given to you:

(1) He occupies 50% of the building for own residence and let out the balance for

residential use at a monthly rent of `̀̀̀ 5,000. The building was constructed during the

year 1997-98, when the housing loan was taken.

(2) Motor car was put to use both for official and personal purpose. One-fifth of the

motor car use is for personal purpose. No car loan interest was paid during the year.

(3) The written down value of assets as on 1-4-2017 are given below:

Furniture & Fittings `̀̀̀ 60,000

Plant & Machinery `̀̀̀ 80,000

(Air-conditioners, Photocopiers, etc.)

Computers `̀̀̀ 50,000

Note: Mr. Rajiv follows regularly the cash system of accounting.

Compute the total income of Mr. Rajiv for the assessment year 2018-19.

Q. 16. Mr. Sukhvinder is engaged in the business of plying goods carriages. On 1st April, 2017,

he owns 10 trucks (out of which 6 are heavy goods vehicles). On 2nd May, 2017, he sold

one of the heavy goods vehicles and purchased a light goods vehicle on 6th May, 2017.

This new vehicle could however be put to use only on 15th June, 2017.

Compute the total income of Mr. Sukhvinder for the assessment year 2018-19, taking note

of the following data:

Particulars `̀̀̀ `̀̀̀

Freight charges collected 12,70,000

Less : Operational expenses 6,25,000

Depreciation as per section 32 1,85,000

Other office expenses 15,000 8,25,000

Net Profit 4,45,000

Other business and non- business income 70,000

Page 147: INTER CA Taxation - CA Study Web€¦ · INTER CA Taxation Head Office Shraddha, 4th Floor, ... 14. Advance Tax & Interest 268 - 271 15. Tax Deducted at Source (TDS) 272 - 294 16

: 144 :

J. K. SHAH CLASSES INTER C.A. - DIRECT TAXES

INCOME FROM BUSINESS & PROFESSION

Ans. 2.(i) The tax has been deducted in February 2018 & deposited with the Central Gov-

ernment before the due date for filing return of income for P.Y. 2017-2018. Hence,

the expenditure shall be allowed as a deduction in P.Y. 2017-2018, whether Mr. A

is a resident or non-resident.

(ii) Since tax is not deducted at source by the payer, the expenditure shall be disal-

lowed fully (if the receiver is a non-resident person) or disallowed to the extent of

30% (if the receiver is a resident person). Subsequently, if tax is deducted by the

payer, the expenditure shall be allowed as a deduction in that financial year in

which the TDS is deposited with the government.

If tax is not deducted by the payer but the resident receiver has paid the income

tax on that amount to the government, then it shall be deemed that the payer has

deducted and deposited TDS on the date of filing return of income by the receiver.

Accordingly the expenditure shall be allowed as a deduction in P.Y. 2017-18.

(iii) Since tax is deducted but deposited after due date of filing return, the expendi-

ture shall be disallowed fully (if the receiver is a NR) or disallowed to the extent

of 30% (if the receiver is a resident). Subsequently if tax is deposited, the ex-

penditure shall be allowed as a deduction in that financial year in which the TDS

is deposited with the government. In the given case 70% is allowed in PY 2017 –

18 and 30% in PY 2018 – 19 (if receiver is resident) and 100% in PY 2018 – 19

(if receiver is NR).

Ans. 4. Mr.X

Computation of income from business for A.Y 18-19

Particulars

Net profit as per P&L Account 53,98,500

Add: Expenses debited to P&L account but disallowed

1) Provision for bad debts 17,000

2) Income tax [Sec 40(a)] 25,000

3) Unpaid Bonus [50,000-27,000] 23,000

4) Outstanding interest 40,000

5) Contribution to URPF 75,000

6) Cost of computer 50,000

7) Unreasonable payment to relative for goods [working] 50,000

8) Cash payment for goods [working] 50,000 3,30,000

Less:Income credited to P&L a/c but exempt/taxable under

different heads

1) Income tax refund, not taxable. 8,500

2) Interest on IT refund, taxable as IFOS 2,000 (10,500)

Less: Expenses not debited to P&L a/c but allowed

1) Interest on loan [40,000-25,000] (Note) 15,000

2) Depreciation u/s 32 on computer [50,000 X 60% X ½] 15,000 (30,000)

Taxable Business Income 56,88,000

CLASS WORK SOLUTIONS

Page 148: INTER CA Taxation - CA Study Web€¦ · INTER CA Taxation Head Office Shraddha, 4th Floor, ... 14. Advance Tax & Interest 268 - 271 15. Tax Deducted at Source (TDS) 272 - 294 16

: 145 :

J. K. SHAH CLASSES INTER C.A. - DIRECT TAXES

INCOME FROM BUSINESS & PROFESSION

Notes & Assumptions

1) It has been assumed that the interest of 15,000 is on loans from private

parties & hence deduction of this amount has been allowed on PAYABLE basis.

2) It has been assumed that the resident receiver has paid income tax on the

commission of 15,000 & therefore deduction of this amount has been allowed

to Mr. X even if tax was not deducted at source by him.

Working for adjustment

Cost of goods = 2,00,000

Market Value = 1,50,000 Unreasonable/Excess 50,000

Account Payee cheque Cash 50,000 Cash

1,00,000

Disallowed u/s. 40A(2)

Allowed Disallowed u/s. 40A(3)

Ans. 5. Mr. Sivam

Computation of Income from Business (on actual basis) for A.Y. 2018-2019

Particulars ` `` `` `` `

Net Profit as per P & L A/c. 50,000

Add: Expenses debited to P & L A/c. but disallowed

1 Depreciation as per books 1,05,000

2 Unreasonable salary to brother [Section 40A(2)] 2,000

3 Printing and Stationery, assumed to be paid in

cash on a SINGLE DAY [Section 40A(3)] 23,200

4 Loss on sale of shares 8,100

5 Penalty for contravention of sales tax 3,000

6 Donation to Public Charitable trust,

allowed u/s. 80G from GTI 2,000 1,43,300

Add: Income not credited to P & L A/c. but taxable

Omission of closing stock 18,000

Less: Incomes credited to P & L A/c. but exempt / not taxable

under this head

Income from UTI (2,400)

Less:Expenses not debited to P & L A/c. but allowed

1 Depreciation u/s. 32

Opening WDV 4,20,000

Add: Addition on 1-7-2017 70,000

Less: Sale on 1-10-2017 (50,000)

Qualifying Amount 4,40,000

x Block rate 15% 66,000

2 Omission of Opening Stock 9,000 (75,000)

INCOME FROM BUSINESS (actual basis) 1,33,900

Page 149: INTER CA Taxation - CA Study Web€¦ · INTER CA Taxation Head Office Shraddha, 4th Floor, ... 14. Advance Tax & Interest 268 - 271 15. Tax Deducted at Source (TDS) 272 - 294 16

: 146 :

J. K. SHAH CLASSES INTER C.A. - DIRECT TAXES

INCOME FROM BUSINESS & PROFESSION

Computation of Income from Business (on presumptive basis) for A.Y. 2018-2019

Particulars `̀̀̀

Sales 12,11,500

Add : Other business receipts 6,100

Turnover / Gross receipts 12,17,600

x Rate of profit u/s. 44AD 8.00%

INCOME FROM BUSINESS (presumptive basis) 97,408

Conclusion :

Since the income calculated on actual basis is greater than the income on

presumptive basis, `̀̀̀ 1,33,900 i.e. income on actual basis shall be treated as the

taxable income from business.

If the income on actual basis would have been lower than the income on presumptive

basis and the assessee wishes to declare the lower amount as his income, it is

possible but the provisions of Section 44AA- Compulsory maintenance of books and

Section 44AB- Compulsory tax audit shall be applicable to him.

Page 150: INTER CA Taxation - CA Study Web€¦ · INTER CA Taxation Head Office Shraddha, 4th Floor, ... 14. Advance Tax & Interest 268 - 271 15. Tax Deducted at Source (TDS) 272 - 294 16

: 147 :

J. K. SHAH CLASSES INTER C.A. - DIRECT TAXES

INCOME FROM BUSINESS & PROFESSION

Ans.8. X & Co. a partnership firm

Before changes

(i) Book profits of the firm

Net profit as per profit & loss A/c 1,00,000

Add :

� disallowed expenses u/s 37(1) 20,000

� interest disallowed u/s 40(b) 30,000

in excess of 12% P.A.

1,50,000 * 3 (15 - 12)

15

� Remuneration to working partners 4,00,000 4,50,000

5,50,000

Less : NIL

Book profits 5,50,000

(ii) Permissible remuneration to partners u/s 37(1) read with u/s 40(b)

Remuneration allowable to working partners as per partnership deed u/s 37(1)

read with (r.w.) 40(b) is Least of :

� Actual remuneration paid 4,00,000

OR

� Maximum allowable as per section 40(b)

* upto Rs.3,00,000 of book profits

- 90% 2,70,000 or 1,50,000 (higher)

* above Rs.3,00,000 @ 60%

i.e. 2,50,000 @ 60% = 1,50,000

(5,50,000 - 3,00,000)

(2,70,000 + 1,50,000) : 4,20,000

(iii) The income of the firm :

Book profits as above 5,50,000

(-) allowable remuneration u/s 37(1)

r. w. 40(b) (4,00,000)

Taxable profits 1,50,000

After changes

(i) Book profits of the firm

Net profit as per profit & loss A/c (loss) (3,10,000)

Add disallowed expenses u/s 37(1) 20,000

interest disallowed u/s 40(b) 30,000

Remuneration to working partner 4,00,000 4,50,000

Book profits 1,40,000

(ii) Permissible remuneration to partners u/s 37(1) r. w. u/s 40(b) which is least of :

Actual remuneration paid 4,00,000

OR

Maximum allowable as per section 40(b)

* upto `̀̀̀1,40,000 of book profits

- 90% or `̀̀̀ 1,50,000 (higher) 1,50,000

(iii) The income of the firm

Book profits as above 1,40,000

(-) allowable remuneration (1,50,000)

u/s 37(1) r. w. 40(b)

Loss of firm (10,000)

Page 151: INTER CA Taxation - CA Study Web€¦ · INTER CA Taxation Head Office Shraddha, 4th Floor, ... 14. Advance Tax & Interest 268 - 271 15. Tax Deducted at Source (TDS) 272 - 294 16

: 148 :

J. K. SHAH CLASSES INTER C.A. - DIRECT TAXES

INCOME FROM BUSINESS & PROFESSION

Ans. 9. Amount Deductible under Section 35AD

Particulars `̀̀̀

A. Amount deductible under section 35AD

1. Expenditure incurred prior to commencement

of operation (to the extent they are capitalized)

� Purchase of land (not qualified for deduction) Nil

� Construction cost of cold storage facility 3,00,000

� Purchase of know-how 6,00,000

2. Expenditure incurred during P.Y. 2016-2017

� Construction cost of cold storage facility 100,00,000

� Purchase of machinery (`̀̀̀2,00,000 + `̀̀̀9,00,000) 11,00,000

Eligible capital expenditure 1,20,00,000

Deduction under section 35AD 1,20,00,000

B. Computation of Income from cold storage facility

Net Profit as per P & L A/c 81,70,000

Add : Depreciation / Amortisation as per books

(i) Building 3,15,000

(ii) Machinery 2,64,000

(iii) Know-how 3,00,000 8,79,000

Less : Deduction u/s 35AD (1,20,00,000)

LOSS FROM COLD STORAGE FACILITY (29,51,000)

Computation of income

`̀̀̀ `̀̀̀

Commission agency 20,15,000 20,15,000

Warehouse Loss c/f u/s 73A (29,51,000)

Business income [loss from operating cold storage facility, being a

specified business under section 35AD cannot be set off against

any other income except income from any specified business of

future, without any time limit]

Total Income 20,15,000

Notes –

1. The business of operating cold storage facility is formed by using new

machinery of `̀̀̀ 9,00,000 and old machinery of `̀̀̀ 2,00,000. Value of old plant and

machinery does not exceed 20% of the total value of plant and machinery. Other

conditions of section 35AD are satisfied. X Ltd. is, therefore, eligible for

deduction for the total value of Plant and Machinery under section 35AD.

2. Loss from operating cold storage facility (by virtue of section 73A) can be set off

only against profit and gains, if any, of any other specified business under section

35AD. In this case, X Ltd. does not have any other specified business. Loss will be

carried forward (without any time-limit) for being set off against income from

operating cold storage facility or any other specified business under section 35AD.

Page 152: INTER CA Taxation - CA Study Web€¦ · INTER CA Taxation Head Office Shraddha, 4th Floor, ... 14. Advance Tax & Interest 268 - 271 15. Tax Deducted at Source (TDS) 272 - 294 16

: 149 :

J. K. SHAH CLASSES INTER C.A. - DIRECT TAXES

INCOME FROM BUSINESS & PROFESSION

Ans. 10.Computation of income from business of Mr. Raju for the A. Y. 2018 – 19

Particulars ` `` `` `` `

Net profit as per the and loss account 5,00,000

Add: 1.Excess commission paid to brother disallowed 10,000

under Section 40A (2)

2. Disallowance under section 40A (3) is not attracted NIL

since the limit for the one time cash payment is

`̀̀̀ 35,000 in respect of payment to transport

operators. Therefore, amount of `̀̀̀ 33,000 paid in

cash to transport carrier is allowable as deduction.

3. Bank term loan interest paid after the due date of filling

of return- disallowed as per section 43B 40,000

4. State VAT penalty paid disallowed 5,000

5. Depreciation debited to profit and loss account 2,00,000 2,55,000

Less: 1. Dividend from domestic companies, exempt 15,000

2.Income from agriculture[ Exempt under section 10(1)] 1,80,000

3.Depreciation u/s 32 [ W.N.] 2,85,000 (4,80,000)

Taxable business Income 2,75,000

Working note :

Computation of depreciation u/s 32

Particulars `̀̀̀

Depreciation @ 15% on `̀̀̀ 14 lakh (Opening WDV of `̀̀̀ 12 lakh plus assets

Purchased during the year and used for more than 180 days `̀̀̀ 2 lakh) 2,10,000

Depreciation (half) @ 15% on `̀̀̀ 2 lakh (Cost of assets used for < 180 days) 15,000

Additional Depreciation ( 2,00,000 x 20% ) + ( 2,00,000 x 20% x ½) 60,000

2,85,000

Ans. 11.Statement showing computation of depreciation aiiowable to Sai Ltd. & Shirdi

Ltd. for A.Y. 2018-19

Particulars `̀̀̀

Written down value (WDV) as on 1.4.2017 40,00,000

Addition during the year (used for less than 180 days) 14,40,000

Total 54,40,000

Depreciation on `̀̀̀ 40,00,000 @ 15% 6,00,000

Depreciation on `̀̀̀ 14,40,000 @ 7.5% 1,08,000

Total depreciation for the year 7,08,000

(a) Apportionment between two companies:

Amalgamating company, Sai Ltd.

`̀̀̀ 6,00,000 x 275/365 4,52,055

`̀̀̀ 1,08,000 x 61/151 43,629

4,95,684

(b) Amalgamated company, Shirdi Ltd.

`̀̀̀ 6,00,000 x 90/365 1,47,945

`̀̀̀ 1,08,000 x 90/151 64,371

2,12,316

Page 153: INTER CA Taxation - CA Study Web€¦ · INTER CA Taxation Head Office Shraddha, 4th Floor, ... 14. Advance Tax & Interest 268 - 271 15. Tax Deducted at Source (TDS) 272 - 294 16

: 150 :

J. K. SHAH CLASSES INTER C.A. - DIRECT TAXES

INCOME FROM BUSINESS & PROFESSION

Notes:

(1) The aggregate deduction, in respect of depreciation allowable to the amalga-

mating company and amalgamated company in the case of amalgamation shall

not exceed, in any case, the deduction calculated at the prescribed rates as if

the amalgamation had not taken place. Such deduction shall be apportioned

between the amalgamating company and the amalgamated company in the ra-

tio of the number of days for which the assets were used by them.

(2) The price at which the assets were transferred i.e. rs. 60,00,000/- has no impli-

cation in computing eligible depreciation.

Ans. 16. Section 44AE would apply in the case of Mr. Sukhvinder since he is engaged in the

business of plying goods carriages and owns not more than ten goods carriages at

any time during the previous year.

Section 44AE provides for computation of business income of such assessees on

a presumptive basis. The income shall be deemed to be ̀ ̀ ̀ ̀ 7,500 from each goods

carriage (whether it is heavy or light vehicle) - for every month or part the month

during which such carriage vehicle is owned by the assessee in the previous year or

such higher sum as declared by the assessee in his return of income.

Mr. Sukhvinder’s business income calculated applying the provisions of section 44AE

is ̀ ̀ ̀ ̀ 9,07,500 (See Notes 1 & 2 below) and his total income would be ` ̀` ` 9,77,500.

However, as per section 44AE(7), Mr. Sukhvinder may claim lower profits and gains

if he keeps and maintains proper books of account as per section 44AA and gets

the same audited and furnishes a report of such audit as required under section

44AB. If he does so, then his income for tax purposes from goods carriages would

be ` ` ` ` 4,45,000 instead of ` ` ` ` 9,07,500 and his total income would be ` ` ` ` 5,15,000.

Computation of total income of Mr Sukhvinder for AY 2018 – 2019

Particulars Presumptive Where books are

Income maintained

Income from business of plying goods carriages 9,07,500 4,45,000

Other income 70,000 70,000

Total Income 9,77,500 5,15,000

Calculation of presumptive income as per section 44AE

Type of carriage No. of months Rate Amount

per month

9 goods carriage – held throughout the year 12 7,500 8,10,000

1 goods carriage – held upto 2nd May 2 7,500 15,000

1 goods carriage – held from 6th May 11 7,500 82,500

Total 9,07,500

Page 154: INTER CA Taxation - CA Study Web€¦ · INTER CA Taxation Head Office Shraddha, 4th Floor, ... 14. Advance Tax & Interest 268 - 271 15. Tax Deducted at Source (TDS) 272 - 294 16

: 151 :

J. K. SHAH CLASSES INTER C.A. - DIRECT TAXES

AGRICULTURE INCOME

Introduction

Section 10(1) exempts agricultural income from tax and also provides for its exclusion

incomputing the total income of the assessee. The reason of exemption of agricultural income

from Central taxation is that the Constitution gives exclusive power to make laws with respect

to taxes on agricultural income to the State Legislatures. From the assessment year 1974-75,

agriculture income is, however taken into account to determine tax on non-agricultural income

in certain cases. This Chapter explains the meaning of agricultural Income and mode of

aggregation of agricultural income with non- agricultural income to determine tax incidence

on the latter.

What is the scheme of partial integration of non-agricultural income with agricultural

income.

The scheme of partial integration of non-agricultural income with agricultural income is applicable

if the following conditions are satisfied-

Condition 1 – The taxpayer is an person liable to pay tax as per slab rates.

Condition 2 – Agricultural income of the taxpayer exceeds Rs 5,000

Condtion 3 – The taxpayer non agricultural income exceeding the basic exemption limit

Computation of tax in cases covered by the scheme

Step 1: Add non-agricultural income with net agricultural income. Compute tax on the

aggregate amount.

Step 2: Add net agricultural income and the maximum exemption limit available to the

assessee (i.e., ` ` ` ` 2,50,000 / ` ` ` ` 3,00,000/ ` ` ` ` 5,00,000). Compute tax on the

aggregate amount.

Step 3: Deduct the amount of income tax calculated in step 2 from the income tax

calculated in step 1 i.e., Step 1 – Step 2.

Step 4: The sum so arrived at shall be increased by surcharge, if applicable. It would be

reduced by the rebate if any available u/s 87A.

Step 5: Thereafter, it would be increase by education cess @2% and secondary and

higher education cess @1%.

Definition of agricultural income [Section 2(1A)]: This definition is very wide and covers

the income of not only the cultivators but also the land holders who might have rented out the

lands. Agricultural income may be received in cash or in kind.

Three ways: Agricultural income may arise in any one of the following three ways:-

(1) It may be rent or revenue derived from land situated in India and used for agricultural

purposes.

(2) It may be income derived from such land through agriculture or the performance of a

process ordinarily employed by a cultivator or receiver of rent in kind to render the

produce fit to be taken to the market or through the sale of such agricultural produce in

the market.

(3) Lastly, agricultural income may be derived from any farm building required for agricultural

operations.

AGRICULTURE INCOME

Page 155: INTER CA Taxation - CA Study Web€¦ · INTER CA Taxation Head Office Shraddha, 4th Floor, ... 14. Advance Tax & Interest 268 - 271 15. Tax Deducted at Source (TDS) 272 - 294 16

: 152 :

J. K. SHAH CLASSES INTER C.A. - DIRECT TAXES

AGRICULTURE INCOME

Now let us take a critical look at the following aspects:

(1) Land has to be situated in India - If agricultural lands are situated in a foreign State,

the entire income would be taxable.

(2) “Agriculture” and “agricultural purposes” - These terms have not been defined in

the Act. However, cultivation of a field involving expenditure of human skill and labour

on the land can be broadly termed as agriculture.

(a) “Agriculture” means tilling of the land, sowing of the seeds and similar operations.

These are basic operations and require the expenditure of human skill and labour

on land itself. Those operations which the agriculturists have to resort to and

which are absolutely necessary for the purpose of effectively raising produce

from the land are the basic operations.

(b) Operations to be performed after the produce sprouts from the land (e.g.,

weeding, digging etc.) are subsequent operations. These subsequent operations

would be agricultural operations only when taken in conjunction with and as a

continuation of the basic operations. Simply performing these subsequent

operations without raising such products is not characterized as agriculture.

(c) “Agriculture” comprises within its scope the basic as well as the subsidiary

operations regardless of the nature of the produce raised on the land. These

produce may be grain, fruits or vegetables necessary for sustenance of human

beings including plantation and groves or grass or pasture for consumption of

beasts or articles of luxury such as betel, coffee, tea, spices, tobacco or

commercial crops like cotton flax, jute hemp and indigo. The term comprises of

products of land having some utility either for consumption or for trade and

commerce and would include forest products such as sal, tendu leaves etc.

(d) However, the term ‘agriculture’ cannot be extended to all activities which have

some distant relation to land like dairy farming, breeding and rearing of live stock,

butter and cheese making and poultry farming. This aspect is discussed in detail

later on.

(3) Process ordinarily employed - The process to which the agricultural produce is

subject should be a process which is ordinarily employed by a cultivator. It may be

manual or mechanical. However, it must be employed to render the produce fit to be

taken to the market. For example, before making rice fit to be taken to the market we

have to remove the basic grain from the hay, we have to remove the chaff from the

grain, we have to properly filter them, we have to remove stones etc. and we have to

pack the grain in gunny bags. In that condition alone the rice can be taken to the market

and sold. This process of making the rice ready for the market may involve manual

operations or mechanical operations. All these operations constitute the process

ordinarily employed to make the product fit for the market. The produce must retain its

original character in spite of the processing unless there is no market for selling it in

that condition.

Page 156: INTER CA Taxation - CA Study Web€¦ · INTER CA Taxation Head Office Shraddha, 4th Floor, ... 14. Advance Tax & Interest 268 - 271 15. Tax Deducted at Source (TDS) 272 - 294 16

: 153 :

J. K. SHAH CLASSES INTER C.A. - DIRECT TAXES

AGRICULTURE INCOME

(4) Income from farm building - Income from any farm building which satisfies the

following conditions would be agricultural income and would consequently be exempt

from tax. Income derived from any such building arising from any other use (other than

those discussed below) shall not be agricultural income.

(a) The building should be on or in the immediate vicinity of the agricultural land;

(b) It should be owned and occupied by the receiver of the rent or revenue of any

such land or occupied by the cultivator or the receiver of rent in kind of any land

with respect to which land or the produce of which land the process discussed

above is carried on;

(c) The receiver of the rent or revenue or the cultivator or the receiver of rent in kind

should, by reason of his connection with such land require it as a dwelling house

or other out building.

In addition to the above three conditions any one of the following two conditions should

also be satisfied:

(i) The land should either be assessed to land revenue in India or be subject to a

local rate assessed and collected by the officers of the Government as such or;

(ii) Where the land is not so assessed to land revenue in India or is not subject to

local rate:-

(a) It should not be situated in any area as comprised within the jurisdiction

of a municipality or a cantonment board and which has a population not

less than 10,000.

(b) It should not be situated in any area within such distance, measured

aerially, in relation to the range of population according to the last

preceding census as shown hereunder –

Shortest aerial distance from Population according to the last

the local limits of a municipality preceding census of which the

or cantonment board referred relevant figures have been published

to in item (a) before the first day of the previous year.

(i) 2 kilometers > 10,000 < 1,00,000

(ii) 4 kilometers > 1,00,000 <10,00,000

(iii) 8 kilometers > 10,00,000

Page 157: INTER CA Taxation - CA Study Web€¦ · INTER CA Taxation Head Office Shraddha, 4th Floor, ... 14. Advance Tax & Interest 268 - 271 15. Tax Deducted at Source (TDS) 272 - 294 16

: 154 :

J. K. SHAH CLASSES INTER C.A. - DIRECT TAXES

AGRICULTURE INCOME

(5) Income from nursery - In the past, there have been court rulings that only if a nursery

is maintained by carrying out the basic operations on land and subsequent operations

in continuation thereof, income from such nursery would be treated as agricultural income

and would qualify for exemption under section 10(1). The Supreme Court has, in CIT v.

Raja Benoy Kumar Sahas Roy (1957) 32 ITR 466, held that the basic operations must

be performed before any income can be called agricultural income. The basic

operations involve cultivation of the ground, in the sense of tilling of the land, sowing of

the seeds, planting and other similar operations on the land. Such basic operations

demand the expenditure of human labour and skill upon the land itself and further, they

are directed to make the crop sprout from the land. Therefore, income derived from

sale of plants grown directly in pots would not be treated as agricultural income.

However, the Madras High Court, in CIT v. Soundarya Nursery (2000) 241 ITR 530,

observed that nursing activity involves carrying out of several operations on land before

the saplings were transplanted in suitable containers including pots and thereafter kept

in shade or green house for further operation and growth. Therefore, income arising

from nursery should be considered as agricultural income.

Explanation 3 to section 2(1A) provides that the income derived from saplings or

seedlings grown in a nursery would be deemed to be agricultural income, whether or

not the basic operations were carried out on land. This Explanation ratifies the view

taken by the Madras High Court in favour of the taxpayer.

Examples of Agricultural income:

For better understanding of the concept, certain examples of agricultural income and

nonagricultural income are given below:

Agricultural income

1. Income derived from the sale of seeds.

2. Income from growing of flowers and creepers.

3. Rent received from land used for grazing of cattle required for agricultural

activities.

4. Income from growing of bamboo.

Non-agricultural income

1. Income from breeding of livestock.

2. Income from poultry farming.

3. Income from fisheries.

4. Income from dairy farming.

Page 158: INTER CA Taxation - CA Study Web€¦ · INTER CA Taxation Head Office Shraddha, 4th Floor, ... 14. Advance Tax & Interest 268 - 271 15. Tax Deducted at Source (TDS) 272 - 294 16

: 155 :

J. K. SHAH CLASSES INTER C.A. - DIRECT TAXES

AGRICULTURE INCOME

Composite income

WHAT IS THE TAX TREATMENT OF INCOME WHICH IS PARTLY AGRICULTURAL AND

PARTLY BUSINESS(Rule 7,7A,7B and 8)

For disintegrating a composite business income which is partly agricultural and partly non-

agricultural, the following rules are applicable -

For tea, coffee & latex :

For any other case [Rule 7]

For disintegrating a composite business income which is partly agricultural and

partly non agricultural, the market value of any agricultural produce, raised by the

assessee or received by him as rent-in-kind and utilised as raw material in his

business, is deducted. No further deduction is permissble in respect of any ex-

penditure incurred by the assessee as a cultivator or receiver of rent in kind.

Page 159: INTER CA Taxation - CA Study Web€¦ · INTER CA Taxation Head Office Shraddha, 4th Floor, ... 14. Advance Tax & Interest 268 - 271 15. Tax Deducted at Source (TDS) 272 - 294 16

: 156 :

J. K. SHAH CLASSES INTER C.A. - DIRECT TAXES

AGRICULTURE INCOME

Q. 1. (a) For the previous year ending March 31, 2018, non-agricultural income of X

is 90,000, whereas agricultural income is 5,70,000. Is he liable to pay

income-tax?

(b) For the assessment year 2018-19, net agricultural income of Mrs. X

(47 years) is 3,40,000 and non-agricultural income is 2,80,000.

Determine her basic tax liability.

(c) For A.Y. 2018 - 19, the agricultural income of ABC Pvt. Ltd. is 7,60,000 and

the non-agricultural income is 6,00,000/-. Calculate the tax payable.

Q. 2. Discuss whether the following items of income constitute "Agricultural Income" for

the purpose of the Income-tax Act.

(a) Income derived from rubber plantation in Singapore.

(b) Rent received from a tenant to whom land in India has been let out and who

uses it for cultivating wheat.

(c) Income derived from sale of timber of spontaneous growth on Indian soil.

Q. 3. X Ltd. grows sugarcane to manufacture sugar. Data for 2017-18 is as follows :

( `̀̀̀ In lakh)

Cost of cultivation of sugarcane 6

Market value of sugarcane when sugarcane is transferred to factory 10

Other manufacturing cost 6

Sales turnover of sugar 25

Salary of managing director who looks after non-agricultural

operations of the company 3

Compute agriculture income and income from business.

Q. 4. Mr. Tenzingh is engaged in composite business of growing and curing (further

processing) coffee in Coorg, Karnataka. The whole of coffee grown in his

plantation is cured. Relevant information pertaining to the year ended 31.03.2018

are given below :

WDV of car as on 1.4.2017 (15% rate) 3,00,000

WDV of machinery as on 1.4.2017 (15% rate) 15,00,000

Expenses incurred for growing coffee 3,10,000

Expenditure for curing coffee 3,00,000

Sale value of cured coffee 22,00,000

The expenses incurred for car running and maintenance are `̀̀̀ 50,000. The

machines & the car were used in coffee curing business operations only.

Compute the income arising from the above activities for the assessment year 2018-19.

Q. 5. Mr. Kamal grows paddy and uses the same for the purpose of manufacturing of rice in his

own Rice Mill. The cost of cultivation of 40% of paddy produce is `̀̀̀ 7,00,000 which id sold

for 15,00,000; and the cost of cultivation of balace 60% of paddy is `̀̀̀ 12,00,000 and the

market valueof such paddy is ̀̀̀̀ 24,00,000.To manufacture the rice, he incurred `̀̀̀ 2,00,000

in the manufacturing process on the balance (60%) paddy.

The rice was sold for ̀̀̀̀ 30,00,000.

Compute the Business income and Agriculture Income of Mr. Kamal.

Page 160: INTER CA Taxation - CA Study Web€¦ · INTER CA Taxation Head Office Shraddha, 4th Floor, ... 14. Advance Tax & Interest 268 - 271 15. Tax Deducted at Source (TDS) 272 - 294 16

: 157 :

J. K. SHAH CLASSES INTER C.A.- DIRECT TAXES

CAPITAL GAINS

Sec. 45 - Charging Section

CAPITAL GAINS / LOSS IS THE GAIN / LOSS WHICH IS COMPUTED [Sec.48] ON THE

TRANSFER [Sec. 2(47)] OF A CAPITAL ASSET [Sec.2(14) WHICH IS CHARGEABLE

TO TAX.

WHAT IS CAPITAL ASSET [Sec.2(14)]:

“Capital asset” is defined to include property of any kind whether fixed, circulating, movable,

or immovable, tangible or intangible. The word "Property" includes any rights in or in relation

to an Indian company including rights of management or control or any other rights whatso-

ever such as equity shares .

W.E.F. P.Y. 14 -15, securities held by a foreign Institutional Investors (F.I.I.) shall also be re-

garded as capital assets u/s 2(14).

The following assets are, however, excluded from the definition of “Capital assets”:

(a) Any stock in trade, consumable stores or raw material held for the purpose of busi-

ness or profession, since its effects would be covered in Income from business & pro-

fession.

(b) Personal Effects of the assessee that is to say movable property (Land, house prop-

erty, commercial property are immovable, therefore they are capital assets) includ-

ing wearing apparel and furniture held for his personal use or for the use of any mem-

bers of his family - dependent upon the assessee. However Jewellery archaeological

collections, drawings, paintings, sculptures, or any work of art, are treated as a capital

asset even though they are meant for personal use of the assessee.

[Gold and silver coins and bars used for puja of deities as a matter of pride and orna-

mentation and normally not intended for personal or household use and securities are

not personal effects and are therefore treated as capital assets.

Furniture, vehicle on which depreciation is not claimed, are also not a capital

asset].

(c) Agriculture land (Non-urban)in India provided it is not situated : In any area which is

comprised within the jurisdiction of any municipality /cantonment board having

population of :

- not being more than 2 kms from the local limits where population

exceeds 10,000 but does not exceed 1,00,000.

- not being more than 6 kms from the local limits where population

exceeds 1,00,000 but does not exceed 10,00,000.

- not being more than 8 kms from the local limits where population

exceeds 10,00,000.

(d) (i) 6.5% Gold bonds, 1977, or

(ii) 7% Gold Bonds, 1980, or

(iii) National Defence Gold Bonds, 1980, or

(iv) Special Bearer Bonds, 1991, or

(v) Gold Deposit Bonds issued under the Gold Deposit scheme, 1999 notified

by the Central Government or deposit certificates issued under the Gold

Monetisation Scheme, 2015.

CAPITAL GAINS

Page 161: INTER CA Taxation - CA Study Web€¦ · INTER CA Taxation Head Office Shraddha, 4th Floor, ... 14. Advance Tax & Interest 268 - 271 15. Tax Deducted at Source (TDS) 272 - 294 16

: 158 :

J. K. SHAH CLASSES INTER C.A.- DIRECT TAXES

CAPITAL GAINS

TRANSFER [Sec. 2(47)]

Transfer, in relation to a Capital Asset, includes sale, exchange or relinquishment of the asset

or the compulsory acquisition thereof under any Law. Transfer also includes disposing of or

parting with an asset or any interest therein, or creating any interest in any asset in any manner

whatsoever, directly or indirectly, absolutely or conditionally, voluntarily or involuntarily, by way

of an agreement (whether entered into in India or outside India) or otherwise, notwithstanding

that such transfer of rights has been characterized as being effected or dependent upon or

flowing from the transfer of a share or shares of a company registered or incorporated outside

India.

The foreign company or entity should have assets (not net of liabilities) exceeding ̀ 10 crs. in

India which represent atleast 50% or more of the fair market value of all the assets (not net of

liabilities) owned by such foreign company or entity.

Transfer also includes the following :

1. From the assessment year 85-86, i.e. with effect from 1.4.84 and onwards, if a ‘Capi-

tal asset’ is converted by the owner thereof into (or is treated by him as) stock in trade

of a business carried on by him, such conversion (or treatment of) the capital asset

shall also be regarded as ‘Taxable transfer’ of the asset.

CASE NAME : CIT v/s. BAI SHIRIN BAI. K. KOOKA - [Section 45(2)].

Note:

(1) If the converted asset is not sold in the year of conversion, there will be no capital

gains tax in such a year of conversion even though it is a taxable transfer, since

there is no liquidity generation to pay the capital gains tax.

(2) Assessee will be brought to both capital gains tax and income from business &

profession in the year of sale of converted stock in trade i.e. the year of

liquidity generation.

(3) For the computation of capital gains, the sale consideration is taken at the market

value on the date of conversion. If Indexation is done, then done up to the date of

conversion only .

2. From the assessment year 88-89, i.e. with effect from 1.4.87 and onwards,the follow-

ing transactions are also regarded as a “Taxable transfer” :

(a) Any transaction involving the allowing of the possession of any immovable

property to be taken or retained in part performance of a contract of the nature

referred to in Section 53A of the TRANSFER OF PROPERTY ACT, 1882.

As per section 53A of the Transfer Of Property Act [TOPA], 1882 in the year in

which all the 3 conditions undermentioned are fulfilled, the buyer would be

considered deemed owner u/s 27 and the assesse seller will be brought to

capital gains tax even though he still is the legal owner of the property.

(i) There should be a contract for consideration, in writing, duly signed, in

relation to transfer of immovable property i.e. “sale deed” is made even

though it is not applied for registry transfer.

(ii) The transferee (i.e. the buyer) should have taken the possession of the

property from the seller.

(iii) The transferee has performed or is willing to perform his part of the contract

(i.e. the buyer has paid the consideration or shall pay it and there is no

dispute regarding the same).

Page 162: INTER CA Taxation - CA Study Web€¦ · INTER CA Taxation Head Office Shraddha, 4th Floor, ... 14. Advance Tax & Interest 268 - 271 15. Tax Deducted at Source (TDS) 272 - 294 16

: 159 :

J. K. SHAH CLASSES INTER C.A.- DIRECT TAXES

CAPITAL GAINS

(b) Any transaction (whether by way of becoming a member of, or acquiring shares in

a co-operative society, company, or other association of persons, etc.) which has

the effect of transferring, or enabling the enjoyment of any immovable property.

Example: Mr. A takes a house from DDA (Delhi Development Authority) i n

installment purchase. The registry (ownership) is not transferred to Mr.A, Mr. A

will be considered deemed owner u/s 27 and DDA would be brought to capital

gains tax. Subsequently when Mr. A Sells the house to Mr.B (with the consent

of DDA being the registered owner still), then Mr. B will be considered deemed

owner and Mr. A will be brought to capital gains tax.

3. W. E. F. 1.6.2005 & onwards the maturity or redemption of a "zero coupon

bond", shall be considered as a taxable transfer.

[Zero coupon bond means a bond :

- issued by any infrastructure capital company or infrastructure capital fund

or public sector company or scheduled bank,

- in respect of which no payment and benefit is received or receivable

before maturity or redemption from the bond issuing organisations.

- such bonds will be notified by the Central Government in the official ga-

zette]

- If held for more than 12 months, then long term capital asset -

which would be taxable @ 10% u/s 112 without indexation under

option 2.

Types of capital Assets

There are two types of Capital Assets :

(i) Long term capital asset

(ii) Short term capital Asset

Long term and Short term depends on holding period which is as follows:

In case of following assets holding period is 12 months

1. Listed shares

2. Securities Listed on RSE in India

3. unit of UTI (Listed or Not)

4. unit of equity oriented MF (Listed or Not)

5. Zero coupon Bonds (Listed or Not)

In Case of following assets holding period is 24 months

1. Unlisted shares

2. Land/ Building

In Case of all other assets holding period is 36 months

Page 163: INTER CA Taxation - CA Study Web€¦ · INTER CA Taxation Head Office Shraddha, 4th Floor, ... 14. Advance Tax & Interest 268 - 271 15. Tax Deducted at Source (TDS) 272 - 294 16

: 160 :

J. K. SHAH CLASSES INTER C.A.- DIRECT TAXES

CAPITAL GAINS

CAPITAL GAINS HOW COMPUTED : [Sec. 48]

(A) COMPUTATION OF SHORT TERM CAPITAL GAINS :

Full value of consideration xx

Less :

(i) Expenditure incurred wholly and exclusively

in connection with such transfer xx

(ii) Cost of acquisition xx

(iii) Cost of improvement. xx (-) xx

GROSS SHORT TERM CAPITAL GAINS xx

Less :

[Exemption u/s. 54B, 54D, 54G]* (-) xx

NET SHORT TERM CAPITAL GAIN xx

(B) COMPUTATION OF LONG TERM CAPITAL GAINS :

Full Value of consideration. xx

Less :

(i) Expenditure incurred wholly and exclusively

in Connection with such transfer. xx

(ii) INDEXED cost acquisition xx

(iii) INDEXED cost of improvement xx (-) x

GROSS LONG TERM CAPITAL GAIN xx

Less :

Exemption u/s. 54, 54B, 54D, 54EC, 54F & 54 G (-) xx

NET LONG TERM CAPITAL GAIN xx

(a) Full value of consideration i.e. sale consideration.

The consideration (in money or in kind) what the transfererrer receives in

place of the asset he transfers. (It is detailed later on).

(b) Expenditure incurred wholly and exclusively in connection with such transfer.

It includes all expenditure by the seller to complete the sale and transfer the

ownership of the asset to the buyer such as brokerage, stamp duty,

registration fees etc.(It is detailed later on).

(c) Cost of acquisition

It is the purchase cost of the asset and it includes all expenses of capital nature

of acquiring title (i.e. ownership) of such asset purchased.

(It is detailed later on and it is the most important aspect in the computation

of capital gains).

(d) Cost of improvement

It includes all the expenditure of capital nature by the seller in increasing the

value of the capital asset (It is detailed later on).

(e) Exemption u/s 54 are based on certain investments of capital gains within a

given time limit.

Page 164: INTER CA Taxation - CA Study Web€¦ · INTER CA Taxation Head Office Shraddha, 4th Floor, ... 14. Advance Tax & Interest 268 - 271 15. Tax Deducted at Source (TDS) 272 - 294 16

: 161 :

J. K. SHAH CLASSES INTER C.A.- DIRECT TAXES

CAPITAL GAINS

What is Cost inflation Index :

“Cost inflation index” in relation to a previous year, means such index as the Central

Government may, having regard to seventy-five percent of average rise in the Consumer

Price Index for (Urban) for the immediately preceding previous year to such previous

year, by notification in the Official Gazette, specify in this behalf.

The Central Government has notified the “Cost inflation index” for the purpose of long-

term capital gain as follows :

Financial Year / Previous Year Cost Inflation Index

2001-2002 100

2002-2003 105

2003-2004 109

2004-2005 113

2005-2006 117

2006-2007 122

2007-2008 129

2008-2009 137

2009-2010 148

2010-2011 167

2011-2012 184

2012-2013 200

2013-2014 220

2014-2015 240

2015 - 2016 254

2016 -2017 264

2017 - 2018 272

Note :

(1) These are normally given in the examination.

(2) No indexation benefit in case of long term capital gains from sale of

debentures, bonds, Global deposit receipts, 0% discount bonds etc. (except

capital indexed bonds of central government and Sovereign Gold Bonds).

(3) If the capital asset (except self-generated asset of goodwill etc. and

depreciable asset under block method) is acquired prior to 1-4-01, then Fair

Market Value (FMV) as decided by approved valuers as at 1-4-01 or the

purchase cost, whichever is more, is taken as cost of acquisition u/s 55(2)

and then subject to indexation.

* INDEXED COST OF ACQUISITION

The amount which bears to the cost of acquisition, the same proportion as

cost inflation index for the year in which the asset is transferred / sold, bears

to the cost inflation Index for the first year in which the asset was held by the

assessee, or for the year beginning on 1 .4. 01, whichever is later.

Page 165: INTER CA Taxation - CA Study Web€¦ · INTER CA Taxation Head Office Shraddha, 4th Floor, ... 14. Advance Tax & Interest 268 - 271 15. Tax Deducted at Source (TDS) 272 - 294 16

: 162 :

J. K. SHAH CLASSES INTER C.A.- DIRECT TAXES

CAPITAL GAINS

Tax Rate

Short term capital gains tax in respect of equity shares/ units of an equity oriented

fund [Section 111A]

(i) This section provides for a concessional rate of tax (i.e. 15%) on the short-term capital

gains on transfer of -

(1) an equity share in a company or

(2) a unit of an equity oriented fund or

(3) a unit of a business trust.

(ii) The conditions for availing the benefit of this concessional rate are –

(1) the transaction of sale of such equity share or unit should be entered into on or

after 1.10.2004, being the date on which Chapter VII of the Finance (No. 2) Act,

2004 came into force; and

(2) such transaction should be chargeable to securities transaction tax under the

said Chapter.

(iii) The proviso to this section provides that in the case of resident individuals or HUF, if

the basic exemption is not fully exhausted by any other income, then the short-term

capital gain will be reduced by the unexhausted basic exemption limit and only the

balance would be taxed at 15%. However, the benefit of availing the basic exemption

limit is not available in the case of non-residents.

(iv) Deductions under Chapter VI-A cannot be availed in respect of such short-term capital

gains on equity shares of a company or units of an equity oriented mutual fund included

in the total income of the assessee.

The expression “equity oriented fund” has the same meaning assigned to it in the

explanation to section 10(38) i.e., “Equity oriented fund” means a fund –

(1) where the investible funds are invested by way of equity shares in domestic

companies to the extent of more than 65% of the total proceeds of such fund;

and

(2) which has been set up under a scheme of a Mutual Fund specified under clause (23D).

Tax on long-term capital gains [Section 112]

(1) Where the total income of an assessee includes long-term capital gains, tax is payable

by the assessee @20% on such long-term capital gains. The treatment of long-term

capital gains in the hands of different types of assessees are as follows -

(i) Resident individual or Hindu undivided family: Income-tax payable at normal

rates on total income as reduced by long-term capital gains plus 20% on such

long-term capital gains. However, where the total income as reduced by such

long-term capital gains is below the maximum amount which is not chargeable

to income-tax then such long-term capital gains shall be reduced by the amount

by which the total income as so reduced falls short of the maximum amount which

is not chargeable to income-tax and the tax on the balance of such long-term

capital gains will be calculated @ 20%.

Page 166: INTER CA Taxation - CA Study Web€¦ · INTER CA Taxation Head Office Shraddha, 4th Floor, ... 14. Advance Tax & Interest 268 - 271 15. Tax Deducted at Source (TDS) 272 - 294 16

: 163 :

J. K. SHAH CLASSES INTER C.A.- DIRECT TAXES

CAPITAL GAINS

(ii) Domestic Company: Long-term capital gains will be charged @ 20%.

(iii) Non-corporate non-resident / foreign company: (i) Long-term capital gains

arising from the transfer of a capital asset, being unlisted securities, or shares

of a company not being a company in which public are substantially interested,

would be calculated at the rate of 10% on the capital gains in respect of such

asset without giving effect to the indexation provision under second proviso to

section 48 and currency fluctuation under first proviso to section 48.

(2) In respect of other long-term capital gains, the applicable rate of tax would be 20%.

(i) Residents (other than those included in (i) above): Long-term capital gains will

be charged @20%.

(ii) Where the tax payable in respect of any income arising from the transfer of a

listed security (other than a unit) or a zero coupon bond, being a long-term capital

asset, exceeds 10% of the amount of capital gains before indexation, then such

excess shall be ignored while computing the tax payable by the assessee.

Consequently, long term capital gains on transfer of units of debt oriented mutual

fund and unlisted securities are not eligible for concessional rate of tax@10%

(without indexation benefit). Therefore, the long-term capital gains, in such cases,

is taxable@20% (with indexation benefit).

However, in case of non-corporate non-residents and foreign companies, long

term capital gains arising from transfer of a capital asset, being unlisted securities

or shares in a company in which public are not substantially interested are eligible

for a concessional rate of tax@10% (without indexation benefit).

(iii) For this purpose, "listed securities" means securities as defined by section 2(h)

of the Securities Contracts (Regulation) Act, 1956.

(iv) The provisions of section 112 make it clear that the deductions under Chapter

VIA cannot be availed in respect of the long-term capital gains included in the

total income of the assessee.

The priority of set off of minimum exemption limit applicable towards N.T.T.I and

the taxability of L.T.C.G u/s 112 as well as S.T.C.G U/S 111A in the case of Individual

& H.U.F.

The minimum exemption limit applicable for individuals of 2,50,000 or 3,00,000 or

5,00,000 and for HUF of 2,50,000 is applied towards the N.T.T.I (Net Taxable Total

Income)in the following priority.

* 1st applied towards the N.T.T.I portion of income from salary, house property,

business & profession, normal short term capital gains taxable at slab rates

and income from other sources all incomes except casual winnings (i.e. excluding

incomes in the nature of L.T.C.G, S.T.C.G taxable u/s 111A and casual winnings).

* 2nd priority : If any portion of minimum limit remains after above, then that is

adjusted towards any long-term capital gains, and the balance is taxable @ 20%

(10%in case of option 2 discussed at Para).

Page 167: INTER CA Taxation - CA Study Web€¦ · INTER CA Taxation Head Office Shraddha, 4th Floor, ... 14. Advance Tax & Interest 268 - 271 15. Tax Deducted at Source (TDS) 272 - 294 16

: 164 :

J. K. SHAH CLASSES INTER C.A.- DIRECT TAXES

CAPITAL GAINS

* 3rd priority: If any portion of minimum limit still remains after above, then that is

set off towards short-term capital gains generated on sale of equity shares / units

of an equity oriented mutual fund sold via stock exchange[S.T.T is paid there on];and

the balance of such short term capital gains will be taxable at a flat rate of 15% u/

s 111A.

Please note that the minimum exemption limit remaining spare cannot be adjusted

against casual winnings.

CERTAIN TRANSACTIONS NOT INCLUDED IN TRANSFER.

The following transactions are not regarded as taxable transfer even though the ownership of

asset has shifted from the transferer to the transferee.

Please note:

* If in these situations the transferer and transferee both are not brought to capital

gains tax because of the under mentioned provisions, and later on the transferee

sells such a capital asset, his cost of acquisition will be the cost of acquisition of

the asset of the previous owner u/s 49(1).

* If the previous owner has acquired the asset prior to 1-4-81, then the option u/s

55(2) of taking the F.M.V as at 1-4-81 or the original purchase cost of the previous

owner(whichever is more), will also be available to the transferee.

* In such cases the period for which the capital asset was held by the previous owner

should be included in the period held by the assessee to know if it is long term or

short term. But indexation is allowed only for the holding period of the assessee

i.e. current owner of the capital asset.

(A) Distribution of capital assets in cash or in kind by a company to its

share holders on its liquidation [Sec. 46(1)].

Note:

* The ‘company’ is not brought to capital gains tax u/s 46(1), since now

the shareholders become the legal owners of the net assets distributed

by company upon liquidation.

* The shareholder is brought to capital gains tax u/s 46(2), since he is

surrendering one asset i.e. share certificates, which has become

worthless, and getting another asset i.e. in cash or in kind.

If received in kind, then F.M.V. of asset received is taken as sale

consideration and the first portion of the sale consideration is received

towards the deemed dividend u/s 2(22)(c)and the balance is received

towards the share capital.

Subsequently when such a shareholder sells such a capital asset ,

whatever was taken as sale consideration in the hands of shareholder

u/s 46(2), becomes his cost of acquisition and the holding period of

the asset will be taken from the date he got the asset till the date he

sold it subsequently.

Page 168: INTER CA Taxation - CA Study Web€¦ · INTER CA Taxation Head Office Shraddha, 4th Floor, ... 14. Advance Tax & Interest 268 - 271 15. Tax Deducted at Source (TDS) 272 - 294 16

: 165 :

J. K. SHAH CLASSES INTER C.A.- DIRECT TAXES

CAPITAL GAINS

(B) Any distribution of Capital Assets in cash or in kind by a Hindu

Undivided Family to its members at the time of partition [Sec. 47(i)].

Note:

* The H.U.F is a collective body of Hindu family members and is

considered an assessee for income tax purposes.

Upon partition, when it distributes assets to its members, it is not

brought to tax as no outsider is involved.

* The receiving H.U.F members is also not brought to tax, since he is only

receiving the capital asset and not surrendering any capital asset.

* The receiving H.U.F member will be brought to tax only when he sells

such a capital asset.

(C) Any distribution of asset in kind on the dissolution of the firm, body

of individuals, association of persons if such dissolution took place

prior to 1-4-87 i.e. till 31.3.87 [sec 47(ii)]

Note:

* Section47 (ii) was valid up to P.Y. 86-87 A.Y. 87-88

- For income tax purposes ‘partner’ and ‘partnership firm’- are

considered separate persons.

* Section 47(ii) (omitted from P.Y. 87-88 A.Y.88-89)

In a situation where the firm distributed the asset to partner upon

dissolution prior to 1-4-87 and it was not brought to tax then because

of section 47(ii)and the partner receiving the asset was also not

brought to tax then, since he was receiving the capital asset only and

not surrendering any capital asset.

If such an asset is sold in the current year by the partner, his cost of

acquisition would be the cost to the firms u/s 49(I) read with 47(ii).

* Whenever a partner gives an asset to the firm as his capital

contribution, the amount recorded in the books of the firm (i.e.

the agreed value) is taken as sale consideration in the hands of the

partner (and not the market value of asset ) and the partner is brought

to capital gains tax.

* On or after 1-4-1987 if the firm distributes assets to its partners upon

dissolution, the market value is taken as sale consideration in the

hands of the firm (i.e. the agreed value is not relevant at all) and the

firm is brought to capital gains tax.

(D) Any transfer of capital asset under a gift or a will or an irrevocable

trust [Sec. 47(iii)].

W.E.F. A.Y. 2001-2002 it has been provided that this clause shall not apply

to transfer under a gift or an irrevocable trust of a capital asset being shares,

debentures or warrants allotted by a company directly or indirectly to its em-

ployees under the Employee's Stock Option Scheme [E.S.O.P] or Employ-

ee's Stock Purchase Scheme framed in accordance with the guidelines is-

sued by Central Government.

Page 169: INTER CA Taxation - CA Study Web€¦ · INTER CA Taxation Head Office Shraddha, 4th Floor, ... 14. Advance Tax & Interest 268 - 271 15. Tax Deducted at Source (TDS) 272 - 294 16

: 166 :

J. K. SHAH CLASSES INTER C.A.- DIRECT TAXES

CAPITAL GAINS

Notes:* The receiver of gift is not brought to capital gains tax, since he is only

receiving and not giving any asset .

* Subsequently when the transferee will sell such an asset in future, hiscost of acquisition will be the cost to the previous owner u/s 49(1) r.w47(iii).

* The E.S.O.P securities are gifted or transferred by way of an irrevocabletrust during the life time of the transferer, then the provisions of section47(iii) are not applicable and the transferer will be brought to capital gains

tax and for that purpose the market value of the securities will be taken assale consideration, i.e. the provisions of section 47 (iii) are not applicable.

* If E.S.O.P securities are sold then agreed value (and not the market value)will be taken a sale consideration for the purpose of capital gains tax.

* If E.S.O.P Securities are transferred by way of will or inheritance afterthe death of the employee, then the provisions of section 47(iii) applyand transferer’s estate (property left behind ) will not be brought totax.

(E) Any transfer by individual of Sovereign Gold Bond issued by R.B.I under theSovereign Gold Bond Scheme, 2015 by way of redemption.[sec47(viic)].

(F) Any transfer by way of conversion of bonds or debentures, debenture Stockor deposit certificate in any form, of an Indian company, into shares or de-bentures of that company (placed with retrospective effect from 1.4.1962)[Sec. 47(x)]NOTES:* If the debentures [Fully Convertible Debenture (F.C.D) or Partly

Convertible Debenture (P.C.D)] get converted into shares andsubsequently such converted shares are sold, then for computing

capital gains on the sale of such converted shares , the cost ofacquisition of the debentures which has been appropriated towardsthe converted shares, shall be the cost of acquisition of such converted

shares [sec 49(2A)]* The holding period of the debentures will not be included in the hold-

ing period of the shares i.e. the holding period of the converted shares

will be from the date they were converted till the date they were sold.(G) Any transfer of a capital asset in a transaction of reverse mortgage under a

scheme made and notified by the Central Government. [Sec. 47(xvi)]

Notes:* ”Reverse Mortgage”

A reverse mortgage (or life time mortgage) is a loan available to

senior citizens. In this a person pledges a property he owns with noexisting loan against it, with a bank. The bank gives a series of fixedmonthly cash flows for a fixed tenure. The normal tenure is 15 years

and the owner of the house and the spouse continue to live in the housetill their death- which can occur later than the tenure of reversemortgage. Thus the senior citizen /spouse will get the amount monthly

for 15 years which will not be taxable in their hands, since it is a loani.e. a capital receipt and hence exempted u/s 10(43)If one of the spouse dies, the other can still continue living in the house. If

both die the bank will give their legal heirs two options:-- Settle the overall outstanding loan and retain the house, or- the bank will sell the house, use the proceeds to settle the

outstanding loan and give the rest to the legal heirs which will

not be taxable in the hands of their legal heirs.

Page 170: INTER CA Taxation - CA Study Web€¦ · INTER CA Taxation Head Office Shraddha, 4th Floor, ... 14. Advance Tax & Interest 268 - 271 15. Tax Deducted at Source (TDS) 272 - 294 16

: 167 :

J. K. SHAH CLASSES INTER C.A.- DIRECT TAXES

CAPITAL GAINS

(H) Conversion of Preference Share into Equity Shares [Sec 47(xb)]

(I) Transfer of Rupee Denominated Bond of Indian Company Issued outside

India, by Non- Resident to another Non- Resident .

Since the above transaction are not covered recognised as “taxable

transfers” for the purpose of Sec. 45, therefore any profit or gain

arising on the above noted transactions is not chargeable to tax u/s.

45, conversely, any loss arising there from is not liable to be set off

against other incomes of the assessee.

’Full value of consideration ‘or ‘Sale consideration’ of various types detailed.

It is the gross due amount (in ‘money or the fair market value ‘if in kind) which the transferer

would receive in lieu of the asset he sells / transfers.

Such amount cannot be reduced by any outgoings by the transferer such as brokerage

commission, stamp duty etc. (such outgoings have to be claimed as transfer expenses).

Normally the agreed sale consideration is taken as sale consideration and not the fair

market value, which could be more or less than agreed sale consideration.

In the following cases however the “Fair Market Value” will be taken as sale consideration

for the computation of capital gains.

(a) While computing capital gains in the case of conversion of capital asset into stock

in trade with effect from 1.4.84 and onwards (i.e. A.Y. 85-86).

The fair market value of the capital asset on the date on which it was converted or treated

as stock in trade shall be deemed to be the full value of the consideration received or

accruing as a result of the transfer of the capital asset.

(b) In the case of transfer by way of distribution of capital assets by a firm, by an

association of persons or by a body of individuals, the fair market value of the

asset as on the date of transfer shall be deemed to be the full value of the consid-

eration as a result of such transfer. In other words the agreed sale consideration is

NOT relevant at all. [Section 45 (4)].

Note: In case the partner gives the asset to the firm as a capital contribution, then

agreed value will become the sale consideration in the hands of the partner.[sec 45(3)]

(c) In case of barter / exchange.

(d) In case of assets distributed in "kind" at the time of liquidation of a company sec.

46(2) taxable in the hands of the share holder as" sale consideration of the

shares"(which have become worthless), the fair market value of the asset received

from the company.

Notes:

* The first portion of the sale consideration is always taken towards the deemed

divided u/s 2(22)(c)& the balance received is towards the share capital.

* The Indian company is brought to tax on behalf of the shareholder u/s 115-0

towards the deemed dividend u/s 2(22)(c), hence exempted in the hands of

the shareholders u/s 10(34).

Page 171: INTER CA Taxation - CA Study Web€¦ · INTER CA Taxation Head Office Shraddha, 4th Floor, ... 14. Advance Tax & Interest 268 - 271 15. Tax Deducted at Source (TDS) 272 - 294 16

: 168 :

J. K. SHAH CLASSES INTER C.A.- DIRECT TAXES

CAPITAL GAINS

(e) Where shares, debentures or warrants referred to in the proviso to sec. 47 (iii) are

transferred under a gift or an irrevocable trust, the market value on the date of such

transfer shall be deemed to be the full value of consideration received or accruing as a

result of transfer for the purposes of this section.

Notes:

* If such E.S.O.P securities are transferred during the life time of the transferer

by way of:

- ”Gift or irrevocable trust”, then market value on the date of transfer

will be taken as sale consideration.

- ”Sale”, then the agreed value will be taken as sale consideration even

if market value is more than agreed value.

* If such E.S.O.P securities are transferred after death of the transferer by

way of will / inheritance, then exempt u/s 47(iii).

* The cost of acquisition ,if taxable, will be the market value of such securities on

the date the employee got it from his employer u/s 49(2AA) [i.e.the cost of

purchase paid to employer (+) taxable perquisite value taxed earlier under salaries

u/s 17(2)(vi)].

(f) Where the consideration received or accruing as a result of the transfer of a capi-

tal asset by an assessee is not ascertainable or cannot be determined, then, for

the purpose of computing income chargeable to tax as capital gains, the fair mar-

ket value of the said asset on the date of transfer shall be deemed to be the sale

consideration. [Section 50D]

(g) Where any person receives at any time during any previous year any money or

other assets under an insurance from an insurer on account of damage to, or de-

struction of, any capital asset, as a result of —

(i) flood, typhoon, hurricane, cyclone, earthquake or other convulsion of nature; or

(ii) riot or civil disturbance; or

(iii) accidental fire or explosion; or

(iv) action by an enemy or action taken in combating an enemy (whether with or

without a declaration of war),

then, any profits or gains arising from receipt of such money or other assets

shall be chargeable to income-tax under the head “Capital gains” and shall

be deemed to be the income of such person of the previous year in which

such money or other asset was received and for the purposes of section 48,

value of any money or the fair market value of other assets on the date of

such receipt shall be deemed to be the full value of the consideration received

or accruing as a result of the transfer of such capital asset.[Sec.45(1A)].

Notes:

* The events of ‘theft’ & normal accident are not covered by above section

45(1A) and therefore in those situations the judgment of supreme court in

the case of Vania silk mills (P) Ltd v/s C.I.T will apply and insurance

indemnification will not be taxable in the hands of the recipient.

Page 172: INTER CA Taxation - CA Study Web€¦ · INTER CA Taxation Head Office Shraddha, 4th Floor, ... 14. Advance Tax & Interest 268 - 271 15. Tax Deducted at Source (TDS) 272 - 294 16

: 169 :

J. K. SHAH CLASSES INTER C.A.- DIRECT TAXES

CAPITAL GAINS

Sale consideration in case of transfer of immovable property [sec 50C]

This section applies to transfer of land or building or both held as capital asset.

(a) If land / building is transferred for a consideration which is lower than the value

adopted or assessed or "assessable" by the State Government authority for

payment of stamp duty, then such assessed value has to be considered as the

sale consideration.

(b) If the assessee claims that the stamp duty value is more than the fair market value

and the stamp duty value has not been disputed, the A.O. may refer the valuation

to the Departmental Valuation Officer.

* If the Departmental valuation officer's valuation is less than the stamp duty

valuation, but more than that declared by the assessee, then adopt the

Departmental Valuation Officer's valuation.

* If the Departmental valuation officer's valuation is more than the stamp duty

valuation, then adopt the stamp duty valuation.

For the purposes of this section, the expression "assessable" means the price which

the stamp valuation authority would have adopted or assessed, if it were referred to

such authority for the purposes of the payment of stamp duty.

Notes:

* Even if the assesse transferee has not gone to stamp duty authorities, the

assessing officer has the means to take the stamp duty valuation, if

necessary.

* If such immovable property is sold as stock in trade by a real estate agent,

then the provisions of section 43CA will apply which is similar to section

50C.

* Basically these provisions are introduced to check the wrong practice of

officially taking less money (by cheque etc) and taking the balance amount

in cash (i.e. black money or unaccounted for money).

Section 50CA Special provision for full value of consideration for transfer of

share other than quoted share.

Where the consideration received or accruing as a result of the transfer by an assessee

of a capital asset, being share of a company other than a quoted share, is less than the

fair market value of such share determined in such manner as may be prescribed, the

value so determined shall, for the purposes of section 48, be deemed to be the full value

of consideration received or accruing as a result of such transfer.

Explanation.—For the purposes of this section, “quoted share” means the share quoted

on any recognised stock exchange with regularity from time to time, where the quotation

of such share is based on current transaction made in the ordinary course of business.’.

Detailed explanation regarding ‘Expenditure on transfer’

It includes all the expenditure by assessee (seller i.e. transferer) to effect the transfer like:-

(i) Brokerage and commission paid for securing a purchaser

(ii) Cost of stamp duty paid

(iii) Registration fees

(iv) Travelling expenses incurred in connection with transfer

Page 173: INTER CA Taxation - CA Study Web€¦ · INTER CA Taxation Head Office Shraddha, 4th Floor, ... 14. Advance Tax & Interest 268 - 271 15. Tax Deducted at Source (TDS) 272 - 294 16

: 170 :

J. K. SHAH CLASSES INTER C.A.- DIRECT TAXES

CAPITAL GAINS

(v) Ligitation expenses incurred to get enhanced compensation

Notes:

* If the buyer incurres such expenses [(i) to(iv)] above , it will be added to the

cost of acquisition of the asset.

* Expenses which are allowed in other heads as a deduction, such as Income

from business or profession or Income from other sources cannot be allowed

as transfer expenses.

* Vague expenses (e.g. medical expenses incurred because of deterioration of

health caused in connection with an asset sale) are not allowed as a deduction.

* Securities Transaction Tax (S.T.T) paid by the assessee seller, is not allowed

as a deduction as transfer expenses.

(It is allowed as a business deduction u/s 36(1)(xv) if securities are sold as stock

in trade via stock exchange)

Detailed explanation regarding cost of acquisition

Cost of acquisition is the value of an asset for which it was acquired by the assessee. Ex-

penses of capital nature for completing or acquiring the title to the property are includable in

the cost of acquisition.

It includes the following :

- Interest on loan taken for acquisition of asset from the date the loan is taken till the

date of acquisition of asset.

- Litigation expenses incurred for compelling the company to register the shares in

the name of the assessee would be of capital nature, forming part of cost of acqui-

sition of the shares.

- Expenses for suits for amending articles of association are of capital in nature

and form part of cost of shares.

- Ground rent paid cannot be cost of acquisition, since they are revenue in nature

i.e. a period’s cost.

(A) COST OF ACQUISITION TO ASSESSEE - BEING COST TO THE PREVIOUS

OWNER: [SEC. 49(1)]

(1) Any distribution of asset in kind on the total or partial partition of a

H.U.F. [Sec.47(i)].

- Cost to the H.U.F. at the time of partition.

(2) Any distribution of asset in kind on the dissolution of the firm, body of

individuals, association of persons if such dissolution took place

PRIOR TO 1.4.87 (i.e. A.Y. 88-89) [Section 47(ii)]

[Sec. 47(ii) was valid upto P.Y. 86-87 / A.Y. 87-88].

- Cost to the firm, B.O.I, A.O.P. at the time of dissolution.

(3) Any transfer of asset under a gift, will, or an irrevocable trust. [Sec. 47(iii)].

- Cost to the transferrer .

(4) Any transfer of asset u/s 47(xiii), 47(xiiib) & 47 (xiv) - cost to the transferor.

(5) Any acquisition of property, by a Hindu undivided family where one of

its members has converted his self acquired property into joint family

property after 31.12.1969.

- Cost to the transferor.

Where the previous owner has acquired the property in the afore-

said manner, the "previous owner" of the property means the last pre-

vious owner who acquired the property by means other than those

discussed above.

Page 174: INTER CA Taxation - CA Study Web€¦ · INTER CA Taxation Head Office Shraddha, 4th Floor, ... 14. Advance Tax & Interest 268 - 271 15. Tax Deducted at Source (TDS) 272 - 294 16

: 171 :

J. K. SHAH CLASSES INTER C.A.- DIRECT TAXES

CAPITAL GAINS

(B) (i) COST OF ACQUISITION WHEN DEBENTURES ARE CONVERTED INTO

SHARES OR BONDS INTO DEBENTURES :

Any transfer by way of conversion of debentures or deposit certificate into shares

or conversion of deposit certificate into debentures of that company will not be

regarded as a taxable transfer [Because of Sec. 47(x)].

For computing capital gains on the sale of such converted shares or debentures

(as the case may be), the cost of acquisition of the old debentures, debenture -

stock, bonds or deposit certificate which has been appropriated towards the

shares or debentures, shall be the cost of acquisition of such converted shares

or debentures. [Sec. 49(2A)]

(ii) SHARES /DEBENTURES GOT AS A PERQUISITE- Section 49(2AA) men-

tions that where the capital gain arises from the transfer of specified secu-

rity or sweat equity shares referred to in section 17(2)(vi), the cost of acqui-

sition of such security or shares shall be the fair market value which has

been taken into account for the purposes of the said section 17(2)(vi).

(iii) Cost of Equity Shares = Cost of Preference shares (conversion of

preference shares x equity shares) [Sec. 49 (2AE)].

(C) COST OF ACQUISITION IN THE CASE OF :

- Depreciable assets under block method :

Under the system of providing deprecation on block of assets, written down

value of any block of assets may be reduced to nil for any of the following

two situations.

(1) Situation 1 : [Sec. 50(1) ] If the sale consideration of any one or more

assets, but not all assets, exceeds the opening W.D.V + Value of as-

sets acquired during the previous year + expenditure in connection with

such transfer; this excess amount shall be deemed to be ALWAYS

SHORT-TERM CAPITAL GAINS.

(Here the cost of acquisition would be the sum total of opening W.D.V.

+ acquisition during the previous year).

(2) Situation 2 : [Sec. 50(2)] Here the block cease to exist because all

the assets in that block are sold during the previous year. The cost of

acquisition in such a case shall be the aggregate of the following :

(a) Opening W.D.V. at the beginning of the previous year.

(b) Actual Cost of any asset, falling within that block, acquired by

the assessee during the previous year.

Here there are 2 possibilities :-

(i) If the sale consideration of all the assets in the block, exceeds the cost of

acquisition above and expenditure of transfer, such excess is chargeable to

tax as SHORT TERM CAPITAL GAIN.

(ii) If the sale consideration of all the assets in the block falls short of the cost of

acquisition above and expenditure of transfer, such short fall is SHORT TERM

CAPITAL LOSS.

Page 175: INTER CA Taxation - CA Study Web€¦ · INTER CA Taxation Head Office Shraddha, 4th Floor, ... 14. Advance Tax & Interest 268 - 271 15. Tax Deducted at Source (TDS) 272 - 294 16

: 172 :

J. K. SHAH CLASSES INTER C.A.- DIRECT TAXES

CAPITAL GAINS

(D) Cost of acquisition in the computation of capital gains in case of slump

sale [Sec. 50B].

(1) Any profits or gains arising from the slump sale effected in the previous

year shall be chargeable to income-tax as capital gains arising from the

transfer of long-term capital assets and shall be deemed to be the income

of the previous year in which the transfer took place:

Provided that any profits or gains arising from the transfer under the slump sale

of any capital asset being one or more undertaking or division owned and

held by an assessee for not more than thirty-six months immediately preceding

the date of its transfer shall be deemed to be the capital gains arising from the

transfer of short-term capital assets.

(2) In relation to capital assets being an undertaking or division transferred by way

of such sale, the “Net worth” of the undertaking or the division, as the case may

be, shall be deemed to be the cost of acquisition and the cost of improvement

for the purposes of sections 48 and 49.

(3) Every assessee, in the case of slump sale, shall furnish in the prescribed

form along with the return of income, a report of an accountant indicating the

computation of the net worth of the undertaking or division, as the case may

be, and certifying that the net worth of the undertaking or division, as the

case may be, has been correctly arrived at in accordance with the provisions

of this section.

No Indexation Benefit will be allowed.

Net worth shall be the aggregate value of total assets of the undertaking or

division (- depreciable assets - WDV - capital asset in respect of which the

whole of the expenditure has been allowed or is allowable as a deduction

under section 35AD - NIL ; - in the case of other assets, the book value of

such assets) as reduced by the value of liabilities of such undertaking or

division as appearing in its books of account ignoring any revaluation of

assets, if already done.

Notes:

* The entire department or division of an organization is sold to an

outsider under “as is where is” condition (i.e. transfer of a part of

business as a going concern; lock, stock and barrel) for a lump sum

price without values being assigned to the individual assets and

liabilities in such sale.

* The buyer uses the department or division as an organization or factory

and the combination of various assets (building, machinery plant etc.)

enables him to do so.

Page 176: INTER CA Taxation - CA Study Web€¦ · INTER CA Taxation Head Office Shraddha, 4th Floor, ... 14. Advance Tax & Interest 268 - 271 15. Tax Deducted at Source (TDS) 272 - 294 16

: 173 :

J. K. SHAH CLASSES INTER C.A.- DIRECT TAXES

CAPITAL GAINS

(E) COST OF ACQUISITION IN THE CASE OF ADVANCE MONEY RECEIVED [Sec. 51] :

Where the capital asset was, on any previous occasion, subject to negotiations for

its transfer, any advance or other money received and forfeited by the assessee

in respect of such negotiation, is to be deducted from the cost for which the asset

was acquired.

W.E.F. p.y. 2014-15, if any sum of money received as advance or otherwise in the course of

negotiations for transfer of capital asset has been forfeited by the assessee , the amount

forfeited shall be taxable under the head "Income from other sources " u/s 56 for the assesee

and hence such amount shall not be reduced as per Sec 51 for determing C.O.A of the

capital asset .

Notes: FOR ADVANCE MONEY FORFEITED PRIOR TO 1- 4-14 :

* Any advance money forfeited “by assessee “(i.e. the seller / transferer) has

to be deducted from the cost of acquisition and then the net cost (if positive)

will be indexed, if long term capital asset.

* Any advance money received and forfeited by the previous owner is not

considered in the hands of the assessee and the cost of acquisition of the

previous owner (before forfeiting) will become the cost of acquisition for the

assessee u/s 49(1).

* Such advance money forfeited by previous owner (before 1.4.2014) is not

taxable under any provisions of income tax in the hands of the previous owner.

(F) OPTION TO SUBSTITUTE THE FAIR MARKET VALUE AS ON 1. 4. 01 AS THE

COST OF ACQUISITION IN CASE THE CAPITAL ASSET, NOT BEING

DEPRECIABLE ASSET AND SELF GENERATED ASSET, IS ACQUIRED BY

THE ASSESSEE PRIOR TO 1.4.01 [Sec. 55(2)] :

The assessee has the option to take either the original cost of acquisition or the

fair market value of the asset as on 1 .4. 01 as the cost of acquisition, whichever is

favourable for him. Please note that no such option is available in respect of

depreciable assets / self generated assets.

Such option is also available to the assessee if he has acquired the capital asset

in any mode referred to in Sec. 49(1) and capital asset in question, became the

property of the previous owner before 1.4.01.

Note:

* Option not available for depreciable asset, since the identity of the asset

gets lost in the common hotch pot of the block under the block method of

depreciation.

* Option not available for self-generated asset because the cost of acquisition

is taken at ‘Nil’

Page 177: INTER CA Taxation - CA Study Web€¦ · INTER CA Taxation Head Office Shraddha, 4th Floor, ... 14. Advance Tax & Interest 268 - 271 15. Tax Deducted at Source (TDS) 272 - 294 16

: 174 :

J. K. SHAH CLASSES INTER C.A.- DIRECT TAXES

CAPITAL GAINS

(G) COST OF ACQUISITION IN THE CASE OF BONUS SHARES AND RIGHT

SHARES OFFER.

(i) Bonus Shares :

* Cost of acquisition of bonus share allotted on or after 1-4-01 will be

taken to be NIL.

* If bonus shares were allotted prior to 1-4-01, the cost of bonus shares

would be the F.M.V as at 1-4-01.

The period of holding shall commence from the date of allotment of such bonus

shares till the date of its sale.

(ii) ’Right shares offer’

Right shares offer is a capital asset u/s 2(14)

Right shares offer and subsequent renouncement of it by the assesssee, the

cost of acquisition would be deemed to be NIL.

Thus the amount received by the original share holder by selling his rights

entitlement will be short term capital gains in his hands, since the cost of

acquisition is taken at NIL.

Note:

* If the shareholder invests money himself in the rights offer, then the

rights offer, which is a capital asset u/s 2(14), gets converted into

another asset i.e. ‘Right shares’.

(H) COST OF ACQUISITION OF GOODWILL OF A BUSINESS, OR A TRADE MARK

OR BRAND NAME ASSOCIATED WITH A BUSINESS OR RIGHT TO MANU-

FACTURE, PRODUCE OR PROCESS ANY ARTICLE OR THING OR RIGHT

TO CARRY ON ANY BUSINESS.

If the capital asset which was purchased ;the cost paid for purchasing the capital asset

shall be the cost of acquisition.

Goodwill, right to manufacture, produce or process any article or thing or the right

to carry on any business or profession will be treated as capital assets, despite

they being self-generated.

In case of a self generated asset, since no cost of acquisition is incurred, while

computing capital gains, the cost of acquisition shall be taken at Nil. In other words

the sale consideration shall be the capital gains.

Page 178: INTER CA Taxation - CA Study Web€¦ · INTER CA Taxation Head Office Shraddha, 4th Floor, ... 14. Advance Tax & Interest 268 - 271 15. Tax Deducted at Source (TDS) 272 - 294 16

: 175 :

J. K. SHAH CLASSES INTER C.A.- DIRECT TAXES

CAPITAL GAINS

Goodwill exchanged by partners of a professional firm is not taxable at all.

Detailed explanation regarding cost of improvement

- Cost of improvement includes all the expenditure of capital nature in making any

addition to the capital asset which increases the value of the capital asset

(e.g.,constructing a first floor on an existing bungalow).

- Improvement cost incurred on or after 1.4.81 by previous owner and / or the

assessee will be considered only, since up to 31.3.81 the effects of improvement,

if any, would be absorbed in the F.M.V option u/s 55(2)

* If assessee acquired asset prior to 1-4-81(by way of purchase or u/s 49(1)

situations), then any improvement cost incurred by assesse only on or after

1.4.81, will be considered only.

* If assessee acquired asset on or after 1-4-81 under section 49(1)

situations, then any improvement cost incurred by previous owner and

/ or assessee on or after 1-4-81, will be considered only.

- Cost of improvement is not allowed towards goodwill, right to manufacture,

produce or process any article or thing and right to carry on business or

profession.

- If any capital expenditure incurred on or after 1-4-81 is already allowed as a deduction

under Income from business or profession, then it will not be considered.

- The holding period of improvement by itself is not considered, if the original

capital asset is long term,the improvement is long term, since improvement

is not a separate capital asset.

- Indexation on improvement cost

Any improvement done on or after 1-4-81 by the previous owner or the

assessee will be considered only and it will be divided by the index factor

of the year in which the improvement took place and multiplied by the

index factor of year of sale.

COMPUTATION OF CAPITAL GAINS IN CERTAIN SPECIAL SITUATIONS:

��� Computation of capital gain in the case of compulsory acquisition of an

asset [Sec. 45(5)].

1. Initial Compensation awarded or determined or approved on compulsory acqui-

sition of property shall be deemed to be the sale consideration and capital gains

shall be computed pertaining to the previous year in which the initial compensa-

tion or the first instalment thereof is received. Full cost of acquisition is to be

reduced from the total initial compensation in computing the aforesaid capital

gains. (Indexing to be done till the year of transfer only).

2. In case of any enhanced compensation is received subsequently, it is taken

as sale consideration and only transfer expenses i.e. litigation expenses

incurred to get enhanced compensation is allowed as a deduction. The excess

of enhanced compensation over transfer expenses, is taxable as capital

gains (short term or long term, depending upon the holding period prior to

its compulsory acquisition, by the assessee).

Page 179: INTER CA Taxation - CA Study Web€¦ · INTER CA Taxation Head Office Shraddha, 4th Floor, ... 14. Advance Tax & Interest 268 - 271 15. Tax Deducted at Source (TDS) 272 - 294 16

: 176 :

J. K. SHAH CLASSES INTER C.A.- DIRECT TAXES

CAPITAL GAINS

3. If the compensation is received by any other person, e.g. legal heir on death

of the assessee, it is taxable as income of the recipient.

4. In case the initial compensation or the enhanced compensation is

subsequently reduced by any court, Tribunal or other authority, after

assessment of capital gains, such assessed capital gain of that year shall

be recomputed by taking the compensation or consideration as so reduced

by such court, Tribunal or other authority to be the full value of consideration.

5. W.E.F. p.y. 2014-15 , any compensation received in pursuance of an interim

order of a Court , Tribunal or authority shall be deemed to be taxable in the

p.y. in which the final order of such Court , Tribunal or authority is made.

� Computation of capital gains in the case of non-resident.

1. Capital gain in case of non-resident arising on sale or transfer of listed shares /

debentures / securities in an Indian company shall be computed by converting

the cost of acquisition, expenditure in connection with such transfer and the full

value of the consideration received or accruing as a result of the transfer of the

capital asset into the same foreign currency as was initially utilised in the pur-

chase of the shares or debentures. The capital gains so computed in such a for-

eign currency shall be reconverted into Indian Currency (Rupees) by the telegraphic

transfer, BUYING RATE of such currency, as on the date of transfer of the capital asset.

2. No relief in terms of INDEXATION will be available in the aforesaid computation.

3. For the aforesaid purpose, telegraphic transfer buying / selling rate in relation to

a foreign currency is rate of exchange adopted by the State Bank of India for pur-

chasing or selling such currency where such currency, is made available by that

bank through telegraphic transfer.

4. If the long term capital gain arises on transfer of unlisted securities or shares of a

company in which the public are not substantially interested [i.e. private limited

company] to the non-resident, then it shall be taxable at a flat rate of 10% without

indexation and without conversion and reconversion into foreign currency / Indian currency.

5. For other long term capital gains, the tax rate will be flat 20% after indexation.

CAPITAL GAINS ON PURCHASE BY COMPANY OF ITS OWN SHARES OR

OTHER SPECIFIED SECURITIES [Sec. 46A].

Where a shareholder or a holder of other specified securities receives any

consideration from any company for purchase of its own shares or other specified

securities held by such shareholder or holder of other specified securities then,

subject to the provisions of section 48, the difference between the cost of

acquisition and the value of consideration received by the shareholder or the

holder of other specified securities, as the case may be shall be deemed to be

the capital gains arising to such shareholder or the holder of other specified

securities, as the case may be, in the year in which such shares or other specified

securities were purchased by the company.

Page 180: INTER CA Taxation - CA Study Web€¦ · INTER CA Taxation Head Office Shraddha, 4th Floor, ... 14. Advance Tax & Interest 268 - 271 15. Tax Deducted at Source (TDS) 272 - 294 16

J.K.SHAH CLASSES INTER C.A. – DIRECT TAXES

: 177 :

Sec. 54 Profit on Sale of House Property

Assessee entitled to claim exemption individual or HUF

Nature of asset transferred Residential House Property (covered by Sec.22)

Nature of gain LTCG

if the aforesaid conditions are satisfied, exemption shall be allowed subject to the following :

Nature of asset to be acquired One residential house in India (either purchase and / or construct)

Amount to be invested LTCG arising on transfer of the original asset

Time for making the investment If purchased – within 1 year before or 2 years from the date of transfer of original asset. If constructed – within 3 years from the date of transfer.

Capital gain account scheme 1988 If the amount is not utilized by the assessee up to the due date of ROI u/s 139, it can be deposited in the said account with any specified bank or institution, before the due date of furnishing such return.

Non utilization of amount from the deposit account

The unutilized amount shall be charged as Capital Gain of the PY in which the period of 3 years from the date of the transfer of the original asset expires.

Lock in period of the new asset The new house cannot be transferred up to a period of 3 years from the date of acquisition or construction, as the case may be.

Withdrawal of exemption if new asset is transferred within lock in period

The cost of new asset shall be reduced by the amount of exemption claimed.

Notes :

1. The unutilized deposit amount in the Capital Gains Account Scheme, 1988 in the case of an individual who dies before the expiry of the stipulated period under sections 54, 54B, 54D, 54F and 54G cannot be taxed in the hands of the deceased. This amount is not taxable in the hands of legal heirs also as the unutilized portion of the deposit does not partake the character of income in their hands but is only a part of the estate devolving upon them. [Circular No.743, dated 6-5-1996]

2. The cost of the land is an integral part of the cost of the residential house, whether purchased or built.

Sec 54B Capital gain on transfer of land used for agricultural purposes

Assessee entitled to claim exemption Individual or HUF

Nature of Asset transferred Land used for agricultural purposes by the individual himself or by any of his parents, or by HUF for at least 2 years preceding the date of transfer

nature of gain LTCG or STCG

Page 181: INTER CA Taxation - CA Study Web€¦ · INTER CA Taxation Head Office Shraddha, 4th Floor, ... 14. Advance Tax & Interest 268 - 271 15. Tax Deducted at Source (TDS) 272 - 294 16

J.K.SHAH CLASSES INTER C.A. – DIRECT TAXES

: 178 :

if the aforesaid conditions are satisfied, exemption shall be allowed subject to the following :

nature of asset to be acquired: Any land for being used for agricultural purposes

amount to be invested Capital gain arising on transfer of the original land

time for making the investment Within 2 years from the date of transfer

capital gain account scheme 1988 Applicable

non utilization of amount from the deposit account

The unutilized amount shall be charged as Capital Gain of the previous year in which the period 2 years from the date of the transfer of the original asset expires

lock in period of the new asset The new land cannot be transferred up to a period of 3 years from the date of acquisition

withdrawal of exemption if new asset is transferred within lock in period

For the purpose of computing capital gain arising from the transfer of the new asset, the cost shall be reduced by the amount of exemption claimed in respect of investment in such new asset

Sec. 54D Capital gain on compulsory acquisition of lands and buildings

Assessee entitled to claim exemption Any assessee

nature of asset transferred Land or building forming part of an industrial undertaking, used by the assessee for business purposes for at least 2 years preceding the date of transfer

nature of gain LTCG or STCG

if the aforesaid conditions are satisfied, exemption shall be allowed subject to the following :

nature of asset to be acquired any land or building or any right in land or building or construction of any building for the purposes of shifting or re-establishing the said undertaking or setting up another industrial undertaking.

Amount to be invested capital gain arising on transfer of the original land and / or building

Time for making the investment Within a period of three years from the date of receipt of compensation

Capital gain account scheme 1988 Applicable

Non utilization of amount from the deposit account

If the amount deposited is not utilized wholly or partly for the purchase / construction of the land / building within the period specified, then the amount not so utilized shall be charged as Capital Gain of the previous year in which the period of 3 years from the date of receipt of compensation expires.

Page 182: INTER CA Taxation - CA Study Web€¦ · INTER CA Taxation Head Office Shraddha, 4th Floor, ... 14. Advance Tax & Interest 268 - 271 15. Tax Deducted at Source (TDS) 272 - 294 16

J.K.SHAH CLASSES INTER C.A. – DIRECT TAXES

: 179 :

Lock in period of the new asset The new land / building cannot be transferred up to a period of 3 years from the date of acquisition

Withdrawal of exemption if new asset is transferred within lock in period

For the purpose of computing capital gain arising from the transfer of the new asset, the cost shall be reduced by the amount of exemption claimed in respect of investment in such new asset.

Sec. 54EC capital gain not to be charged on investment in certain bonds

Assessment entitled to claim exemption

any Assessee

Nature of asset transferred any capital asset

Nature of gain long term capital gain (LTCG)

If the aforesaid conditions are satisfied, exemption shall be allowed subject to the following :

Nature of asset to be acquired Bonds redeemable after three years, issued by NHAI or RECL or any bond notified by central government .

Amount to be invested LTCG arising on transfer of the original asset. (maximum investment in the PY of transfer + next P Y Rs.50 lakhs pa)

Time for making the investment Within 6 months from the date of transfer of the original asset.

Capital gain account scheme 1988 not applicable

Lock in period of the new asset the bonds cannot be transferred or converted into money up to a period of 3 years from the date of acquisition.

Withdrawal of exemption if new asset is transferred within lock in period

the amount of exemption allowed in respect of such investment shall be deemed to be the long term capital gains of the previous year in which the specified asset is transferred or converted into money.

Sec. 54F capital gain on transfer of certain capital assets not to be charged in case of investment in residential house

Assessee entitled to claim exemption individual or HUF

Nature of asset transferred any asset other than a residential house

Nature of gain long term capital gain (LTCG)

if the aforesaid conditions are satisfied, exemption shall be allowed subject to the following :

Nature of asset to be acquired one Residential House in India (either purchase or construct)

Amount to be invested net sale consideration of the original asset

Time for making the investment if purchased – within 1 year before or 2 years from the date of transfer of original asset. If constructed – within 3 years from the date of transfer.

Page 183: INTER CA Taxation - CA Study Web€¦ · INTER CA Taxation Head Office Shraddha, 4th Floor, ... 14. Advance Tax & Interest 268 - 271 15. Tax Deducted at Source (TDS) 272 - 294 16

J.K.SHAH CLASSES INTER C.A. – DIRECT TAXES

: 180 :

Capital gain account scheme 1988 applicable

Non utilization of amount from the deposit account

if the amount deposited is not utilized wholly or partly for the purchase or construction of the new house within the period specified, then (the un-utilized amount * LTCG / NSC) shall be charged as long term capital gain of the previous year in which the period of 3 years from the date of the transfer of the original asset expires

Lock in period of the new asset the new house cannot be transferred up to a period of 3 years from the date of acquisition or construction, as the case may be

Withdrawal of exemption if new asset is transferred within lock in period

the amount of exemption allowed in respect of such investment (LTCG * investment / NSC) shall be deemed to be the long term capital gains of the previous year in which the specified asset is transferred or sold.

Restriction on number of hoses no exemption shall be allowed under this section, if the assessee, -

(i) Owns more than one residential house, other than the new asset, on the date of transfer of the original asset; or

(ii) Purchases any residential house, other than the new asset, within a period of 2 year after the date of transfer of the original asset; or

(iii) Construct any residential house, other than the new asset, within a period of 3 year after the date of transfer of the original asset; or

Withdrawal of exemption on acquisition / construction of second house

where the assessee purchases, within the period of 2 years after the date of the transfer of the original asset, or constructs, within the period of 3 years after such date, any residential house, (the income from which is chargeable under the head “Income from house property”), other than the new asset, the amount of exemption allowed under this section shall be deemed to be Long Term Capital Gains of the previous year in which such residential house is purchased or constructed.

Page 184: INTER CA Taxation - CA Study Web€¦ · INTER CA Taxation Head Office Shraddha, 4th Floor, ... 14. Advance Tax & Interest 268 - 271 15. Tax Deducted at Source (TDS) 272 - 294 16

J.K.SHAH CLASSES INTER C.A. – DIRECT TAXES

: 181 :

Sec. 54G Exemption of capital gains on transfer of assets in case of shifting of industrial undertaking from urban area to rural area.

Assessee entitled to claim exemption any assessee

Nature of asset transferred capital asset, being machinery or plant or building or land or any rights in building or land used for the purposes of an industrial undertaking situate in an urban area, effected in the course of, or in consequence of, the shifting of such industrial undertaking to any rural area.

Nature of gain long term or short term capital gain (LTCG or STCG)

if the aforesaid conditions are satisfied, exemption shall be allowed subject to the following :

Nature of asset to be acquired (i) Purchase new machinery or plant for the purposes of industrial undertaking;

(ii) Acquire building or land or construct building for the purposes of industrial undertaking;

Amount to be invested capital gain arising on transfer of the original land, building, plant or machinery.

Time for making the investment within a period of 1 year or before or 3 years after the date of transfer

Capital gain account scheme 1988 applicable

Non utilization of amount from the deposit account

if the amount deposited is not utilized wholly or partly for the purchase / construction of the land, building, plant or machinery within the period specified, then the amount not so utilized shall be charged as capital gain of the previous year in which the period of 3 years from the date of receipt of compensation expires.

Lock in period of the new asset the new land, building, plant or machinery cannot be transferred up to a period of 3 years from the date of acquisition

Withdrawal of exemption if new asset is transferred within lock in period

for the purpose of computing capital gain arising from the transfer of the new asset, the cost shall be reduced by the amount of exemption claimed in respect of investment in such new asset

Page 185: INTER CA Taxation - CA Study Web€¦ · INTER CA Taxation Head Office Shraddha, 4th Floor, ... 14. Advance Tax & Interest 268 - 271 15. Tax Deducted at Source (TDS) 272 - 294 16

J.K.SHAH CLASSES INTER C.A. – DIRECT TAXES

: 182 :

Sec. 54GA Exemption of capital gains on transfer of assets in cases of shifting

of industrial undertaking from urban area to any Special Economic Zone

Assessee entitled to claim exemption any assessee

Nature of asset transferred capital asset, being machinery or plant

or building or land or any rights in

building or land used for the purposes of

the business of an industrial undertaking

situate in an urban area, effected in the

course of, or in consequence of, the

shifting of such industrial undertaking to

any Special Economic Zone.

Nature of gain long term or short term capital gain

(LTCG or STCG)

if the aforesaid conditions are satisfied, exemption shall be allowed subject to the

following :

Nature of asset to be acquired : a) Purchase machinery or plant for the

purposes of business of the

industrial undertaking in the Special

Economic Zone to which the said

undertaking is shifted;

b) Acquire building or land or construct

building for the purposes of his

business in the Special Economic

Zone;

c) Shift the original asset and transfer

the establishment of such

undertaking to the Special Economic

Zone; and

d) Incur expenses on such other

purposes as may be specified in a

scheme framed by the Central

Government for the purposes of this

section,

Amount to be invested capital gain arising on transfer of the

original land, building, plant or

machinery.

Time for making the investment within a period of 1 year or before or 3

years after the date of transfer

Capital gain account scheme 1988 applicable

Page 186: INTER CA Taxation - CA Study Web€¦ · INTER CA Taxation Head Office Shraddha, 4th Floor, ... 14. Advance Tax & Interest 268 - 271 15. Tax Deducted at Source (TDS) 272 - 294 16

J.K.SHAH CLASSES INTER C.A. – DIRECT TAXES

: 183 :

Non utilization of amount from the

deposit account

if the amount deposited is not utilized

wholly or partly for the aforesaid

purposes within the period specified,

then the amount not so utilized shall be

charged as capital gain of the previous

year in which the period of 3 years from

the date of transfer expires

Lock in period of the new asset the new land, building, plant or

machinery cannot be transferred up to a

period of 3 years from the date of

acquisition.

Withdrawal of exemption if new asset

is transferred within lock in period

for the purpose of computing capital gain

arising from the transfer of the new

asset, the cost shall be reduced by the

amount of exemption claimed in respect

of investment in such new asset.

Sec. 54 EE Exemption of capital gains on investment in certain mutual funds :

• Objective:

For incentivising the start-up ecosystem in India, the 'start-up India Action Plan'

envisages establishment of a Fund of Funds which intends to raise ` 2,500

crores annually for four years to finance the start-ups.

• Exemption of LTCG invested in units of specified fund:

In order to achieve this objective, new section 54EE has been inserted to provide

exemption from capital gains tax if the long term capital gains proceeds are

invested by an assessee in units issued before 1st April, 2019 of such fund, as

may be notified by the Central Government in this behalf. The lower of the

capital gains or the amount so invested would be exempt under section 54EE.

Quantum of Exemption:

Case Amount exempted

If amount invested in notified units of specified

fund ≥ Capital gains Entire capital gains is exempt

If amount invested in notified units of specified

fund < Capital gains Capital gains to the extent of cost of amount

invested in notified units is exempt

• Time limit for investment:

The investment has to be made within 6 months after the date of transfer.

Page 187: INTER CA Taxation - CA Study Web€¦ · INTER CA Taxation Head Office Shraddha, 4th Floor, ... 14. Advance Tax & Interest 268 - 271 15. Tax Deducted at Source (TDS) 272 - 294 16

J.K.SHAH CLASSES INTER C.A. – DIRECT TAXES

: 184 :

• Ceiling limit for investment in units of the specified fund:

The maximum investment in units of the specified fund in any financial year is `

50 lakh. Further, the investment made by an assessee in the units of specified

fund out of capital gains arising from the transfer of one or more capital assets,

cannot exceed ` 50 lakh, whether the investment is made in the same financial

year or subsequent financial year or partly in the same financial year and partly

in the subsequent financial year.

• Consequence of transfer of units before 3 years:

Where the units are transferred at any time within a period of three years from

its acquisition, the capital gains, to the extent exempt earlier, would be

chargeable as capital gains in the year of transfer.

• Deemed transfer of notified units:

Further, if the assessee takes any loan or advance on the security of such

units, he shall be deemed to have transferred such units on the date on which

such loan or advance is taken.

Page 188: INTER CA Taxation - CA Study Web€¦ · INTER CA Taxation Head Office Shraddha, 4th Floor, ... 14. Advance Tax & Interest 268 - 271 15. Tax Deducted at Source (TDS) 272 - 294 16

: 185 :

J. K. SHAH CLASSES INTER C.A. - DIRECT TAXES

CAPITAL GAINS

Sec.54GB : Transfer of residential property [house / plot of land] and investment in

equity shares of an eligible company.

I. Available to : Individual & HUF (referred to as "the assessee")

II. What is to be invested : Net Sale Consideration [Sale consideration –

Transfer expenses]

III. Qualifying asset (long term or both) : Long term

IV. Time Limit for Purchase: Subscription to equity shares before the due

date of filing return of income u/s 139(1) applicable for the assessee.

V. Time Limit for Completion of Construction: N.A.

VI. Case of Non utilization : L.T.C.G.

VII. Case of Misutilisation : Residential Property : LTCG,

Equity Shares : Both,

“New Asset” : STCG

VIII. Special conditions :

1. The Company should be incorporated in the relevant P.Y. till the

due date of filing return of income u/s 139(1) for that relevant P.Y.

The company is engaged in business of manufacture of article or

thing and qualifies to be a “Small or Medium enterprise”.

2. The share capital/ voting rights of the assessee in this company

after subscription to these shares should exceed 50%.

3. The “new asset” should be purchased by the company within one year

from the date of subscription in equity shares by the assessee.

4. The equity shares of the company as well as the new asset

acquired by the company should be retained for a period of five

years from the date of their respective acquisition, else the

provision of misutilisation would be attracted.

IX. Remarks :

1. (a) If the amount invested in acquisition of “new asset” by the

company is less than the net sale consideration then :

(b) If the amount invested in acquisition of “new asset” by the

company is equal to or more than the net sale consideration,

the Gross LTCG is fully exempt from tax u/s 54GB.

Page 189: INTER CA Taxation - CA Study Web€¦ · INTER CA Taxation Head Office Shraddha, 4th Floor, ... 14. Advance Tax & Interest 268 - 271 15. Tax Deducted at Source (TDS) 272 - 294 16

: 186 :

J. K. SHAH CLASSES INTER C.A. - DIRECT TAXES

CAPITAL GAINS

2. Case of non-utilisation

Exemption allowed

Taxable LTCG = u/s 54 GB

x unutilized amount

Total investment i.e. cost in the deposit scheme

of new asset + amount of deposit

This amount shall be taxable for the assessee in the year in

which the period of 1 year from the date of subscription in

equity shares expires.

3. Case of Misutilisation

In case of misutilisation, the amount of LTCG, which was exempt

earlier for the assessee, shall be taxable as LTCG for the assessee

in the year of misutilisation and the capital gain arising on the

transfer of equity shares (for the assessee) and / or "new asset" (for

the company) shall also be taxable in the year of misutilisation.

4. " New Asset" means new plant and Machinery which satisfies all the

conditions for claim of additional depreciation u/s 32. In the case of

eligible start-up, it also includes computers and computer software.

5. This exemption does not apply to any transfer of residential property

made after 31-3-2017. For eligible start-ups, this section shall apply

upto 31-3-2019

6. "Eligible business" means a business which includes innovation,

development, deployment or commercialisation of new products,

processes, or services driven by technology or intellectual property.

7. "Eligible start up" Means a company incorporated on or after 1.4.2016

but before 1.4.2019 and the turnover in any P.Y. between 2016-17 to

2020-21 does not exceed ` ` ` ` 25 crores.

Page 190: INTER CA Taxation - CA Study Web€¦ · INTER CA Taxation Head Office Shraddha, 4th Floor, ... 14. Advance Tax & Interest 268 - 271 15. Tax Deducted at Source (TDS) 272 - 294 16

: 187 :

J. K. SHAH CLASSES INTER C.A. - DIRECT TAXES

CAPITAL GAINS

CAPITAL GAINS ACCOUNT SCHEME

If the investment in the new asset has not been made by the assessee upto the date of

filing of return of income for that P.Y., the assessee shall transfer the capital gain to a

special bank account as per this scheme on or before the date of filing return of income.

The amount so transferred to the special bank account shall be considered as “cost of

the new asset” for calculation of exemption.

The amount in the special bank account should be utilized by the assessee for investing in the

new asset after the date of filing return but upto the time limit specified in the section.

If any amount remains unused in the special bank account, it is termed as a case of

“non-utilization” and the unutilized amount shall be taxable as capital gain in the year in

which the time limit of the section expires.

Any interest accrued on the amount deposited in the special bank account shall be taxable for

the assessee as “Income from Other Sources.”

EXTENSION OF TIME LIMIT FOR ACQUIRING NEW ASSET. [Sec. 54 H]

Section 54 H been inserted to provide that where the transfer of the original asset is by way of

compulsory acquisition under any law and the amount of compensation awarded for such acquisi-

tion is not received by the assessee on the date of such transfer, the period of acquiring the new

asset under section 54, 54B [Compulsory Acquisition by State Government] 54D, 54 EC and 54F

by the assessee or the period for depositing or investing the amount of capital gain shall be ex-

tended in relation to such amount of compensation as is not received on the date of transfer. The

extended period shall be reckoned from the date of receipt of the amount of compensation.

Page 191: INTER CA Taxation - CA Study Web€¦ · INTER CA Taxation Head Office Shraddha, 4th Floor, ... 14. Advance Tax & Interest 268 - 271 15. Tax Deducted at Source (TDS) 272 - 294 16

: 188 :

J. K. SHAH CLASSES INTER C.A. - DIRECT TAXES

CAPITAL GAINS

Q. 1. X had bought a car in 2010 for `̀̀̀ 85,000 and used it for business purposes. He

sold the car in March, 2018 for `̀̀̀55,000 after deducting depreciation of `̀̀̀50,000

claimed by him,has he made a short term capital gain of `̀̀̀20,000?

Q. 2. X owns Gold Deposit Certificates, 2015 worth `̀̀̀25 lakhs. Y owns shares of Hindustan

lever whose market price is `̀̀̀25 lakhs. X & Y swap their investments, since the transfer

of property does not result in any gain, nothing is taxable in the hands of X & Y.

Q. 3. Discuss the tax rates applicable in the following cases

Sr. Capital Asset STCG LTCG

1 Property

2 Listed Debentures

3 Unlisted equity shares

4 Listed preference shares

5 Zero Coupon Bonds

6 Listed shares of a company relinquished

on liquidation

7 Unlisted debentures of a company

8 Urban Agricultural Land

9 Listed equity shares of a company sold

through BSE.

10 Units of equity oriented mutual fund sold

through BSE.

11 Units of debt oriented mutual fund sold

through BSE.

12 Bonus equity shares of a listed company

allotted on

21-9-2009 and sold on 16-2-2018 outside BSE

13 Gold Bonds

Q. 4. X acquires a capital asset on April 1, 2007 for `̀̀̀ 40,000. he converts the capital asset

into stock-in-trade on April 1, 2009 (fair market value on the day of conversion 1,62,000).

The stock-in-trade is sold by X on March 10, 2018 for ̀̀̀̀ 5,86,000. Determine the amount

of chargeable profit.

Q. 5. X intends to sell his house property to Y. The consideration is fixed at `̀̀̀10 lakhs

and the possession is given to Y. Half the consideration has already paid by by Y

and the balance is to be paid shortly. However no sale deed has been executed till

31.03.2018. Nevertheless, for purposes of Sec.45, X has transferred the property.

Is this statement correct?

CLASS WORK PROBLEMS

Page 192: INTER CA Taxation - CA Study Web€¦ · INTER CA Taxation Head Office Shraddha, 4th Floor, ... 14. Advance Tax & Interest 268 - 271 15. Tax Deducted at Source (TDS) 272 - 294 16

: 189 :

J. K. SHAH CLASSES INTER C.A. - DIRECT TAXES

CAPITAL GAINS

Q. 6. On 29th August, 2003, G acquired 500 shares of `̀̀̀100 each for `̀̀̀ 60,000 in Blue Ltd.

On 23rd March, 2017, the company by resolution went in to voluntary liquidation. G

received `̀̀̀1,90,000 in May, 2017 as final settlement from the company. There were no

accumulated profits with the company immediately before liquidation. Decide the

taxability of the receipt in the hands of G.

Q. 7. Ramdev HUF is partitioned in 03-04 and assets distributed to both Ram & Dev

equally properties worth `̀̀̀50 lakhs each, the properties had been acquired by HUF

in 1961 at `̀̀̀1 lakh (fmv.1.4.01 - 25,00,000). Ram sells off his property in March,

2018 for `̀̀̀ 2 crore.Calculate the amount of Capital Gains chargeable to tax in the

hands of H.U.F. and Ram.

Q. 8. X and Y are two partners of a firm : A Co. On June 1, 2017 B joins the firm and brings listed

shares in a company as his capital contribution. Fair market value of these shares (listed)

on June 1, 2017 is `̀̀̀1,86,000, whereas amount credited in B's account in the firm is

`̀̀̀ 2,20,000. Assuming that cost of acquisition in 2007-2008 of these shares is `̀̀̀ 45,000,

find out the amount of chargeable capital gain for the assessment year 2018-19.

Q. 9. A Co. (a firm which deals in chemical goods and has partners: X, Y and Z) acquires

gold on May 10,2004 for `̀̀̀ 40,000. This is taken over at the time of dissolution by Y on

March 31, 2018. Though on March 31st, 2018 its market value is `̀̀̀ 5,60,000, agreed

value as per dissolution deed is `̀̀̀ 6,00,000. Determine the amount of capital gain

chargeable to tax for the assessment year 2018-19.

Q. 10.X purchases debentures in A Ltd. on January 1, 2005 for `̀̀̀ 20,000. He gifts these

debentures to his friend Y, on June 10, 2005 (fair market value on June 10, 2005.

`̀̀̀33,000). Y dies on March 13, 2007 and as per his will, debentures are

transferred to his son Z (Fair market value on March 13.2007, `̀̀̀ 40,000). Z sell

these debentures on June 10, 2017 for `̀̀̀ 86,000. Determine the amount of capital

gain arising to X, Y and Z from the aforesaid transactions.

Q. 11. X gets shares from his employer - ABC Ltd., under ESOP (whose market value at

acquisition was `̀̀̀ 3,00,000) for `̀̀̀ 80,000 on 4.4.2017.

X sells such shares on 7.1.18 for `̀̀̀ 7,00,000, when its FMV was 9,00,000. Com-

pute effects. Describe the changes if :

Such shares were gifted on 7.1.18.

Such shares were given by way will in P.Y. 17-18.

Page 193: INTER CA Taxation - CA Study Web€¦ · INTER CA Taxation Head Office Shraddha, 4th Floor, ... 14. Advance Tax & Interest 268 - 271 15. Tax Deducted at Source (TDS) 272 - 294 16

: 190 :

J. K. SHAH CLASSES INTER C.A. - DIRECT TAXES

CAPITAL GAINS

Q. 12.X, a non-resident, remits US $ 40,000 to India on April 4, 2007. This amount is

partly utilised in purchasing the following assets on April 10, 2007.

Assets Quantity Amount

`̀̀̀

Silver 10 Kg. 30,000

Listed shares in an Indian Company 2,000 24,000

(Sold privately)

X transfers these assets on May 10, 2017 for total consideration of `̀̀̀16,00,000

(silver : `̀̀̀ 3,70,000; shares : `̀̀̀ 12,30,000) transfer expenses incurred for shares

`̀̀̀ 22,000/-. Find out the amount of capital gains chargeable to tax for the

assessment year 2018-2019, on the assumption that telegraphic transfer buying /

selling rate of US dollar adopted by State Bank of India is as follows:

Buying Selling

(1 US $) (1 US $)

` `` `` `` `

April 10, 2007 27.50 28.10

May 10, 2017 48.30 49.10

Q. 13.X had purchased convertible debenture of Essar Gujarat in March, 2007 at `̀̀̀100

per debenture. The debenture were converted to 2 equity shares in April, 2016.

The market price in April, 2016 was `̀̀̀80 per share. The shares were sold in June,

2017 at `̀̀̀120 per share. Compute Capital Gains.

Q. 14. K had purchased 100 shares of ICICI Bank in March, 2003 at ̀̀̀̀ 200 per share. He gets a

bonus in March, 2006 in the ratio of 1 : 1 and again in March, 2007 in ratio of 1 : 1. He sells

off 300 shares in June, 2017 at `̀̀̀1,200 per share. Securities transaction tax is not

attracted.Compute Capital Gains.

Q. 15.Mr. P holds 500 shares of ABC Ltd., which were allotted to him on 22.4.2005 @

`̀̀̀30 per share. On 22nd July, 2017 ABC Ltd., made right issue to the existing share-

holders at the rate of one share for every five shares held @ `̀̀̀100 per share. Mr. P,

instead of exercising his right to obtain right shares, has exercised his right of

renouncement by renouncing the said right entitlement in favour of Mr. Q @ `̀̀̀70

per right share entitlement on 4th August, 2017.

Determine the nature and acquisition of capital gain, if any, taxable in the hands of

Mr. P. What is the C.O.A. of the shares for Mr. Q?

Q. 16.X purchases a property on April 1, 1995 for `95,000. He enters into agreement for

sale of the property to A on November 1, 2005 and receives `10,000 as advance.

A could not, however keep his promise and advance of `10,000 given by him is

forfeited by X. Later on he gifts the property to his friend Y on May 15, 2007. The

following expenses are incurred by X and Y for renewal of the property :

Page 194: INTER CA Taxation - CA Study Web€¦ · INTER CA Taxation Head Office Shraddha, 4th Floor, ... 14. Advance Tax & Interest 268 - 271 15. Tax Deducted at Source (TDS) 272 - 294 16

: 191 :

J. K. SHAH CLASSES INTER C.A. - DIRECT TAXES

CAPITAL GAINS

Cost ( `̀̀̀)

Addition of two rooms by X during 1999-2000 25,000

Addition of first floor by X during 2004-05 40,000

Addition of second floor by Y during 2009-10 1,25,000

Fair market value of the property on April 1, 2001 is `1,15,000.

Y enters into an agreement to sell the property for `12,50,000 to B on April 1, 2011

after receiving an advance of ` 50,000. B could not pay the balance within the

stipulated time of two months and Y forfeits the advance ` 50,000 as per

agreement with B. Y ultimately finds a buyer in C to whom property is transferred

for ` 73,75,000 on December 1, 2017. The Valuation adopted by the registration

authorities for charge of stamp duty was `76,50,000 which was not contested by

the buyer, but as per the assessee's request, the Assessing Officer made a

reference to valuation officer. The value adopted by valuation officer was

`75,62,000/- Brokerage at 1% of sale consideration was paid by Mr. Y to Mr. H.

Compute the capital gain chargeable to tax in the hands of Y.

Q. 17. Mr. A is proprietor of Chirag Enterprises having 2 units. He transferred on 1.4.2017 his unit

1 by way of slump sale for a total consideration of ̀̀̀̀ 25 Lacs. The expenses incurred for this

transfer were ̀̀̀̀ 28,000/-. His Balance Sheet as on 31.3.2017 is as under :

Liabilities Total Assets Unit 1 Unit 2 Total

` ` ` `` ` ` `` ` ` `` ` ` `

Own Capital 15,00,000 Building 12,00,000 2,00,000 14,00,000

Revaluation Reserve 3,00,000 Machinery 3,00,000 1,00,000 4,00,000

(for building of Unit 1) Debtors 1,00,000 40,000 1,40,000

Bank Loan 2,00,000 Other Assets 1,50,000 60,000 2,10,000

(70% for Unit I)

Trade Creditors 1,50,000

(25% for Unit 1)

TOTAL 21,50,000 TOTAL 17,50,000 4,00,000 21,50,000

Other Information :

(i) Revaluation reserve is created by revising upward the value of the building

of Unit 1.

(ii) No individual value of any asset is considered in the transfer deed.

(iii) Other assets of Unit I include patents acquired on 1.7.2015 for `̀̀̀50,000/- on

which no depreciation has been charged. (Depreciation rate 25%).

Compute the capital gain for the assessment year 2018-2019.

Page 195: INTER CA Taxation - CA Study Web€¦ · INTER CA Taxation Head Office Shraddha, 4th Floor, ... 14. Advance Tax & Interest 268 - 271 15. Tax Deducted at Source (TDS) 272 - 294 16

: 192 :

J. K. SHAH CLASSES INTER C.A. - DIRECT TAXES

CAPITAL GAINS

Q. 18.The Government of Kerala acquires a commercial building owned by X Ltd. on

March 10,2006. X Ltd. receives first instalment of `̀̀̀ 4,20,000 of the initial

compensation of `̀̀̀14,00,000 on September 25, 2009 (cost of acquisition on May

5, 2002 `̀̀̀ 6,70,000). On appeal of X Ltd. the Kerala high court increased the

compensation to `̀̀̀ 26,25,000. The additional compensation `̀̀̀12,25,000 is

received by X Ltd. on May 6, 2017, X Ltd. incurred litigation expenses of `̀̀̀1,15,000

to get the enhanced compensation. Find out the capital gain chargeable to the tax

for the assessment year 2006-07, 2010-2011 and 2018-2019.

Q. 19.Mr. Malik owns a factory building on which he had been claiming depreciation for

the past few years. It is the only asset in the block. The factory building and land

appurtenant thereto were sold during the year. The following details are available.

`̀̀̀

Building completed in September, 2005 for 10,00,000

Land appurtenant thereto purchased in April, 2003 for 12,00,000

Advance received from a prospective buyer for land in May, 2004, 50,000

forfeited in favour of assessee, as negotiations failed

WDV of the building block as on 1.4.2017 8,74,800

Sale value of factory building in November 2017 8,00,000

Sale value of appurtenant land 46,00,000

The assessee is ready to invest in long - term specified assets under section 54EC,

within specified time in order to bring the tax liability to NIL.

Compute the amount of taxable capital gain for the assessment year 2018-19 and the

amount to be invested under section 54EC for availing the maximum exemption.

Q. 20.Mr. Subash Nagre had purchased a plot of land in Aug. 1999 for `̀̀̀12,00,000/-.

This land was sold by him on 19 June 2017 for `̀̀̀47,00,000/-. On sale of this land,

he had to pay brokerage of 1.5% of the sale consideration. The stamp valuation

authority assessed the value of the land at `̀̀̀51,00,000 which was neither disputed

by the buyer nor by him.

On 6th Oct. 2017, he incorporated a private limited company in which he held

70.14% of the total equity share capital. These shares were purchased by him by

utilizing the net sale consideration from sale of land. The shares were allotted on

17 Dec. 2017 to all the shareholders.

The private limited company is engaged in manufacturing fertilizers. It purchased

new machinery worth `̀̀̀40,00,000 by utilizing the share money received from

Mr. Nagre. The machinery was purchased and put to use on 29th January, 2018.

Compute the amount of exemption under section 54GB available to Mr. Nagre for

A.Y. 2018-2019.

Describe the effects if –

1. The equity shares are sold by him on 31 Jan 2019; or

2. The machinery is sold by the company on 20th June, 2025.

Page 196: INTER CA Taxation - CA Study Web€¦ · INTER CA Taxation Head Office Shraddha, 4th Floor, ... 14. Advance Tax & Interest 268 - 271 15. Tax Deducted at Source (TDS) 272 - 294 16

: 193 :

J. K. SHAH CLASSES INTER C.A. - DIRECT TAXES

CAPITAL GAINS

Q. 21.Mr. A is an individual carrying on business. His stock and machinery were

damaged and destroyed in a fire accident in December, 2017.

The value of stock lost (total damaged) was `̀̀̀ 6,50,000. Certain portion of the

machinery could be salvaged. The opening WDV of the block as on 1.4.2017 was

`̀̀̀ 10,80,000.

During the process of safeguarding machinery and in the fire fighting operations,

Mr. A lost his gold chain and a diamond ring, which he had purchased in April,

2008 for `̀̀̀1,20,000. The market value of these two items as on the date of fire

accident was `̀̀̀ 4,00,000.

Mr. A received the following amounts from the insurance company :

(i) Towards loss of stock `̀̀̀ 4,80,000

(ii) Towards damage of machinery `̀̀̀ 6,00,000

(iii) Towards gold chain and diamond ring `̀̀̀ 4,00,000

You are requested to briefly comment on the tax treatment of the above three items

under the provisions of the Income - tax Act, 1961.

Q. 22.Mr. ‘X’ furnishes the following data for the previous year ending 31.3.2018:

(a) Unlisted Equity Shares of AB Ltd., 10,000 in number were sold on 31.5.2017,

at `500 for each share.

(b) The above shares of 10,000 were acquired by ‘X’ in the following manner:

(i) Received as gift from his father on 1.6.2000 (5,000 shares) the fair

market value on 1.4.2001 `50 per share.

(ii) Bonus shares received from AB Ltd. on 21.7.2005 (2,000 shares).

(iii) Purchased on 1.2.2004 at the price of ̀ 125 per share (3,000 shares).

(c) Purchased one residential house at `37 lakhs, on 1.5.2018 from the sale

proceeds of shares.

(d) ‘X’ is already owning a residential house, even before the purchase of above

house.

You are required to compute the taxable capital gain. He has no other source of

income chargeable to tax.

Q.23.Mr. Ayush Goenka owns two houses at Amritsar and Pune. He transfers the

following long term assets during P.Y. 2017-2018:

Residential Gold Silver

property at Pune

Date of sale 10.4.2017 20.4.2017 24.4.2017

Sale consideration 10,00,000 8,00,000 6,00,000

Indexed cost of acquisition 4,00,000 7,00,000 2,50,000

No transfer expenses were incurred for any sale. Mr. Ayush purchases the

following assets:

(i) Residential property at Mumbai for Rs. 7,00,000 on October 11, 2017.

(ii) Bonds of National Highways Authority of India for Rs. 2,50,000 on 27th

September 2017.

Compute the amount of capital gains chargeable to tax for A.Y. 2018-19.

Page 197: INTER CA Taxation - CA Study Web€¦ · INTER CA Taxation Head Office Shraddha, 4th Floor, ... 14. Advance Tax & Interest 268 - 271 15. Tax Deducted at Source (TDS) 272 - 294 16

: 194 :

J. K. SHAH CLASSES INTER C.A. - DIRECT TAXES

CAPITAL GAINS

Ans. 1. Assuming that the motor car is the only asset in the block, on sale of the motor

car, the block ceases to exist. The capital gain arising in this situation is always

short term even if the asset has been actually held for more than 36 months. This is

because once an asset enters the block; it loses its identity, individuality and

personality in the block. All calculations are made on the basis of “value” of the

block and such value keeps on changing every year (due to depreciation). This

implies that the value has been held for less than 36 months (1 year) and

accordingly the gain or loss shall be always be treated as short term

Ans. 2.True for Mr. X but False for Mr. Y, Gold Deposit Certificates 2015 are not consid-

ered as “capital asset” u/s. 2(14) and hence any profit arising on exchange of the Gold

Bonds is not taxable for Mr. X.

Shares of Hindustan Lever are capital assets as per Section 2(14) and Exchange is a

method of Transfer as per Section 2(47). Capital gain shall be taxable for Mr. Y.

Ans.5.False, Sec 53A of transfer of property Act would be attracted if all the conditions

Mentioned below are satisfied.

(i) There should be a contract for consideration in writing

(ii) The transferee has obtained possession of the property

(iii) The transferee has performed or agreed to perform his part of the contract.

In the given case the sale deed has not been executed till 31st March, 2017. This

implies that the contract for sale is not “in writing” and therefore it cannot be treated as

a transfer u/s 2(47). Therefore, capital gains is not taxable in P.Y. 2016-17. As and

when Mr X and Mr Y execute the contract, capital gain will be taxable for Mr X in that year.

Ans.9.When a partnership firm distributes a capital asset to the partner (on dissolution or

otherwise), the F.M.V. of the capital asset on the date of transfer shall be taken as Full

Value of consideration for the firm. The taxable capital gain for the firm is as follows:

Particulars `̀̀̀

Full value of consideration 5,60,000

Less: Indexed Cost of Acquisition

(LTCA as h.p. > 36 months from May. 2004 to Mar. 2018)

(40,000 x )

Taxable LTCG

CLASS WORK SOLUTIONS

Page 198: INTER CA Taxation - CA Study Web€¦ · INTER CA Taxation Head Office Shraddha, 4th Floor, ... 14. Advance Tax & Interest 268 - 271 15. Tax Deducted at Source (TDS) 272 - 294 16

: 195 :

J. K. SHAH CLASSES INTER C.A. - DIRECT TAXES

CAPITAL GAINS

Ans. 12. Mr. X

Computation of Capital Gains for A.Y. 2017-18

a) Silver

Full Value of Consideration 3,70,000

Less : Indexed Cost of Acquisition

(LTCA as hp > 36m from 10-4-2007 to 10-5-2017)

30,000 x

LTCG taxable u/s 112

Tax @ 20% =

b) Listed Shares in an Indian Company $

Full Value of Consideration

12,30,000 ÷ 48.30 + 49.10 25,256.67

2

Less : Cost of Acquisition

(LTCA as hp > 12m from 10.4.2007 to 10.5.2017)

24,000 ÷ 27.50 + 28.10 (863.31)

2

Less : Expenditure in Connection with Transfer

22,000 ÷ 48,30 + 49.10 (451.75)

2

LTCG 23,941.61

LTCG (in INR) = LTCG (in $) x Buying Rate on 10-5-16

= $ 23,941.61 x `̀̀̀ 48.30 / $

= `̀̀̀ 11,56,380/-

Page 199: INTER CA Taxation - CA Study Web€¦ · INTER CA Taxation Head Office Shraddha, 4th Floor, ... 14. Advance Tax & Interest 268 - 271 15. Tax Deducted at Source (TDS) 272 - 294 16

: 196 :

J. K. SHAH CLASSES INTER C.A. - DIRECT TAXES

CAPITAL GAINS

Ans. 14. False.

March 2003 Shares purchased @ `̀̀̀ 200/ share 100 shares

March 2006 Bonus 1 : 1 100 shares

200 shares

March 2007 Bonus 1 : 1 200 shares

Total 400 shares

June 2017 Sale of shares @ `̀̀̀ 1,200/- share (300 shares)

Balance 100 shares

FIFO method

In this case, the 100 original shares purchased in March 2003, 100 bonus shares

allotted in March 2006 and 100 bonus shares allotted in March, 2007 have been

sold in June 2017. The LTCG on 100 original shares is exempt from tax

u/s. 10(36). The LTCG on 200 bonus shares is taxable as follows :

Taxable LTCG = 200 bonus shares x (`̀̀̀ 1,200 -- NIL) = `̀̀̀ 2,40,000/-

Tax u/s. 112 under option 2 = `̀̀̀ 2,40,000 x 10% = `̀̀̀ 24,000/-

LIFO method

In this case, all the bonus shares allotted in March 2006 and March 2007

have been sold in June 2017. The LTCG is taxable as follows:

Taxable LTCG = 300 bonus shares x (`̀̀̀ 1,200 -- NIL) = `̀̀̀ 3,60,000/-

Tax u/s. 112 under option 2 = `̀̀̀ 3,60,000 x 10% = `̀̀̀ 36,000/-

Ans. 15. When Mr. P renounces the right entitlement in favour of Mr. Q, it results into

“relinquishment of the asset” for Mr. P. The capital gain for Mr. P and the

cost of acquisition for Mr. Q are calculated as follows :

Particulars `̀̀̀

Full value of consideration

(100 right share entitlement x ̀̀̀̀ 70/ entitlement) 7,000

Less:Cost of Acquisition

(STCA as h.p. < 12 months from 22-7-2017 to 4-8-2017)

COA of right renounced NIL

Taxable STCG 7,000

C.O.A. for Mr. Q

Cost of the share (paid to ABC Ltd.) `̀̀̀ 100/ share

Add: Cost of the Right (paid to Mr. P) `̀̀̀ 70/ share

`̀̀̀ 170/ share

X Number of shares 100 shares

C.O.A. of the shares `̀̀̀ 17,000

Page 200: INTER CA Taxation - CA Study Web€¦ · INTER CA Taxation Head Office Shraddha, 4th Floor, ... 14. Advance Tax & Interest 268 - 271 15. Tax Deducted at Source (TDS) 272 - 294 16

: 197 :

J. K. SHAH CLASSES INTER C.A. - DIRECT TAXES

CAPITAL GAINS

Ans.18. P.Y. 2005-2006, A.Y. 2006-2007

Nothing shall be taxable for X Ltd. as this is the year of compulsory

acquisition. Capital gain shall be taxable in the year in which the initial

compensation (or any installment) is received for the first time.

P.Y. 2009-2010, A.Y. 2010—2011

Particulars `̀̀̀

Full value of consideration

(Always Initial compensation AWARDED/ DETERMINED) 14,00,000

Less: Indexed Cost of Acquisition

(L.T.C.A. as h.p. > 36 months from 5-5-1992 to 10-3-1998)

6,70,000 x

TAXABLE LTCG u/s. 112 @ 20%

P.Y. 2017-2018, A.Y. 2018—2019

Particulars `̀̀̀

Full value of consideration

(Always Enhanced compensation awarded) 12,25,000

Less: Indexed Cost of Acquisition NIL

Less: Indexed Cost of Improvement NIL

Less: Transfer expenses

Litigation expenses (1,15,000)

TAXABLE LTCG u/s. 112 @ 20% 11,10,000

Ans.17. Mr. A

Computation of Capital Gains for A.Y. 2018-2019

Particulars `̀̀̀

Full Value of Consideration 25,00,000

Less: Expenditure in connection with transfer (28,000)

Less: Cost of Acquisition (W.N.) (12,50,625)

TAXABLE STCG/LTCG 12,21,375

W.N. Cost of Acquisition- Net Worth of Unit 1

Page 201: INTER CA Taxation - CA Study Web€¦ · INTER CA Taxation Head Office Shraddha, 4th Floor, ... 14. Advance Tax & Interest 268 - 271 15. Tax Deducted at Source (TDS) 272 - 294 16

: 198 :

J. K. SHAH CLASSES INTER C.A. - DIRECT TAXES

CAPITAL GAINS

Particulars ` `` `` `` `

Assets related to Unit 1

- Building (excluding revaluation reserve) 9,00,000

- Machinery 3,00,000

- Debtors 1,00,000

- Other Assets (excluding Patents) 1,00,000

- Patents (See working below) 28,125 14,28,125

Less: Liabilities related to Unit 1

- Bank Loan (2,00,000 x 70%) 1,40,000

- Trade Creditors (1,50,000 x 25%) 37,500 (1,77,500)

NET WORTH 12,50,625

Patents – WDV on 1-4-2017 (Depreciation rate 25%)

Actual Cost on 1-7-2015 50,000

Less: Depreciation u/s. 32 for

- P.Y. 2015-2016 (50,000 x 25%) 12,500

- P.Y. 2016-2017 [(50,000-12,500) x 25%] 9,375 (21,875)

W.D.V. on 1-4-2017 28,125

Ans.19. Mr. Malik

Computation of capital gains for A.Y. 2018-2019

i. Building- Depreciable asset

Particulars `̀̀̀

Full value of consideration 8,00,000

Less: Cost of Acquisition

BLOCK CEASES TO EXIST- Sec 50(2)

Opening WDV 8,74,800

Add: Additions to the block during the year NIL (8,74,800)

STCL (74,800)

ii. Land- Non Depreciable asset

Particulars `̀̀̀

Full value of consideration 46,00,000

Less: Indexed Cost of Acquisition

(L.T.C.A. as h.p.> 36 months from April

2000 to Nov 2016)

Cost (as incurred) 12,00,000

Less:Advance money forfeited (Sec 51) (50,000)

COA eligible for indexation 11,50,000

11,50,000 x

Gross LTCG

Less: Exemption u/s 54EC Note

Taxable LTCG

Page 202: INTER CA Taxation - CA Study Web€¦ · INTER CA Taxation Head Office Shraddha, 4th Floor, ... 14. Advance Tax & Interest 268 - 271 15. Tax Deducted at Source (TDS) 272 - 294 16

: 199 :

J. K. SHAH CLASSES INTER C.A. - DIRECT TAXES

CAPITAL GAINS

Note: Investment required in Bonds- Section 54EC

Gross LTCG on Land

Less: STCL on Building, set off

Exemption required

Investment in bonds

(rounded to multiples of `̀̀̀ 1,000 as face value of the Bond is `̀̀̀ 1,000)

Ans.20. i. Sale of equity shares on 31st January 2019

If the equity shares are sold on 31.1.2019, it implies that the shares

have not been retained for 5 years from the date of their acquisition

(Misutilization). Accordingly, the LTCG on sale of land which was

claimed exempt u/s. 54GB in P.Y. 2017-18 shall now become taxable

in P.Y. 2018-2019. Additionally, the STCG on sale of equity shares

shall also be taxable for the assessee in P.Y. 2018-2019.

ii. Sale of Machinery on 20th June 2025

If the machinery is sold on 20-6-2025, the provisions of misutilization

are not applicable as the machinery has been retained by the

company for 5 years from the date of its acquisition. The exemption

u/s. 54GB shall not be withdrawn for the assessee.

On sale of the machinery by the company in P.Y. 2025-2026, if the block

of machinery turns negative or the block ceases to exist, the STCG on the

machinery shall be taxable for the company. If the block continues to exist

after the sale, the profit or loss on sale of this individual machinery shall

not be recognized separately as per the block system of depreciation

(Section 32). The profit or loss on sale of this individual machine shall be

absorbed in the calculation of the block.

Ans.21. The tax treatment of the various insurance claims received by Mr. A are as follows :-

(i) Towards Stock

The amount received as insurance claim towards loss of stock is taxable

under the head “Profits and Gains of Business or Profession”u/s 28.

The value of stock lost by fire will be allowed as a deduction u/s 37(1) while

computing business income.

(ii) Towards Machinery

Full value of consideration [Amount received u/s 45(1A)] 6,00,000

Less : Cost of acquisition u/s 50(2)

(Block ceases to exist)

Opening WDV 10,80,000

Add : Additions during the year NIL (10,80,000)

S.T.C.L. (4,80,000)

(iii) Towards gold chain and diamond ring

Full value of consideration 4,00,000

Less : Indexed cost of acquisition

L.T.C.A. as h.p. > 36 months

from April 2007 to December 2017

1,20,000 x

L.T.C.G. u/s 112

Note : S.T.C.L. shall be set off against L.T.C.G. and the balance S.T.C.L. shall be carried

forward for maximum 8 assessment years.

Page 203: INTER CA Taxation - CA Study Web€¦ · INTER CA Taxation Head Office Shraddha, 4th Floor, ... 14. Advance Tax & Interest 268 - 271 15. Tax Deducted at Source (TDS) 272 - 294 16

: 200 :

J. K. SHAH CLASSES INTER C.A. - DIRECT TAXES

CAPITAL GAINS

Ans.22. Computation of taxable capital gain of Mr. ‘X’ for A.Y. 2018-19

Particulars ` `` `` `` `

Sale consideration received on sale of 10,000 shares @ `̀̀̀ 500 each 50,00,000

Less : Indexed cost acquisition

(a) 5,000 shares received as gift from father on 1.6.2000

Indexed cost rs 5,000 x 50 x

(b) 2,000 bonus shares received from AB Ltd Nil

Bonus shares are acquired on 21.7.2005 (i.e. after the year

2000 when the original shares were purchased). Hence,

the cost is Nil

(c) 3000 shares purchased on 1.2.2004 @ `̀̀̀125 per share.

The indexed cost is 3000 x 125 x

G.L.T.C.G.

Less : Exemption under section 54F(See Note below)

37,00,000 x

50,00,000

Taxable long term capital gain

Note :

Exemption under section 54F can be availed by the assessee subject to

fulfillment of the following conditions :

(a) The assesse should not own more than one residential house on the

date of transfer of the long –term capital asset ;

(b) The assessee should purchase a residential house within a period of

1 year before or 2 years after the date of transfer or construct a

residential house within a period of 3 years after the date of transfer

of the long-term capital asset.

In this case, the assessee has fulfilled the two conditions mentioned above.

Therefore, he is entitled to exemption under section 54F.

Page 204: INTER CA Taxation - CA Study Web€¦ · INTER CA Taxation Head Office Shraddha, 4th Floor, ... 14. Advance Tax & Interest 268 - 271 15. Tax Deducted at Source (TDS) 272 - 294 16

: 201 :

J. K. SHAH CLASSES INTER C.A. - DIRECT TAXES

CAPITAL GAINS

Ans. 23.

Nil 1,41,667

Working for exemption u/s. 54F

The exemption u/s. 54F can be claimed for LTCG on Gold as well as Silver. The ratio of

Gross LTCG to Net Sale Consideration is 12.5% in case of Gold [1,00,000 / 8,00,000] and

is 58.33% in case of Silver [3,50,000 / 6,00,000]. Hence the balance cost of the house

property at Mumbai Rs. 1,00,000 (after claiming exemption under section 54) should be

utilised for claiming exemption under section 54F for LTCG on Silver in order to claim

higher exemption and reduce tax liability.

The exemption under section 54EC for bonds of NHAI Rs. 2,50,000 can be claimed for

LTCG on Gold as well as Silver. The LTCG on Gold is fully exempt by investment into these

bonds and the balance cost of NHAI bonds of Rs. 1,50,000 has been utilised for claiming

exemption in case of LTCG on Silver.

It may be noted that any other option of claiming exemptions under sections 54EC and 54F

shall result into higher tax liability than the option described above.

Page 205: INTER CA Taxation - CA Study Web€¦ · INTER CA Taxation Head Office Shraddha, 4th Floor, ... 14. Advance Tax & Interest 268 - 271 15. Tax Deducted at Source (TDS) 272 - 294 16

: 202 :

J. K. SHAH CLASSES INTER C.A. - DIRECT TAXES

INCOME FROM OTHER SOURCES

INCOME FROM OTHER SOURCES

SECTION DESCRIPTION

56 Charging section

*56(1) General Incomes charging section

*56(2) Specific eleven incomes charging section

57 Deductions permissible in the Computation of Income from other sources

58 Specific disallowances

59 Subsequent recovery of expenses previously allowed u/s 57, such

subsequently recovery is fully taxable. [Similar to section 41(1)].

CHARGING SECTION 56 :

GENERAL INCOMES TAXABLE U/S 56 (1):

Section 56(1) specifies that if an income is not taxable under the first four heads of in-

come, it should be brought to tax under the head Income from Other sources, provided

the income is not exempt. One can describe these as general incomes from other sources.

Few of them under certain broad classification:-

* The return on investment type:

- Interest on loans (If it is not an Income from business)

- Income from royalty (if it is not an income from business / profession)

Agriculture Income from a place outside India (Not exempt u/s 10(I), since

not from India)

- Rent of plot of land (vacant land)

- Ground rent (Rent of land on which house property is constructed)

- Annuity payable under a will, contract, trust deed (excluding annuity payable

by employer, which is chargeable under the head “salaries”)

- Interest on employee’s contribution to unrecognized Provident Fund.

- Annuity payable to the lender of a trademark.

* Income which do not have employer- employee relationship:

- Director’s fees (of non employee directors)

- Directors commission for standing as a guarantor to bankers / underwriting shares

of new company.

- Examinations fees received by a teacher from a non-employer (college professor

getting remuneration from university)

- Salaries payable to a Member of Parliament (MP) or Member of Legislative

Assembly (MLA).

- Gratuities paid to director who is not an employee of the company.

- Pension received under a family pension scheme, by a non- employee i.e.

spouse of the deceased employee.

Page 206: INTER CA Taxation - CA Study Web€¦ · INTER CA Taxation Head Office Shraddha, 4th Floor, ... 14. Advance Tax & Interest 268 - 271 15. Tax Deducted at Source (TDS) 272 - 294 16

: 203 :

J. K. SHAH CLASSES INTER C.A. - DIRECT TAXES

INCOME FROM OTHER SOURCES

* OTHER TYPES:

- Income from subletting (provided tenant is not deemed owner u/s 27).

Since there is no ownership of property .

- Income from racing establishments such as income from activity of

owning and maintaining race horses:

Actually it is a business income (computed at stake money received (-)

expenses u/s 57) and not casual winnings, but taxable under Income from

other sources, since it promotes gambling.

SPECIFIC INCOMES TAXABLE ONLY UNDER INCOME FROM OTHER

SOURCES U/S 56(2)

Section 56 (2) specifies eleven incomes & puts a compulsion that these incomes

must be included in the head Income from Other Sources, even though they may

have certain features of Income from House Property & Income from Business.

These specific incomes are:

(1) Dividend income

Dividend on shares held as investment or as stock in trade (in which case it is

basically business income in nature) such dividend income is always taxable

under Income from other sources as per the Income tax Act.

(2) Interest on securities (if held as investment and not as stock in trade, because in

that case income is taxable under Income from business & profession).

(3) Winning from lottery, crossword puzzles, card game or any other sort of gambling.

“Lottery" includes winnings from prizes awarded to any person by draw of lots or by

chance or in any other manner whatsoever, under any scheme or arrangement by

whatever name called "Card game and other game of any sort" includes any game

show, an entertainment programme on television or electronic mode, in which

people compete to win prizes or any other similar game.

(4) Hire charges of plant, machinery and furniture, if let out as idle capacity.

(5) Letting out of building alongwith plant and machinery as idle capacity.

(6) Any sum received under "Keyman insurance policy" including sum allocated by

way of bonus on such policy [if not taxable u/s 17(3) in the hands of employee

under Income from salary or not taxable u/s 28 in the hands of employer].

(7) Any sum received by the assessee from his employees as contribution to

any staff welfare scheme [i.e. Employer does not have income taxable under

business or profession and has income taxable under Income from house

property / Income from other sources / capital gains ; and such employer collects

from employees their contribution towards provident fund, gratuity fund etc. Such

amount collected is considered as income in the hands of employer u/s 56 (just

like section 28) and when the employer deposits the amount collected in the

relevant fund on or before the fund's due date, it is allowed as a deduction u/s 57

in the hands of the employer [just like section 36(1)(va)].

Page 207: INTER CA Taxation - CA Study Web€¦ · INTER CA Taxation Head Office Shraddha, 4th Floor, ... 14. Advance Tax & Interest 268 - 271 15. Tax Deducted at Source (TDS) 272 - 294 16

: 204 :

J. K. SHAH CLASSES INTER C.A. - DIRECT TAXES

INCOME FROM OTHER SOURCES

(8) Any interest on compensation or enhanced compensation received on compulsory

acquisition (as compensation for loss of opportunity because of delay in payment).

The assessee is entitled to claim deduction u/s 57 at 50% of the amount of

interest received.

Note : Compensation amount or enhanced compensation amount will be

taxable under capital gains , here only interest received is taxable.

(9) Gifts received by an any preson will not be taxable in relation to any sum

of money or any property received :

(a) ���� From any relative :Where relative means :

* For Individual :

(i) Spouse of the individual

(ii) brother or sister of the individual including their

spouses,

(iii) brother or sister of the spouse of the individual

including their spouses,

(iv) brother or sister of either of the parents of the

individual including their spouses,

(v) any lineal ascendant or descendant of the indi-

vidual including their spouses,

(vi) any lineal ascendant or descendant of the spouse

of the individual including their spouses,

* For HUF:

Any member of the family

(b) ���� On the occasion of marriage of the individual

(c) ���� Under a will or by way of inheritance

(d) ���� In contemplation of death of the payer or donor (i.e. a gift of

personal property by a person expecting to die soon due to ill

health or age)

(e) ���� From any local authority, covered under section 10(20)

(f) ���� From any fund, foundation, university, other educational institu-

tion, hospital, medical institution, any trust or institution referred

to in section 10(23C).

(g) ���� From a charitable institute registered under section 12AA.

If the gift is not falling in the above exceptions, then it is

taxable here if not taxable under other heads of Income.

(A) When an any person receives in any previous year, from any person

or persons –

(i) Any sum of money without consideration, the aggregate value

of which exceeds ̀ ̀ ̀ ̀ 50,000/- the total amount received is taxable

under this head (and not the excess over `̀̀̀ 50,000 /-).

Page 208: INTER CA Taxation - CA Study Web€¦ · INTER CA Taxation Head Office Shraddha, 4th Floor, ... 14. Advance Tax & Interest 268 - 271 15. Tax Deducted at Source (TDS) 272 - 294 16

: 205 :

J. K. SHAH CLASSES INTER C.A. - DIRECT TAXES

INCOME FROM OTHER SOURCES

(ii) Any immovable property of land or building or both

(a) Received without consideration :

The entire stamp duty value of the property is taxable under

this head if such stamp duty value exceeds `̀̀̀ 50,000.

Note: For each property gifted the limit of `̀̀̀ 50,000 is

applicable.

(b) Receivable for inadequate consideration

If received for inadequate consideration where the

difference between stamp duty value and amount paid

exceeds `̀̀̀ 50,000 per gift of immovable property, the

entire difference is taxable.

Note:

* If such difference is less than or equal to `̀̀̀ 50,000 per

gift of immovable property, then nothing is taxable under

these provisions.

* In the case of immovable property if the stamp duty value

is disputed by the assessee as mentioned u/s 50C, the

Assessing Officer may refer the valuation of such prop-

erty to a Valuation Officer as per section 50C.

(iii) Any movable property of :

- Shares and securities

- Jewellery

- Archeological collections

- Drawings

- Paintings

- Sculptures

- Any work of art

- bullion

(a) Received without consideration

If the aggregate fair market value of such property

exceeds `̀̀̀ 50,000 in the relevant previous year, the

whole of the aggregate fair market value of the

property is taxable under this head.

(b) Received for inadequate consideration

If the difference between the aggregate fair market value

of the property and the aggregate consideration paid for

the property exceeds `̀̀̀ 50,000/-, the entire differences

is taxable under this head.

Page 209: INTER CA Taxation - CA Study Web€¦ · INTER CA Taxation Head Office Shraddha, 4th Floor, ... 14. Advance Tax & Interest 268 - 271 15. Tax Deducted at Source (TDS) 272 - 294 16

: 206 :

J. K. SHAH CLASSES INTER C.A. - DIRECT TAXES

INCOME FROM OTHER SOURCES

(10) When a closely held company (i.e. Private limited) receives in any

previous year, from any resident person any consideration for issue of shares

that exceeds the face value of such shares, the difference between the

aggregate consideration received and the fair market value of the shares

shall be taxable under this head.

(11) Any sum of money received as an advance or otherwise in the course of

Negotiations for transfer of a capital asset, if,- (a) such sum is forfeited; and

(b) the negotiations do not result in transfer of such capital asset.”.

Dividend income [Section 8]

Dividend

Notes :

1. Indian company is brought to tax on behalf of the shareholders u/s 115-0 therefore

exempt in the hands of the shareholders u/s 10(34) for interim, final & deemed

dividend u/s 2(22) (a) to u/s 2(22)(d).

The shareholder is brought to tax for deemed dividend u/s 2(22)(e) only.

2. Dividend received from a foreign company or a co-operative society is taxable in

the hands of the shareholders u/s 56 i.e. INCOME FROM OTHER SOURCES as

R.O.I. (Return on Investment).

Deemed Dividend

It is a method by which the Income tax Act attempts to plug the loop holes in the Act, or to

check the practice of departing from the natural and ordinary term of “interim dividend”

and “final dividend”, so as to ensure that Government is not deprived of the taxes by way

of any alluring activity which cannot be otherwise brought into the tax net in the ordinary

course.

Page 210: INTER CA Taxation - CA Study Web€¦ · INTER CA Taxation Head Office Shraddha, 4th Floor, ... 14. Advance Tax & Interest 268 - 271 15. Tax Deducted at Source (TDS) 272 - 294 16

: 207 :

J. K. SHAH CLASSES INTER C.A. - DIRECT TAXES

INCOME FROM OTHER SOURCES

Deemed Dividend is taxable to the extent of “Distribution“ or “accumulated profits“

whichever is less. It is essential to know the meaning of accumulated profits (i.e.

the shareholders’ funds).

* Accumulated Profits :

On the basis of different judicial decisions, the following conclusions can be

drawn in respect of accumulated profits :

1. Accumulated profits include all profits of a company upto the date of

distribution or payment. In the case of liquidation of company,

however, it includes all profits upto the date of liquidation.

2. Accumulated profits are computed on the basis of commercial profits and

not on the incomes calculated by assessing officers (except concealed

incomes detected by them).

3. W hi le calculat ing "Accumulated pro f i ts " an al lowance for

depreciation at the rates provided by the Income-tax Act itself has to

be made by way of deduction.

4. Accumulated profits include tax-free income e.g. agricultural income.

However, receipts of capital nature are included in accumulated profits

only if such receipts are chargeable to tax under the head "Capital gains."

5. Accumulated profits include general reserve.

Provisions for taxation and dividends do not form part of accumulated

reserve.

6. Subsidy from government on the basis of investment in capital assets

cannot be treated as accumulated profits.

In Income tax deemed dividend is defined u/s 2(22) as follows:

(a) Distribution of accumulated profits which results into release of all or

any part of the assets of the company (prior to liquidation) to its

shareholders [sec. 2(22)(a)].

(b) (i) Any distribution to the extent of the accumulated profits in the

form of debentures, debenture-stock, deposit certificates in any

form with or without interest to its shareholders. [Sec. 2(22)(b)].

(ii) Any bonus shares to preference shareholders [Sec. 2(22)(b)].

Notes:

If bonus shares of equity shares are issued out of accumulated profits,

then not taxable on receipts of such shares, since such shares will not

be redeemed over the life time of the company.

(c) Any distribution to shareholders on the liquidation of the company to the

extent of distribution attributable to accumulated profits [Sec. 2(22)(c)].

(d) Any distribution on reduction of share capital to the extent the company

possesses accumulated profits [Sec. 2(22)(d)].

Page 211: INTER CA Taxation - CA Study Web€¦ · INTER CA Taxation Head Office Shraddha, 4th Floor, ... 14. Advance Tax & Interest 268 - 271 15. Tax Deducted at Source (TDS) 272 - 294 16

: 208 :

J. K. SHAH CLASSES INTER C.A. - DIRECT TAXES

INCOME FROM OTHER SOURCES

(e) Distribution of accumulated profits excluding capitalized profits (i.e. bonus

shares) by way of advance or loan by a private limited (not being a money

lending or banking company), to its registered shareholder holding at least

10% of the equity capital beneficially on or after 1-6-87.

* The loan amount given to the registered share holder or to any other person

by the private limited company on behalf of, or for the individual benefit of

the registered shareholder, or the accumulated profits as at the date of

loan, whichever is less, is taxable as deemed dividend u/s 2(22)(e) in the

hands of the registered shareholder.

* If loan / advance is given to a concern (firm / company / AOP/ BOP) in which a

registered shareholder (beneficially holding at least 10% of the equity capital) of

the loan giving company, has substantial interest (i.e. 20% or more in the loan receiving

concern), then such a payment is taxable as deemed dividend in the hands of the

registered shareholder u/s 2(22)(e) and not the loan receiving concern.

Deemed Dividend provisions are not applicable u/s 2(22)(e) in the following cases:

(a) Any loans or advance given to the shareholders in the ordinary course of

money lending business .

(b) Any dividend payable to a registered share holder is adjusted against the

outstanding loan [which was considered as deemed dividend in the hands

of the registered shareholder u/s 2(22)(e)], instead of paying it to the

registered shareholder .

Note :

* Such dividend adjusted will not be taxable u/s 115-O (D.D.T) in the hands of

the company.

* Repayment of loan is not considered in the deemed dividend provisions

u/s 2(22) (e).

* If the accumulated profits are say 20 lakhs and one of the registered share

holder “A” beneficially holding at least 10% or more in the voting powers –

takes a loan of 15 lakhs , then entire 15 lakhs is considered as deemed

dividend in the hands of the registered shareholder “A”. Subsequently a

similar registered shareholder “B” takes a loan of 8 lakhs, then deemed

dividend taxable in his hands will be only the remaining accumulated profits

of 5 lakhs u/s 2(22)(e). Subsequently a similar registered shareholder “C”

takes a loan of 4 lakhs , nothing is taxable in his hands u/s 2(22)(e), since

there is no accumulated profit left for taxability.

Now if a company after above, actually distributes “interim” or “final” dividend

out of accumulated profits, nothing would be taxable in its hands u/s 115-0 i.e.

D.D.T in the hands of the company, since such accumulated profits are already

taxed as deemed dividend u/s 2(22)(e) in the hands of the shareholder.

Page 212: INTER CA Taxation - CA Study Web€¦ · INTER CA Taxation Head Office Shraddha, 4th Floor, ... 14. Advance Tax & Interest 268 - 271 15. Tax Deducted at Source (TDS) 272 - 294 16

: 209 :

J. K. SHAH CLASSES INTER C.A. - DIRECT TAXES

INCOME FROM OTHER SOURCES

DEDUCTIONS PERMISSIBLE FROM INCOME FROM OTHER SOURCES u/s. 57

(1) Commission or remuneration for realising dividend or interest on

securities : Any reasonable sum paid by way of commission or remuneration to

a banker or any other person for the purpose of realising dividends, other than

dividends referred to in section 115-O or interest on securities on behalf of the

assessee is deductible.

(2) Deduction in respect of employees contribution towards staff welfare schemes :

Deduction in respect of any sum received by a tax payer as contribution from his

employees towards any welfare fund of such employees is allowable only if such sum is

credited by the tax payer to the employees account in the relevant fund on or before the

due date of the fund under the provisions of any law or terms of contract of service or

otherwise. [Similar to sec 36(1)(va)].

(3) Repairs, depreciation in the case of letting out of plant, machinery,

furniture, building : In the case of income chargeable under section 56, the fol-

lowing expenses are deductible :

(a) Current repairs & Insurance premium for building (similar to Sec. 30).

(b) Repairs and insurance of machinery, plant and furniture (similar to sec.31).

(c) Depreciation (similar to sec.32) allowed as wear & tear.

(4) Standard deduction in the case of family pension : In the case of income in the

nature of family pension, the amount deductible is `̀̀̀15,000 or 1/3 of such income, which-

ever is less.

For this purpose "Family pension" means a regular monthly amount payable by the

employer to a person belonging to the family of an employee, in the event of his

death.

(5) In the case of interest received on compensation or on enhanced compen-

sation - a deduction of a sum equal to 50% of such interest income will be allowed

and no other deduction shall be allowed towards the same.

(6) Any other expenses for earning income : Any other expenditure is deductible

under section 57 if the following five basic conditions are satisfied :

(a) The expenditure must be laid out or expended wholly and exclusively

for the purpose of making or earning the income ;

(b) The expenditure must not be in the nature of capital expenditure ;

(c) It must not be in the nature of personal expenses of the assessee ;

(d) It must be laid out or expended in the relevant previous year and not

in any prior or subsequent year ;

(e) It must not be prohibited by law.

Page 213: INTER CA Taxation - CA Study Web€¦ · INTER CA Taxation Head Office Shraddha, 4th Floor, ... 14. Advance Tax & Interest 268 - 271 15. Tax Deducted at Source (TDS) 272 - 294 16

: 210 :

J. K. SHAH CLASSES INTER C.A. - DIRECT TAXES

INCOME FROM OTHER SOURCES

Commissioner of Income tax v Rajendra Prasad Moody (1978)

The Supreme Court held that the expenditure incurred for the purpose of

earning the income shall be allowed as a deduction, even if the source of in-

come has not yielded any income during the previous year. This principle can be

applied only for the expenditure incurred in earning "taxable" income. If the

income is exempt from tax [e.g. dividend income u/s 10(34) and 10(35)],

expenditure incurred towards such exempt income cannot be allowed as a deduction.

What are specific disallowances u/s 58 -The following amounts are not deductible while

computing income under the head "Income from other sources":

(a) Personal Expenses are not deductible.

(b) Interest : Any interest chargeable under the Act which is payable outside India on

which tax has not been deducted at source is not deduct [similar to sec 40(a)].

(c) Salary : Any payment of salaries outside India is not deductible if tax has not

been paid or deducted therefrom [similar to sec 40(a)].

(d) Direct Tax: Any sum paid on account of income -tax / wealth -tax is not deductible,

since they are “application of income “[similar to sec 40(a)].

(e) Amount specified by section 40A: Any amount specified by sections :

* Section 40A(2) - Payment to relatives in excess of market value of goods

or services received.

* Section 40A(3) - Payment in excess of `̀̀̀ 10,000 by cash / bearer cheque

etc. in relation to deduction claimed .

* Section 40A(7) - Contribution to unapproved gratuity fund of employees

* Section 40A - Contribution to unrecognized provident fund of employees

(9,10,11) is not deductible while calculating income under the head

"income from other sources“.

(f) Expenditure in respect of winnings from lottery etc. While computing the afore-

said incomes the following are not deductible :

1. Expenses covered by section 57.

2. Losses cannot be set off under sections 70, 71, 71B, 72,32(2), 73, 73A, 74

and 74A against the aforesaid incomes.

3. No deduction is permissible under sections 80C to 80U (though winnings from

lotteries, etc., form part of gross total income).

4. It is directly taxable @ flat rate 30% u/s. 115BB (TDS is done u/s194B

and 194BB).

Page 214: INTER CA Taxation - CA Study Web€¦ · INTER CA Taxation Head Office Shraddha, 4th Floor, ... 14. Advance Tax & Interest 268 - 271 15. Tax Deducted at Source (TDS) 272 - 294 16

: 211 :

J. K. SHAH CLASSES INTER C.A. - DIRECT TAXES

INCOME FROM OTHER SOURCES

However, the following are deductible.

(a) exemption under section 10(32), i.e. minors income of casual winnings

or `̀̀̀ 1,500, whichever is less.

(b) amount deductible from lottery prize etc. for the benefit of the

Government / agency conducting the lotteries. (Like contribution for

social welfare), since such an amount deducted from casual winnings

has not been received by the assessee.

Income taxable u/s 59: Any subsequent recovery of expenses by assessee in current

year , which were allowed to him as a deduction in the earlier previous year u/s 57, such

subsequent recovery is taxable u/s 59 in the current year because it is reversal of an

expenditure previously allowed by way of considering it income. This is similar to section 41(1).

Other Aspects :

Section 115BBD: Tax on dividends received from foreign companies

(1) When an Indian company receives dividends declared, distributed or paid

by a "specified foreign company", such dividend income shall be taxable in the

hands of the Indian company at the rate of 15% of such dividends.

(2) No deduction of any expenditure shall be allowed to the Indian company under any

provision of the Income tax Act.

(3) (i) Dividend shall have the same meaning under section 2(22)(a) to (d).

(ii) A "specified foreign company" means a foreign company in which them

Indian company holds 26% or more in nominal value of the equity share

capital of the company.

Section 115BBDA: Tax on dividends received from domestic companies

1) Dividends declared by domestic companies u/s 2(22)(a) to 2(22)(d) are covered

by the provisions of D.D.T. u/s 115-0. The basic rate of D.D.T is 15%. This

creates inequality amongst different shareholders as those who earn high

dividend income are being subjected to tax at 15%. If such dividend would have

been taxed for the shareholder, it would been taxed at 30%

2) To remove such inequality, section 115BBDA has been introduced from P.Y.

2016-17, A.Y. 2017-18. If a shareholder, being specified assessee receives

dividends covered by section 2(22)(a) to section 2(22)(d) exceeding Rs. 10 lakhs,

then such dividend shall also be taxable for the shareholder at the rate of 10%.

3) No deduction in respect of expenditure, allowance or set off of losses shall be

allowed from such dividend. This income tax is in addition to the D.D.T. pay-

able by domestic company on such dividend.

4) It is worthwhile to note that this section does not apply to dividend u/s 2(22)(e).

Therefore the rate of tax for dividend u/s 2(22)(e) shall be the normal tax rates

applicable.

Specified assessee means a person other than

domestic company,

institution or trust, university, hospital claiming exemptions under sec.10(23C).

Trust registered under sec. 12A & 12AA.

Page 215: INTER CA Taxation - CA Study Web€¦ · INTER CA Taxation Head Office Shraddha, 4th Floor, ... 14. Advance Tax & Interest 268 - 271 15. Tax Deducted at Source (TDS) 272 - 294 16

: 212 :

J. K. SHAH CLASSES INTER C.A. - DIRECT TAXES

INCOME FROM OTHER SOURCES

Method of Accounting

Regarding Dividend Income, the Method of Accounting is irrelevant. Every assessee has

to follow section 8. Final dividend is taxable in the financial year of declaration whereas

interim dividend is taxable in the financial year of receipt.

Computation of any other income depends on the method of accounting.

1) If Mercantile method is followed income should be computed considering income

receivable and expense payable.

2) If Cash method is followed, income should be computed considering only actual

income received & actual expenses paid.

3) If no method is decided by the taxpayer, he is supposed to follow mercantile

method.

(A) Income from composite letting of building, machinery, plant or furniture

as idle capacity let out.

If an assessee lets on hire machinery, plant or furniture and also building

and letting of building is inseparable from letting of machinery, plant or

furniture, income from such letting is taxable as income from other sources,

if the same is not chargeable to tax under the head "Profits and gains of

business or profession", i.e. let out of idle capacity.

This rule is applicable even if the sum receivable for the two lettings is fixed separately.

Page 216: INTER CA Taxation - CA Study Web€¦ · INTER CA Taxation Head Office Shraddha, 4th Floor, ... 14. Advance Tax & Interest 268 - 271 15. Tax Deducted at Source (TDS) 272 - 294 16

: 213 :

J. K. SHAH CLASSES INTER C.A. - DIRECT TAXES

INCOME FROM OTHER SOURCES

Q. 1. Mr. X has received the following gifts payments in the previous year 2017-18,

select the taxable amount / not taxable from them.

Amt. recd. Gift / Amount received fromTaxable / Remarks

Non - taxable/

`̀̀̀ 30,000 Gift from Cousin brother

`̀̀̀ 25,500 Gift from Sister's son

`̀̀̀ 40,000 Gift from Sister

`̀̀̀ 25,800 Gift from Sister's daughter's husband

`̀̀̀ 15,000 Gift from Wife's sister's daughter

`̀̀̀ 30,000 Gift from Maternal uncle

`̀̀̀ 10,000 Gift from Grand father

`̀̀̀ 50,000 Gift from Wife's sister's husband

`̀̀̀ 26,000 Gift from Wife's sister's husband's mother

`̀̀̀ 35,000 Gift from Brother of grand father

`̀̀̀ 2,00,000 Gift on the occasion of marriage

`̀̀̀ 60,000 Of concession on the acquisition of capital asset

i.e. immovable property from a friend in relation to

its market value

`̀̀̀ 4,00,000 Worth of jewellery received as gift from a friend

Q. 2. If Mr. A purchased house in year 1 for `̀̀̀ 8,00,000 and transferred /sold the same in

3 independent situations as under.

1. - sold the same for `̀̀̀22,00,000 in year 3.

2. - gifted the same in year 2 to a ‘relative‘ when its F.M.V. was `̀̀̀15,00,000.

Subsequently the relative sold it for `̀̀̀ 22,00,000 in year 3.

3. - gifted the same in year 2 to a ‘friend’ when its F.M.V. was `̀̀̀15,00,000

subsequently the friend sold it for `̀̀̀22,00,000 in year 3.

Compute effects of all 3 situations.

Q. 3. PQR private Ltd. a closely held company, involved in the manufacture of goods,

has given the following independent loans to registered shareholders holding

atleast 10% in the voting powers of PQR private Ltd. beneficially :

(1) To MR. A of `̀̀̀15,00,000 when the general reserve & current profits were

`̀̀̀16,00,000 and `̀̀̀ 8,00,000 respectively.

(2) To MR. B of `̀̀̀ 26,00,000 when the general Reserve & current profi t were

`̀̀̀16,00,000 and `̀̀̀ 8,00,000 respectively.

(3) To XYZ bros. of `̀̀̀ 15,00,000 in which Mr. A share holder of PQR (P) Ltd., is

substantially interested i.e. having 20% or more in the profit sharing in the

loan receiving f i rm, when the general reserve was `̀̀̀18,00,000 of

PQR (P) Ltd..

(4) PQR (P) Ltd. has a general reserve of `̀̀̀15,00,000, and the company has given

a loan of `̀̀̀12,00,000 on 5.5.16 to Mr. A & `̀̀̀ 10,00,000 to MR. B on 6.6.16.

Compute the deemed dividend taxable u/s 2(22)(e) in each of the independent situations.

CLASS WORK PROBLEMS

Page 217: INTER CA Taxation - CA Study Web€¦ · INTER CA Taxation Head Office Shraddha, 4th Floor, ... 14. Advance Tax & Interest 268 - 271 15. Tax Deducted at Source (TDS) 272 - 294 16

: 214 :

J. K. SHAH CLASSES INTER C.A. - DIRECT TAXES

INCOME FROM OTHER SOURCES

Q. 4. Ronak Industries Private Limited (RIPL), incorporated in India, gives you the

following details regarding P.Y. 2017-18. Compute income tax payable.

(a) It has invested in shares of Microsoft Ltd., a foreign company, to the extent

of 30% of total equity share capital of the foreign company.

Microsoft Ltd. declared an interim dividend and RIPL received `̀̀̀1,20,000/-

on 28.2.2018. Collection charges incurred `̀̀̀ 5,000 and interest on loan to

purchase above shares `̀̀̀ 40,000/-.

(b) It also received dividend from Nissan Inc. Ltd., another foreign company, of

`̀̀̀ 65,000. The shareholding of RIPL in this foreign company is 15%. Collection

charges `̀̀̀ 1,300 and interest on loan to purchase above shares `̀̀̀ 20,000/-.

(c) Dividend from shares of Reliance Industries Ltd. `̀̀̀ 60,000/-.

(d) Its income from business computed as per I.T. provisions is `̀̀̀ 95,00,000/-

and income from house property (computed) is `̀̀̀ 30,00,000/-.

(e) It issued 50,000 equity shares of face value of `̀̀̀ 10 per share at a premium

of `̀̀̀ 60 per share to various resident persons. The fair market value of the

share as per prescribed rule is `̀̀̀ 50 per share.

Q. 5. Dr. Eknath is a Professor of Taxation and is a resident of India. He submits before

you the following detail for computing his income under the head 'Income from

other sources' for the assessment year 2018-2019 :

1. He is a author of a text book of a school which fetched him a gross royalty of

`̀̀̀ 1,50,000. He claims the following deductions from this amount:

(a) Salary to a clerk who collects for him necessary data and goes through

the final proofreading `̀̀̀ 5,000.

(b) Purchased books worth `̀̀̀ 4,000 in connection with the revision of the book.

(c) Telephone expenses of `̀̀̀ 2,000 attributed to the publication and sale

of his book and other matters in connection with the printing of the

new edition of the book.

2. He borrowed `̀̀̀ 24,00,000 on 1.4.2017 from a Bank @ 18% p.a. He invested

the money in the purchase of plot of land meant to be let out on hire. During

the year, no rental income was received from the land. He, however, claimed

that the interest paid to the bank must be allowed as an expense.

3. He lives in a rented house paying rent of `̀̀̀10,000 p.m. The house is too big

for his family. Hence he has sub - let one - third portion of the house on a rent

of `̀̀̀ 7,000 p.m. Dr. Eknath has undertaken the liability of paying municipal

taxes of `̀̀̀ 5,400 on the whole house and also the current repair of the whole

house amounting to `̀̀̀ 6,000.

4. He gets a family pension of `̀̀̀12,500 p.m. He is an examiner of a number of

Universities. This source gave him a remuneration of `̀̀̀ 24,000.

5. He owns race horses, the expenses on their maintenance are `̀̀̀ 1,20,000. He has

earned stake money of `̀̀̀ 9,00,000 when one of the horses stood first in the race.

6. He has received dividend of `̀̀̀ 40,000 from mutual fund notified u/s 10(23D)

and dividend from co-operative society `̀̀̀ 10,000/-.

Page 218: INTER CA Taxation - CA Study Web€¦ · INTER CA Taxation Head Office Shraddha, 4th Floor, ... 14. Advance Tax & Interest 268 - 271 15. Tax Deducted at Source (TDS) 272 - 294 16

: 215 :

J. K. SHAH CLASSES INTER C.A. - DIRECT TAXES

INCOME FROM OTHER SOURCES

Q. 6. Mr. Chaturvedi, R & OR, gives following information regarding his income in

P.Y. 2017-2018.

(A) Dividend Income Details

(1) He has earned a dividend of `̀̀̀1,20,000 from Britney Ltd., Canada &

he has recorded fo llowing expenses to earn this div idend :

(Equity Shareholding = 35%)

Interest on loan taken to invest in above shares sent abroad without TDS ` ` ` ` 60,000

Interest on loan (taken to invest in above shares) paid in India by cash `̀̀̀ 70,000

Collection Charges to realise dividend warrants `̀̀̀ 140

(It is Final dividend declared in AGM held on 1 March 2017 at Torronto but

received on 10 April 2017. Chaturvedi follows Cash System of accounting

regarding dividend incomes).

(2) Dividend from Shares in Saraswat Co-op Bank (held as stock-in-trade)

`̀̀̀ 15,000.

(3) He has taken a loan from Kalasangam Private Limited in which he is 15%

equity shareholder. The loan amount is `̀̀̀ 6,00,000 & accumulated prof-

its of the company are `̀̀̀ 45,60,000.

(B) Interest Income details :

(a) Interest earned on Govt. Bonds `̀̀̀ 1,23,000 (held as stock in trade)

(b) Interest on Income tax refund `̀̀̀ 2,000 & income tax refund `̀̀̀ 20,000.

(C) Other Income details :

(i) Winning from Lottery `̀̀̀ 70,000 Net (Tax deducted at Source `̀̀̀ 30,000)

(He claims the cost of lottery tickets `̀̀̀ 20,000 as expenses)

(ii) He has received `̀̀̀12,00,000 on assignment & immediate surrender

of Key man Insurance Policy. (On resignation as head of Sales Department).

(iii) He received `̀̀̀ 5,000 on his birthday from each of his eleven friends

and a diamond ring of `̀̀̀ 5,00,000/- from his wife.

You are required to calculate Income from Other Sources for Assessment

Year 2018-2019.

Q. 7.What is the priority of set off of minimum exemption limit applicable against N.T.T.I ?

Page 219: INTER CA Taxation - CA Study Web€¦ · INTER CA Taxation Head Office Shraddha, 4th Floor, ... 14. Advance Tax & Interest 268 - 271 15. Tax Deducted at Source (TDS) 272 - 294 16

: 216 :

J. K. SHAH CLASSES INTER C.A. - DIRECT TAXES

INCOME FROM OTHER SOURCES

Ans.3. (1) This is deemed dividend u/s 2(22)(e) in the hands of Mr. A,a

registered shareholder, holding atleast 10% in the voting powers ben-

eficially, of `̀̀̀ 15,00,000, since the accumulated profits are more than

`̀̀̀ 15,00,000. i.e `̀̀̀ 24,00,000.

(2) This is deemed dividend in the hands of Mr. B a registered shareholder,

holding atleast 10% in the voting powers beneficially, u/s 2(22)(e) of

`̀̀̀ 24,00,000, since the accumulated profits are 24,00,000, thus restricted

to such an extent, even though the loan amount is `̀̀̀ 26,00,000.

(3) This is deemed dividend of `̀̀̀ 15,00,000 in the hands of Mr. A

u/s 2(22)(e), since Mr. A is a registered shareholder of PQR(P) Ltd.

holding atleast 10% in the voting powers beneficial ly, and is

substantially interested in the loan receiving organization. This is so

because the accumulated profits of PQR(P) Ltd. is `̀̀̀ 18,00,000.

(4) Deemed Dividend of `̀̀̀ 12,00,000 in the hands of Mr. A, a registered

shareholder, holding atleast 10% in the voting powers beneficially, on

5.5.2015, since the accumulated profits at this moment of time are

`̀̀̀ 15,00,000.

(5) Deemed Dividend of `̀̀̀ 3,00,000 in the hands of Mr. B a registered

shareholder, holding atleast 10% in the voting powers beneficially, on

6.6.2015 although the loan amount is `̀̀̀10,00,000, since the

accumulated profits that remains now as at 6.6.2015 is `̀̀̀ 3,00,000

( `̀̀̀15,00,000 total `̀̀̀12,00,000 taken for A).

Ans.4. Ronak Industries Private Limited

Computation of Total Income and tax liability for A.Y. 2018-2019

Particulars `̀̀̀

Income from House Property 30,00,000

Income from Business 95,00,000

Income from Other Sources (W.N.) 11,63,700

GROSS TOTAL INCOME (G.T.I.) 1,36,63,700

Less: Deductions u/s. 80C to 80U NIL

NET TAXABLE TOTAL INCOME (N.T.T.I.) 1,36,63,700

TAX LIABILITY

Basic Tax on:

- Dividend u/s. 115BBD `̀̀̀ 1,20,000 @ 15% 18,000

- Other income `̀̀̀ 1,35,43,700 (balancing figure) @ 30% 40,63,110 40,81,110

Add: Surcharge @ 7% of basic tax 2,85,678

Total 43,66,788

Add: Education cess @ 3% 1,31,004

TAX PAYABLE 44,97,792

CLASSWORK SOLUTION

Page 220: INTER CA Taxation - CA Study Web€¦ · INTER CA Taxation Head Office Shraddha, 4th Floor, ... 14. Advance Tax & Interest 268 - 271 15. Tax Deducted at Source (TDS) 272 - 294 16

: 217 :

J. K. SHAH CLASSES INTER C.A. - DIRECT TAXES

INCOME FROM OTHER SOURCES

W.N. Income from Other Sources

Particulars ` `` `` `` `

Dividend from shares of :

- Microsoft Ltd. (Note) 1,20,000

- Nissan Inc Ltd. 65,000

Less: Expenses u/s. 57

(a) Collection charges (1,300)

(b) Interest on loan (20,000) 43,700

- Reliance Industries Ltd., exempt u/s. 10 (34) NIL

Issue of equity shares to residents at premium

[(10 + 60)/ share – 50/ share] x 50,000 shares 10,00,000

INCOME FROM OTHER SOURCES 11,63,700

Note: The dividend received from Microsoft Ltd. (foreign company) by R.I.P.L.

(Indian Company) is covered by the provision of Sec. 115BBD and hence it shall

be taxable at 15% without allowing any deduction for expenses incurred.

Ans. 5. Dr. Eknath

Computation of Income from Other Sources for A.Y. 2018-2019

Page 221: INTER CA Taxation - CA Study Web€¦ · INTER CA Taxation Head Office Shraddha, 4th Floor, ... 14. Advance Tax & Interest 268 - 271 15. Tax Deducted at Source (TDS) 272 - 294 16

: 218 :

J. K. SHAH CLASSES INTER C.A. - DIRECT TAXES

INCOME FROM OTHER SOURCES

Ans.7.The basic exemption limit applicable shall be first applied towards incomes other

than capital gains and casual winnings (i.e. normal incomes taxable as per slab

rates). The normal income in excess of basic exemption limit shall be taxable as

per slab rates. Capital gains and Casual winnings shall be taxable as per the ap-

plicable special rate.

When the normal incomes (taxable as per slab rates) is less than the basic ex-

emption limit applicable, the difference between the normal income and exemp-

tion limit is called as "Unabsorbed Exemption Limit." Such unabsorbed limit shall

be adjusted towards STCG u/s. 111A or LTCG u/s. 112, whichever has a higher tax

rate. The balance capital gains shall be taxed as per the applicable special rate.

Such unabsorbed exemption limit cannot be adjusted against casual winnings.

Casual winnings shall be fully taxable at the rate of 30%.

Page 222: INTER CA Taxation - CA Study Web€¦ · INTER CA Taxation Head Office Shraddha, 4th Floor, ... 14. Advance Tax & Interest 268 - 271 15. Tax Deducted at Source (TDS) 272 - 294 16

J.K.SHAH CLASSES INTER C.A.- DIRECT TAX

: 219 :

INTRODUCTION :

Generally, an assessee is taxed in respect of his own income. In some cases, however, the Income-tax Act deviates from this principle and the assessee may be taxed in respect of income which legally belongs to some other person. Provisions incorporated in sections 60 to 65 deal with cases where taxpayers make an attempt to reduce their tax liability by transferring their assets in favour of their family members or by arranging their sources of income in such a manner that tax incidence falls on others, whereas benefit of income, directly or indirectly, is derived by them. In order to counteract these practices of tax avoidance, necessary provisions have been made in sections 60 to 65 to tax the incomes in the hands of an individual, even though such incomes belong to other persons. This is called as clubbing of income.

SECTION 60 - TRANSFER OF INCOME WITHOUT TRANSFER OF ASSET

Section 60 is applicable if the following conditions are satisfied —

Condition 1 The taxpayer owns an asset.

Condition 2 The ownership of asset is not transferred by him.

Condition 3 The income from the asset is transferred to any person under a settlement, trust, covenant, agreement or arrangement.

Condition 4 The above transfer of income may be revocable or irrevocable.

Condition 5 The above transfer may be effected at any time (maybe before the commencement of the Income-tax Act or after the commencement of the Act, i.e., before or on or after April 1,1962).

If the above conditions are satisfied, the income from the asset would be received by the transferee but it shall be clubbed in the hands of the transferor i.e. it will be included in the taxable income of the transferor.

Example

X owns land, he transfers rental income to Y, his friend, without transferring the ownership of this land. Although, during P.Y. 2016-17, rent of Rs. 96,000 is received by Y, it is taxable in the hands of X, as he has transferred income without transferring the ownership of the asset.

SECTION 61 - REVOCABLE TRANSFER OF ASSETS

If an asset is transferred under a “revocable transfer”, income from such asset is taxable in the hands of the transferor. In case of a revocable transfer, the ownership of the asset is transferred to the transferee, but the transfer is cancellable at any time by the transferor. Since the asset is with the transferee, the income from the asset will be received by the transferee during the previous year. However, such income shall be clubbed in the hands of the transferor.

It may be noted that the income is taxable for the transferor as and when the power to revoke arises, even if the power to revoke has not been exercised by the transferor.

SECTION 62 - IRREVOCABLE TRANSFER OF ASSETS FOR A SPECIFIED PERIOD

In the following cases, the transfer of assets shall be treated as irrevocable for the period mentioned therein (called as “specified period”). In other words, the power to

CHAPTER 10 CLUBBING OF INCOME

Page 223: INTER CA Taxation - CA Study Web€¦ · INTER CA Taxation Head Office Shraddha, 4th Floor, ... 14. Advance Tax & Interest 268 - 271 15. Tax Deducted at Source (TDS) 272 - 294 16

J.K.SHAH CLASSES INTER C.A.- DIRECT TAX

: 220 :

revoke or cancel the transfer arises to the transferor only after the specified period is over. The cases covered are as follows- (a) The transfer of asset is made before 1st April 1961 and is irrevocable for a

period of atleast 6 years. (b) The transfer of asset is irrevocable during the lifetime of the transferee. (c) The transfer of asset is made to a trust and is irrevocable during the lifetime of

the beneficiary/ beneficiaries.

In the above cases, the transferor should not derive any direct or indirect benefit from the income from the asset during the specified period.

During the specified period, the income from the asset shall be taxable for the transferee. After the specified period is over, the income from the asset shall be clubbed with the transferor.

Sr. No.

Situation Tax implication

1 X transfers a house property to a trust for the benefit of A and B. However, X has a right to revoke the trust during the lifetime of A and/or B.

In this case, the house property is transferred under a trust and it is revocable during the lifetime of the beneficiary. It is a revocable transfer and income arising from the house property is taxable in the hands of X.

2 X transfers a house property to A. X has a right to revoke the transfer after the death of A.

In this case, the house property is transferred to a person and it is irrevocable during the lifetime of transferee. Income arising from the house property is taxable in the hands of A during his lifetime and after death of A, income shall be taxable for X.

3 X transfers debentures to A in PY 2010-2011. X has a right to revoke the transfer after 10 years.

In this case, the debentures are transferred to a person and it is not irrevocable during the lifetime of transferee. It is revocable after a fixed period. Since the given case does not get covered by section 62, it is regarded as a “revocable transfer” and income arising from the debentures is taxable in the hands of X as per section 61.

4 X transfers an asset to Y on January 1, 1960. It is revocable on or before June 6, 1965. It is a revocable transfer.

In this case, the asset is transferred before April 1, 1961 but it is revocable within six years. Income arising from the asset is taxable in the hands of X.

Page 224: INTER CA Taxation - CA Study Web€¦ · INTER CA Taxation Head Office Shraddha, 4th Floor, ... 14. Advance Tax & Interest 268 - 271 15. Tax Deducted at Source (TDS) 272 - 294 16

J.K.SHAH CLASSES INTER C.A.- DIRECT TAX

: 221 :

5 X transfers an asset to Y on April 1, 1958. It is revocable on or after March 31, 2018.

It is an irrevocable transfer as the transfer was made before 1-4-1961 and is irrevocable for atleast 6 years (in this case, 60 years). Income arising from the asset upto 31st March 2018 is taxable in the hands of Y. Thereafter the income will be taxable for X, even if the power to revoke has not been exercised.

6 X transfers an asset to Z. Under the terms of transfer, on or after April 1, 2014, he has a right to utilize the income of the asset for his benefit. However, he has not exercised this right as yet.

If the transfer contains any provision to re-transfer the asset (or income therefrom) to the transferor directly or indirectly, wholly or partly, such a transfer cannot be regarded as an irrevocable transfer u/s. 62. It will be considered as a “revocable transfer” u/s. 61. On or after April 1, 2014, income of the asset would be taxable in the hands of X, even if he has not exercised the aforesaid right.

WHEN AN INDIVIDUAL IS ASSESSABLE IN RESPECT OF REMUNERATION OF SPOUSE [SEC. 64(1)(ii)]

Section 64(1)(ii) is applicable if the following conditions are satisfied —

Condition 1 The taxpayer is an individual.

Condition 2 He/she has a substantial interest in a concern.

Condition 3 Spouse of the taxpayer (i.e., husband/ wife of the taxpayer) is deriving salary, fees, commission or remuneration from the above-mentioned concern.

Condition 4 Spouse does not possess any technical or professional knowledge or experience.

If the aforesaid conditions are satisfied, then salary, fees, commission or remuneration of the spouse will be clubbed in the hands of the taxpayer.

Example

X has a substantial interest in A Ltd. and Mrs. X is employed by A Ltd. without any technical or professional qualification to justify the remuneration. In this case, salary income of Mrs. X shall be taxable in the hands of X.

Other points : One has to keep in view the following points —

(a) Meaning of Substantial Interest: An individual has a “substantial interest” in any of the following two situations —

(i) In the case of a company - If the individual beneficially holds (individually or along with his relatives) atleast 20 per cent of voting power in the company at any time during the previous year.

(ii) In the case of a concern other than company - If an individual is entitled to atleast 20 per cent share in profit in the concern (individually or along with his relatives) at any time during the previous year.

(b) Relative- Relative, in relation to an individual, means the husband, wife, brother or sister or any lineal ascendant or descendant of that individual.

Page 225: INTER CA Taxation - CA Study Web€¦ · INTER CA Taxation Head Office Shraddha, 4th Floor, ... 14. Advance Tax & Interest 268 - 271 15. Tax Deducted at Source (TDS) 272 - 294 16

J.K.SHAH CLASSES INTER C.A.- DIRECT TAX

: 222 :

(c) Concern: The expression “concern” covers both business concern and professional concern and both proprietary and non-proprietary concerns.

(d) If once clubbing is done in the hands of X, salary of X and Mrs. X will be included in the income of X (in the subsequent years), even if income of X is lower than that of Mrs. X in that year. In such a case, the Assessing Officer can club the income of X and Mrs. X in the hands of Mrs. X only if the Assessing Officer is satisfied that it is necessary to do so. The Assessing Officer can take such action only after giving Mrs. X an opportunity of being heard.

TWO WAY CLUBBING: When both husband and wife have substantial interest in a concern, both are in receipt of the remuneration from such concern and such remuneration is received without any technical and professional qualification, then both the remunerations will be clubbed in the hands of husband or wife whose total income, excluding such remuneration, is greater.

Question: X holds 30 per cent equity share capital in Y Ltd. Mrs. X is employed by Y Ltd. (salary being Rs. 1,40,000 per month) as general manager (finance). She does not have any professional qualification to justify the remuneration. Ascertain in whose hands salary income is chargeable to tax. Does it make any difference if Mrs. X was employed by Y Ltd. even prior to her marriage?

Solution: In this case, X has substantial interest in Y Ltd. where Mrs. X is employed. Mrs. X does not have any professional qualification to justify the remuneration of Rs. 1,40,000 per month. Her salary income of Rs. 16,80,000 (i.e., Rs. 1,40,000 x 12) will be taxable in the hands of X. It does not make any difference even if Mrs. X was employed by Y Ltd. prior to her marriage.

WHEN AN INDIVIDUAL IS ASSESSABLE IN RESPECT OF INCOME FROM ASSETS TRANSFERRED TO SPOUSE [SEC. 64(1)(iv)]

The following conditions should be satisfied —

Condition 1 The taxpayer is an individual.

Condition 2 He/she has transferred an asset (other than a house property).

Condition 3 The asset is transferred to his/her spouse for no consideration or inadequate consideration.

Condition 4 The transfer may be direct or indirect.

If the above conditions are satisfied, any income from such asset shall be deemed to be the income of the individual who is the transferor of the asset.

It is irrelevant whether the asset is held by the transferee-spouse in the same form or in a different form. For instance, where cash is gifted by an assessee to his wife and the wife purchases debentures using such cash, interest income is included in the assessee’s total income.

Example

X transfers 100 debentures of IFCI to his wife without adequate consideration. Interest income on these debentures will be included in the income of X.

The income from asset transferred must be calculated in the same way as it would be if the asset has not been transferred. Exemptions, deductions or tax incentives in respect of such income shall be allowed to the transferor and only the net income shall be clubbed.

Page 226: INTER CA Taxation - CA Study Web€¦ · INTER CA Taxation Head Office Shraddha, 4th Floor, ... 14. Advance Tax & Interest 268 - 271 15. Tax Deducted at Source (TDS) 272 - 294 16

J.K.SHAH CLASSES INTER C.A.- DIRECT TAX

: 223 :

ANALYSIS OF CONDITIONS

Condition 1 - Asset is transferred by an individual : This provision of clubbing is applicable if the transferor is an individual (i.e., husband or wife). If the transferor is a person other than an individual then the above provisions are not applicable.

Condition 2 - An asset other than a house property is transferred : To attract this section, an asset other than a house property should be transferred. If a house property is transferred for no or inadequate consideration, then the transferor is “deemed” as owner of the property under section 27.

Condition 3 - Relationship of husband and wife: The relationship of husband and wife should subsist both at the time of transfer of asset and at the time when income is accrued (generally at the end of the previous year). It means that transfer of asset before marriage is outside the scope of this section.

For instance, X transfers 1,000 debentures of IFCI without consideration to his fiancee Miss Y on April 10, 2014. Interest income from these debentures will not be taxable in the hands of X even after their marriage.

Similarly, if transferor-spouse dies, the income, although continued to be enjoyed by the transferee, cannot be included in the income of deceased transferor’s heir, as a widow or widower is not a spouse.

Condition 4 - Transfer includes “indirect” transfer: If two or more transfers are inter-connected and are parts of the same transaction, the aforesaid rule of clubbing is applicable.

For instance, if X gifts or cross transfers Rs.10,000 to Mrs. A and A gifts property worth Rs.10,000 to Mrs. X, the transaction would be indirect transfer without consideration by X to Mrs. X and by A to Mrs. A. The clubbing provisions shall apply in the same manner in which they would have applied in case of direct transfer.

If an individual transfers an asset without consideration to his wife who sells it at a profit, capital gain arising to wife on sale of asset is chargeable to tax in the hands of the individual.

CLUBBING WHEN TRANSFERRED ASSET IS INVESTED IN A BUSINESS: An asset (maybe in cash or kind) is transferred by individual to his/her spouse (directly or indirectly) for no or inadequate consideration. The spouse invests the asset in a business. The amount of income that will be clubbed in the hands of the individual will be determined as follows —

Step one Find out total investment of transferee-spouse in the business on the first day of the previous year.

Step two Find out the amount invested by the transferee-spouse in the said business out of the assets transferred to him/her for no or inadequate consideration by the individual on the first day of the previous year.

Step three Find out the taxable income (exempt income is not included) of the transferee-spouse from the business. If the transferee-spouse becomes a partner of a firm by investing the aforesaid asset then only interest income from the firm is considered under Step three. Share of profit from the firm is not considered under Step three as it is exempt under section 10(2A).

Step four The amount which shall be included in the hands of transferor is determined as follows — Step three x Step two ÷ Step one.

Page 227: INTER CA Taxation - CA Study Web€¦ · INTER CA Taxation Head Office Shraddha, 4th Floor, ... 14. Advance Tax & Interest 268 - 271 15. Tax Deducted at Source (TDS) 272 - 294 16

J.K.SHAH CLASSES INTER C.A.- DIRECT TAX

: 224 :

When clubbing is not applicable : On the basis of the aforesaid discussion and judicial pronouncements, section 64(1)(iv) is not applicable in the following cases and the income will be taxable for the spouse:

� If assets are transferred before marriage.

� If assets are transferred for adequate consideration.

� If assets are transferred in connection with an agreement to live apart.

� If on the date of accrual of income, transferee is not spouse of the transferor.

� If asset is acquired by the spouse out of pin money

� Income arising from accretions to transferred assets. If an assessee gifts debentures of a company to the spouse and, subsequently, the company issues bonus debentures to the spouse, interest on bonus debentures will not be includible in the hands of the assessee under section 64(1)(iv) as there is no transfer of bonus debentures by the assessee to the spouse.

WHEN AN INDIVIDUAL IS ASSESSABLE IN RESPECT OF INCOME FROM ASSETS TRANSFERRED TO SON’S WIFE [SEC. 64(1)(vi)]

One has to satisfy the following conditions —

Condition 1 The taxpayer is an individual.

Condition 2 He/she has transferred an asset.

Condition 3 The asset is transferred to his/her son’s wife for no consideration or inadequate consideration.

Condition 4 The transfer may be direct or indirect.

If the above conditions are satisfied, then income from the asset is included in the income of the taxpayer who has transferred the asset.

WHEN AN INDIVIDUAL IS ASSESSABLE IN RESPECT OF INCOME FROM ASSETS TRANSFERRED TO A PERSON FOR THE BENEFIT OF SPOUSE [SEC. 64(1)(vii)]

One has to satisfy the following conditions—

Condition 1 The taxpayer is an individual.

Condition 2 He/she has transferred an asset.

Condition 3 The transfer may be direct or indirect.

Condition 4 The asset is transferred to a person or an association of persons for no or inadequate consideration.

Condition 5 It is transferred for the immediate or deferred benefit of his/her spouse.

If the aforesaid conditions are satisfied then income from such asset to the extent of such benefit is taxable in the hands of the taxpayer who has transferred the asset.

WHEN AN INDIVIDUAL IS ASSESSABLE IN RESPECT OF INCOME FROM ASSETS TRANSFERRED TO A PERSON FOR THE BENEFIT OF SON’S WIFE [SEC. 64(1)(viii)]

One has to satisfy the following conditions —

Condition 1 The taxpayer is an individual.

Condition 2 The asset is transferred to any person or an association of persons for no or inadequate consideration..

Condition 3 Transfer may be direct or indirect.

Condition 4 The asset is transferred for the immediate or deferred benefit of his/her son’s wife.

If the above conditions are satisfied, then income from the asset to the extent of such benefit is included in the income of the taxpayer who has transferred the asset.

Page 228: INTER CA Taxation - CA Study Web€¦ · INTER CA Taxation Head Office Shraddha, 4th Floor, ... 14. Advance Tax & Interest 268 - 271 15. Tax Deducted at Source (TDS) 272 - 294 16

J.K.SHAH CLASSES INTER C.A.- DIRECT TAX

: 225 :

WHEN AN INDIVIDUAL IS ASSESSABLE IN RESPECT OF INCOME OF HIS MINOR CHILD [SEC. 64(1A)]

All income which arises or accrues to the minor child shall be clubbed in the income of his parent.

Clubbing in the hands of father or mother: The income of minor will be included in the income of that parent whose total income [excluding the income includible under section 64(1A)] is greater.

The following points should be noted —

1. A is minor child of X and Mrs. X. During P.Y. 2017-18, income of A is Rs. 2,500 (this is the first income of A during his life time). During P.Y. 2017-18 income of X is higher than that of Mrs. X. Consequently, income of A will be included in the income of X for P.Y. 2017-18. In the subsequent years (during the minority of A), income of A will be included in the income of X, even if income of Mrs. X is higher than that of X in any of the subsequent years. However, there is one exception. If in the subsequent year, the Assessing Officer wants to include the income of minor child A in the hands of Mrs. X, it can be done only if it is necessary to do so and that too after giving an opportunity of being heard to Mrs. X.

2. Where the marriage of the parents does not subsist, the income of minor will be includible in the income of that parent who maintains the minor child in the relevant previous year.

When clubbing is not attracted: In the cases given below, clubbing provisions of section 64(1A) are not applicable —

1. Income of minor child (from all sources) suffering from any disability of the nature specified under section 80U is not subject to clubbing provision given above.

2. Income of minor child on account of any manual work.

3. Income of minor child on account of any activity involving application of his skill, talent or specialized knowledge and experience.

Exemption under section 10(32): In case the income of an individual includes an income of his or her minor child in terms of section 64(1A), such individual shall be entitled to a maximum exemption of Rs. 1,500 in respect of each minor child.

SECTION 65 – POWER OF ASSESSING OFFICER TO RECOVER TAX FROM TRANSFEREE

The Assessing Officer has the power for the recovery of tax from the person to whom the income actually accrued if the Assessing Officer so desires.

General Rule of computation for clubbing of income u/s. 60 to 64

First compute the income in the hands of the actual recipient under the relevant head of income as if the actual recipient of income is liable to pay tax. All the deductions, exemptions, reliefs, allowances etc shall be allowed as per Income Tax Act. The net income shall be included under the same head of income in the hands of other person.

Page 229: INTER CA Taxation - CA Study Web€¦ · INTER CA Taxation Head Office Shraddha, 4th Floor, ... 14. Advance Tax & Interest 268 - 271 15. Tax Deducted at Source (TDS) 272 - 294 16

: 226 :

J. K. SHAH CLASSES INTER C.A.- DIRECT TAXES

CLUBBING OF INCOME

Q. 1. Mrs. G holds 7% equity shares in B Ltd., where her married sister, Mrs. N also

holds 14% equity shares. Mr. G is employed with B Ltd., without holding technical

professional qualification. The particulars of their income for the Previous Year

2017-2018 are given as follows:

Income of Income of

Mr. G Mrs. G

`̀̀̀ `̀̀̀

(i) Gross Salary from B Ltd. 2,02,000 ----

(ii) Dividend from B Ltd. --- 6,000

(iii) Income from House Property 90,000 ----

Q.2. The following are the particulars of income earned by Mr. Chandrapal and his

family members :

Particulars `̀̀̀

(i) Income from Chandrapal’s profession 2,50,000

(ii) Mrs. Chandrapal’s salary as primary teacher 1,06,000

(iii) Minor son Arav (interest on fixed deposits with a bank which

were gifted to him by his uncle) 12,000

(iv) Arav also has income by way winnings from lottery (gross) 2,20,000

(v) Minor daughter Pallavi’s earnings from sports 1,05,000

(vi) Cash gift received by minor married daughter Garima

from friend of Mrs. Chandrapal 55,000

(vii) Income of minor son Arvind, who suffers from disability

specified in section 80U 1,20,000

Discuss the tax implications in the hands of Mr. Chandrapal and Mrs. Chandrapal.

Q.3. Mr. Ramesh gifted a sum of 5 lacs to his brother ’s minor son on 16-4-2017. On

18-4-2017, his brother gifted debentures worth 6 lacs to Mrs. Ramesh. Son of

Mr. Ramesh’s brother invested the amount in fixed deposit with Bank of in India

9% p.a. interest and Mrs. Ramesh received interest of 45,000 on debentures

received by her.

Discuss the implications under the provisions of the Income-tax Act. 1961.

Q. 4. R held 12% shares in a private limited company. He gifted all the shares to his

wife, Mrs. R on 1.11.2011. Mrs. R obtained a loan of 2,17,000 on 4.11.2017 from

the company when the accumulated profits were 1,16,000.What are the tax

implications of the above transactions?

Page 230: INTER CA Taxation - CA Study Web€¦ · INTER CA Taxation Head Office Shraddha, 4th Floor, ... 14. Advance Tax & Interest 268 - 271 15. Tax Deducted at Source (TDS) 272 - 294 16

: 227 :

J. K. SHAH CLASSES INTER C.A.- DIRECT TAXES

CLUBBING OF INCOME

Q. 5. A Proprietary Business was started by Smt. Rani in the year 2010. As on 1.4.2016

her capital in business was 3,00,000.

Her husband gifted 2,00,000, on 10.4.2016, which amount she invested in her

business on the same date. She earned profits from her proprietary business for

the Financial Year 2016-17 1,50,000 and Financial Year 2017-18 3,90,000.

Compute the income to be clubbed in the hands of Rani's husband for the

A.Y. 2018-2019 with reasons.

Q. 6. Mr. Ram gifted 20,00,000 to his wife. She purchased a house worth 50,00,000.

The source of funds were :

(i) 20,00,000 gift from her husband.

(ii) 10,00,000 personal assets.

(iii) 20,00,000 loan from a private party on interest of 10% p.a.

She rented the building at 80,000 per month. After payment of the Interest on

Loan, she invested surplus amount of rental income with a merchant, which yielded

an interest income of 70,000.

The Assessing Officer wants to add the rental income and interest income in the

income of her husband. Is the Assessing Officer justified in his action?

Q. 7. Divya is born on 7.6.2003. Her betrothal took place on 5.3.2017. On the said day she

received cash gifts of 50,000 each from her father, father's mother, father's father,

mother, mother's mother, and mother's father. All the six relatives made similar gifts on

the day of marriage i.e., on 1.4.2017. The amount so received is deposited in a private

limited company in which her husband has substantial interest. For the year ending

31.3.2018, the company has paid her interest @ 14% i.e. 84,000.

Discuss how this income will be assessed to income-tax.

Q. 8. Mr. & Mrs. X own the following house properties

Place owner M.V Rent pm M.T paid Occupancy

Mumbai X 2,00,000 20,000 40,000 Let out

Panvel X 1,50,000 N A 12000 SOP

Thana Mrs. X 2,20,000 N A 20,000 Mrs. X Parents SOP

Bengaluru Mrs. X 1,80,000 21,000 22,000 Let out

Note: Thane house owned by Mrs X is gifted by Mr. X in the year 05-06, Whereas

Bengaluru house is acquired by cash gifted by Mr. X in the year 07-08.The Interest

on loan for Thana house is `̀̀̀1,20,000 and for Panvel house is `̀̀̀2,60,000/-.

Compute Income from house property in the hands of Mr. X and Mrs. X.

Page 231: INTER CA Taxation - CA Study Web€¦ · INTER CA Taxation Head Office Shraddha, 4th Floor, ... 14. Advance Tax & Interest 268 - 271 15. Tax Deducted at Source (TDS) 272 - 294 16

: 228 :

J. K. SHAH CLASSES INTER C.A.- DIRECT TAXES

CLUBBING OF INCOME

Q. 9. On 21-3-2017 Mr. Janak gifted to his wife Mrs.Thilagam 200 listed shares, which had

been bought by him on 19.4.2013 at `̀̀̀ 2,000 per share. On 1.6.2017, bonus shares

were allotted in the ratio of 1 : 1. All these shares were sold by Mrs.Thilagam as under:

Date of sale Manner of sale No. of Net sales

shares Value ( `̀̀̀)

21-5-2017 Sold in recognized stock exchange, STT paid 100 2,20,000

21-7-2017 Private sale, to an outsider All bonus 1,250

shares per share

28-2-2018 Private sale, to her friend Mrs. Hema 100 1,70,000

(Market value on this date was `̀̀̀ 2,40,000)

Briefly state the income tax consequences in respect of the sale of the shares by

Mrs.Thilagam, showing clearly the person in whose hands the same is chargeable, the

quantum and the head of income in respect of the above transactions. Detailed

computation is not required.

Net Sales value represents the amount credited after all taxes, levies, brokerage,

etc.,and the same may be adopted for computing capital gains.

Page 232: INTER CA Taxation - CA Study Web€¦ · INTER CA Taxation Head Office Shraddha, 4th Floor, ... 14. Advance Tax & Interest 268 - 271 15. Tax Deducted at Source (TDS) 272 - 294 16

: 229 :

J. K. SHAH CLASSES INTER C.A.- DIRECT TAXES

CLUBBING OF INCOME

Ans.1. Computation of Total Income of Mr. G & Mrs. G for the A.Y. 2018-2019.

Particulars Mr. G `̀̀̀ Mrs. G `̀̀̀

Gross Salary 2,02,000

Taxable Salary to be included in the total income of ---- 2,02,000

Mrs G [Sec. 64 (i) (iii)]

Add : Income from House Property 90,000

Add: Income from Other Sources : Dividends to

Mrs.G, but exempt under Sec. 10(34) ---- ----

Total Income 90,000 2,02,000

Note :

1. In the Instant case, Mrs. G along with his sister, holds substantial in B Ltd, and

Mr. G does not hold professional qualification. Accordingly, remuneration of Mr.

G has been included in the total income of Mrs. G.

2. If the requisite conditions of clubbing are satisfied, clubbing provision will apply

even if their application results into lower incidence of tax.

Ans. 2.

As per the provisions of section 64(1A), in case the marriage of the parents subsist,

the income of a minor child shall be clubbed in the hands of the parent whose total

income, excluding the income of the minor child to be clubbed, is greater. In this prob-

lem, it has been assumed that the marriage of Mr. Chandrapal and Mrs. Chandrapal

subsists.

However, in case the income arises to the minor child on account of any manual work

done by the child or as a result of any activity involving application of skill, talent, spe-

cialized knowledge or experience of the child, then, the same shall not be clubbed in

the hands of the parent.

Further, the income of minor child suffering from disability of the nature specified under

section 80U shall also not be included in the hands of parents.

Tax implications

(i) Income of `̀̀̀ 2,50,000 from Mr. Chandrapal’s profession shall be taxable in the

hands of Mr. chandrapal under the head ‘Profits and gains of business or

profession’.

(ii) Salary of `̀̀̀1,06,000 received by Mrs. Chandrapal as a Primary teacher shall be

taxable as ‘Salaries’ in the hands of Mrs. Chandrapal.

(iii) Income from fixed deposit of `̀̀̀ 12,000 arising to the minor son Arav, shall be

clubbed in the hand of the father, Mr. Chandrapal as “Income from other sources”.

since Mr. Chandrapal’s income is greater than the income of his wife before

including the income of the minor child.

As per section 10(32), income of a minor child which is includible in the income of

the parent shall be exempt to the extent of `̀̀̀ 1,500 per child. The balance income

would be clubbed in the hand of the parent as “ Income from other sources”.

Therefore, `̀̀̀ 10,500 would be clubbed in the hands of Mr.Chandrapal.

Page 233: INTER CA Taxation - CA Study Web€¦ · INTER CA Taxation Head Office Shraddha, 4th Floor, ... 14. Advance Tax & Interest 268 - 271 15. Tax Deducted at Source (TDS) 272 - 294 16

: 230 :

J. K. SHAH CLASSES INTER C.A.- DIRECT TAXES

CLUBBING OF INCOME

(iv) income of `̀̀̀ 2,20,000 arising to minor son Aray from lottery shall be included in

the hands of Mr. Chandrapal as “Income from other sources”’ since Mr.

Chandrapal’s income is greater than the income of his wife before including the

income of minor child.

Note – Mr. Chandrapal can reduce the tax deducted at source from such lottery

income while computing his net tax liability.

(v) Income of `̀̀̀ 1,05,000 arising to the minor daughter Pallavi from sports shall not

be included in the hands of the parent, since such income has arisen to the mi-

nor daughter on account of an activity involving application of her skill.

(vi) The clubbing provisions are attracted even in respect of income of minor mar-

ried daughter. As per section 56(2)(vii), cash gifts received from any person/

persons exceeding `̀̀̀ 50,000 during the year in aggregate is taxable. Since the

cash gift in this case exceeds `̀̀̀ 50,000, the amount of `̀̀̀ 55,000 shall be tax-

able under section 56(2)(vii). This amount shall be clubbed in the hands of Mr.

Chandrapal and exemption under section 10(32) of `̀̀̀ 1,500 per child shall be

allowed in his hands.

(vii) Income of minor son Arvind, who suffers from disability specified under section

80U, shall not be included in the hands of either of his parents.

Ans.3. In the given case, Mr. Ramesh gifted a sum of `̀̀̀ 5 lacs to his brother’s minor son on

16-4-2017 and simultaneously, his brother gifted debentures worth `̀̀̀ 6 lacs to Mr.

Ramesh’s wife on 18-4-2017. Mr. Ramesh’s brother’s minor son invested the gifted

amount of ̀ 5 lacs in fixed deposit with Bank of India.

These transfers are in the nature of cross transfers. Accordingly, the income from the

assets transferred would be assessed in the hands of the deemed transferor because

the transfers are so intimately connected to form part of a single transaction and each

transfer constitutes consideration for the other by being mutual or otherwise.

As per section 64(1A), all income of a minor child is includible in the hand of the parent,

whose total income, before including minor’s income is higher. Accordingly, the inter-

est income arising to Mr. Ramesh’s brother’s son from fixed deposits would be

included in the total income of Mr. Ramesh’s brother, assuming that Mr. Ramesh’s broth-

er’s total income is higher than his wife’s total income, before including minor’s

income. Mr. Ramesh’s brother can claim exemption of `̀̀̀ 1,500 under section 10(32).

Interest on debentures arising in the hands of Mrs. Ramesh would be taxable in the

hands of Mr. Ramesh as per section 64(1)(iv).

This is because both Mr. Ramesh and his brother are the indirect transferors of the

income to their spouse and minor son, respectively, with an intention to reduce their

burden of taxation.

In the hands of Mr. Ramesh Interest received by his spouse on debentures of `̀̀̀5 lacs

alone would be included and not the entire interest income on the debentures of ` ` ` ` 6

lacs, since the cross transfer is only to the extent of `̀̀̀ 5 lacs.

Hence, only proportional interest (i.e., 5/6th of interest on debentures received) `̀̀̀ 37,500

would be includible in the hand of Mr. Ramesh.

The provisions of section 56(2)(vii) are not attracted in respect of sum of money transferred

of value of debentures transferred, since in both the cases, the transfer is from a relative.

Page 234: INTER CA Taxation - CA Study Web€¦ · INTER CA Taxation Head Office Shraddha, 4th Floor, ... 14. Advance Tax & Interest 268 - 271 15. Tax Deducted at Source (TDS) 272 - 294 16

: 231 :

J. K. SHAH CLASSES INTER C.A.- DIRECT TAXES

CLUBBING OF INCOME

Ans.4. Gift of shares by Mr. R to Mrs. R is a non-taxable transfer for Mr.R. The

shares received are not taxable for Mrs. R too as they are received from

a "relative" [spouse of individual].

The loan obtained from the private limited company would be taxable as

dividend u/s 2(22)(e) since all the conditions are fulfilled. However, since

the shares have been gifted, such dividend income will be clubbed in the

hands of Mr. R u/s 64(1)(iv) of ` ` ` ` 1,16,000.

Ans. 5. There will be no clubbing of income in the hands of Rani’s husband for

P.Y. 2016-2017 since the gift amount has been invested by Smt. Rani in the

business during P.Y. 2016-17 and not on the first day of the year. The

entire profit of `̀̀̀ 1,50,000 shall be taxable for Smt. Rani.

The profit for P.Y. 2017-2018 shall be clubbed in the hands of Rani’s

husband u/s. 64(1)(iv) as follows:

Total investment in business on 1-4-2017

Opening Balance on 1-4-2016 3,00,000

Add: Gift amount invested in business on 10-4-2016 2,00,000

Add: Profit for P.Y. 2016-2017 1,50,000

Closing Balance on 31-3-2017 i.e. Balance on 1-4-2017 6,50,000

Amount to be clubbed =

Gift amount invested in business on the first day of P.Y. x Taxable Profits

Total investment in business on the first day of P.Y.

= ` ` ` ` 2,00,000

x `̀̀̀ 3,90,000 = `̀̀̀ 1,20,000`̀̀̀ 6,50,000

Amount taxable for Smt. Rani= `̀̀̀ 3,90,000 – `̀̀̀ 1,20,000 = `̀̀̀ 2,70,000/-

Ans.6. Mr. Ram has gifted cash which was subsequently utilized to purchase a house

property. So the provision of deemed owner shall not apply. The rent income

from the property shall be clubbed in the hands of Mr. Ram u/s. 64(1)(iv).

The interest income is generated out of investment of surplus rent income. This

implies that interest is “Income on already clubbed Income” and accordingly

cannot be clubbed in the hands of Mr. Ram. The interest shall be taxable for Mrs. Ram.

Ans. 7. Such interest income will be clubbed u/s 64(1A) in that parent’s hands whose

income prior to such clubbing is higher, after allowing exemption u/s 10(32)

of 1,500 or the income clubbed (whichever is least).

Even if the amount is invested in an organisation in which her husband is

substantially interested, the clubbing provisions u/s 64(1)(ii) will not be

attracted as interest is not remuneration but return on investment. The

amount invested by Divya is not gifted by her husband and hence,

clubbing u/s. 64(1)(iv) is also not attracted. All income of the minor,

including minor married daughter, is clubbed with the parent u/s 64(1A).

After attaining majority, such interest income will be taxable in the hands of Divya.

Page 235: INTER CA Taxation - CA Study Web€¦ · INTER CA Taxation Head Office Shraddha, 4th Floor, ... 14. Advance Tax & Interest 268 - 271 15. Tax Deducted at Source (TDS) 272 - 294 16

: 232 :

J. K. SHAH CLASSES INTER C.A.- DIRECT TAXES

CLUBBING OF INCOME

Ans. 9. The Shares gifted by Mr Janak on 21st March 2017 is a non-taxable transfer

for him. The shares received as a gift shall not be taxable in the hands of Mrs

Janak as the gift is received from relative.

The shares sold on 21st May 2017 are long term capital asset (after

including the holding period of the previous owner). Since the shares are sold

via recognised stock exchange the long term capital gain is exempt under

section 10(38). As the gain is exempt the question of clubbing doesn’t arise.

Bonus shares represent accretion to the assets and hence capital gains on

transfer of bonus share shall not be clubbed, it shall be taxable in the hands

of Mrs Thilagam.

Computation of Income from Capital Gains in the hands of Mrs

Thilagam For Assessment year 2018-2019

Particulars `̀̀̀

Full value of Consideration 1,25,000

Less: Cost of acquisition (Nil)

(STCA as the holding period is less than 12months

i.e. 1.6.2017 to 21.07.2017)

Short term Capital gain 1,25,000

The Shares sold on 28.02.2018 are long term capital asset (after including

the holding period of the previous owner). Since the share are sold privately

exemption under section 10(38) is not available. The capital gains or loss on

transfer of shares shall be clubbed in the hands of Mr Janak as per section

64(1)(iv)

Computation of Income from Capital Gains

Particulars `̀̀̀

Full value of Consideration 1,70,000

Less: Indexed Cost of acquisition (Nil)

(LTCA as the holding period is more than 12months

i.e. 19.04.2013 to 28.02.2018)

Long term Capital gain

Page 236: INTER CA Taxation - CA Study Web€¦ · INTER CA Taxation Head Office Shraddha, 4th Floor, ... 14. Advance Tax & Interest 268 - 271 15. Tax Deducted at Source (TDS) 272 - 294 16

: 233 :

J. K. SHAH CLASSES INTER C.A.- DIRECT TAXES

WHAT IS THE MODE OF SET OFF AND CARRY FORWARD

The process of setting off of losses and their carry forward may be covered in the following

steps:

Step 1 : Inter-source adjustment under the same head of income.

Step 2 : Inter-head adjustment in the same assessment year. Step 2 is applied only if a

loss cannot be set off under step 1.

Step 3 : Carry forward of a loss. Step 3 is applied only if a loss cannot be set off under

Steps 1 and 2.

Inter source adjustment (Sec. 70):

General rule – If the net result for any assessment year, in respect of any source under

any head of income, is a loss, the assessee is entitled to have the amount of such loss

set off against his income from any other source under the same head of income for the

same assessment year.

Exceptions

���� Loss from speculation business : Loss in a speculation business can be set off only

against the profits in a speculation business.

Note:

* Loss from non-speculation business can be set off against other non-speculation

business profits and if not fully set off, then it can be set off against speculation

business profits also.

* Loss from a specified business – Any loss, computed in respect of any specified

business referred to in section 35 AD, shall not be set off except against profits and

gains, if any other specified business.

���� Loss from the activity of owning and maintaining race horses - Loss incurred in

the business of owning and maintaining race horses cannot be set off against income,

if any, from any other source, except, income from such business in the future.

Note:

* Although it is a business income, but it is taxable under income from sources,

since it promotes gambling.

� Loss cannot be set off against winnings from lotteries,crossword puzzles,

etc.- As per section 58 a loss cannot be set off against winnings from lotteries,

crossword puzzles, races including horse races, card game and other games of

any sort or from gambling or betting of any form or nature.

� Long - term capital loss can be set off only against long - term capital gains

i.e. it cannot be set off against short term capital gains.

Note :

* Short term capital loss can be set off against any other short term capital

gains as first priority and if not fully set off, then set off against long term

capital gains if any.

SET OFF & CARRY FORWARD OF LOSSES

Page 237: INTER CA Taxation - CA Study Web€¦ · INTER CA Taxation Head Office Shraddha, 4th Floor, ... 14. Advance Tax & Interest 268 - 271 15. Tax Deducted at Source (TDS) 272 - 294 16

: 234 :

J. K. SHAH CLASSES INTER C.A.- DIRECT TAXES

INTER-HEAD ADJUSTMENT – HOW MADE [SEC. 71]

General rule – Where the net result of computation made for any assessment year in

respect of any head of income is a loss, the same can be set off against the income from

other heads.

Provisions illustrated – X has two non-speculative businesses – Business A and Busi-

ness B. Besides he has income from house property. The result of the three sources of

income is given below –

Business income Property income

` `

Business A (-) 2,90,000

Business B 70,000

Income from house property 5,10,000

Total (-) 2,20,000 5,10,000

In this case, business loss of Rs. 2,20,000 can be adjusted against property income of

Rs. 5,10,000. Consequently, the property income is reduced to Rs. 2,90,000. It may be

noted that X does not have any option to set off (or not to set off) the business loss

against property income.

Exceptions- The following are the exceptions to the aforesaid rule—

���� Loss in a speculation business - Loss in a speculation business cannot be set

off against any other income.

���� Loss in a business specified under section 35AD - Loss, computed in respect

of any specified business referred to in section 35AD cannot be set off against

any other income.

���� Loss under the head "Capital gains" - Losses under the head "Capital gains"

cannot be set off against any income except income under the head "Capital gains".

���� Loss from the activity of earning and maintaining race horses - Losses from

the activity of owning and maintaining race horses cannot be set off against any

other income.

���� Business loss cannot be set off against salary income' - Loss from business

or profession (including depredation) cannot be set off against income under the

head "Salaries".

���� Loss cannot be set off against winnings lotteries, etc. - By virtue of section

58(4) a loss cannot be set off against winnings from lotteries, crossword puzzles,

races (including horse races), card games and other games of any sort or from

gambling or betting of any form or nature.

���� Loss from house property cannot be set off more than 2 lakhs against any other

head of income

Page 238: INTER CA Taxation - CA Study Web€¦ · INTER CA Taxation Head Office Shraddha, 4th Floor, ... 14. Advance Tax & Interest 268 - 271 15. Tax Deducted at Source (TDS) 272 - 294 16

: 235 :

J. K. SHAH CLASSES INTER C.A.- DIRECT TAXES

Sec. 71B :Carry forward and set off of loss from house property

Loss to the extent not set off u/s 70 & 71, is allowed to be carried forward for set off against

future income from house property only, for a maximum of 8 assessment years immediately

after the end of the relevant assessment year in which the loss was suffered.

Sec. 72 : Carry forward and set off of non-speculation business losses :

Non-speculation business loss to the extent not set off u/s 70 &71, is allowed to be carried

forward for set off against the profits and gains of any business (non-speculation as well as

speculation) and not necessarily the same business income of the future. If not fully set off,

then against any ‘business income’ taxable under other heads such as:-

* Income from house property (where it is the business of owning and letting out

residential houses)

* Income from other sources

- Income from other activity of owning & maintaining race horses.

- Dividend on foreign company shares or co-operative society shares held

as stock in trade.

Even if the business to which such loss relates is discontinued, yet the loss is entitled to be

carried forward for a maximum period of 8 assessment years from the end of

assessment year of loss, failing which the unabsorbed loss is lost i.e. not allowed to be

carried forward

Unabsorbed depreciation under sec. 32(2): Unabsorbed depreciation allowance

(U.A.D.) will be set off and carried forward as follows :

(1) Normally entitled depreciation is allowed as a deduction to the maximum extent of

available profits prior to claiming depreciation, but after claiming all other

deductions, i.e. to the maximum extent to bring the profits to NIL.

A person cannot create a loss or cannot increase an already existing cash loss, by

claiming depreciation.

The portion of depreciation which could not be allowed as a deduction is known as

unabsorbed depreciation (i.e. U.A.D).

(2) Such U.A.D can be set off u/s 70 against any business income (non-speculation

or speculation)

(3) If not fully set off u/s 70 as above, then the unabsorbed portion can be set off u/s 71

against any another income of the assessee for the same year except salary and

casual winnings.

(4) U.A.D. (after sec 70 & 71) will be carried forward u/s 32(2) to be set off against

any income (except casual winnings & salaries) till it is fully set off, i.e. without

any time limit, even if the business to which such U.A.D. relates, is no longer

in existence in the year of set off.

Page 239: INTER CA Taxation - CA Study Web€¦ · INTER CA Taxation Head Office Shraddha, 4th Floor, ... 14. Advance Tax & Interest 268 - 271 15. Tax Deducted at Source (TDS) 272 - 294 16

: 236 :

J. K. SHAH CLASSES INTER C.A.- DIRECT TAXES

However the priority of set off of U.A.D. in the subsequent years against subsequent

year’s income from business or profession would be :

* 1st priority to current depreciation of the subsequent year.

* 2nd priority to subsequent year’s any other business loss u/s 70

* 3rd priority to any other head’s loss u/s 71 (i.e. loss under The head “income from

house property“ or “income from other sources” which remains unabsorbed after

applying section 70 provisions) of the subsequent year.

* 4th priority to brought forward business loss u/s 72 within the time limit of 8 years.

* If still some positive income remains after above set off, under subsequent year’s

income from business , then the 5 th priority to unabsorbed depreciation and the

amount not set off can be set off against any other positive income of subsequent

year except income from salaries & casual winnings.

Sec. 73 : Loss in speculation business.

(1) This section allows such loss to be set off only against profit and gains of any other

such speculation business of the same previous year, under sec. 70.

(2) Unabsorbed speculation business loss will be carried forward and set off against

speculation business profit of subsequent year u/s. 73.

(3) Thus, speculative business loss cannot be set off against any other head of Income.

(4) It is allowed to be carried forward and set off for a period of 4 years from the end

of assessment year of loss.

The business in which speculation loss is incurred may be discontinued in the rel-

evant year of set off.

Sec.73A : Losses in specified business u/s 35AD

���� Any loss, computed in respect of any specified business referred to in section

35AD shall not be set off except against profits and gains, if any, of any other

specified business. In case of Hotel and Hospital business, such loss can be set

off against the profit of another specified business, whether OR NOT the latter is

eligible for deduction u/s. 35AD.

���� If any loss remains unabsorbed as above, it shall be carried forward and set off

against any income of any specified business of the next assessment year / years

till it is fully set off, without any time limit.

Sec. 74 : Loss under the head Capital Gains :

This section permits loss under the head capital gains to be carried forward and set off

against Income under the head Capital Gains of subsequent year as under:

(i) ���� Unabsorbed short term capital losses can be carried forward and set off

against short term or long term capital gains.

���� Unabsorbed long term capital loss can be carried forward and set off against

long - term capital gains only.

(ii) Thus loss under capital gains cannot be set off against any other head of income.

(iii) Carry forward is allowed for a maximum of 8 years from the end of assessment year of loss

Page 240: INTER CA Taxation - CA Study Web€¦ · INTER CA Taxation Head Office Shraddha, 4th Floor, ... 14. Advance Tax & Interest 268 - 271 15. Tax Deducted at Source (TDS) 272 - 294 16

: 237 :

J. K. SHAH CLASSES INTER C.A.- DIRECT TAXES

Sec. 74 A. : Loss from the activity of owning and maintaining horses for racing :

(a) This section permits it to be carried forward in the subsequent years.

(b) It cannot be adjusted against any other head of income in the same year.

(c) In the subsequent years it can be set off only against the income of the same activity.

(d) Such carry forward is allowed only for period of 4 years from the end of

assessment year of loss.

Sec 78: Change in constitution of partnership firm and its effect on carry forward

of losses:

When there is retirement or death of a partner (outgoing partner) in a P.Y., the partnership

firm shall NOT be allowed to carry forward and set off losses proportionate to the share

of the outgoing partner.

This restriction does not apply to Unabsorbed Depreciation u/s. 32(2) i.e. U.A.D. shall

be carried forward and set off FULLY without any reduction.

Sec 80: Submission of return of losses

According to this section carry forward and set off of only “Assessed Loss” will be

allowed (and not the “returned loss” as per the return of income).

It is compulsory to file return of income u/s 139(3) in case of a loss

[u/s 139(3) filing of ‘return of income ‘on or before the due date as specified u/s 139(1):-

* 30/9 of assessment year if audit is compulsory

* 30/11 of assessment year if there is an international transaction

31/7 of assessment year in any other case].

However in the case of the following, the loss will be allowed to be carried forward and

set off, even if the return is not filed on or before its due date :-

- House property loss u/s 71B

- U.A.D u/s 32(2)

- Specified business loss u/s 73A

Measures to Curb dividend stripping u/s 94(7).

If a person buys securities or units within a period of 3 months prior to the record date

and subsequently sells or transfers such securities or units within a period of 3 months

(for securities) or 9 months (for units) after the record date, the person may suffer a short

term capital loss (STCL) on sale of securities or units. Such securities or units generate

dividend income which is exempt u/s. 10(34) or 10(35).

In such a case, the STCL, to the extent of dividend income, shall not be allowed to be set off

u/s. 70 (Inter Source Adjustment) or carried forward u/s. 74.

"Record date" means a date fixed by the company or mutual fund for declaration of divi-

dend. If a person is the holder of securities or units on the record date, he shall be enti-

tled to receive the dividend income.

Page 241: INTER CA Taxation - CA Study Web€¦ · INTER CA Taxation Head Office Shraddha, 4th Floor, ... 14. Advance Tax & Interest 268 - 271 15. Tax Deducted at Source (TDS) 272 - 294 16

: 238 :

J. K. SHAH CLASSES INTER C.A.- DIRECT TAXES

Short term against short term or long term, but long term only against long term.

# Yes, only against any other specified business income, with the exception

hotel and hospital specified business .

Page 242: INTER CA Taxation - CA Study Web€¦ · INTER CA Taxation Head Office Shraddha, 4th Floor, ... 14. Advance Tax & Interest 268 - 271 15. Tax Deducted at Source (TDS) 272 - 294 16

: 239 :

J. K. SHAH CLASSES INTER C.A.- DIRECT TAXES

Q. 1. Compute the GTI of Mr. Madhavan for the Assessment year 2018-19 from the

following particulars :

(i) Net house property income as computed under the head 'Income from

House Property' `̀̀̀ 2,70,000

(ii) Income from business before adjusting the following ` ` ` ` 90,000

(a) Carried forward business loss of A.Y. 2016-17 `̀̀̀ 1,70,000

(b) Current depreciation `̀̀̀ 30,000

(c) Carried forward unabsorbed depreciation of earlier years `̀̀̀ 1,40,000

(iii) Short - term capital gains - Jewellery `̀̀̀ 1,60,000

(iv) Long - term capital loss - Shares ` ` ` ` 40,000

(v) Long - term capital gains - Debentures ` ` ` ` 2,00,000

(vi) Dividend on shares of a co-operative society held as stock in trade ` ` ` ` 10,000

(vii) Dividend on shares held as investment from a foreign company carrying

on agricultural operation `̀̀̀ 12,000

(viii) Income from growing and manufacturing coffee (cured and roasted) `̀̀̀ 1,00,000

Q. 2. Mr. Yeshwant submits the following information for the financial year ending 31st

March, 2018 He desires that you should (a) compute the gross total income and

(b) ascertain the amount of losses that can be carried forward :

(I) He has two houses : `̀̀̀

(a) House No. I - Computed income 36,000

(b) House No. II - Current year loss 10,000

(c) Brought forward loss of assessment year 2013- 14 of second house 30,000

(II) He has three proprietary businesses :

(a) Textile business :

(i) Discontinued from 31st October, 2017 current year loss 25,000

(ii) Brought forward business loss of the assessment year 2012-13 80,000

(b) Chemical business :

(i) Discontinued from 1st March, 2017 hence no profits / loss in current year NIL

(ii) Bad debts allowed in earlier years recovered during this year 30,000

(iii) Carried forward business loss for the assessment year 2017-18 20,000

(c) Leather business :

Profit for the current year 70,000

(III) Capital Gains

(i) Short - term capital gains 20,000

(ii) Long - term capital loss 15,000

CLASS WORK PROBLEMS

Page 243: INTER CA Taxation - CA Study Web€¦ · INTER CA Taxation Head Office Shraddha, 4th Floor, ... 14. Advance Tax & Interest 268 - 271 15. Tax Deducted at Source (TDS) 272 - 294 16

: 240 :

J. K. SHAH CLASSES INTER C.A.- DIRECT TAXES

Q. 3. The following are the details relating to Mr. Srivatsan, a resident Indian, aged 57,

relating to the year ended 31-3-2018 :

`̀̀̀

Income from salaries 2,20,000

Loss from House property 1,90,000

Loss from cloth business 2,40,000

Income from speculation business 30,000

Loss from specified business covered by section 35AD 20,000

Long-term capital gains from sale of listed shares in recognized

stock exchange (STT paid) 1,10,000

Long term capital gains on sale of urban land 2,50,000

Loss from card games 32,000

Income from betting 45,000

Life Insurance Premium paid 1,20,000

Compute the total income and show the items eligible for carry forward.

Q. 4. X purchases on 10.5.2017 1,000 equity shares of `̀̀̀ 10 each in A Ltd. @ `̀̀̀ 55.55.

On 20.10.2017 he transfers 800 shares @ `̀̀̀ 37/- per share and remaining 200

shares are transferred on 20.12.2017 @ `̀̀̀ 20 per share. A Ltd., declares 50%

dividend on record date 3.8.2017. During previous year 2017-18, X has

long - term capital gains of `̀̀̀ 76,000.

Compute the loss to be disallowed u/s 94(7) and net changeable long - term capital gains.

Q. 5. M/s. Vivitha & Co., a partnership firm, with four partners A, B, C and D having

equal shares, furnishes the following details, summarized from the valid returns of

income filed by it :

Assessment year Item eligible for carry forward and set off

2016 - 17 : Unabsorbed business loss `̀̀̀ 1,20,000

2017 - 18 : Unabsorbed business loss `̀̀̀ 1,90,000

2017 - 18 : Unabsorbed depreciation `̀̀̀ 1,20,000

2017 - 18 : Unabsorbed long - term Capital Loss :

- from shares `̀̀̀ 1,10,000

- from building `̀̀̀ 1,90,000

C who was a partner during the last three years, retired from the firm with effect from 1.4.2017.

The summarized results of the firm for the assessment year 2018 - 19 are as under :

`̀̀̀

Income from house property 70,000

Income from business

Speculation 2,20,000

Non - speculation (-) 50,000

Capital gains

Short - term (from sale of shares) 40,000

Long - term (from sale of building) 2,10,000

Income from other sources 60,000

Briefly discuss, how the items brought forward from earlier years can be set off in

the hands of the firm for the assessment year 2018-19, in the manner most

beneficial to the assessee. Also show the items to be carried forward.

Page 244: INTER CA Taxation - CA Study Web€¦ · INTER CA Taxation Head Office Shraddha, 4th Floor, ... 14. Advance Tax & Interest 268 - 271 15. Tax Deducted at Source (TDS) 272 - 294 16

: 241 :

J. K. SHAH CLASSES INTER C.A.- DIRECT TAXES

Ans. 4. Mr. X

Computation of capital gains for A.Y. 2018-2019

(a) 800 shares

Particulars `̀̀̀

Full value of consideration (800 shares x `̀̀̀37/ share) 29,600

Less: Cost of Acquisition (800 shares x `̀̀̀ 55.55/share)

(STCA as h.p. < 12 months from 10-5-17to 20-10-17) (44,440)

STCL (14,840)

Less: Loss to be disallowed u/s 94(7): LEAST OF

STCL 14,840

or

Dividend (800 shares x `̀̀̀ 5 / share) 4,000 4,000

STCL, eligible for carry forward and set off (10,840)

(b) 200 shares

Particulars `̀̀̀

Full value of consideration (200 shares x `̀̀̀ 20/ share) 4,000

Less: Cost of Acquisition (200 shares x `̀̀̀ 55.55/ share)

(STCA as h.p. < 12 months from 10-5-2017 to 20-12-2017) (11,110)

STCL (7,110)

Less: Loss to be disallowed u/s 94(7) N.A.

STCL, eligible for carry forward and set off (7,110)

Capital Gains for A.Y. 2018-2019:

L.T.C.G. (as given) `̀̀̀ 76,000

Less: Set off u/s. 70 of STCL on

(a) 800 shares `̀̀̀ 10,840

(b) 200 shares `̀̀̀ 7,110 (`̀̀̀ 17,950)

TAXABLE L.T.C.G. `̀̀̀ 58,050

Ans. 5. M/s. Vivitha & Co.

Statement showing losses eligible for set off and carry forward

as per Sec 78 for A.Y. 2018-19

A.Y. of Loss Particulars Total Loss C’s share Eligible

(1/4th) Loss

2016-17 Business Loss 1,20,000 30,000 90,000

2017-18 Business Loss 1,90,000 47,500 1,42,500

2017-18 Unabsorbed Depreciation (Sec 78 N.A.) 1,20,000 NIL 1,20,000

2017-18 L.T.C.L.- shares 1,10,000 27,500 82,500

2017-18 L.T.C.L.- building 1,90,000 47,500 1,42,500

CLASS WORK SOLUTIONS

Page 245: INTER CA Taxation - CA Study Web€¦ · INTER CA Taxation Head Office Shraddha, 4th Floor, ... 14. Advance Tax & Interest 268 - 271 15. Tax Deducted at Source (TDS) 272 - 294 16

: 242 :

J. K. SHAH CLASSES INTER C.A.- DIRECT TAXES

Computation of Total Income for A.Y. 2018-19

Particulars ` ` `` ` `` ` `` ` `

Income from House Property 70,000

Income from Business

Speculation profit 2,20,000

Less : Non speculation losses, set off u/s. 70

- Inter Source Adjustment (50,000)

Net 1,70,000

Less : Set off of b/f Business loss of A.Y. 2016-17 u/s. 72

- Eligible Loss set off (90,000)

Net 80,000

Less: Set off of b/f Business loss of A.Y. 2017-18 u/s. 72 (80,000) NIL

- Eligible Loss b/f (1,42,500)

- Set off possible 80,000

C/f loss u/s 72 upto to Max. A.Y. 2025 - 26 (62,500)

Capital Gains

L.T.C.G. 2,10,000

Less: Set off of b/f L.T.C.L. – shares of A.Y. 2017-18 u/s. 74

- Eligible LTCL set off (82,500)

Net 1,27,500

Less: Set off of b/f L.T.C.L. – building of A.Y. 2017-18 u/s. 74 (1,27,500)

NIL

- Eligible LTCL b/f (1,42,500)

- Set off possible 1,27,500

L.T.C.L. c/f u/s upto Max. A.Y. 2025-26 to be set off against

future L. T. C.G. only (15,000)

S.T.C.G. 40,000 40,000

Income from Other Sources 60,000

Total 1,70,000

Less: Set off of b/f U.A.D. of A.Y. 2017-18 u/s. 32(2)

- Eligible U.A.D. fully set off (1,20,000)

TAXABLE INCOME 50,000

Page 246: INTER CA Taxation - CA Study Web€¦ · INTER CA Taxation Head Office Shraddha, 4th Floor, ... 14. Advance Tax & Interest 268 - 271 15. Tax Deducted at Source (TDS) 272 - 294 16

J.K. SHAH CLASSES INTER C.A.- DIRECT TAXES

: 243 :

DEDUCTION IN RESPECT OF CERTAIN DONATIONS FOR SCIENTIFIC RESEARCH

OR RURAL DEVELOPMENT [SECTION 80GGA]

Who can claim deduction – An assessee (other than an assessee whose gross total in-

come includes income chargeable under the head “ Profits and gains of business or profes-

sion) can claim deduction under section 80GGA.

What is the qualifying expenditure – Donation should be given to an approved research

association, university, college or other institution to be used for scientific research. A contri-

bution can also be given for the purpose of eligible project under section 35AC or for the

purpose of notified National Fund For rural development or notified National Urban Poverty

Eradication Fund.

Amount of deduction – 100% for the aforesaid donation is deductible. Donation can be

given in cash or by cheque or draft. However, no deduction shall be allowed under section

80GGA in respect of a cash contribution exceeding ̀ 10,000.

DEDUCTION IN RESPECT OF CONTRIBUTIONS GIVEN TO POLITICAL PARTIES OR

ELECTORAL TRUST - TO WHAT EXTENT DEDCUTIBLE [SECS. 80 GGB AND 80GGC]

While computing the total income of an assessee (including an Indian company), any sum

contributed uring previous year to any political party or electoral trust shall qualify for deduc-

tion.

However, the deduction is not available to a local authority and every artificial juridical person

wholly or partly unded by lire Government).

DEDUCTTON IN RESPECT OF EMPLOYMENT OF NEW EMPLOYEES [SEC. 80JJAA]

The provisions of section 80JJAA (as substituted with effect from the assessment year 2017-

18) are given below-

Conditions - One has to satisfy the following conditions -

1. The assessee has income from business and is subject to tax audit under section

44AB.

2. The business of the assessee is not formed by splitting up, or the reconstruction, of an

existing business.

However, this condition is not applicable in respect of a business which is formed as a result

of the re-establishment, reconstruction or revival by the assessee of the business of any such

undertaking as referred to section 33B.

Business is not acquired by the assessee by way of transfer from any other person or as a

result of any business organisation.

DEDUCTIONS FROM GROSS TOTALINCOME - CHAPTER - VIA (PART II)

Page 247: INTER CA Taxation - CA Study Web€¦ · INTER CA Taxation Head Office Shraddha, 4th Floor, ... 14. Advance Tax & Interest 268 - 271 15. Tax Deducted at Source (TDS) 272 - 294 16

J.K. SHAH CLASSES INTER C.A.- DIRECT TAXES

: 244 :

Amount of deduction - If the above conditions are satisfied, an amount equivalent to 30

per cent of additional employee cost (incurred in the course of such business in the

previous year) is deductible under section 80JJAA three assessment years including the

assessment year relevant to the previous year in which such employment provided. Books

of account should be audited and report of audit should be submitted along with the

return income. Deduction should be claimed in the return of income (otherwise deduction

is not available). The wing points should be noted ,

1. Additional employee cost" means total emoluments paid or payable to additional

employees employed r.ng the previous year.

2. In the case of an existing business, the additional employee cost shall be nil, if—

(a) there is no increase in the number of employees from the toted number of

employees employed as on the last day of the preceding year;

(b) emoluments are paid otherwise than by an account payee cheque or ac-

count payee bank draft or by use of electronic clearing system through a

hank account.

3. In the first year of a new business, emoluments paid or payable to employees em-

ployed during that previous LT shall be deemed to be the additional employee

cost.

4. “Additional employee" means an employee who has been employed during the

previous year and whose employment has the effect of increasing the total number

of employees employed by the employer as on the last of the preceding year, but

does not include, —

(a) an employee whose total emoluments are more than Rs. 25,000 per month;

or

(b) an employee for whom the entire contribution is paid by the Government

under the Employees' Pension Scheme notified in accordance with the pro-

visions of the Employees' Provident Funds and Miscellaneous Provisions

Act, 1952;

(c) an employee employed for a period of less than 240 days’!' during the pre-

vious year; or

(d) an employee who does not participate in the recognised provident fund.

5. "Emoluments" mean any sum paid or payable to an employee in lieu of his em-

ployment by whatei called, but does not inc lude employer's contribution to pen-

sion fund/provident fund/any other fun benefit of employee under any law. Further,

it does not include lump sum payment at the time of ten of service, or superannua-

tion or voluntary retirement, such as gratuity, severance pay, leave enca voluntary

retrenchment benefits, commutation of pension, and the like.

6. The provisions of old section 80JJAA shall apply to an assessee who is eligible to

claim any deductio section 80JJAA for the assessment year 2016-17 (or any ear-

lier assessment year).

Page 248: INTER CA Taxation - CA Study Web€¦ · INTER CA Taxation Head Office Shraddha, 4th Floor, ... 14. Advance Tax & Interest 268 - 271 15. Tax Deducted at Source (TDS) 272 - 294 16

J.K. SHAH CLASSES INTER C.A.- DIRECT TAXES

: 245 :

Deduction in respect of royalty income of authors - To what extent available

[Sec.80QQB]

The provisions of section 80QQB are given below —

Conditions - The following conditions should be satisfied —

���� Resident individual - The taxpayer is an individual resident in India. He may be an

Indian citizen at citizen. He may be resident and ordinarily resident or resident but

not ordinarily resident. But he s a non-resident in India.

���� Author or joint author -He is an author or joint author.

���� Literary work - The book authored by him is work of literary, artistic or scientific nature.

However, shall not include brochures, commentaries, diaries, guides, journals, maga-

zines, newspapers, pam books for schools, tracts and other publications of similar

nature, by whatever name called.

���� Income includes royalty - The gross total income of the taxpayer includes the following—

a. royalty or copyright fees (payable in lump sum or o then vise) in respect of afore-

said book (it also advance payment which is not returnable); and

b. lump sum consideration for transfer (or grant) of any interest in the copyright of

the book.

���� Furnishing of Form No. 10CCD - The taxpayer shall have to obtain a certificate in Form

No. 10CCD person responsible for paying the income and furnish it along with the

return*.

���� Return of income - Deduction under section 80QQB is not available unless it is claimed

in the return of income.

Amount of deduction - If the aforesaid conditions are satisfied, then the amount of de-

duction

a. Rs. 3,00,000; or

b. income from royalty as stated above,

whichever is lower.

Other points - One should also keep in view the following points—

���� Remittance from abroad - Where the eligible income is earned outside India, the de-

duction shall be allowed so much of the income earned in foreign exchange, which is

brought in India within 6 months from the end of previous year (or within extended

period as permitted by RBI or competent authority). Moreover, dedcution not be al-

lowed unless the taxpayer furnishes* a certificate in Form No. 1011.

���� Rate of royalty not to exceed 15 per cent - Where the income by way of royalty (or the

copyright fee), is not lump sum consideration (in lieu of all rights of the assessee in the

book) so much of the income (before allow expenses attributable to such income) as is

in excess of 15 per cent of the value of such books sold during: previous year, shall be

ignored.

���� double deduction not available - Whore a deduction under section 80QQB, for any

previous year has been claimed allowed, no deduction in respect of such income shall

be allowed under any other provision of the Act in any assessment year.

Page 249: INTER CA Taxation - CA Study Web€¦ · INTER CA Taxation Head Office Shraddha, 4th Floor, ... 14. Advance Tax & Interest 268 - 271 15. Tax Deducted at Source (TDS) 272 - 294 16

J.K. SHAH CLASSES INTER C.A.- DIRECT TAXES

: 246 :

DEDUCTION IN RESPECT OF ROYALTY ON PATENTS - TO WHAT EXTENT AVAIL-

ABLE [SEC. 80RRB]

The provisions of section 80RRB are given below —

Conditions - The following conditions should be satisfied in order to claim deduction under

section 80RRB--

���� Individual - The taxpayer is an individual (may be an Indian citizen or foreign citizen).

���� Resident in India -He is resident in India (he may be ordinarily resident or not ordinar-

ily resident but deduction under section 80RRB is not available if he is non-resident).

���� Owner/co-owner of patent - He is a patentee (he maybe a co-owner of patent). Pat-

entee means the person (being the true and first inventor of the invention), whose name

is entered on the patent register as the patentee, in accordance with the Patents Act, 1970.

���� Royalty from patent -He is in receipt of any income by way of royalty in respect

of patent, which is registered - under the Patent Act after March 31,2003. It in-

cludes advance royalty which is not returnable. However, it does not include any

consideration for sale of product manufactured with the use of patented process

or of the patented article per se for commercial use. Further, any consideration,

which is chargeable under the head

“Capital gains” is not royalty.

���� Furnish Form No. 10CCE - The assessee shall have to furnish a certificate in

Form No. 10CCE, duly signed by the prescribed authority [i.e., die controller un-

der section 1(b) of the Patents Act] along with the return of income.

���� Return of income - Deduction under section 80RRB is not available unless it is

claimed in the return of income.

Amount of deduction - If the aforesaid conditions are satisfied,then the amount of de-

duction is —

(a) Rs. 3,00,000; or

(b) income from "royalty" as stated above,

whichever is lower.

Other points - One should also keep in view the following points—

���� Royalty from foreign sources - Where any income is earned from sources outside

India on which the deduction r der this section is claimed, only so much of the

income shall be considered, as is brought into India by, (or on behalf of) the asses-

see in convertible foreign exchange within a period of 6 months from the end of the

previous i (or within such further period as RBI may allow in this behalf).

Where any income is earned from sources outside India, a certificate* certifying

that the deduction has bed correctly claimed in accordance with the provision of

this section (in Form No. 10H), is required.

Page 250: INTER CA Taxation - CA Study Web€¦ · INTER CA Taxation Head Office Shraddha, 4th Floor, ... 14. Advance Tax & Interest 268 - 271 15. Tax Deducted at Source (TDS) 272 - 294 16

J.K. SHAH CLASSES INTER C.A.- DIRECT TAXES

: 247 :

���� Subsequent revocation of patent - In case the patent is subsequently revoked by

the Controller or the High Court or the name of the assessee is subsequently

excluded from the patents register as patentee in respect of that patent, the de-

duction relatable to royalty income in respect of the period for which the paten-

tee's claim wasm1 valid, shall be withdrawn. For this purpose, the assessment

may be rectified within a period of 4 years from ‘is* end of the previous year in

which such order is passed by High Court or Controller.

���� Double deduction not possible - Where a deduction for any previous year has been

claimed and allowed undo section 80RRB in respect of any income referred to

above, no deduction in respect of such income shall be allowed, under any other

provision of lire Act in any assessment year)

SECTION : 80GG - Deduction in respect of rent paid.

ALLOWED TO : Individual

DEDUCTION : Least of : 1. Rent Paid (-)10% of A.T.I

2. 25% of A.T.I.

3. 5,000 p.m.

CONDITIONS : 1. Assessee should not have claimed exemption u/s 10(13A) or

receiving any H.R.A.

2. Assessee or his spouse or minor child or his H.U.F. should not

own any residential accommodation at the place of work where

he ordinarily resides or performs duties.

3. Assessee should not claim exemption u/s 23(2)(a) or 23(4)(a) in

respect of any other residential accommodation at any other place.

REMARKS : A.T. I. =Adjusted Total Income which is G.T.I. (-) All other deductions u/s 80

except 80GG, long term capital gains and short - term capital gains

u/s 111A.

Page 251: INTER CA Taxation - CA Study Web€¦ · INTER CA Taxation Head Office Shraddha, 4th Floor, ... 14. Advance Tax & Interest 268 - 271 15. Tax Deducted at Source (TDS) 272 - 294 16

J.K. SHAH CLASSES INTER C.A.- DIRECT TAXES

: 248 :

SECTION 80 G --DEDUCTION FOR DONATIONSThis deduction is applicable to all types of assessees.

1 ) Donations given in Kind/by goods e.g.Donations byornaments,books,utensils,foodgrains,cloth,building,animals etc

2 ) Donations given to Individuals ,families & unregistered organisations3 ) Donations sent to Foreign organisations/Foreign govt (except 18 & 19)

4 ) Any donation promised but not supported by cash payment is not eligible for deduction.(Receipt is must)

5) No deduction shall be allowed under this section for any donation exceeding `̀̀̀ 2,000 unless such sum

Eligible Donations (cash donations matching with the list)Donations actually paid by cash /cheque by assessee to the following funds , institutions etc will be eligiblefor deduction :

OTHER POINTS

1 Donations listed under 1to 24 are unlimited category donations because any amount can be donated.2 Donations listed under 25to31 are limited category donations because total amount should not exceed

10% of Adjusted Gross Total Income (AGTI).3. AGTI means

Gross Total Income XXXLess : Long term capital Gain -XXXLess : Deductions U/s 80C to 80U - XXX

AGTI XXX

4. Donations listed under1to 4 & 27 to 31 are eligible for 50 % deduction.

5 . Donations listed under 5 to 24 & 25 -26are eligible for 100% deduction.

1) Jawaharlal Nehru Memorial Fund

2) Indira Gandhi Memorial Trust3) Rajiv Gandhi Foundation

4) Prime Minister’s Drought Relief Fund

5. National Children’s Fund6 . National Defence Fund set up by the Central Gov-

ernment

7 . Prime Minister’s National Relief Fund8 . National Blood Transfusion Council and State Coun-

cil for Blood Transfusion.

9 . National Trust for welfare of persons withAutism,Cerebralplasi,MentalRetardation,Multiple Disabilites

10 . National Sports Fund / National Cultural Fund

1 1 . National Illness Assistance Fund1 2 . National Foundation for Communal Harmony1 3 . Cheif Minister’s Earthquake Relief Fund/Gujarat

Chief Minister’s Earthquake Relief Fund1 4 . Andhra. Pradesh Chief Minister’s Cyclone Relief

Fund15 . State Fund for the medical relief of the POOR1 6 . Chief Minister’s Relief Fund or Lieutenant

Governor’s Relief Fund1 7 . Zila Saksharta Samiti1 8 . Prime Minister’s Armenia Earthquake Relief Fund1 9 . Africa Fund (Public Contributions of India)20. An Approved university/educational institute of

National Popularity/Eminence2 1 . Army Central Welfare Fund// the Indian Naval/

Navy BenevolentFund/Airforce Central WelfareFund /Kargil Fund

2 2 . Swachh Bharat Kosh2 3 . Clean Ganga Fund (amount donated by resident

only)2 4 . National fund for control of drug abuse

25.Donations for family planning given to

� Central Government

�State Government

� Local Government/Municiplity

�Approved Organisation/Association /Hospital.

26.Donations to Indian Olympic Association/or anynotified association for sports infrastructure

development (only companies are eligible)

27 . Donation to an authority established for TOWNPLANNNG /housing development/( e.g MHADA)

28. Donation , for repair & renovation ,given to noti-fied TEMPLE, mosque, gurdwara, church, his-toric place,archeological place

29. Donation given for charitable purposes to:

� APPROVED SCHOOLS,colleges,educational insti-

tutions (not nationally eminent)

� Universities approved by Income tax authorities but

not by UGC (not covered by 20 above)

� Approved trusts,organisations,associations

� Approved Hostel,old age homes,library,hospitals etc

� Approved Funds & Foundations

30. Donation given for charitable purposes to

� Central Government

� State Government

� Local Government/Municipality

31.

MINORITY Community

Page 252: INTER CA Taxation - CA Study Web€¦ · INTER CA Taxation Head Office Shraddha, 4th Floor, ... 14. Advance Tax & Interest 268 - 271 15. Tax Deducted at Source (TDS) 272 - 294 16

J.K. SHAH CLASSES INTER C.A.- DIRECT TAXES

: 249 :

Q. 1. Mr. X has a G.T.I. of `̀̀̀ 4,50,000, which includes LTCG under section 112 `̀̀̀ 3,00,000

and he makes the following payments during previous year 2017-18 :

Mediclaim insurance premium paid for himself and wife, by cheque `̀̀̀ 7,000

Donation to approved charitable institution `̀̀̀ 16,000

Rent paid per month and fulfils the condition of section 80GG `̀̀̀ 3,000

Calculate the deduction u/s 80G & 80GG.

Q. 2. Determine the amount deductible under section 80QQB in the following cases

pertaining to the assessment year 2018-19.

Q. 3. Mr Pradeep Bhushan donated as follows through account payee cheque in 2017-18 :

Prime Minister’s National Relief Fund 82,000

Africa (Public Contributions - India) Fund 22,000

University of Newyork, approveed by IETS, USA 30,000

Andhra Pradesh Chief Minister’s Cyclone Relief Fund (paid in cash) 54,000

Balaji Trust of Tirupati for renovation 1,03,000

BMC to be utilised for the purpose of promoting family planning 15,000

Indian Olympic Association 6,000

National Blood Transfusion Council and State Council for Blood Transfusion. 1,15,000

National Defence Fund set up by the Central Government 80,000

Prime Minister's Drought Relief Fund 13,000

Zila Saksharta Samiti (paid through debit card) 24,000

Lieutenant Governor’s Relief Fund Pondicherry 11,000

Swach Bharat Kosh 10,000

Jawaharlal Nehru Memorial Fund 27,000

Indian National Congress 2,000

National Fund for Rural Development (paid in cash) 7,000

CLASS WORK PROBLEMS

Page 253: INTER CA Taxation - CA Study Web€¦ · INTER CA Taxation Head Office Shraddha, 4th Floor, ... 14. Advance Tax & Interest 268 - 271 15. Tax Deducted at Source (TDS) 272 - 294 16

J.K. SHAH CLASSES INTER C.A.- DIRECT TAXES

: 250 :

Other Information :

1. He has salary income of `̀̀̀ 6,00,000 and long term capital gain on sale of a

commercial property `̀̀̀ 11,00,000/-.

2. He has invested in fixed deposits of various banks on which interest accrued during

the year amounts to `̀̀̀ 30,000/-. Interest on saving bank account `̀̀̀ 6,500/-

3. He has deposited `̀̀̀ 60,000 in Public Provident Fund and has incurred an

expenditure of `̀̀̀ 15,000 on disabled brother (fully blind) dependant on him.

4. Interest on education loan for son’s MBA course `̀̀̀ 80,000/-. He has paid

medical insurance premium of `̀̀̀ 27,000 in cheque for himself and his wife.

Compute total income and tax payable.

Q. 4. Mr. Manjesh, resident senior citizen, aged 62 years, gives the following inforomation

regarding his income in P.Y. 2017-18. Calculate income tax payable.

(1) He sold a commercial property on 16.08.2017 for `̀̀̀ 25,00,000/-. This

property was acquired by him on 27.04.2001 for `̀̀̀ 7,00,000. Brokerage paid

on sale was 1.5% of sale consideration.

(2) He also sold an urban land for `̀̀̀ 16,00,000 on 09.09.2017. This land was

purchased 3 months ago for `̀̀̀ 12,50,000/- Brokerage on sale 1%.

(3) He has paid medical insurance premium for himself `̀̀̀ 7,000, for wife `̀̀̀ 5,000 by

account payee cheque and for his elder son `̀̀̀ 6,000. His elder son is employed

in a company at a salary of `̀̀̀ 6,00,000 p.a. He has also paid `̀̀̀ 6,000 in cash on

account of preventive health check-up of himself and wife.

(4) Deposit under Senior Citizen saving scheme, 2004, `̀̀̀ 20,000/-. Deposit with

SBI for 6 years `̀̀̀ 50,000/-.

Page 254: INTER CA Taxation - CA Study Web€¦ · INTER CA Taxation Head Office Shraddha, 4th Floor, ... 14. Advance Tax & Interest 268 - 271 15. Tax Deducted at Source (TDS) 272 - 294 16

J.K. SHAH CLASSES INTER C.A.- DIRECT TAXES

: 251 :

Ans. 3 Mr. Pradeep Bhushan

Computation of total income and tax liability for A.Y. 2018-2019

Particulars ` `` `` `` `

Income from Salary 6,00,000

Capital Gains

L.T.C.G. on sale of commercial property, taxable u/s. 112 @ 20% 11,00,000

Income from Other Sources

Interest on Bank F.D. 30,000

Interest on savings bank account 6,500 36,500

GROSS TOTAL INCOME (G.T.I.) 17,36,500

Less:Deductions u/s. 80C to 80U

u/s. 80C: Contribution to PPF 60,000

u/s. 80D: Mediclaim premium paid (27,000 or max 25,000) 25,000

u/s. 80DD: Maintenance of dependant (note) 1,25,000

u/s. 80E: Interest on loan for higher education 80,000

u/s. 80GGA: Contribution to National Fund for Rural Development 7,000

u/s. 80GGC: Donation to political party 2,000

u/s. 80G: Donation to specified funds/ organisaitions (W.N. 1) 3,88,050

u/s. 80TTA: Interest on savings bank accounts 6,500

Total Deductions u/s. 80C to 80U (I) 6,93,550

GTI available (excluding L.T.C.G.) (II) 6,36,500

Deductions allowable (I or II, whichever is less) (6,36,500)

NET TAXABLE TOTAL INCOME (N.T.T.I.) 11,00,000

TAX LIABILITY

Basic tax (W.N. 2) 1,70,000

Add: Education cess @ 3% 5,100

TAX PAYABLE 1,75,100

CLASSWORK SOLUTION

Page 255: INTER CA Taxation - CA Study Web€¦ · INTER CA Taxation Head Office Shraddha, 4th Floor, ... 14. Advance Tax & Interest 268 - 271 15. Tax Deducted at Source (TDS) 272 - 294 16

J.K. SHAH CLASSES INTER C.A.- DIRECT TAXES

: 252 :

W.N. 1 Deduction u/s. 80G

Category Donations made Donations eligible % Deduction

Unlimited -100% 3,44,000 3,44,000 100 3,44,000

Unlimited -50% 40,000 40,000 50 20,000

Limited -100% 15,000 15,000 100 15,000

Limited -50% 1,03,000 18,100 50 9,050

3,88,050

A.T.I. = 17,36,500 – 11,00,000 - 60,000 - 25,000 - 1,25,000 - 80,000 -7,000- 2,000

- 6,500 = 3,31,000

Total donations made to Limited category (15,000 + 1,03,000) 1,18,000

OR

10% of A.T.I. of `̀̀̀ 3,31,000 33,100

Whichever is less

Eligible amount = `̀̀̀ 33,100

100% Category 50% Category

`̀̀̀ 15,000 `̀̀̀ 18,100 (Balancing figure)

W.N. 2: Basic tax

N.T.T.I. (consisting of LTCG only) `̀̀̀11,00,000

Less: basic exemption limit, remaining fully unabsorbed ( `̀̀̀ 2,50,000)

Taxable L.T.C.G. `̀̀̀ 8,50,000

x Tax rate u/s. 112 20%

Basic tax payable `̀̀̀ 1,70,000

Note: It has been assumed that the disabled brother does not claim deduction

u/s. 80U and therefore Mr. Pradeep Bhushan is entitled to deduction u/s. 80DD.

Page 256: INTER CA Taxation - CA Study Web€¦ · INTER CA Taxation Head Office Shraddha, 4th Floor, ... 14. Advance Tax & Interest 268 - 271 15. Tax Deducted at Source (TDS) 272 - 294 16

J.K. SHAH CLASSES INTER C.A.- DIRECT TAXES

: 253 :

Ans.4. Mr. Manjesh

Computation of Total Income and tax liability for A.Y. 2018-2019

Particulars ` `` `` `` `

Capital Gains, which becomes G.T.I.

- LTCG (W.N. 1)

- STCG (W.N. 2) 3,34,000

Less: Deductions u/s. 80C to 80U

u/s. 80C: Deposit with:-

(a) Senior Citizen Savings Scheme, 2004 20,000

(b) SBI for 6 years (within the overall limit of Sec 80C of ̀̀̀̀ 1,50,000) 50,000 (70,000)

u/s. 80D: Mediclaim insurance premium paid for :-

(a) Self 7,000

(b) Spouse 5,000

(c) Elder Son N.A.

(d) Preventive health check up [Maximum] 5,000 (17,000)

(within the overall limit of Sec 80D of `̀̀̀ 30,000)

NET TAXABLE TOTAL INCOME (N.T.T.I.)

TAX LIABILITY

Basic Tax (W.N. 3)

Add: Education cess @ 3%

TAX PAYABLE

W.N.1 L.T.C.G

Particulars `̀̀̀

Full value of consideration 25,00,000

Less: Transfer expenses: Brokerage= 1.5% x `̀̀̀ 25,00,000 (37,500)

Less: Indexed Cost of Acquisition

(LTCA as h.p. > 36 months from 27.4.2001 to 16.8.2017)

(7,00,000 )

LTCG, taxable u/s. 112 @ 20%

W.N.2 S.T.C.G

Full value of consideration 16,00,000

Less: Transfer expenses: Brokerage= 1% x `̀̀̀ 16,00,000 (16,000)

Less: Cost of Acquisition(STCA as h.p. < 36 months from

10.6.2017 to 9.9.2017 i.e. 3 months) (12,50,000)

Page 257: INTER CA Taxation - CA Study Web€¦ · INTER CA Taxation Head Office Shraddha, 4th Floor, ... 14. Advance Tax & Interest 268 - 271 15. Tax Deducted at Source (TDS) 272 - 294 16

J.K. SHAH CLASSES INTER C.A.- DIRECT TAXES

: 254 :

W.N.3: Basic tax

N.T.T.I. 8,60,908

Less: L.T.C.G. (6,13,908)

Normal Income 2,47,000

Less: Basic Exemption Limit applicable (3,00,000)

UNABSORBED Exemption Limit 53,000

L.T.C.G 6,13,908

Less: Unabsorbed Exemption Limit (53,000)

Taxable L.T.C.G. 5,60,908

X Tax rate u/s. 112 20%

Basic tax payable 1,12,182

Page 258: INTER CA Taxation - CA Study Web€¦ · INTER CA Taxation Head Office Shraddha, 4th Floor, ... 14. Advance Tax & Interest 268 - 271 15. Tax Deducted at Source (TDS) 272 - 294 16

: 255 :

J. K. SHAH CLASSES INTER C.A. - DIRECT TAXES

ASSESSMENT PROCEDURE

ASSESSMENT PROCEDURE

SECTION - WISE INTRODUCTION AND BRIEF PARTICULARS OF

ASSESSMENT PROCEDURE

Types of Assessments :

Note 1 – Summary Assessment

Assessment is completed on the basis of return submitted by the assessee. Assessing

Officer can make the following adjustments to the total Income declared in the return of

Income:

1. Any arithmetical error

2. Any incorrect claim

An Intimation under section 143(1) should not be sent after expiry of 1 year from the end

of Financial year in which return is filed

Note 2 – Regular assessment

Assessing officer will issue a notice to assessee under section 143(2) to take the case in

scrutiny. Such notice shall be served on the assessee within 6 months from the end of

financial year in which return is filed. After hearing such evidence as the assessee may

produce and after taking into account all relevant material which assessing officer has

gathered, he shall pass an order under section 143(3) determining the sum payable or

refundable to the assessee.

Note 3 – Best Judgement assessment

Assessing officer after considering all materials which he has gathered is under an obli-

gation to make assessment of total income or loss to the best of his judgement in some

cases

1. If person fails to respond to a notice under section 142(1) (for Filing ROI)

2. If Person fails to respond to a scrutiny notice under section 143(2).

SECTION : 139(1) - Due dates for filing voluntary returns

PARTICULARS :

(a) Every company or a firm shall furnish on or before the due date the return in respect of

its income or loss in every previous year.

(b) Every person, being an individual or a HUF or an A.O.P. or a B.O.I. or an artificial

juridical person, if his total income or the total income of any other person in respect of

which he is assessable under this Act during the previous year, without giving effect to

the provisions of Section 10(38) or Section 10A or Section 10B or Section 10BA or

Chapter VI-A, exceeded the maximum amount which is not chargeable to income tax

shall, on or before the due date, furnish a return of his income or the income of such

other person during the previous year, in the prescribed form and verified in the pre-

scribed manner and setting forth such other particulars as may be prescribed.

Page 259: INTER CA Taxation - CA Study Web€¦ · INTER CA Taxation Head Office Shraddha, 4th Floor, ... 14. Advance Tax & Interest 268 - 271 15. Tax Deducted at Source (TDS) 272 - 294 16

: 256 :

J. K. SHAH CLASSES INTER C.A. - DIRECT TAXES

ASSESSMENT PROCEDURE

Provided that a person, being resident and ordinarily resident, who is not required

to furnish a return under this sub-section (i.e. GTI ���� Basic Exemption Limit) and

who during the previous year has any asset (including any financial interest in any

entity) located outside India or any signing authority in any account located outside

India; such a person shall furnish on or before the due date, a return in respect of

his income or loss for the previous year in such form and verified in such manner

and setting forth such other particulars as may be prescribed.

Explanation : In this sub-section, “Due date” means :

(a) Where the assessee has entered into an international transaction

dur ing the previous year, the 30th day of November of the

assessment year.

(b) where the assessee [ not covered by point (a) above] is -

(i) a company; or

(ii) a person (other than a company) whose accounts are required to be au-

dited under this Act or under any other law for the time being in force ; or

(iii) a working partner of a firm whose accounts are required to be audited

under this Act or under any other law for the time being in force,

the 30th day of September of the assessment year ;

(c) in the case of any other assessee, the 31st day of July of the assessment year.

SECTION : 139(3) - Return of Loss

PARTICULARS :

If any person who has sustained a loss in any previous year under the head “Profits and

Gains of business or profession” (including losses of specified businesses) or under the

head “Capital Gains” or under "Activity of owning and maintaining Race Horses" and

claims that the loss or any part thereof should be c/f in the relevant sections, he has to

submit his return as per Sec. 139(1) i.e. on or before the due dates or else the loss might

not be allowed to be carried forward.

Section 139(3) shall not apply to loss under the head "Income from House Property" and

Unabsorbed Depreciation

SECTION : 139(4) - Belated return i.e. return furnished after the due date

PARTICULARS :

Any person who has not furnished a return within the time allowed to him under Sec.

139(1) or within the time allowed under a notice u/s. 142(1)(i) may furnish the return

for any previous year at any time before the end of the relevant assessment year

OR before completion of the assessment, whichever is earlier. However he will

be liable to interest u/s. 234A.

Page 260: INTER CA Taxation - CA Study Web€¦ · INTER CA Taxation Head Office Shraddha, 4th Floor, ... 14. Advance Tax & Interest 268 - 271 15. Tax Deducted at Source (TDS) 272 - 294 16

: 257 :

J. K. SHAH CLASSES INTER C.A. - DIRECT TAXES

ASSESSMENT PROCEDURE

SECTION : 139(4A) - Return of income of charitable trusts and institutions

PARTICULARS :

Every person in receipt of income derived from property held under trust or other

legal obligation wholly for charitable or religious purposes or in part only for such

purposes, or of income being voluntary contributions, shall, if the total income in re-

spect of which he is assessable as a representative assessee (the total income for

this purpose being computed under this act without giving effect to the provisions

of section 11 and 12) exceeds the minimum exemption limit , furnish return of such

Income of the previous year, as per section 139(1).

SECTION : 139(4B) - Return of income of political parties

PARTICULARS :

The chief executive officer of every political party shall, if the total income in respect

of which the political party is assessable (total income for this purpose being com-

puted under this Act without giving effect to section 13A) exceeds the minimum ex-

emption limit, furnish a return of such income of the previous year as per section

139(1).

SECTION : 139(4C) -Return by certain assessees whose income is exempt u/s.10

PARTICULARS :

Institution covered u/s. 10 i.e. Scientific research association u/s10(21), news agency

u/s 10(22B), professional institutes u/s 10(23A), Khadi institution u/s.10(23B), any

university, educational institution etc. u/s.10(23C), trade union u/s10(24), any body or

authority or Board or Trust or Commission u/s 10(46), infrastructure debt fund u/s

10(47) whose total income, without giving effect of sec.10, exceeds the minimum

exemption limit, must file a return u/s 139(1).

SECTION : 139(5) -Revised return i.e. submitting a fresh return subsequent to

discovery of any error in the original return

PARTICULARS :

If any person, having furnished a return u/s.139(1) or u/s 139(4) or in pursuance of a

notice issued u/s. 142(1)(i), discovers any omission or any wrong statement therein,

he may furnish a revised return at any time before the expiry of one year from the

end of the relevant assessment year OR before completion of the assessment,

whichever is earlier.

SECTION : 139(9) - Defective return

PARTICULARS :

When a return could be considered as defective :-

a) the annexures, statements and columns in the return of income relating to computation

of income chargeable under each head of income are not duly filled in, or

b) computation of tax is not accompanying the return, or

c) where tax audit is compulsory, no tax audit report u/s. 44AB is submitted along

with the return, or

Page 261: INTER CA Taxation - CA Study Web€¦ · INTER CA Taxation Head Office Shraddha, 4th Floor, ... 14. Advance Tax & Interest 268 - 271 15. Tax Deducted at Source (TDS) 272 - 294 16

: 258 :

J. K. SHAH CLASSES INTER C.A. - DIRECT TAXES

ASSESSMENT PROCEDURE

d) the return is not accompanied by proof of T. D. S. or T. C.S. (Certificate), advance tax

payment (Challans) and tax on self-assessment (Challans) claimed to have been paid.

Provided that where the return is not accompanied by proof of the tax, if any,

claimed to have been deducted or collected at source, the return shall not be

regarded as defective if :

a) A certificate for tax deducted or collected was not furnished u/s 203 or u/s

206C to the person furnishing the return of income.

b) Such certificate is produced within a period of two years from the end of

the relevant assessment year, or

e) where regular books of account are maintained by the assessee, the return is not

accompanied by copies of Manufacturing account, Trading account, Profit and Loss

account as the case may be, Income and Expenditure account or any other similar

account and Balance Sheet alongwith the personal account of the proprietor, (in

case of proprietary business) the personal accounts of partners / members (in

case of firm / AOP / BOI) respectively, or

f) where the accounts of the assessee have been audited, the return is not

accompanied by copies of audited P & L & Balance Sheet and the auditor’s re-

port, or

g) where the regular books of accounts are not maintained by the assessee, the re-

turn is not accompanied by a statement indicating the amounts of turnover or, as

the case may be, gross receipts, gross profit, expenses and net profit of the busi-

ness or profession and the basis on which such amounts of total sundry debtors,

sundry creditors, stock in trade and cash balance at the end of the previous year are computed.

Where the A.O. considers that the return of income furnished by the assessee is

defective, he may intimate the defect to the assessee and give him an opportunity

to rectify the defect within a period of 15 days from the date of service of such

intimation, or within such extended time as he may think proper.

If the defect is not rectified within the above period, the return would be treated as

invalid and it would be assumed that the assessee had failed to furnish the return.

Provided that where the assessee rectifies the defect after the expiry of the said

period of 15 days or the further period allowed, but before the assessment is made,

the A.O. may condone the delay and treat the return as a valid return.

SECTION : 139A - Permanent Account Number [ P.A.N.]

PARTICULARS :

Every person, if his total income or if he is assessable in respect of some other person’s income, if

that other person’s income exceeds the tax free limit must apply to A.O. for the allotment of P.A.N.

within the prescribed time (i.e. upto 31st May of the assessment year).

In case the person is carrying on a business whose total sales, turnover or gross receipts

are likely to exceed `̀̀̀ 5,00,000 in an accounting year or who is required to furnish a return

u/s. 139(4A) (if the person already has a P.A.N, need not apply again) and has not been

allotted P.A.N, he must apply for it within the prescribed time i.e. before the end of the

previous year as per Rule 114.

Page 262: INTER CA Taxation - CA Study Web€¦ · INTER CA Taxation Head Office Shraddha, 4th Floor, ... 14. Advance Tax & Interest 268 - 271 15. Tax Deducted at Source (TDS) 272 - 294 16

: 259 :

J. K. SHAH CLASSES INTER C.A. - DIRECT TAXES

ASSESSMENT PROCEDURE

The Central Government may, by notification in the Official Gazette, specify any class

or classes of persons by whom tax is payable under this Act or any tax or duty is

payable under any other law for the time being in force (including importers and

exporters whether any tax is payable by them or not) and such persons shall, within

such time as mentioned in that notification, apply to the Assessing Officer for the

allotment of a P.A.N.

The Central Government may, for the purpose of collecting any information which may be

useful or relevant to the purposes of this Act, by notification in the Official Gazette, specify,

any class or classes of persons who shall apply to the Assessing Officer for the allotment

of the P.A.N. and such persons shall, within such time as mentioned in that notification,

apply to the Assessing Officer for the same.

The Assessing Officer having regard to the nature of the transactions as may be

prescribed, may also allot a P.A.N, to any other person (whether any tax is payable

by him or not) in accordance with the prescribed procedure .

The following transactions have been prescribed :

(a) Purchase and Sale of Immovable Property valued at `̀̀̀ 5 lacs or more

(b) Purchase and sale of Motor Vehicles (other than two wheeled vehicles)

(c) Transactions in securities exceeding `̀̀̀ 1,00,000

(d) Opening a new bank account (in case of minor, PAN of parent / guardian is required).

(e) Fixed Deposit of more than 50,000

(f) Application of allotment of telephone connection (including cellular phone)

(g) Payment to hotels exceeding `̀̀̀ 25,000/- at any one time.

(h) Deposit exceeding `̀̀̀ 50,000 in any account with Post Office Savings Bank.

(i) Cash payment for purchase of bank drafts or pay orders or bankers cheque from a

banking company for an amount aggregating `̀̀̀ 50,000 or more during any one day.

(j) Cash deposits aggregating `̀̀̀ 50,000 or more with a banking company during any

one day.

(k) Cash payment exceeding `̀̀̀ 25,000 in connection with travel to any foreign

country at any one time.

(l) Making an application to any bank or banking institution or company or any

institution for issue of a credit card.

(m) Making an application to any mutual fund for purchase of its units for an amount

of `̀̀̀ 50,000 or more.

(n) Making an application to a company for acquiring shares for an amount of

`̀̀̀ 50,000 or more.

(o) Making an application to a company or institution for acquiring debentures for

an amount of `̀̀̀ 50,000 or more.

(p) Making an application to Reserve Bank of India for acquiring its bonds for an

amount of `̀̀̀ 50,000 or more.

Page 263: INTER CA Taxation - CA Study Web€¦ · INTER CA Taxation Head Office Shraddha, 4th Floor, ... 14. Advance Tax & Interest 268 - 271 15. Tax Deducted at Source (TDS) 272 - 294 16

: 260 :

J. K. SHAH CLASSES INTER C.A. - DIRECT TAXES

ASSESSMENT PROCEDURE

Every buyer or licensee or lessee referred to section 206C shall intimate his P.A.N.

to the person responsible for collecting tax referred to in that section.

The assessee should quote G.I.R. (General Index Register) number till such time

P. A.N. is not allotted to such assessee.

Once a P.A.N. i s a l lo t ted , such number must be quoted in a l l returns,

correspondence with Income-tax authorities, challans for payment and in all

documents prescribed by the Board.

He must intimate the A. O. about any change in the address, name or nature of busi-

ness carried on by him. W.E.F 1.6.2001 it is compulsory for every person

receiving any sum, income, or amount from which T.D.S has been deducted, to

intimate his P. A. Number to the person responsible for deducting T.D.S. The person

who has deducted T.D.S. / T.C.S., shall also quote the above P. A. N. in all

statements, certificates, statements & returns filed by him.

If a person without reasonable cause, fails to apply for the allotment of P.A.N. within

the prescribed time or fails to quote it on the relevant document, the A.O. shall

impose a penalty of a sum which shall be minimum of `̀̀̀ 500 and maximum of `̀̀̀ 10,000

u/s. 272 A.

SECTION : 140 - Return under section 139, by whom to be signed.

Persons authorized to verify the Return of Income [Section 140]

Assessee Circumstance Authorised Persons

1. Individual (i) In circumstances not covered the individual himself

under (ii), (iii) & (iv) below

(ii) where he is absent from India - the individual himself or

- any person duly authorised

by him in this behalf holding a

valid power of attornery from the

individual (Such power of attorney

should be attached to the return

of income)

(iii) where he is mentally - his guardian; or

incapacitated from attending to

- any other person competent to

his affairs.

act on his behalf

(iv) where, for any other reason, it - any person duly authorised by

is not possible for the individual to him in this behalf holding a valid

verify the return power of attorney from the

individual (Such power of attorney

should be attached to the return

of income)

Page 264: INTER CA Taxation - CA Study Web€¦ · INTER CA Taxation Head Office Shraddha, 4th Floor, ... 14. Advance Tax & Interest 268 - 271 15. Tax Deducted at Source (TDS) 272 - 294 16

: 261 :

J. K. SHAH CLASSES INTER C.A. - DIRECT TAXES

ASSESSMENT PROCEDURE

2. Hindu (i) in circumstances not covered - the karta

Undivided under (ii) and (iii) below

Family (ii) where the karta is absent from - any other adult member of the

India HUF

(iii) where the karta is mentally - any other adult member of the

incapacitated from attending to HUF

his affairs

3. Company (i) in circumstances not covered - the managing directors of the

under (ii) to (v) below company

(ii) (a) where for any unavoidable - any director of the company

reason such managing directors

is not able to verify the return; or

(b)where there is no managing - any director of the company

director

(iii) where the company is not - a person who holds a valid

resident in India power of attorney from such

company to do so (such power of

attorney should be attached to the

return.)

(iv) (a) Where the company is - Liquidator

being wound up (whether under the

orders of a court or otherwise); or

(b) Where any person has been - Liquidator

appointed as the receiver of any

assets of the company.

(v) Where the management of the - the principal officer of the

company has been taken over by company

the Central Government or any

State Government under any law

4. Firm (i) in circumstances not covered - the managing partner of the firm

under (ii) below

(ii) (a) where for any unavoidable - any partner of the firm, not

reason such managing partner is being a minor

not able to verify the return or

(b) where there is no managing - any partner of the firm, not

partner. being a minor

5. Local authority ---- - the principal officer

6. Political party ---- - the chief executive officer of

[referred to in - such party (whether he is known

section 139 as secretary or by any other

(4B] designation)

7. Any other ---- - any member of the association

association or the principal officer of such

association

8. Any other ---- - that person or some other

person person competent to act on his

behalf.

Page 265: INTER CA Taxation - CA Study Web€¦ · INTER CA Taxation Head Office Shraddha, 4th Floor, ... 14. Advance Tax & Interest 268 - 271 15. Tax Deducted at Source (TDS) 272 - 294 16

: 262 :

J. K. SHAH CLASSES INTER C.A. - DIRECT TAXES

ASSESSMENT PROCEDURE

SECTION : 140 A - Self Assessment

PARTICULARS :

Any amount of tax which is payable on the basis of return u/s 139 or in response to

notice u/s 142(1)(i) [including any interest payable by the assessee] after deducting

the amount of any tax already paid or deducted or collected at source, any relief of tax

claimed u/s 90 or 91 on account of tax paid in a country outside India, any relief of tax

claimed u/s 90A on account of tax paid in any specified territory outside India and any

tax credit claimed to be set - off as per section 115JD, must be paid before the filing

of the return, and proof of such payment must be attached with the return.

Where any amount paid u/s. 140A falls short of tax and interest actually payable by the

assessee, the amount paid u/s. 140A shall first be adjusted towards interest and balance

shall be adjusted towards tax. If the assessee does not pay any part of tax and interest as

provided above, he will be deemed to be an assessee in default.

Page 266: INTER CA Taxation - CA Study Web€¦ · INTER CA Taxation Head Office Shraddha, 4th Floor, ... 14. Advance Tax & Interest 268 - 271 15. Tax Deducted at Source (TDS) 272 - 294 16

: 263 :

J. K. SHAH CLASSES INTER C.A. - DIRECT TAXES

ASSESSMENT PROCEDURE

PRACTICAL PROBLEMS ON RETURN OF INCOME & ASSESSMENT PROCEDURE

Q. 1. Following information is given regarding certain persons.

`̀̀̀ `̀̀̀

`̀̀̀

`̀̀̀

`̀̀̀

`̀̀̀

`̀̀̀

Discuss whether the above persons are required to submit return of income for the

assessment year 2018-19 (if yes, indicate due date for furnishing the return).

Q. 2. Discuss the correctness or otherwise of the following :

1. Mr. Pinakin having business loss of `̀̀̀ 1,20,000 (after providing for depreciation of

`̀̀̀ 80,000) furnishes his return of income on 5.8.2018 (turnover for 2017-18 `̀̀̀ 100

lacs). He wants to carry forward `̀̀̀ 1,20,000.

2. Mr. Anil has furnished his return of income for the assessment year 2018-19 on

31st March, 2019. The assessment was completed on the basis of such return

on 10th May, 2020.

3. Mr. Quadro has furnished his return of income for the assessment year

2018-19 on 11th April, 2020. The assessment was completed on the basis of

such return on 8th May, 2020.

4. Mr. Siddhu didn't f ile any return of income for the assessment year

2018-19 and his assessment for that year was completed on 31st

December, 2018. He furnishes return of income u/s 139(4) on 18.1.2019.

5. Mr. Chinumama having loss from other sources of `̀̀̀ 10,000 furnishes his return on

30.7.2018. He wants to carry forward such loss for set off in subsequent years.

Page 267: INTER CA Taxation - CA Study Web€¦ · INTER CA Taxation Head Office Shraddha, 4th Floor, ... 14. Advance Tax & Interest 268 - 271 15. Tax Deducted at Source (TDS) 272 - 294 16

: 264 :

J. K. SHAH CLASSES INTER C.A. - DIRECT TAXES

ASSESSMENT PROCEDURE

Q. 3. Which of the following returns can be revised u/s 139(5).

(a) Return of loss filed under section 139(3) .

(b) Return filed by a trust u/s 139(4A) or a Political party u/s 139(4B) or Association

u/s 139(4C) .

(c) Return filed within the time extended by CBDT under section 119 .

(d) A Revised Return filed under section 139(5) .

(e) Defective return filed u/s 139(9) .

Q. 4. State whether filing of income-tax return is mandatory for the assessment year 2018-19

in respect of the following cases:

(i) Research association eligible for exemption under section 10(21) having total

income of `̀̀̀ 3,10,000

(ii) Registered trade union eligible for exemption under section 10(24) having fol-

lowing incomes:

Income from house property (computed) `̀̀̀ 60,000

Income from other sources (computed) `̀̀̀ 40,000

(iii) A charitable trust registered under section 12AA, having total income of

`̀̀̀ 2,60,000.

(iv) A Limited Liability Partnership (LLP) with business loss of `̀̀̀ 1,30,000.

Q. 5. Mr. Vineet submits his return of income on 12-09-2018 for A.Y 2018-19 consisting of

income under the head salaries, “Income from house property” and bank interest. On

21-01-2019, he realized that he had not claimed deduction under section 80TTA in

respect of his interest income on the Savings Bank Account. He wants to revise his

return of income, since one year has not elapsed from the end of the relevant assess-

ment year. Discuss.

Page 268: INTER CA Taxation - CA Study Web€¦ · INTER CA Taxation Head Office Shraddha, 4th Floor, ... 14. Advance Tax & Interest 268 - 271 15. Tax Deducted at Source (TDS) 272 - 294 16

: 265 :

J. K. SHAH CLASSES INTER C.A. - DIRECT TAXES

ASSESSMENT PROCEDURE

Ans. 2.

1. Mr. Pinakin can carry forward unabsorbed depreciation of `̀̀̀ 80,000 as Section

139(3) does not apply to unabsorbed depreciation.

The due date for filing the return of income for Mr. Pinakin u/s. 139(1) for A.Y.

2018-2018 is 31st July 2018 (since the turnover for P.Y. 2017-18 is exactly

`̀̀̀ 1 crore, tax audit u/s. 44AB is not applicable). The ROI has been filed on

5.8.2018 i.e. after the due date u/s. 139(1). Accordingly, he cannot carry forward

business loss of `̀̀̀ 40,000/-

2. The last date for filing a belated return for A.Y. 2018-19 u/s. 139(4) is as follows:-

End of the assessment year 31-3-2019

OR

Completion of Assessment 10-5-2020

Whichever is earlier

In the given case, since the ROI has been filed on 31-3-2019, it is a valid

belated return.

3. The last date for filing a belated return for A.Y. 2018-19 u/s. 139(4) is 31-3-2019

(same as point 2 above). Since the ROI has been filed on 11-4-2020, it is an

invalid belated return.

4. The last date for filing a belated return for A.Y. 2018-19 u/s. 139(4) is 31-3-2019

or 31-12-2018, whichever is earlier i.e. 31-12-2018. Since the ROI has been

filed on 18-1-2019, it is an invalid belated return.

The assessment performed by the Income tax department in this case is Best

Judgment Assessment since the ROI was filed by the assessee after comple-

tion of the assessment. In other words, at the time of performing assessment,

there was no ROI filed by the assessee.

5. If the loss of `̀̀̀ 10,000 is from the activity of owning and maintaining race horses,

then the assessee will be allowed to carry forward such loss (as per Section

74A) since the ROI was filed before the applicable due date.

However, if this loss is any “Other Loss” i.e. not from the activity of owning and

maintaining race horses, there is no section in the Income tax Act, 1961 which

allows or authorizes the carry forward of such loss. Thus the assessee will not be

allowed to carry forward such loss, irrespective of the date of filing ROI.

Ans. 3.

(a) A return of loss filed u/s. 139(3) implies that the return has been filed as per

Section 139(1) and hence it can be revised under section 139(5)

(b) A trust [Section 139(4A)], a political party [Section 139(4B)] or an association

[Section 139(4C)] enjoy exemption from income tax but are required to file their

return of income as per Section 139(1) if their income (before exemption)

exceeds the basic exemption limit of `̀̀̀ 2,50,000. If the return is filed as above, it

implies that the return can be revised u/s. 139(5).

CLASS WORK SOLUTIONS

Page 269: INTER CA Taxation - CA Study Web€¦ · INTER CA Taxation Head Office Shraddha, 4th Floor, ... 14. Advance Tax & Interest 268 - 271 15. Tax Deducted at Source (TDS) 272 - 294 16

: 266 :

J. K. SHAH CLASSES INTER C.A. - DIRECT TAXES

ASSESSMENT PROCEDURE

(c) If the return of income for an assessment year has been filed within the extended

time limit specified by C.B.D.T., it implies that the return of income for that year

is as per Section 139(1) and hence it can be revised u/s. 139(5).

(d) A revised return replaces the original return filed as per Section 139(1). The

fact that a revised return has been filed implies that the original return was filed

by the assessee as per Section 139(1). Thus a revised return can also be re-

vised within the time limit allowed by Section 139(5).

(e) A return of income is termed as a defective return if some information or data is

missing in the return. Examples- submitting computation of income but not com-

putation of tax, not submitting Final Accounts along with ROI, not submitting proof

of tax paid (TDS certificate, tax challans etc).

If a defective return is filed by the assessee, the Assessing Officer informs the

assessee of the defect and allows a time limit of 15 days for rectifying the de-

fect. If the defect is rectified by the assessee, the return of income shall be treated

as a return filed u/s. 139(1). Accordingly, such a return can be revised u/s. 139(5).

If the defect is not rectified by the assessee, the return of income is an incom-

plete return and cannot be treated as a return filed u/s. 139(1). Accordingly, such

a return cannot be revised u/s. 139(5).

Ans. 4.(i) As per section 139(4C), a research association referred to in section 10(21)

must file its return of income within the due date under section 139(1) if its total

income, without giving effect to the provisions of section 10, exceeds the maxi-

mum amount which is not chargeable to income-tax.

Since the total income of the research association exceeds the basic exemp-

tion limit of ` ` ` ` 2,50,000, it has to file its return of income for the A.Y.2018-19.

(ii) As per section 139(4C), a registered trade union referred to in section 10(24)

must file its return of income if the total income exceeds the basic exemption

limit without giving effect to the provisions of section 10.

Since the total income of the trade union is less than the basic exemption limit of

` ` ` ` 2,50,000, it need not file its return of income for the A.Y. 2018-19.

(iii) As per section 139(4A), a charitable trust registered under section 12AA must

file its return of income, if its total income computed as per the provisions of the

Income-tax Act, 1961, without giving effect to the provisions of sections 11 and

12, exceeds the maximum amount which is not chargeable to income-tax. Since

the total income of the charitable trust exceeds ` ` ` ` 2,50,000, it has to file its return

of income for the A.Y. 2018-19.

(iv) As per third proviso to section 139(1), every company or firm shall furnish on or

before the due date the return in respect of its income or loss in every previous

year. Since LLP is included in the definition of “firm” under the Income-tax Act,

1961, it has to file its return mandatorily, even though it has incurred a loss.

Page 270: INTER CA Taxation - CA Study Web€¦ · INTER CA Taxation Head Office Shraddha, 4th Floor, ... 14. Advance Tax & Interest 268 - 271 15. Tax Deducted at Source (TDS) 272 - 294 16

: 267 :

J. K. SHAH CLASSES INTER C.A. - DIRECT TAXES

ASSESSMENT PROCEDURE

Ans. 5. Since Mr. Vineet has income only under the heads “Salaries”, “Income from house

property” and “Income from other sources”, he does not fall under the category of

a person whose accounts are required to be audited under the Income-tax Act,

1961 or any other law in force.

Therefore, the due date of filing return for A.Y.2018-19 under section 139(1), in

his case, is 31st July, 2018. Since Mr. Vineet had submitted his return only on

12.9.2018, the said return is a belated return under section 139(4).

As per section 139(5), a return furnished under section 139(1) or a belated re-

turn u/s 139(4) can be revised. Thus, a belated return under section 139(4) can

also be revised. Therefore, Mr. Vineet can revise the return of income filed by

him under section 139(4), to claim deduction under section 80TTA, since the

time limit of one year from the end of the relevant assessment year has not

elapsed.

Page 271: INTER CA Taxation - CA Study Web€¦ · INTER CA Taxation Head Office Shraddha, 4th Floor, ... 14. Advance Tax & Interest 268 - 271 15. Tax Deducted at Source (TDS) 272 - 294 16

: 268 :

J. K. SHAH CLASSES INTER C.A. - DIRECT TAXES

ADVANCE TAX & INTEREST

ADVANCE TAX

Sec. 207 - Liability of Payment of advance tax

(1) Under the provisions of advance tax, tax is payable in advance during the previous

year itself in which the current income is earned, although it will be assessed and

chargeable to tax in the assessment year.

(2) Advance tax is not payable by a resident individual of age 60 years or more in

the relevant P.Y. who does not have any income under the head "Profits and

Gains of Business or Profession."

Sec. 208 - Advance tax is payable in a previous year / financial year as per the

provisions if the amount of advance tax is ì ì ì ì 10,000/- or more.

Sec. 209 - Calculation of advance tax by assessee

Step 1 : Estimate the entire current income inclusive of long - term short - term

capital gains, casual winnings and consider agriculture income as well

for rate of tax purposes.

Step 2 : Compute the basic tax on above

Step 3 : Add surcharge if applicable and education cess

Step 4 : Give relief u/s. 89 if applicable

Step 5 : Deduct the TDS / TCS actually deducted/collected.

Step 6 : The balance amount if it is ì ì ì ì 10,000 or more is the advance tax payable

as per section 211.

In other words, if tax was deductible but not deducted by the payer, the receiver shall

have to pay advance tax towards such income.

Sec. 210(1) - Every person who is liable to pay advance tax, shall do so whether or not

he was subjected to regular assessment earlier.

Sec. 210(2) - He may adjust the remaining advance tax instalments by way of increase

or reduction in them, should the need arise.

Sec. 211 : Instalment of advance tax and due dates

On or Before

ADVANCE TAX & INTEREST

Page 272: INTER CA Taxation - CA Study Web€¦ · INTER CA Taxation Head Office Shraddha, 4th Floor, ... 14. Advance Tax & Interest 268 - 271 15. Tax Deducted at Source (TDS) 272 - 294 16

: 269 :

J. K. SHAH CLASSES INTER C.A. - DIRECT TAXES

ADVANCE TAX & INTEREST

For an eligible assessee covered by Section 44AD [Presumptive Taxation], the above

installments shall not apply and the assessee can pay the whole amount of advance tax on

or before 15th March of the previous year.

Any amount paid after 15th of March but before of 31st March of the relevant previous year

shall be considered as advance tax payment only, but shall be liable to interest.

If on the last day for payment of any instalment of advance tax the bank is closed, the

assessee should make the payment on the immediately following working day and the

interest leviable u/s. 234B & 234C will not be charged.

INTEREST

Sec. 234A - Interest when return is furnished after the due date.

Rate of Interest : 1% per month and part of a month is considered one full month.

Period of Interest : Commencing from the date immediately following the due

date of filing the return till :

� the date of actually filing the return

� date of completion of assessment u/s. 144 (best

judgement assessment), if no return is filed.

Amount on which interest : Assessed tax (-) Advance tax (-) TDS / TCS

shall be calculated but not tax paid u/s. 140A, i.e. after 31st of March

For calculating interest element paid u/s. 234A

in the total tax + interest payment u/s. 140A,

returned tax shall be considered.

Rule 119A : Any fraction beyond the nearest multiple of ì ì ì ì 100/- will be

ignored in the calculation of interest payable or receivable by

the assessee.

Sec. 234B - Interest on default of payment of advance - tax payable.

� Either no advance tax is paid or paid less than 90% of assessed tax.

� If no advance tax is paid then interest is payable on assessed tax @ 1% per month

for every month or part of a month which is considered as a full month, starting from

the 1st day of April of the assessment year till the date of determination of income

under summary assessment - u/s. 143(1) or regular assessment - u/s. 143(3).

� If advance tax is paid then as above except that assessed tax will be reduced by

the advance tax so paid.

� Rule 119A to be considered.

Page 273: INTER CA Taxation - CA Study Web€¦ · INTER CA Taxation Head Office Shraddha, 4th Floor, ... 14. Advance Tax & Interest 268 - 271 15. Tax Deducted at Source (TDS) 272 - 294 16

: 270 :

J. K. SHAH CLASSES INTER C.A. - DIRECT TAXES

ADVANCE TAX & INTEREST

Sec. 234C - Interest on deferment of advance tax

� When assessee does not pay advance tax or underestimates instalments of

advance tax.

For all assessees except assessee covered by Sec 44AD :

(1) 15% Returned tax - TDS / TCS Advance tax x

1 x 3 months

paid till 15th June. 100

��� ��% Returned tax - TDS / TCS Advance tax x

1 x 3 months

paid till 15th Sept. 100

rate of interest

(3) 75% Returned tax - TDS / TCS Advance tax x

1 x 3 months

paid till 15th Dec. 100

��� 100% Returned tax - TDS / TCS Advance tax x

1 x 1 month

paid till 15th March 100

���� For an asseessee covered by section 44AD, interest will be calculated only for

the instalment of March as follows :

100% Returned tax - TDS / TCS Advance tax x

1 x 1 month

paid till 15th March 100

���� Rule 119A to be considered.

���� Interest provisions are not attracted if the shortfall in the payment of advance tax

is due to underestimation or non - estimation of capital gains or casual winnings,

and the assessee has paid the entire tax payable in respect of such income, as

part of remaining instalments of advance tax which are due or if no instalment is

due, then such tax is paid before the end of the financial year.

���� Interest provisions shall also not be attracted if income arises under the head

"Business or Profession" for the FIRST TIME.

���� If the advance tax paid by the assessee on or before 15th June is atleast 12% of

the returned tax or if the advance tax paid on or before 15th September is atleast

36% of returned tax, then no interest shall be charged on the shortfall on those

dates.

Sec. 234D - Interest on excess refund

If excess refund is granted at the time of summary assessment, in relation to either

lesser refund or no refund on regular assessment.

Rate of Interest : 0.5% per month or part of a month

Period of Interest : Date of grant of refund under summary assessment to the

date of completion of regular assessment.

Calculated on : Full refund or excess refund amount as the case may be.

Sec. 244A - Interest payable to assessee on refund

��� If the refund arises due to TDS or TCS or advance tax, interest shall be payable

to the assessee @ 0.5% per month or part of a month for the period -

(a) From 1st April of the A.Y. to date of receiving refund, if the R.O.I for that

P.Y. has been furnished on or before the due date u/s 139(1); or

(b) in any other case, from the date of furnishing R.O.I. to the date of receiv-

ing refund.

(2) If the refund arises due to any self assessment tax paid u/s 140A, the period

shall begin from the date of furnishing R.O.I. or date of payment of tax, whichever

is later.

Page 274: INTER CA Taxation - CA Study Web€¦ · INTER CA Taxation Head Office Shraddha, 4th Floor, ... 14. Advance Tax & Interest 268 - 271 15. Tax Deducted at Source (TDS) 272 - 294 16

: 271 :

J. K. SHAH CLASSES INTER C.A. - DIRECT TAXES

ADVANCE TAX & INTEREST

Q. 1. Mr. S during the financial year 2017-18 aged 48 years pays the following instalments of

advance tax :

15.9.2017 21,000

15.12.2017 14,000

16.3.2018 12,300

The regular assessment for the A.Y. 2018-19 is completed on 10.3.2019 and income

determined by the Assessing Officer without any addition is ì ì ì ì 6,97,200. He is also

entitled to tax credit of ì ì ì ì 9,476 on account of TDS.

Find out the amount of interest payable under section 234B & 234C.

Q. 2. Compute the advance tax payable by Ms. Shraddha from the following estimated

income submitted for the financial year 2017-2018

Income from Salary 5,80,000

Rent from house property (per annum) 3,60,000

Interest on government securities 25,000

Interest on saving bank deposit 3,000

Receipt from lottery (net) (TDS @ 30%) 14,000

Agriculture Income 90,000

Contribution to PPF 60,000

Tax Deducted at source by the employer on salary 37,080

Page 275: INTER CA Taxation - CA Study Web€¦ · INTER CA Taxation Head Office Shraddha, 4th Floor, ... 14. Advance Tax & Interest 268 - 271 15. Tax Deducted at Source (TDS) 272 - 294 16

J.K.SHAH CLASSES INTER C.A. – DIRECT TAX

: 272 :

SEC 192. SALARY

Person responsible to deduct tax

All Assesses who are employers (resident as well as non-

resident)

Rate of deduction of tax

Average of income-tax computed on the basis of the rates in force for the financial year (Part III of Schedule I of the Finance Act) in which the payment is made, on the estimated income of the assessee under the head salary.

The employer shall, for the purposes of estimating income of the employee or computing tax deductible, obtain from the assessee the evidence or proof or particulars of prescribed claims (including claim for set-off of loss) under the provisions of the Act in such form and manners as may be prescribed.

Exemption limit Basic exemption available to the assessee

Time for deduction of tax

At the time of payment i.e. as and when salary is paid

Time for deposit of TDS

Refer to Table A

Deduction of tax at lower rate or non-deduction of tax

Application in Form No.13 shall be made to the AO. Certificate in the prescribed form shall be issued by the AO to the Assessee.

Inclusion of income / loss from other heads

The employer shall include the income from any other head on a declaration filed by the employee, but shall not consider loss under any other head except “House Property”. However, such inclusion of other income shall not in any case have the effect of reducing the tax deductible on salary income.

Payment of tax by employer on non-monetary perquisites Sec.192(1A)

The employer may, at his opinion, pay income tax on the whole or part of perquisites provided by way of non-monetary payments. Such tax may be determined at the average of income tax computed on the income chargeable under the head ‘Salaries’, including the non-monetary payments. The tax so payable shall be construed as if it were a tax deductible at source.

PAYMENT OF ACCUMULATED BALANCE FROM PROVIDENT FUND :- SEC 192A [inserted by FA 2015 w-e-f 1-6-15

Person responsible to deduct tax

The trustee of the Payment of Employees’ Provident Fund Scheme, 1952

Category of payee Employee

Circumstance In a case where the accumulated balance due to an employee participating in a recognized provident fund is includible in his total income owing to the provisions of rule 8 of Part A of the Fourth Schedule not being applicable.

CHAPTER NO. 15 TAX DEDUCTION AND COLLECTION AT SOURCE

Page 276: INTER CA Taxation - CA Study Web€¦ · INTER CA Taxation Head Office Shraddha, 4th Floor, ... 14. Advance Tax & Interest 268 - 271 15. Tax Deducted at Source (TDS) 272 - 294 16

J.K.SHAH CLASSES INTER C.A. – DIRECT TAX

: 273 :

Rate of deduction of tax

10%. If PAN is not provided then MMR (34.608%)

No TDS The payment is < Rs.50,000/-

Time for deduction of tax

At the time of payment

Time for deposit of TDS

Refer to Table A

Payees declaration for non-deduction of tax

Declaration in Form 15G [seniors (age > 60) – form 15H] by an Assessee in duplicate. Once copy of such form is to be delivered to income tax department on or before 7

th of the

following month in which declaration is furnished.

INTEREST ON SECURITIES :- SEC. 193

Person responsible for tax deduction

Central or State Government, Local Authority, Company or Corporation established under the Central or State Act

Category of payee and Rate of deduction of tax

Resident Non-corporate Assessee and Domestic Company : @ 10%

Time for deduction of tax

At the time of credit or payment whichever is earlier. It includes credit to “Interest payable account” or “suspense account”.

Time for deposit of TDS

Refer to Table A

Deduction at lower rate or no deduction.

Application in Form No.13 shall be made to the AO.

Certificate in the prescribed form shall be issued by the AO to the Assessee.

Non-deduction of tax Declaration in Form No.15G [seniors (age > 60) – form 15H) by an Assessee other than Company or Firm in duplicate. One copy of such form is to be delivered to income tax department on or before 7th of the month following the month in which declaration is furnished.

Interest on which no tax is deducted at source

i) Any interest payable on National Development Bonds; or

ii) Any interest payable on notified debentures, issued by any institution or authority, or any public sector company, or any co-operative society (including a co-operative land mortgage bank or a co-operative land development bank)

iii) Any interest payable on any security of the Central Government or a State Government [excluding interest > Rs.10,000/- payable on 8% Savings (Taxable) Bonds,2003];

iv) Any interest payable to an individual or HUF, who is resident in India, on listed debentures issued by a company in which the public are substantially interested, if –

Page 277: INTER CA Taxation - CA Study Web€¦ · INTER CA Taxation Head Office Shraddha, 4th Floor, ... 14. Advance Tax & Interest 268 - 271 15. Tax Deducted at Source (TDS) 272 - 294 16

J.K.SHAH CLASSES INTER C.A. – DIRECT TAX

: 274 :

(a) The interest is paid by the company by an account payee cheque; and

(b) The aggregate amount of interest paid or likely to be paid during the FY does not exceed Rs.5,000/-

v) Any interest payable to the LIC, on any securities owned by it or in which it has full beneficial interest, or

vi) Any interest payable to the GIC, or to any of its 4 subsidiary companies, on any securities owned by the Corporation or such company or in which the Corporation or such company has full beneficial interest; or

vii) Any interest payable to any other insurer on any securities owned by it or in which it has full beneficial interest.

viii) Any interest payable on any security issued by a company, where such security is in dematerialized form and is listed on a recognized stock exchange in India in accordance with the Securities Contracts (Regulation) Act, 1956 and the rules made there under.

DIVIDENDS :- SEC. 194

The principal officer of a domestic company shall, before making any payment in cash

or before issuing any cheque or warrant in respect of any dividend or before making any

distribution or payment to a shareholder, who is resident in India, of any dividend within

the meaning of section 2(22)(e), deduct from the amount of such dividend, income-tax

@10%.

Exceptions : No such deduction shall be made in the following cases:-

In case of a shareholder, being an individual if –

a. The dividend is paid by the company by an account payee cheque; and

b. The amount of such dividend or, as the case may be, the aggregate of the

amounts of such dividend distributed or paid or likely to be distributed or paid

during the financial year by the company to the shareholder, does not exceed

Rs.2,500/-

INTEREST OTHER THAN “INTEREST ON SECURITIES”:- SEC.194A

Person responsible for tax deduciton

a) All persons other than individual and HUF, and b) Individual and HUF who are required to get their

accounts audited u/s 44AB (a) or (b), in the preceding FY.

Category of payee Any resident in India

Rate of deduction of tax

Non-corporate Assesses or Domestic companies @10%

No TDS i) Where the aggregate of interest credited or paid or likely to be credited or paid during the FY does not exceed – a) Rs.10,000/- in case of interest paid by banking

Page 278: INTER CA Taxation - CA Study Web€¦ · INTER CA Taxation Head Office Shraddha, 4th Floor, ... 14. Advance Tax & Interest 268 - 271 15. Tax Deducted at Source (TDS) 272 - 294 16

J.K.SHAH CLASSES INTER C.A. – DIRECT TAX

: 275 :

company, co-operative bank or post office on notified deposit schemes, and

b) Rs.5,000/- in any other case Provided that in respect of the income credited or paid in respect of – (a) Time deposits with a banking company; or (b) Time deposits with a co-operative bank; (c) Deposits with a public company providing long-term

finance for construction or purchase of residential houses in India and which is eligible for deduction under section 36(1)(viii), The aforesaid amount shall be computed with reference to a branch;

Provided further that the amount referred to in the first proviso shall be computed with reference to the total income credited or paid by the banking company or the co-operative society or the public company (and not branch wise), where such banking company or the co-operative society or the public company has adopted core banking solutions [inserted by FA 2015 w-e-f 1-6-15] “TIME DEPOSITS” means deposits including recurring deposits repayable on the expiry of fixed periods

ii) Interest credited or paid to –a) Any banking company, or any co-operative bank

(including a co-operative land mortgage bank), or b) Any financial corporation established by or under a

Central, State or Provincial Act, or c) The Life Insurance Corporation of India, or d) The Unit Trust of India, or e) Any company or co-operative society carrying on the

business of insurance, or f) Notified institution, association or body or class of

institutions, associations or bodies; iii) Interest credited or paid by a firm to a partner of the firm;iv) To such income credited or paid by a co-operative

society (other than a co-operative bank) to a member thereof or to such income credited or paid by a co-operative society to any other co-operative society. [Amended by FA 2015 w-e-f 1-6-15]Explanation – For the purposes of this clause, “co-operative bank” shall have the same meaning as assigned to it in Part V of the Banking Regulation Act, 1949 (10 of 1949)

v) Interest credited or paid by the Central Government under any provision of this Act or, or the Estate Duty Act, 1953, or the Wealth-tax Act, 1957;

vi) Interest paid or payable by an infrastructure capital

Page 279: INTER CA Taxation - CA Study Web€¦ · INTER CA Taxation Head Office Shraddha, 4th Floor, ... 14. Advance Tax & Interest 268 - 271 15. Tax Deducted at Source (TDS) 272 - 294 16

J.K.SHAH CLASSES INTER C.A. – DIRECT TAX

: 276 :

company or infrastructure capital fund or a public sector company or scheduled bank in relation to a zero coupon bond issued on or after 1st June, 2005.

Time for deduction of tax

At the time of credit or payment whichever is earlier. It

includes credit to a suspense account

Time for deposit of TDS

Refer to Table A

Deduction of tax at lower rate or non-deduction of tax

Application in Form No.13 shall be made to the AO. Certificate in the prescribed form shall be issued by the AO to the Assessee.

Payees declaration for non-deduction of tax

Declaration shall be furnished in Form 15G [seniors (age > 60) – form 15H] by an Assessee other than Company of Firm in duplicate. One copy of such form is to be delivered to INCOME TAX DEPARTMENT on or before 7th of the following month in which declaration is furnished.

WINNINGS FROM LOTTERY OR CROSSWORD PUZZLE : SEC 194B

Person responsible to deduct tax

Any person paying the sum

Category of payee Any assessee

Rate of deduction of tax

30%

Winning in kind Person responsible for paying shall, before releasing the winnings, ensure that tax has been paid in respect of the winnings.

No TDS The payment =< Rs.10,000/-

Time for deduction of tax

At the time of payment. Where prize money is in kind, before releasing the prize, the payer should ensure that tax has been paid in respect of the winnings.

Time for deposit of TDS

Refer to Table A

WINNINGS FROM HORSE RACE :- SEC 194BB

Person responsible to deduct tax

Any person being a licensed bookmaker

Category of payee All Assesses

Rate of deduction of tax

30%

No TDS The payment =< Rs.10,000/-

Time for deduction of tax

At the time of payment

Time for deposit of TDS

Refer to Table A

Page 280: INTER CA Taxation - CA Study Web€¦ · INTER CA Taxation Head Office Shraddha, 4th Floor, ... 14. Advance Tax & Interest 268 - 271 15. Tax Deducted at Source (TDS) 272 - 294 16

J.K.SHAH CLASSES INTER C.A. – DIRECT TAX

: 277 :

PAYMENTS TO CONTRACTORS AND SUB-CONTRACTORS:- SEC. 194C

Person responsible to

deduct tax

a) The Central Government of any state Government; or b) Any local authority; or c) Any corporation established by or under a Central, State or

Provincial Act; or d) Any company; or e) Any co-operative society; or f) Any authority, constituted in India by or under any law,

engaged either for the purpose dealing with and satisfying the need for housing accommodation or for the purpose of planning, development or improvement of cities, towns and villages, or for both; or

g) Any society registered under the Societies Registration Act, 1860, or under any law corresponding to that Act in force in any part of India; or

h) Any trust; or i) Any university established or incorporated by or under a

Central, State or Provincial Act and an institution declared to be a university under section 3 of the University Grants Commission Act, 1956 (3 of 1956); or

j) Any Government of a foreign State or a foreign enterprise or any association or body established outside India; or

k) Any firm; or l) Any person, being an individual or a Hindu undivided family or

an association of persons or a body of individuals, if such person – A. Does not fall under any of the preceding sub-clauses; and B. Is liable to audit of accounts under clause (a) or clause (b)

of section 44AB during the preceding financial year; Exception for individual and HUF : Contract of personal nature

Nature of payment

Payment of any sum to any resident or contractor for carrying out any work including supply of labour for carrying out any work in pursuance of any contract, including –

a) Advertising b) Broadcasting and telecasting including production of

programs for such broadcasting or telecasting c) Carriage of goods and passengers by any mode of

transport other than by railways d) Catering e) Manufacturing or supplying a product according to the

requirement or specification of a customer by using material purchased from such customer, but does not include manufacturing or supplying a product according to the requirement or specification of a customer by using material purchased from a person, other than such customer.

f)

Category of payee Any person

Page 281: INTER CA Taxation - CA Study Web€¦ · INTER CA Taxation Head Office Shraddha, 4th Floor, ... 14. Advance Tax & Interest 268 - 271 15. Tax Deducted at Source (TDS) 272 - 294 16

J.K.SHAH CLASSES INTER C.A. – DIRECT TAX

: 278 :

Rate of deduction

of tax

Payment to an individual or HUF – 1%

Payment to any other person – 2%

Where any sum is paid or credited for carrying out any work

mentioned in sub-clause (e), above, tax shall be deducted at

source –

i) On the invoice value excluding the value of material, if

such value is mentioned separately in the invoice; or

ii) On the whole of the invoice, if the value of material is not

mentioned separately in the invoice

No TDS

a) Contracts the consideration for which does not exceed

Rs.30,000/- or

b) Aggregate of the sums credited or paid or likely to be credited

or paid during the FY does not exceed Rs.1,00,000/-, or

c) Payment to a contractor during the course of business of

plying, hiring or leasing goods carriages, not owning more

than 10 goods carriages anytime during the PY and who

furnishes his PAN to the payer. [Amended by FA 2015 we-f

1-6-15]

Time for

deduction of tax

At the time of credit or payment whichever is earlier (including

Cr to suspense a/c)

Time for deposit

of TDS

Refer to Table A

Deduction at

lower or nil rate

Application in Form No.13 shall be made to the AO

INSURANCE COMMISSION – SEC.194D

Person responsible to

deduct tax

Insurance companies

Category of payee Resident Assessee

Rate of deduction of

tax

Non Corporate Assessee -5% and Domestic Company –

10%

Exemption limit Payment in aggregate =, Rs.15,000/- during FY

Time for deduction of

tax

At the time of credit or payment whichever is earlier

Time for deposit of

TDS

Deduction of tax at

lower / nil rate

Application in form No.13 shall be made to the Assessing

Officer Certificate in the prescribed form shall be issued by

the AO.

Page 282: INTER CA Taxation - CA Study Web€¦ · INTER CA Taxation Head Office Shraddha, 4th Floor, ... 14. Advance Tax & Interest 268 - 271 15. Tax Deducted at Source (TDS) 272 - 294 16

J.K.SHAH CLASSES INTER C.A. – DIRECT TAX

: 279 :

PAYMENT FROM LIFE INSURANCE POLICY – SEC.194 DA

[Inserted by FA 2014]

Person responsible to deduct tax

Insurance Companes

Category of payee Resident Assessee

Rate of deduction of tax

1% of payment including bonus

Exemption Amount which is exempt u/s 10(10D), or Aggregate amounts of payments in a FY < 1,00,000/-

Payee declaration for non-deduction of tax

Declaration shall be furnished in Form 15G [seniors (age > 60) – form 15H] by an Assessee other than company or Firm in duplicate. One copy of such form is to be delivered to INCOME TAX DEPARTMENT on or before 7th of the

following month in which declaration is furnished.

PAYMENTS TO NON-RESIDENT SPORTSMEN OR SPORTS ASSOCIAITONS. SEC.

194E

Where any income referred to in section 115BBA is payable to a non-resident sportsman (including an athlete), or an entertainer, who is not a citizen of India or a non-resident sports association or institution, the person responsible for making the payment shall, at the time of credit of such income to the account of the payee or at the time of payment thereof in cash or by issue of a chque or draft or by any other mode, whichever is earlier, deduct income-tax thereon @20%.

COMMISSION, ETC., ON SALE OF LOTTERY TICKETS – SEC.194G

Any person who is responsible for paying to any person, who is or has been stocking, distributing, purchasing or selling lottery tickets, any income by way of commission, remuneration or prize (by whatever name called) on such tickets in an amount > Rs.15,000/- shall, at the time of credit of such income to the account of the payee or at the time of payment of such income in cash or by the issue of a cheque or draft or by any other mode, whichever is earlier, deduct income-tax thereon @5%

COMMISSION OR BROKERAGE – SEC. 194H

Person responsible to deduct tax

a) All persons other than individual and HUF, and b) Individual and HUF who are required to get their

accounts audited u/s 44AB (a) or (b), in the preceding FY

Category of payee Any person

Nature of payment Commission or Brokerage includes any payment received or receivable, directly or indirectly, by a person acting on behalf of another person for services rendered (not being professional services) or for any services in the course of buying or selling of goods or in relation to any transaction

Page 283: INTER CA Taxation - CA Study Web€¦ · INTER CA Taxation Head Office Shraddha, 4th Floor, ... 14. Advance Tax & Interest 268 - 271 15. Tax Deducted at Source (TDS) 272 - 294 16

J.K.SHAH CLASSES INTER C.A. – DIRECT TAX

: 280 :

relating to any asset, valuable article or thing, not being securities. Exception: Commission or Brokerage payable by BSNL or MTNL to their public call office franchisees.

Rate of deduction of tax

5%

No TDS Aggregate payment =, Rs.15,000/- during the FY

Time for deduction of tax

At the time of payment of credit whichever is earlier, including credit to a suspense account

Time for deposit of TDS

Refer to Table A

Deduction of tax at lower / nil rate

Application in Form No. 13 shall be made to the AO. Certificate in the prescribed form shall be issued by the AO.

RENT – SEC. 194-I

Person responsible to deduct tax

a) All persons other than individual and HUF, and b) Individual and HUF who are required to get their

accounts audited u/s 44AB in the preceding FY

Category of payee Any person being resident in India

Nature of payment Rent means any payment, by whatever name called, under nay lease, sublease, tenancy or any other agreement or arrangement for the use of (either separately or together) any, -

i. Land; or ii. Building (including factory building); or iii. Land appurtenant to a building (including factory

buildings); or iv. Machinery; or v. Plant; or vi. Equipment; or vii. Furniture; or viii. Fittings, Whether or not any or all of the above are owned by the payee;

Rate of deduction of tax

a) Machinery, plant or equipment - @2%b) Land or building (including factory building) or land

appurtenant to a building (including factory building) or furniture or fittings - @10%

No TDS i. Aggregate amount of rent paid or credited < Rs.1,80,000/- during the FY, or the payee is the

government or local authority.

Time for deduction of tax

At the time of payment or credit whichever is earlier, including credit to suspense account

Time for deposit of TDS

Refer to table A

Deduction of tax at lower nil / rate

Application in Form No.13 shall be made to the Assessing Officer Certificate in the prescribed form shall be issued by the AO to the Assessee.

Page 284: INTER CA Taxation - CA Study Web€¦ · INTER CA Taxation Head Office Shraddha, 4th Floor, ... 14. Advance Tax & Interest 268 - 271 15. Tax Deducted at Source (TDS) 272 - 294 16

J.K.SHAH CLASSES INTER C.A. – DIRECT TAX

: 281 :

RENT SECTION 194 IB

(1) Any person, being an individual or a Hindu undivided family (other than those

referred to in the second proviso to section 194-I), responsible for paying to a

resident any income by way of rent exceeding fifty thousand rupees for a month or

part of a month during the previous year, shall deduct an amount equal to five per

cent of such income as income-tax thereon.

(2) The income-tax referred to in sub-section (1) shall be deducted on such income at

the time of credit of rent, for the last month of the previous year or the last month

of tenancy, if the property is vacated during the year, as the case may be, to the

account of the payee or at the time of payment thereof in cash or by issue of a

cheque or draft or by any other mode, whichever is earlier.

(3) The provisions of section 203A shall not apply to a person required to deduct tax in

accordance with the provisions of this section.

(4) In a case where the tax is required to be deducted as per the provisions of section

206AA, such deduction shall not exceed the amount of rent payable for the last

month of the previous year or the last month of the tenancy, as the case may be.

Explanation.—For the purposes of this section, “rent” means any payment, by whatever

name called, under any lease, sub-lease, tenancy or any other agreement or

arrangement for the use of any land or building or both.’.

PAYMENT ON TRANSFER OF CERTAIN IMMOVABLE PROPERTY OTHER THAN AGRICULTURAL LAND:- SEC. 194IA

Person responsible to deduct tax

Any person being a transferee and responsible to pay the consideration

Category of payee Any person 9transferor) being resident in India

Nature of payment Consideration for transfer of immovable property being land (other than rural agricultural land in India) or building or part of a building.

Rate of deduction of tax

1% of consideration

No TDS Consideration < Rs.50 lakhs

Time for deduction of tax

at the time of credit of such sum to the account of the transferor or at the time of payment of such sum in cash or by issue of a cheque or draft or by any other mode, whichever is earlier

Time for deposit of TDS

Compulsory electronic payment within 7 days of the following month

TAN and PAN Numbers?

Deductor is not required to have TAN number. However, he must obtain PAN of the Deductee. Otherwise tax will have to be deducted @20% instead of 1%. Section 206AA will apply in such case.

Page 285: INTER CA Taxation - CA Study Web€¦ · INTER CA Taxation Head Office Shraddha, 4th Floor, ... 14. Advance Tax & Interest 268 - 271 15. Tax Deducted at Source (TDS) 272 - 294 16

J.K.SHAH CLASSES INTER C.A. – DIRECT TAX

: 282 :

FEES FOR PROFESSIONAL OR TECHNICAL SERVICES :- SEC.194J

Person responsible to deduct tax

c) All persons other than individual and HUF, and d) Individual and HUF who are required to get their

accounts audited u/s 44AB in the preceding FY

Category of payee Any person

Nature of payment (a) “Professional Services” means services rendered by a person in the course of carrying on legal, medical, engineering or architectural profession or the profession of accountancy or technical consultancy or interior decoration or advertising or such other profession as is notified by the Board for the purposes of section 44AA or of this section;

(b) “Fees for technical services” shall have the same

meaning as in Explanation 2 to clause (vii) of sub-section (1) of section 9

(c) any remuneration or fees or commission by whatever name called, other than those which tax is deduction u/s 192, to a director of a company, or

(d) royalty, or (e) any sum referred to in clause (va) of section 28

Rate of deduction 10% (in case payee engaged only in the business of operation of call center = 2%)

No TDS i. Aggregate of payments =< Rs.30,000/- during the FYii. Payment in respect of services for personal purpose

of individual or any member of Hindu undivided family

Time for deduction of tax

At the time of payment or credit whichever is earlier, including credit to suspense account

Time for deposit of TDS

Refer to table A

Deduction of tax at lower nil / rate

Application in Form No.13 shall be made to the Assessing Officer for obtaining the certificate in the prescribed form.

PAYMENT OF COMPENSATION ON ACQUISITION OF CERTAIN IMMOVABLE PROPERTY : SEC. 194LA

Person responsible to deduct tax

Any person responsible for payment of compensation

Category of payee Any resident person

Nature of payment Compensation or the enhanced compensation or the consideration or the enhanced consideration on account of compulsory acquisition, under any law for the time being in force, of any immovable property (other than agricultural land)

Rate of deduction 10% of such compensation

No TDS Where the aggregate amount of such payments during the FY =< Rs.2,50,000/-

Time for deduction At the time of payment in cash or by cheque or draft or by any other mode

Page 286: INTER CA Taxation - CA Study Web€¦ · INTER CA Taxation Head Office Shraddha, 4th Floor, ... 14. Advance Tax & Interest 268 - 271 15. Tax Deducted at Source (TDS) 272 - 294 16

J.K.SHAH CLASSES INTER C.A. – DIRECT TAX

: 283 :

OTHER SUMS. SEC. 195.

Payment in respect of which tax is to be deducted

Payment to a Non-Resident or a Foreign Company of: � Any interest (other than covered by 194LC) including

interest on securities; or � Any other sum chargeable to income tax in India, not beign

salaries

Time for deduction At the time of payment or credit to the account of payee, whichever is earlier, including credit to interest payable account or suspense account

Time for deposit Refer Table A

Certificate of TDS Form 16A, refer Table C

Quarterly Statements Form No. 26Q. refer table B for time

Deduction of tax at lower / nil rate

Application in Form No. 15C/15D shall be made to the AO

TAX DEDUCTED IS INCOME RECEIVED SEC.198.

All sums deducted in accordance with the provisions of the foregoing provisions of this

Chapter shall, for the purpose of computing the income of an assessee, be deemed to

be income received.

Provided that the sum being the tax paid, u/s.192(1A) shall not be deemed to be income

received.

CREDIT FOR TAX DEDUCTED SEC.199.

1) Any deduction made in accordance with the foregoing provisions of this Chapter and

paid to the Central Government shall be treated as a payment of tax on behalf of the

person from whose income the deduction was made, or of the owner of the security,

or of the depositor or of the owner of property or of the unit-holder, or of the

shareholder, as the case may be.

2) Any sum referred to in sub-section (1A) of section 192 and paid to the Central

Government shall be treated as the tax paid on behalf of the person in respect of

whose income such payment of tax has been made.

DUTY OF PERSON DEDUCTING TAX SEC.200.

1) Any person deducting any sum shall pay such sum within the prescribed time, to the

credit of the Central Government or as the Board directs.

2) Any person being an employer, referred to u/s 192(1A) shall pay, within the

prescribed time, the tax to the credit of the Central Government or as the Board

directs

2A) in case of an office of the Government, where the sum deducted in accordance with

the foregoing provisions of this Chapter or tax referred to in sub-section (1A) of section

Page 287: INTER CA Taxation - CA Study Web€¦ · INTER CA Taxation Head Office Shraddha, 4th Floor, ... 14. Advance Tax & Interest 268 - 271 15. Tax Deducted at Source (TDS) 272 - 294 16

J.K.SHAH CLASSES INTER C.A. – DIRECT TAX

: 284 :

192 has been paid to the credit of the Central Government without the production of a

challan, the Pay and Accounts Officer or the Treasury Officer or the Cheque Drawing

and Disbursing Officer or any other person, by whatever name called, who is

responsible for crediting such sum or tax to the credit of the Central Government, shall

delivery or cause to be delivered to the prescribed income-tax authority, or to the person

authorized by such authority, a statement in such form, verified in such manner, setting

for the such particulars and within such time as may be prescribed. [Inserted by FA 15]

3) Any person deducting any sum in accordance with the foregoing provisions of this

Chapter or, as the case may be, any person being an employer referred to in sub-

section (1A) of Section 192 shall, after paying the tax deducted to the credit of the

Central Government within the prescribed time, prepare such statements for such

period as may be prescribed, and deliver or cause to be delivered to the prescribed

income-tax authority or the person authorized by such authority such statement in

such form and verified in such manner and setting forth such particulars and within

such time as may be prescribed.

Provided that the person may also deliver to the prescribed authority a correction

statement for rectification of any mistake or to add, delete or update the information

furnished in the statement delivered under this sub-section in such form and verified

in such manner as may be specified by the authority.

Table A : Due date for deposit of TDS or TCS Amount :- Sec.192(1A)

SI. No. Deductor Cases Due Date for TDS

Due Date for TCS

1

Government

Tax paid without production of an income-tax challan

Same day Same Day

2 Tax paid accompanied by an income-tax challan

7 days from end of month

7 days from end of month

3 Any other person

Deduction made in the months of April to Feb

7 days from end of month

7 days from end of month

4 If income is credited or paid in the month of March

30th April 7 days from end of month

Failure to deduct / collect or pay tax at source (Sec. 201 and 206C)

If any person liable to deduct / collect tax at source fails to collect or fails to pay it to the

Department then the following consequences arise :-

* Interest - The defaulter shall be liable to pay simple interest u/s 201(1A) as under :

(i) at 1% p.m. or part thereof on the amount of tax not deducted from the date the tax

was deductible to the date the tax was actually deducted.

AND

(ii) at 1.5% p.m. or part thereof on the amount of tax deducted but not paid from the

date the tax was actually deducted to the date the tax was actually paid.

Page 288: INTER CA Taxation - CA Study Web€¦ · INTER CA Taxation Head Office Shraddha, 4th Floor, ... 14. Advance Tax & Interest 268 - 271 15. Tax Deducted at Source (TDS) 272 - 294 16

J.K.SHAH CLASSES INTER C.A. – DIRECT TAX

: 285 :

In case TDS is not deducted by the payer but as per section 191, tax is paid by the

resident receipient on such income, then interest will be levied for the period from the

date tax was deductible to the date of filing return of income by the receipient. The rate

of interest will be 1% p.m. or part thereof.

* Penalty - Levy of penalty equal to the amount of TDS / TCS not deducted /

collected u/s. 271C & u/s. 271CA.

* Rigorous imprisonment - Rigorous imprisonment for a term not less than 3 months

but extending upto 7 years, with fine for failure to pay to the credit of the Central

Government the tax deducted / collected at source, u/s. 276B.

Penalty and rigorous imprisonment can be waived if the defaulter proves that there

was reasonable cause for such failure.

REQUIREMENT TO FURNISH PERMANENT ACCOUNT NUMBER :- SEC 206AA

1) Any person entitled to receive any sum or income or amount, on which tax is

deductible shall furnish his PAN to the person responsible for deducting such tax,

failing which tax shall be deducted at the higher of the following rates, namely:-

(i) At the rate specified in the relevant provision of this Act; or

(ii) At the rate or rates in force; or

(iii) At the rate of 20%

2) Form 15G shall not be valid unless the person furnishes his PAN in such declaration

3) In case any declaration becomes invalid under sub-section (2), the deductor shall

deduct the tax at source in accordance with sub-section (1)

4) No certificate shall be granted on the basis of form 13 unless the application

contains the PAN of the applicant

5) The deductee shall furnish his PAN to the deductor and both shall indicate the same

in all the correspondence, bills, vouchers and other documents which are sent to

each other.

6) Where the PAN provided to the deductor is invalid or does not belong to the

deductee. It shall be deemed that the deductee has not furnished his PAN to the

deductor and the provisions of sub-section (1) shall apply accordingly.

7) The provisions of this section shall not apply in respect of payment of interest on

long-term bonds as referred to in section 194LC, to a foreign company or any

other non-resident person.

Page 289: INTER CA Taxation - CA Study Web€¦ · INTER CA Taxation Head Office Shraddha, 4th Floor, ... 14. Advance Tax & Interest 268 - 271 15. Tax Deducted at Source (TDS) 272 - 294 16

J.K.SHAH CLASSES INTER C.A. – DIRECT TAX

: 286 :

Tax Collection at Source

PROFITS AND GAINS FROM THE BUSINESS OF TRADING IN ALCOHOLIC LIQUOR, FOREST PRODUCE, SCRAP, ETC.:- SEC 206C

1) Every seller shall, at the time of debiting of the amount payable to the account of the

buyer or at the time of receipt of such amount whichever is earlier, collect from the

buyer of following goods, a sum equal to the specified percentage of such amount

as income-tax.

Table

SI.No.

Nature of Goods Percentage

Category A: Sale of the following

i. Alcoholic liquor for human consumption 1%

ii. Tendu leaves 5%

iii. Timber obtained under a forest lease 2.5%

iv. Timber obtained by nay mode other than under a forest lease

2.5%

v. Any other forest produce not being timber or tendu leaves

2.5%

vi. Scrap 1%

vii. Minerals, being coal or lignite or iron ore 1%

Category B: Grant of Lease/License of the following

i. Parking lot 2%

ii. Toll plaza 2%

iii. Mining and quarrying 2%

Category C: Sale of the following where any amount of consideration is received in Cash.

i. Bullion > 2,00,000/- 1%

ii. Jewellery > Rs.5,00,000/- 1%

Category D: Sale of the following whether payment is received by Cheque or by any mode

i. Motor Vehicle of the value exceeding Rs. 10 Lakh 1%

(1A) Exception: No collection of tax shall be made in the case of a resident buyer, if he

furnishes to the person responsible for collecting tax, a declaration in duplicate in Form

27C to the effect that the goods are to be utilized for the purpose of manufacturing,

processing or producing articles or things or for the purposes of generation of power

and not for trading purposes.

(1B) The Seller shall deliver to the CCIT or CIT one copy of such declaration on or

before the 7th day of the month next following the month in which the declaration is

furnished to him.

Page 290: INTER CA Taxation - CA Study Web€¦ · INTER CA Taxation Head Office Shraddha, 4th Floor, ... 14. Advance Tax & Interest 268 - 271 15. Tax Deducted at Source (TDS) 272 - 294 16

J.K.SHAH CLASSES INTER C.A. – DIRECT TAX

: 287 :

(1C) Every lessor of parking lot, toll plaza or mine or quary, shall at the time of debiting of the amount payable to the account of lessee or at the time of receipt of such amount from lessee in cash or by the issue of a cheque or draft or by any other mode, whichever is earlier, collect a sum equal to the prescribed percentage, of such amount as income-tax.

Explanation : - “mining and quarrying” shall not include mining and quarrying of mineral oil (including petroleum and natural gas) (1D) Every seller, who receives any consideration in cash for sale of –

� Bullion > 2,00,000/- or

� Jewellery > Rs.5,00,000/-Shall, at the time of receipt of such amount in cash, collect from the buyer, a sum = 1% of sale consideration as income tax.

Time for deposit of Tax collected at source – same as TDS Time for issue of TCS certificate – same as TDS Time for filling quarterly statement in form 27Q – same as TDS In case of an office of the Government, where the amount collected has been paid to the credit of the Central Government without the production of a challan, the Pay and Accounts Officer or the Treasury Officer or the Cheque Drawing and Disbursing Officer or any other person, by whatever name called, who is responsible for crediting such tax to the credit of the Central Government, shall deliver or cause to be delivered to the prescribed income-tax authority, or to the person authorized by such authority, a statement in such form, verified in such manner, setting forth such particulars and within such time as may be prescribed.

Assessee in default – same as TDS Exception : Provided that any person, other than referred to in sub-section (1D), shall not be deemed to be an assessee in default in respect of such tax if such buyer or license or lessee –

i) Has furnished his ROI u/s 139; ii) Has taken into account such amount for computing his income; and iii) Has paid the tax due on such income

And the person furnishes a certificate to this effect from CA in prescribed form:

Interest on Delay in deposit of TCS : simple interest @ 1% per month or part

thereof on the amount of such tax from the date on which such tax was collectible to the date on which the tax was actually paid, and such interest shall be paid before furnishing the quarterly statement.

Provided that in case any person, other than referred to in sub-section (1D), who is not deemed to be an assessee in default, the interest shall be payable from the date on which such tax was collectible to the date of furnishing of return of income by such buyer or license or lessee.

Page 291: INTER CA Taxation - CA Study Web€¦ · INTER CA Taxation Head Office Shraddha, 4th Floor, ... 14. Advance Tax & Interest 268 - 271 15. Tax Deducted at Source (TDS) 272 - 294 16

J.K.SHAH CLASSES INTER C.A. – DIRECT TAX

: 288 :

Lesser or no collection of Tax at source : Where the AO is satisfied that the total income of the buyer or lessee justifies the collection of the tax at any lower rate than the relevant rate, he shall, on an application [form 13] made by the buyer or lessee in this behalf, give to him a certificate for collection of tax at such lower rate.

Explanation – For the purposes of this section, -

(a) “Buyer” with respect to – (i) Sub-section (1) means a person who obtains in any sale, by way of auction,

tender or any other mode, goods of the nature specified in the Table in sub-section (1) or the right to receive any such goods but does not include, - (B) a public sector company, the Central Government, a State Government, and an embassy, a High Commission, legation, commission, consulate and the trade representation, of a foreign State and a club; or (C) a buyer in the retail sale of such goods purchased by him for personal consumption,

(ii) sub-section (1D) means a person who obtains in any sale, goods of the nature specified in the said sub-seciton

(b) “Scrap” means a waste and scrap from the manufacture or mechanical working of

materials, which is definitely not usable as such because of breakage, cutting up, wear and other reasons;

(c) “Seller” means the Central Government, a State Government or any local authority or corporation or authority established by or under a Central, State or Provincial Act, or nay company or firm or co-operative society and also includes an individual or a Hindu undivided family whose total sales, gross receipts or turnover from the business or profession carried on by him exceed the monetary limits specified under clause (a) or clause (b) of section 44AB during the financial year immediately preceding the financial year in which the goods of the nature specified in the Table in sub-section (1) are sold.

Other Sections

Bar against direct demand on assessee – sec 205

Where the tax deductible at source under the provisions of section 192 to 196D, the assessee shall not be called upon to pay the tax himself to the extent to which tax has been deducted from that income.

Where the payer, who has deducted tax at source, has not paid tax to the credit of the CG only such payer can be considered as an assessee in default in respect of the amount so deducted and the department cannot recover the amount from the payee.

Tax deducted is income received [Section 198] (1) All sums deducted in accordance with the foregoing provisions shall, for the

purpose of computing the income of an assessee, be deemed to be income received.

(2) However, the tax paid by an employer under sub-section (1A) of section 192 on nonmonetary perquisites provided to the employees, shall not be deemed to be income received by the assessee.

Page 292: INTER CA Taxation - CA Study Web€¦ · INTER CA Taxation Head Office Shraddha, 4th Floor, ... 14. Advance Tax & Interest 268 - 271 15. Tax Deducted at Source (TDS) 272 - 294 16

J.K.SHAH CLASSES INTER C.A. – DIRECT TAX

: 289 :

Credit for tax deducted at source [Section 199]

(1) Tax deducted at source in accordance with the above provisions and paid to the

credit of the Central Government shall be treated as payment of tax on behalf of

the-

(i) person from whose income the deduction was made; or

(ii) owner of the security; or

(iii) depositor; or

(iv) owner of property; or

(v) unit-holder; or

(vi) shareholder.

(2) Any sum referred to in sub-section (1A) of section 192 and paid to the Central

Government, shall be treated as the tax paid on behalf of the person in respect of

whose income, such payment of tax has been made.

(3) The CBDT is empowered to frame rules for the purpose of giving credit in respect

of tax deducted or tax paid under Chapter XVII. The CBDT also has the power to

make rules for giving credit to a person other than the persons mentioned in (1)

and (2) above. Further, the CBDT can specify the assessment year for which such

credit may be given.

Page 293: INTER CA Taxation - CA Study Web€¦ · INTER CA Taxation Head Office Shraddha, 4th Floor, ... 14. Advance Tax & Interest 268 - 271 15. Tax Deducted at Source (TDS) 272 - 294 16

�������

���������������� �������� ��������� ��

������������������������

����� ������������ ������������ ����������������������������������� ����

������������������������������������������������ ��������� �!

"�����#���� ��������$�%�����������

&������'������� $�(����"�����#����

"���� ��������)�%���

&������������������� ��������� ����*����������������� ����!

+������������,��� ����������%���

,,- ��������.�%���

&�����������,����������/��0��������1����-��� ���������.%���

2�������������������3�������4������������������������������� �������������%���

����������������������������������%�����������4 ����!

+���������������������,����� ��������.�%���

5����������������� �6�����7 �������� �%���

+��������4������ ���������.%���

1� ��� ����� ��������8�%����69&#����������������8%���7�

������ &���:�������������������������������,�����������������������������������������������

��� ���������;���������������������������.���*����9���������������������� ��������

;������������� ������*� �����4��������� ���*����&������,�<�������� ����*��������������

�� �������������������������������������������������!

�������� ������������������ �������������� �� ������!

�� �������� ������!���!

' ����= �������>�����

����������������� ����������$.%���

" ����= �����������������

>����������������������� ��������%.�%���

2 ����< 2������������������������� ���� %���

& ����< 2������������������������� �����%���

? ����< 2������������������������� ���.�%���

?3�������4���� ������������3��@���)$2A

���"� '"2�+�������*������4�������4������������B%��������������%�����������������)����������

����������� ��%�������������������*�������������� �����B�������������������������������

��������)%��������'"2�+�������������������������������������B�������4������

4���������������������������� ��9���������4�������������������������������� ����

���������� ��������� �������������������%$�%�������������.$%������������ ����������

�����������������B��������4��*����������������;���;���� ��?3��������4������������'"2

+���������������3��������������������������������������������

���#� �����%��������������%����������������4������������������ ��������������.%�����9�������

���������3��@���)$�����������������������%�4����������������������3���������������

���2������C����������������������������������������'�<����� ��)����������

��������������������������������4��������������������4��������3�������������

������������*����������������3�����������������4 �����������9��'�������

D���������������������������� ������3�������3���������3�����������������������

'��������D��������������������������A

�$ ���%&'� (&)$*�

Page 294: INTER CA Taxation - CA Study Web€¦ · INTER CA Taxation Head Office Shraddha, 4th Floor, ... 14. Advance Tax & Interest 268 - 271 15. Tax Deducted at Source (TDS) 272 - 294 16

�������

���������������� �������� ��������� ��

������������������������

���+����=��������������������� ����"���������������������������������������������������

���������������������8����*�����������.���*�%��������� %��������<������ ��������

��������������������������� ������������������� ������8���������$����*�����

���������*�%��������� ��9���������� ��������������������������%����%��� �����

��� ���� .� ��*�� ���� ������� ��*�� ���� ��� ����� ������ � ���� ������ ������������� ����%

�������� ��&����������3��������������������������������=��������<�������

9&#������������%������ %�������������������<%��������������4�������=��������<

������������������

���,� 2�������������������3������������������������������������ �������4

���������������� �����������

��� ���� (����� -���

���

� ��������� ,� ������������%��%�����������E����%�������������%����������

������������������������������������� �������������������

,'0��������� ��

� ��������� ,� ���������������������������.%��������1� ��� ��� ����������%���

�������#�����������������������,'0�

; ;��8����� ,� ������������;8%��������@���=<F�,������+�������������� ��

4��������

$ ������� ,� �������������%��%��������������1�������������������

�����������������������������������������������E�����

���������������������������������������������4 �����E����

�������1����

. ������� ,� ������������������������.%�������4 �����:�������

����'��� �

8 �������� ,� �����������������������������&"��"��*������;8%.��@�

� ;��������� ,� ������������.�%��%�����������'��� ��#��������%���

��������������2?D�������&�����#�������������������+�����

����#����������������: ���������������� �

��� .��?3���� ��� 9&#� ������������ ����� ������� �)$'� ��� ��� ����� ������

�������G

�7 D�����������%������������������3��������3������������1�������*��H

)(�����������'"2�2����������"��*��9����3�����������������

;��;���� �

��7 D����8�����%����C�����������������������3�������������1������*�

�������� �����������H�)(������&���*��"�����%�B���*�����"���������

1������"����������=<F�"��*%���4��*������������������2"#��9����3�

����������������� ������ �

���7 D����$�����%����1�������������� ����������������������1����%������

�����H (� ����� ����� ,I1� "��*�� 9�� ��������� ������� ������ ��

;��;���� �

���/� #��������������4���� ����9&#����������������9&#���������������������������!

6�7 1������������������������� �4 �"�+�����������1�������������%��%����

647 -���������&���#���������4 �#������6�J-7���������;.%������������� �������

�����4������������ �

6�7 �'"2�����2���+������������������)%�����������������&������������������������

�������� �

Page 295: INTER CA Taxation - CA Study Web€¦ · INTER CA Taxation Head Office Shraddha, 4th Floor, ... 14. Advance Tax & Interest 268 - 271 15. Tax Deducted at Source (TDS) 272 - 294 16

�������

���������������� �������� ��������� ��

������������������������

���� ��&��� :������� ��� �� K�������� �����L� ����� �� ��� ������ 4 � ��3� ������ ���

,�<�����8������6������������������ ��7��9��9&#��������������@����)$2��������� �

�������4 �����������,�<���������� ��������������!

6�7 2��������'�!�#����������������� �������������=�3����������;�%���%����3

������4��������H��(���������������������$.%���@�

647 2��������"�!�0��9&#����������������������������������������

6�7 2��������2�M�&�!�9���� �������������������������������3��������;�%���

��������������� �������4�������������������������3���������%��%�����9�����

��3����������4���������������������*������ ��������������������������

6�7 2��������?�! 9���� ���������������������3���������;�%�������������������

����������������������������������<�3����������%��%�����'��������� ���3�H

�(��������������������������������������������4��>��������4��������

����"��9���������4��������������������������%$�%���������������������������� �����8

�����������.$%��������4����������������BN�������������;��;���� ��#������������

��������������������3����������% �%���%�9&#��@����)$���������4�������������

�������!

� D����������"�������!��%$�%����3���( ������$%���

� D������������������������ !�.$%����3��( ������%� �

9���� ������.%� �

9�������������9&#�������4���������4 �'"2�+������������2������C������������

4����;����'����%���� �������������������@������6�7�

����+�

6�7 ��0����-�������������1��1��!�����*������'������������.�2%������������ �����

�������������� �6������������� .���*�7�������4�������4�����������������������������

���������������������������� ��9�����%������$.���*��6���������� .���*��G������$����*�%�4���

���������������7�������4���3�4����������������������������������'�<�� ��)�

#�������������������������������������������������%���������������������������������

��������������3�4���������������������=�

6��7 ��0����-�������������1��1��!�����*��2���������������4�������� ���������

��������>����������������%������������4���������������������������

�������������������4���3�4��������������.86�76���7%�����������������3���

����.�%������9�����%���������������������.���*��6O� .���*��G��������8����*�7�������4���3�4�

�������������������<�������������.86�76���7���#��������������������������������������

����%��������������������������.86�76���7�������������������������������������

���������������������������>����������������%����������������������K������ L�����

��������.86�76���7������������ �������������������������������

�$ ���%&'� �&$3��&��

Page 296: INTER CA Taxation - CA Study Web€¦ · INTER CA Taxation Head Office Shraddha, 4th Floor, ... 14. Advance Tax & Interest 268 - 271 15. Tax Deducted at Source (TDS) 272 - 294 16

����"��

���������������� �������� ��������� ��

������������������������

6���7 �������-�������������1��1��!�����*��2����#�������������������������������

������ �3���������.����*�%����<�����>����������������3���������������������

�)$��'��9����3����4����������������������)$��'�������4������8�%���%�4�����(

���O�8����*����9&#�������������������������)$��'����������������������������

������������������������������������

����,��� 0����3�����>��������4������������������������������)$2�4 ������ �������

�� �������������E����%������������������������������������������!�

6�7 ������������������������������������� ���������������������� ��P

647 �����������������4������������ ���%���������������������������������

6�7 ��������4���������,�'�0���������� ��

�� 0����3�����>��������4������������������������������)$B���������K����������L

�� ��������� ��� ������������������������3��������;�%���@�

;� 9�3�����>��������4������������������������������)$2�H��(����������;8%�������

�������������������� �������������������4��������3���������;�%���@��

$� '��������������������)$2%�����������������K���*L�������������������������������

�������� �����������������������������������������4 �������%�����������������

�����������������������������������������������������������%�������%

����E�������������>����������������3��������� �������������1����%�����

����������������������K���L�����������K���*����������L

.� 0����3�����>��������4������������������������������)$������������ ��

�����������3���������.%���@�

8� 0����3�����>��������4������������������������������)$'���������������

������������4��*��������� �

�� '�������������)$&'%���3�������4������������(��������� �������������

��������������������������������������������3�4������������������������

�������3�'����'��������� %������������%����9&#�������4��(���������.�%��%�������

������%��%���@�

����.��� '"2�2����������"��*����������������3����������H���(����������������

1��$.%����6)(�3�1�������*��3�Q7��������������)$'��9����3�������4����������

�������������)$'�������������������%�������%�1��$.��

��� =<F�"��*����������������3����������H���(��������������)$'%��������

��������������������3���������������������4�������������4��*�����������%�.�

R�%��%����3�;�3�)(�3�)@��S%�������3������������������������1����%�����#���

=<F�4��*������������2"#%��������������������������@������4 �����4������

�������4�����������#��������������������������������%�.��3���������������

��������1����%������3��������4��������H���(��������������)$'�

Page 297: INTER CA Taxation - CA Study Web€¦ · INTER CA Taxation Head Office Shraddha, 4th Floor, ... 14. Advance Tax & Interest 268 - 271 15. Tax Deducted at Source (TDS) 272 - 294 16

����#��

���������������� �������� ��������� ��

������������������������

���� 9�3��������4���������������������)$'�4 �,I1�"��*������������������������%$��

���������������������������������;��;������������1����%������G

6�7 K���������������L�����4�����������������������������K���������LP����

6�7 #�����������3������������������������1����%���

����/�6�7 #��������������������������������� �4 �"��+�����������1����3����������% �%���%

�������������������������)$���������������������3�����������������������9�����

�������4��������������������3�����������������������)$�������������������������

���������������� �����(������������������1���������������������������

����������4�����"�+���

9�����%���������������3����4����������������!�T�������%��%����3��(�T������$����

0��!������������1�����������������������������������������4�����"�+���%

��3�������4��������H���(����������%��%���%�4 ���������������������������������8''�

647 '������������������������������)$B%���������J��������-��� �����>�������

��������3����������������������������������������������� ������������4���������3

������������������$$'"����������������� ������������������������������� ���

�����%����������� ��������������������������������3������� ���������������

������������ �4�����������J��������-��� %����%�������4���� ������������3���

�������������

9�����%���������������%�������#������6�J-7�������4�������3������������

������ ������������������� ��%���� ���4���� ������������3����������������

�������������������������%��������������������������������&���#����������������

��������������������������������� ������4������������ �

6�7 #�������)$B���������������������������3����������H��(������� ���������4

�� ������ ����������������������������%�4 ����������������%������������

�������%���������������������������������� ������������3����������4�������������

�)���9������������������������;�%����������������������������������3������������

������������������������������������ �������� ��������������������

�)$B���%������%������������4�����������������������������������

9�����%���3H��(��������4������������������������������)$B������������

���������������)%���������4 �'"2�+�������������������

Page 298: INTER CA Taxation - CA Study Web€¦ · INTER CA Taxation Head Office Shraddha, 4th Floor, ... 14. Advance Tax & Interest 268 - 271 15. Tax Deducted at Source (TDS) 272 - 294 16

�������

���������������� ������� ���������� ��

���� �������� ����

�����������

������������ ��������� �������� ������ ����������� ���������� ���������������

���� ��������������� ������������ ������ ��

����� � ������������������������������� ����������

����� ! �"#���$%"#&

����� ���������������� ������������

����� ��������� � � ����������������������� �� ����� �!�"��#� � ���������

����� ���������������� ���� �� ����� ��������� $ ���������$��%��

�!�"����������� ��$�� � ���

������ %��� ������������������ ����������&

%�� �� �'��� �����"������$ ��� ���� �������� ���������� ����������

����� ��� �� ���� � � �� ��� � � ����� �������'�"���%���'��� ����� ���

��� �� ��������������� ������ ������ ���� $ ���������$��%�� ��

'�"���%����������� ��$�� � ���

���(� �� �������� � ����' ��

���)��� %�������������������� ����� ��&

* ��� ����� � � � �� ��� �� � ����� ��� ��� ���� �� ��� ��� �������� ���

����������� ����������������������� ����� ���� �� �� � ���������

��� ����� ��������� �� ��������������������� � ������������� � $ �������

�$������� ������������������ �� �+����� ���� �� � ��������������

������ �� �������� � ����' ��

������� �� �������� � ����' ��

�������� �� �������� � ����' ��

�����,� �� �������� � ����' ��

�����-� �� �������� � ����' ��

�����--� �� �������� � ����' ��

�����.� ������ � � � �� ��� /� � �������� � '����� �� ������� $ ��� ����� �$

���������������� � �����

��������� � �!��(��) %% *�!+��"&�&,��(��"- $!��#����.�/�$!/�#��(����)���!&$#"!��

� %��0��&��"1"2%��

��� ��������� � � ���0���1�..23* � ������ ����42���

���� ���������� � � ����� ����5 ������������� �'����2�3* � ������ ���4��

����� ��������� � � ����� ������������ ���������� �� ! #�2�) #��3���3�����

��� �� ����������� �4#�-�$-�4"0"2%�������������� � �����������

������� ��������1���/&���5� )��(���"4��"%�&$-�"&&$#�/6� #

��.� ��������� � � ����� ������������ ��������&&$�/� !� #�")��#���7�����

��� �� ����������� �4#�-�$-�4"0"2%�������������� �� �����������

������� ��������1���/&���5� )��(���"4��"%�&$-�"&&$#�/�

����������������

Page 299: INTER CA Taxation - CA Study Web€¦ · INTER CA Taxation Head Office Shraddha, 4th Floor, ... 14. Advance Tax & Interest 268 - 271 15. Tax Deducted at Source (TDS) 272 - 294 16

����8��

���������������� ������� ���������� ��

���� �������� ����

�.� ��������� � � ����� ������������ ���������� �� !� #�")��#���7����3�

�� � � � ������� �� ���� ������� ���� �� ���� � ���� �� ����� ���� �

� � �� ��� ���� � ������ ���� �� ���� ���� ����� �������� ��� �� �

� ����1�!������ �� �� ��� ������ ����1�..,����� �� ���������

� ��� ���������� ������������� �� ����������� � ������� ������

$� ����(6����� ���������������� ��

�� � ��� � $ ���������� �������� �������������� � � ������ �� �������

� �������� �������� ����� ����� � ����� �������������������������� �

��������������� ����������������7-�������������� �7�� ���������

�������� ������ � ��

9 8� ����� ���������� �������� ����� �� ��� ����

9 � ����� ��������� � ����������������������� ��� ��� ���������� ��

����������������� ������������� ��������� �� � � ����� ��� ������

�������� �����

�5 ������������ �������� ������/� ��������� �'������9 ������

� ��������� ��� �������� ��� ������������������� � ����� ���

� � ����� ��� ��� �������� ������������� � �� ���������� �

����� � ������ ����� ������� ������ ��� ��� �����������:

���� ����������������������� ������� ������� �������������

������� � ������� ���������������������������� ������

�������� ���� ���� �� ����� ��� ��������� � �� �� ��������� ���� � � %�9����

%����������������� ������;���� ���� ��� �<�� �� ���%������,��9���

�1=>���������� $ ���

������?������ �� �������� � ����' ��

����>� �� �������� � ����' ��

����>�� �� �������� � ����' ��

����;� �� �������� � ����' ��

����(� �� � ������ ��� ��&:

�� $ ���������$�������� ��� ��� ��

�� � ������ ������������ $ ���������$�&

9 �� � �������� ��� ��� ������������� ���� ��������� &���:� ���

@������%�����������- ���� �2�@������. � �� �<����,��������A1�2

%� ����, �� ��,�������AA�2�8� ������%������. �����- ���� �2�'��

B��� �-����- ���� ���(�� ����2�@������'����- ���� ������� ����2

@������'����%�����- ���� ������� ����2�'���B��� �@������%��:

����- ���� ������� ���0�=�� ����2�'���B��� �%�����,��9���������:

�����2�'���������������'���B��� �%�������������*�� ���� � �

���� �(�����2�'����B��� �-8.2�"$ ��. ����3<�� ��� ��%�����

- ���� ���"$ ��. �����*�� ���A)1����'���B��� ��"$ ��. ����

��� �� �A)142� %� ���� . ���� %�� � �� �A1�2� ���� @��:* �� �

�@��:� ������ ��*�� �. ����%�� � �

9 �� � �����=6�-�������� �� ��,�������� ������������������

���� ���� !���� �� "����� ����� �� � �� �� �� ��� � � - ����

<�� ��� ��� ��� ���)�������������� ����

Page 300: INTER CA Taxation - CA Study Web€¦ · INTER CA Taxation Head Office Shraddha, 4th Floor, ... 14. Advance Tax & Interest 268 - 271 15. Tax Deducted at Source (TDS) 272 - 294 16

����:��

���������������� ������� ���������� ��

���� �������� ����

9 �� � �������6�* � ��,��������� ���� ������������������ ���

����� ��������

9 �� � �� ������ � �� ���� ��� ��� ���9� � �������� � ����� ���9��

������������ ������

9 �� � �������� ��������0�� � ��� �������������� ������������

9 �� � ������ ��� ��� ������� �# ���� �-������� ���,������<��

C�����,����������� � ������ ���������� �� � ������ ��������

� �,����������� �9����� ��� ��������������������� �* � �� �,��9

����������������������� �������9������ �- �����<�� ��� ���������

����������� �B������<�D ���� ���������� �����

9 �� � �� ��� <���� . ���� ,����� ��� �� ��� �� � � <���� . ���

%�� � ��EAA���� ������ �- �����<�������� ����� ���� ����� �

��� ��<����F�� �����%�� � �����(�

����)� G�������%����������&

%�������������� ����� �� �������� ��������� $ ���������$�

����=� .������������ ����F �� ������'����� ��&

�� $ ���������$���� �� ������ ������������������ ��

��� ������������� �� � � ���������F �� �����'��� �� ��������

%� �-��� �� � ����

��� ���� �������� � � � � �� ��� �� F �� �� ��� '����� �� ��� �� �

F �� ������'����� ���-���� ������������ ��*�� ����A1)����

��� ��������� ������������ �� � � ���������� ��������� ����������

� �� ������������%� �/ ������ ���� ��������������� ����� ���

���%� �/ ������ �

����=�� �������&

8� �������������������� � ������������������9������ � $ ���������$�

9 �������� ����� ����������� ����������������� ����� ������

� � ������ �- �����<�� ��� ���������%� �<�� ��� �������� �

��������� ����������������� ������ �- �����<�� ��� ��

9 �������� ����� ����� ��������� �- �����<�� ��� ���������%�

<�� ��� �� ���� ����� ������ �� ������� � � ������ ����� � �- ����

<�� ��� �������� �������� �������� � ��

����1� �������� �����������:

�� � ������ � � ������������������������� ����� �� ��� ����� �- ����

���%� �<�� ��� ����������� ������� ��7'�����C��-��9��7����7F���

C��-��9��7����7C��-��9��7����������� ��������������������� �- ����

<�� ��� ��������������������� �B������<�D ���� ���������� ����2

�� ������� ������ � � ���������� �� ������ ��������������������

� � �� ���������:����� ����

����A� "������ ������ � � ������ �������������� ���������� ��� �������� ����

����� �������� �� ������ ���� ������ ����������������:����������� �������

�������� � �� �� ������������ �� ������������� ����� ������ ������ ������

�� ������������������� ���������+ �������������������������� ��� ���� ��

����A�� "��� ����� ������������%� ��&

�������C��� ���������� ������ ����� ������������������ ����� $ ���������$�

Page 301: INTER CA Taxation - CA Study Web€¦ · INTER CA Taxation Head Office Shraddha, 4th Floor, ... 14. Advance Tax & Interest 268 - 271 15. Tax Deducted at Source (TDS) 272 - 294 16

����;��

���������������� ������� ���������� ��

���� �������� ����

������ ����� ����/�������������&

!��#������ ��������������������� $ ���������$� $� ������ ��������

����� ��� ��� ����� �� �� �������� ������������������������� ��� ���� �

������ ����� � ���������� �������+������������ ��

����>��,� �������� ������������������ � ����� ����� ���� �������/���-���������� ����1�A)

��������� ������ ����� ����� �������� � �&

�� 8��������������������� ��������� ���������� ������� ����� � ���

� �����������������������2

�� #������������ ������ �-������ ������������� ����� ��������� �* �������

����� � ���� ��������� ������ ���0��>���������*�.���������AAA

������� � $ ���������$�

����>.� ����� ����F�����"���

����� � ��� ��� ��F����� "����� � � ��� ��� ������ � ���� ���9�� �� ���9

����� �� �� � � �� ����� ��� 7�� �� ������ � ���� ���9�7� ��� � � *�,���� ��

������������������������ ������ �% ��� ������G$����� �,�������

*�,������ $ ���������$�

����;� ����� ����8��� �!�����&

�������� ������� ��� ���� ��� �� ����7����� ���������� ����� ��7����

����� �������� ������� �7�����&

��� ��� �� � �������� �� � ����� ��� �������8��� �!����������A�)

��)�����A�)������ ��������������� ������� ����� �������� �� �����

� � �����9� ������ ����� ������� � �����9� ���������9� ���

$ ���������$�

��� ���������������� �� � ��������� � �� ���������:����� ��������� �

����(� ����� ����'���� ��"�����&

������������� ���� � $ ���������$���� ���������� �&:

��� � � ������ ��� ��� �������������������� ��������������������

����������������� ���� �� $����� ��������� ����������� ��� ��

��� �������� �� � � ������ ���� ������ ����������� ����� ������� �

��������������� ����� ��������������������������� �������������

��� �������� �� � � ������ �,��������8��� ������ ��������. ����:

��9 ���������� �"����

�����)���� ��������� ����������� �������� ����������������� �����%99� � �

��� �������������� ���� ��� ����%99��2���

��� ������������� ������� � ������ ��� ��

'���� ���������������� ������������ �����������������%99� � �������

�������� ������������������������%99� � ���������� ��������������1�

"���� ������� ������������� ��7%99� � 7�������� ���:

��� ��������������� ���� ���� ���� ����� �� �� ������� ����� ��%99�

%��+ ���* ����������A)�

��� ������ ��������������� ���� ��� �������� ������ �� �� ����

��� ������ ��� �������������� ���� ����������������E����� �

���������������� ������������� ��������� �������� ���� ���� �

����� ��� ���� ����� �� �� ��

Page 302: INTER CA Taxation - CA Study Web€¦ · INTER CA Taxation Head Office Shraddha, 4th Floor, ... 14. Advance Tax & Interest 268 - 271 15. Tax Deducted at Source (TDS) 272 - 294 16

�������

���������������� ������� ���������� ��

���� �������� ����

���>�� ����� ����F����&

������ �� ����� �������������������� ��� ����� ������������������

�������� ����);�������������������������� � �� ���� $ �������

��$�����������(��0:���� �� ����� ��������������

���>>� �������� ������������� ������ ���������������� ����������:� ��������

�������������:� ������������������� ������������ �!��%�� � ���A);

� � �� ��������� ��� ������ �!��8�������������8����� �������� ��9������

* � �������������������� � �� ������ ������������� ���9 ������ ��������� �

��;������� ��#�G�"����� ��� ��H ������>:�;�

�%�"&��! ����

I .� ������J�����:������

I .� ��������� ��:��������� ����;�����

I !���������� �� ����������������� ��������������9������ �

���>;� �������� �������������� ����� � �� ������� ������(:B�

�� � ����������������� �!�"���������� �� ����������� � � ����� ���

�������� ��������� �� $� ����*������������0:���� �'�H���� ��� �������

� ����� ����������� ��$����� ���� ���� � � ����� ��� ������6��8����$

����� ������������ �.�� ���.�������8�$��..8�������� ����� ���� �:

�������� ������������� ���

���>;�� �������� ��������������� �� �� ��������� ���� �����������������������9

������� �������� ����� �������� ����� �����9� $����� ������ �����������

� � �� ������� ������(K���G�� �� �������:):���>��

��������< ��=�"1� !���&�#�2$��/��!� -�=�

8� ���� � �� �� � ��� ��������9���� �������� ���� ����� ����� ����

���� �����7.���� ������� &7�8� ���������������� ����� ��������������

���� :�$�>���5����� ������ ������ �������� ����� ���� ����������:

����� �L����6����� �������� ���L�>�6�8� �������������� :�$�������

���� ���������&("%%�2���(��� �/�.�/�!/�/�&�#�2$�� !��"1��8� ����� ������

��� �&("#�( %/�#&���� �� �����������������9����� ���������������� ���1�

�-4��$!/�#�&���� !����37 ����� �� ��������������� �������� ��������

���� :�$����� ����:���9�������� ��

���>(� �������� �����������&

��� ���� �� � � ����� �� ������ ����������F�����"������ �� �

��� ������>.��2���

��� ���� � � � � �� �� � �� �� ��� ���� ����� � ����������� ��� �

�� �� ����� ��9���2���

��� ���� �� � � ����� �� ��������������� ��� �� ����������

'� �� ��� ������� $ ������������ ����� �����������������

�������������� ������ �����

���>)� �������� ������������� ������ �����������: ����"4��"%�"&&����� ��

J������� ���������������� ������� ����� �����9� $����� �������

��������������������� ����>����>����� ��� ���>����;��� ������A�������;�

�%�"&��! �����

I 8� � ����� ��������� ���� �� ��� ������ ���� �� ������ ��� �

�� ���� $ �����

I 8� � $ ��������������� ������ �������� ���� �� ���������9������ �

Page 303: INTER CA Taxation - CA Study Web€¦ · INTER CA Taxation Head Office Shraddha, 4th Floor, ... 14. Advance Tax & Interest 268 - 271 15. Tax Deducted at Source (TDS) 272 - 294 16

��3����

���������������� ������� ���������� ��

���� �������� ����

���>=� ���� ���� ���������� �� ��� ������������������� ��������� ��������

�������� �������:� �����������:� ��������� ��� ���� ��� �� ���7-����

����7������������� ������ ������������������������ � �:

�� ��������������������� ���������� ��� � �� ������� �������;��

�� ������������������ �� ����������� ������ �� ����� � �����

�� ��������� �������� ����� ����������������������� ���������

��������� ��������������������������� ��������2

�� ���������� ���������������������������J�������� ���������������

����� ��� ������ ����������������� ��� ����������� ������

- �����<�� ��� ������ �* � �� �,��9���������2

��� ��������� �������� �������� ����� �������������� �������������

����� ��� � � ������������� �� � !� #�")��#���7����7�

��� �� �� ������ � � � � �� ��� $ ����� ������ � � ������� � ���

���� ������ � � � ���� ����;����;�� �� �� �� � ��� � ��� � ��

����� ��� � �� � �� �� � � ��������� ����������� ��J�� �� � ����� �

��;����;������������ ������0���� ����� ������ ����� � � �

�������� ����;����;������������� � $ ������������ ��$�

14%"!"�� !���"���� ������� ������������� ��� � $�� �����7���� �����

��� ����� ����7� ����� �� � ����� ��������� ����� ����� ����� ����

���� �� ����� ��������������������������� ���������

����� !����3: �

�������� ������ ��� ���� ��� �� ���M-���������N���� �� ����

����� ��������� �� ����������� ��������������� �� ��� �������������

����� ��������� ������������������ ���� ������������ �� �����������

������� ������������O�� �����;���������� �������� �� ����������� ���� �

� �/����'������%�� � ��� � ��� � �� �������M� ���� � N����� � ����� �

� ��������'��� ���-�����-��/����'������%�� � ��"����������������� :

� ������*�� ������(���� ���� ��� �������������� ��������'��� ���-��:

���* ����. � ���� ������������� ���;����� � � ��� ��� � �����������

%�� � ����� ���� ��� ��������

G$��������P"���� ������� ������������� ��M�� �� ����������� N

� ����P

��� � �������������������������� ������ ���� �� �������� �����������

O�� �����;��������������� ������� �� ����� ��� ���� � 2���

��� � ���������������� ������ ���� ���� ����� ��� ���� � ����

��� �� ���� �� ���������������������������� � �� ����������� ����2���

��� �� �� ����� ������������������� ���� ������ ��� � � ������ ���:

� �� ���� ���������������������������� � �� ����������� ���������:

������� ����� ���� � �������������������������� ���� ������ ����

� � � ��������� �� ���������� ���������� � ������� ���������� ��

�������� ����� ��������������������������������� ���� ������2Q2

Page 304: INTER CA Taxation - CA Study Web€¦ · INTER CA Taxation Head Office Shraddha, 4th Floor, ... 14. Advance Tax & Interest 268 - 271 15. Tax Deducted at Source (TDS) 272 - 294 16

��3����

���������������� ������� ���������� ��

���� �������� ����

���>1� ���� ���� ������� ����� � � ����� �� ��� �� ����� :� ��� ������ ��� ���� ��

����� � ������ ��� J������� ��������������������������� J����� � �������

����� ���������������� ���������� ��� ���� � � �������� ����� �����9

$����� ���������������� ��� ��� ������������� ��C������� �"���� ��@���������

���;���� ��������� ��� ������������$����� ��������� �� ����������;�

� �E J����� � ������E�� �����������&

�� #� � �� ��� ��� ��������� ��� � ������������ J������� ������� ��

������ ����� � $ �������� �����)(6����� ��������� ����������������

�� ����� ����� �������� $ ����0������>.��

'���� �� ��� � �6� ��� J��� ���� � ������� ��� � � ����� ������ �

����� �� ��� � � � �� � �� � � ������� �� ��� � ��� � � ������

�� ��� ������ ��� ������������������� ��

M'���� ��������������������� ������������ ������������������

���� ������������� ������ �����������: ������������ ��� ������ J��

���� ����������������� ���������������J�������� ������� ���J�:

������� ������ �- �����<�� ��� �������� �������������� J��

���� ��� � � ������������� ��� �����������B��� ������;���������

������������������� ��� ���� ��� ������������$���� ��-��� �

C������� �"���� ��@�������������;�N2

���;>� ���� ������ � � � �� ��� ��� �������� ��� �� ������ � �� �� ����� ������� �

������ �������������������� � �� ������� �� � �� ������� ����;=�$���

���;;� �������� �� � � ���������� ����������������� ���������� �@ ��' �����%�� �

8���� ������ ������=�������1���� ��� �������������� �������8����������11��

����;(� ������������ ����� �J�� ���������� ���� ������ �- �����<�� ��� �����

B������<�D ������� ������������� �-������������� � ��-�������������

�� ��� �� ������ � ��� �� ������ �!����'�����% ��� �-��������

����;)� ������ �� �� ���� �����������������������������,��������8������

-��������������� � ����� ����� ��������:

��� ����� �� ������ ���������� ����������� ����- ������%� ���

'������������ �������� ����� � �- �����<�� ��� �����%�

<�� ��� ������� ���+ ������ ��������������� �����������:

������� �� � ������ �� � ���������2

��� ����� ���� ������������ ���������2����

��� ����� ������ �- �����<�� ��� ����� �B������<�D ������

������ ������������� �

14%"!"�� !�:�"���� ������� ������������ ��7�� �� ������ 7�� ����� ���:

��� ������ ����� �������� � $ ������������������������������,��������8���

������������������� � ����� ����� ���� � �� �������������� �������� �- �:

����<�� ��� ��������������������� �B������<�D ���� ���������� ����2

G$���� �&�'��+���%� �G� �����* ��������-��������� �������G� ����

* ��������-���������*�+������%� �'�������- �����,������5�����9�

/� ���9�. � ���� ���� ���� �

Page 305: INTER CA Taxation - CA Study Web€¦ · INTER CA Taxation Head Office Shraddha, 4th Floor, ... 14. Advance Tax & Interest 268 - 271 15. Tax Deducted at Source (TDS) 272 - 294 16

��3����

���������������� ������� ���������� ��

���� �������� ����

����;=� �������� ����������������� �� ��������� ��������������� ����� ���� �� �

�������� ��� ���� ������������� ������ �- �����<�� ��� ����� �B�����

<�D ������ ������� ������������� �

���;1� ���� ���� � � � � �� �� ����� �� ������ ���� ���� ��� �� ��� ��� �������� ��

������������� �������� ����������� ����������������� ������������� ��

������������� ���� ���������� �������

���;1�� �������� ������������������������� ����������������������������� ���

���� ���������������������������� ������������� ����������� �� ��� ������

������������ � $ ���%��������� �������� ��������� ����� ����� � �� �:

� ��������� �- �����<�� ��� ��?��������� ������� ��������� ����

- �����<�� ��� ��

����� �"1�( %�/"0�) #�!�*%0��&�"2%�&(�/�$!��&��!��4���"%�� ! -���? !�&���?�

#� � ������� �� �� ��������������� ���������� ����� ������������������ ����

� ��� ��������� ��'�H�����(:���)��������'�H�����A�:��������������� ������%�G�R��

��� ��������������������(����� ��� �� ����������� ������ �������������

��� ���6����������� �� �������� � $���������������� ������������������� ��� �

����(����� ��� �� ����� ������������ �� ����������� ����� ����������:

����� ���������� ��������� �� ��� �

��� (�6���������������� � $���������������� ������������������� ��� ������� �� $

(�� ����

�>� (��6�������������������� $��������� �� $�(�� ���������� ��� ��������� �����:

� �� �������� ����� � �� ����� �����M�%� ����G�������R�� �* :�� �� �

* � �� �������N������� ���� ������� ��� ���� �������� ��

- $!�� )��1�-4�� !

'�������������� ������� ����1���G$����8����� ������ ���

������������8����8����� ������ ���

$������6���������(�� ����

�@. $��(��6������� $�(�� ����

�@. $��(�6������� $�(�� ��������+ ��������� ����� � �� �

<$�&�� !

F@B�/��������� ���� ��9�����%� ����G�������R�� ��%GR���"����������

� �� ������ ������������� ���������� ������):�=�

!����%GR

8����%�� � ���

G$����%�� � �(�

@ �'��� ;�

-���� � � �J��������� ���� �� ��������� ��� ������������� � ���H�

���=:�1���� �����������������&

�� ���� ��������� ��������� �����������������������(:�=:���A

�� ���� ��������� ��������� �����������������������:�;:���>

Page 306: INTER CA Taxation - CA Study Web€¦ · INTER CA Taxation Head Office Shraddha, 4th Floor, ... 14. Advance Tax & Interest 268 - 271 15. Tax Deducted at Source (TDS) 272 - 294 16

��3�3��

���������������� ������� ���������� ��

���� �������� ����

� %$�� !��

�� ���!����%GR������ ��������� ����������������������(:�=:���A�&

%�� ����� �1��� �������� ��������� � ���� ������������ � ����

����� ����������(�6����� ������������������������������� ��� �

���������� �� ����� ����������� ������� ��

S '��������!����%GR��$���G$��������� �����!����%GR���

$��(�6��������8��������� �����!����%GR

S ;����9���$����(����9���

�$��(�6 S��(��9���������������������������9��

�� ���!�����%GR������ ��������� �������������������������;����>�&

%�� ����;��� �������� ��������� � ���� ������������� � ���� ����

� �������������6��������������������

S '��������!����%GR��$��G$��������� �����!����%GR��

$����6���������8��������� �����!����%GR

S ;����9����$���(����9���

�$�����6 S >����9�����������������9��

��������������������������� ��������������������

��������������� ������� �������������������������

�������������� � ��������� �������������

������ �!�"����##�� $�"��% !

Page 307: INTER CA Taxation - CA Study Web€¦ · INTER CA Taxation Head Office Shraddha, 4th Floor, ... 14. Advance Tax & Interest 268 - 271 15. Tax Deducted at Source (TDS) 272 - 294 16

J.K.SHAH CLASSES INTER C.A. – DIRECT TAXES

: 304 :

Q. 1.R, who is carrying on a business whose accounts have been subject to tax audit regularly, submits his profit & loss account for the year ending 31.3.2018.

`̀̀̀ `̀̀̀

Office expenses 25,600 Gross profit 5,23,600

Audit fees 32,000 Sundry receipt 11,000Legal expenses 18,000 Customs duties recovered

from the government (Earlier not allowed as deduction) 22,000

Depreciation on machinery 24,000

Salary to staff 84,000

Bonus to staff 45,000 Bad debts recovered (earlier allowed as deduction) 6,000Contribution to an approved

gratuity fund 24,000

Outstanding liability in respect of excise duty

24,000 Gift from son 30,000

Real payable to railways 1,20,000General expenses 36,000Net Profit 1,60,000

5,92,600 5,92,600

Other relevant particulars :

1. Bonus payable to employees according to the payment of Bonus Act, 1965, comes to `40,000.

2. Depreciation on machinery shown in the Profit and Loss Account is calculated according to the income-tax provisions.

3. General expenses include payment of ` 12,000 to an approved and notified education institute for the purpose of carrying on research in social sciences. The research is, however, not related to the business of the assesse.

4. During the previous year 2017-18 R also makes a capital expenditure of ` 25,000 for the purpose of carrying on a scientific research related to his business. This expenditure is, however, not recorded in the Profit and Loss Account.

5. Outstanding liability in respect of excise duty is paid as follows: ` 5,000 on 11.4.2018 ` 3,000 on 5.5.2018 ` 6,000 on 30.6.2018 and the balance on 10.11.2018.

6. Audit fee of 32,000 was credited on 31.3.2018. No tax has been deducted at source.

7. Outstanding liability in respect of rent payable to railways is paid as follows: ` 90,000 on 15.6.2018 and the balance on 14.12.2018.

Compute his income from business for the assessment year 2018-19. Assume the due date of filing return of income is 30th September.

Solution:

Computation of total income:

`̀̀̀ `̀̀̀

Net Profit 1,60,000

Less : Items credit to P & L A/c but not treated as income:

Customer duties recovered

Gift from son 22,000

30,000 52,000

1,08,000

CHAPTER NO. 17 HOMEWORK PROBLEMS

Page 308: INTER CA Taxation - CA Study Web€¦ · INTER CA Taxation Head Office Shraddha, 4th Floor, ... 14. Advance Tax & Interest 268 - 271 15. Tax Deducted at Source (TDS) 272 - 294 16

J.K.SHAH CLASSES INTER C.A. – DIRECT TAXES

: 305 :

Add : (1) Expenses disallowed: Outstanding liability for excise duty not paid before filing the return (30.9.2017) disallowed u/s 43B (24,000 – 14,000)

10,000

(2) Outstanding liability of rent payable to railway not paid before filing the return i.e. 30.9.2017 disallowed u/s 43B

30,000

(3) 30% Audit fee as tax has not been deducted at source (Section 40(a) (ia)

9,600 49,600

1,57,000

Less:(i) Capital expenditure incurred on scientific research related to business but not debited to P & L A/c

25,000

(ii) 25% extra deduction for donation to education institute for research in social sciences.

3,000 28,000

Business income being total income. 1,29,000

Q. 2. Shri Batra is the owner of a small manufacturing unit. He gives you the following details drawn from his books of account for the year 2017-18. 1. Computed net profit, after charging the following items, ` 27,500

2. Provisions and reserves debited to Profit and Loss Account. ` ` ` `

(i) Provision for doubtful debts 15,000 (ii) Depreciation reserve 20,000

3. Household expenses ` 46,000.

4. Donations to Prime Minister National Relief Fund `10,000; Other charitable donations ` 20,000.

5. Account payee cheques issued for purchases 60,000. 6. Audit fee charged ` 20,000, including expenses on income-tax assessment

` 15,000.

7. Patents purchased for ` 70,000 during the previous year on 5.7.2017.

8. Incomes credited to Profit and Loss Account were: (i) Bank interest on F.D. ` 5,000.

(ii) Interest on Post-Office Savings Bank Account ` 3,000

(iii) Interest on UTI units ` 2,000.

9. Opening stock is valued at cost plus 10% basis, whereas closing stock was valued at cost minus 10% basis. Opening stock valued was ` 66,000; Closing stock valued was ` 72,000.

Compute the net Business Income for the assessment year 2018-19

Solution

`̀̀̀ `̀̀̀

Computed net profit

Add : (A) Inadmissible items, debited to Profit & Loss A/c

27,500

(1) Provision for doubtful debts 15,000

(2) Depreciation reserve 20,000

(3) Household expenses 46,000

(4) Donations (allowed from GTI) 30,000

Page 309: INTER CA Taxation - CA Study Web€¦ · INTER CA Taxation Head Office Shraddha, 4th Floor, ... 14. Advance Tax & Interest 268 - 271 15. Tax Deducted at Source (TDS) 272 - 294 16

J.K.SHAH CLASSES INTER C.A. – DIRECT TAXES

: 306 :

(5) Patents (for separate consideration) 70,000 1,81,000 2,08,500

(B) Difference on account of valuation of opening and closing stock

14,000

2,22,500 Less : Items credited to Profit and Loss A/c but not being income from business (5,000 + 3,000 + 2,000)

10,000

2,12,500

Less : Depreciation on patents @ 25% on 70,000 17,500 1,95,000 Notes : If opening stock is taken on cost, valuation should be

60,000

If closing stock is taken on cost, valuation should be 80,000 If these valuation are taken, the profit will increase by 14,000

Q. 3.From the following profit & Loss Account of Mr. Vinay Jain ascertain his income from business and Gross Total Income for the assessment year 2018-19.

`̀̀̀ `̀̀̀

To salary (including proprietor’s salary)

45,000 By gross profit By bad debt recovered

1,22,000

To general expenses 5,000 (No allowed earlier by Assessing Officer due to lack of evidence)

2,000

To advertisement 3,000 By rent received 3,000To interest on proprietor’s capital

2,000 By interest on fixed deposit in a bank

1,000

To provision for bad debts 2,000To Fire Insurance Premium (on house property)

100

To depreciation 4,000To reserve for VAT 10,000

To advance income-tax 9,500

To donation to Central Library 1,000

To corporation-tax on house property

500

To motor car expenses 1,000To stationery 1,900Net profit 38,500

1,28,000 1,28,000

Other information

1. General expenses include ` 300 given to a poor student to enable him to pursue his studies.

2. Depreciation is in excess by ` 1,500.

3. Motor car expenses include ` 300 for personal purpose.

4. Central library is an approved institution.

5. The proprietor draws ` 500 p.m. by way of salary.

Page 310: INTER CA Taxation - CA Study Web€¦ · INTER CA Taxation Head Office Shraddha, 4th Floor, ... 14. Advance Tax & Interest 268 - 271 15. Tax Deducted at Source (TDS) 272 - 294 16

J.K.SHAH CLASSES INTER C.A. – DIRECT TAXES

: 307 :

Solution:

`̀̀̀ `̀̀̀

Profit as per Profit and Loss A/c 38,500Add : Inadmissible expenses:

(i) Charity to poor student 300(ii) Excess depreciation 1,500

(iii) Out of Motor Car expenses 300(iv) Donation to Central Library (Allowed from GTI) 1,000

(v) Proprietor/s salary 6,000(vi) Interest on proprietors capital 2,000(vii) Provision for bad debts 2,000

(viii) Insurance of house property 100(ix) Reserve for VAT 10,000

(x) Advance income-tax 9,500

(xi) Corporation tax on house property 500 33,200

71,700

Less : Items credited to Profit and Loss A/c but not to be treated as income, or taxable under other heads (i) Bad debts recovered 2,000(ii) Rent received 3,000

(iii) Interest on fixed deposit in a bank 1,000 6,000

Income from business 65,700

Income from House Property Rent received Less: Municipal taxes 3,000 500

Less : Standard deduction @ 30% 2,500Income from other sources 750 1,750

Interest on fixed deposit in a bank 1,000

Gross total income 68,450

Q. 4. Dr. J.L. Gupta is a renowned medical practitioner who maintains books of account on cash basis. The following is the receipts and payments A/c for the financial year 2017-18.

`̀̀̀ `̀̀̀

Balance brought forward 44,000 Rent of clinic 2017-18 10,24,800

Consultation fees 2015-17 5,000

2018-19 51,200

2017-18 51,35,000 Water & Electricity Bills 2,000Visiting fees 30,000 Purchase of professional

books 40,000

Loan from bank 1,25,000 Household expenses 47,800Sale of medicines 60,000 Collection charges for

dividend income 100

Gifts and presents 5,000Royalties for articles published in various journals

6,000 Motor car purchased 1,30,000

Page 311: INTER CA Taxation - CA Study Web€¦ · INTER CA Taxation Head Office Shraddha, 4th Floor, ... 14. Advance Tax & Interest 268 - 271 15. Tax Deducted at Source (TDS) 272 - 294 16

J.K.SHAH CLASSES INTER C.A. – DIRECT TAXES

: 308 :

Dividend 10,000 Surgical equipment purchased

24,800

Income-tax 7,000

Audit fee 53,000Interest on Government Securities

7000 Salary to staff 15,15,000

Life Insurance Premium 15,000Gift to son 5,000Interest on loan 11,000

Car expenses 15,000Purchase of medicines 40,000Balance c/d 24,45,300

54,27,000 54,27,000

Compute his income from profession for the assessment year 2018-19 after taking into account the following information:

1. Books worth ` 25,000 were purchased on 15.5.2017, which were annual publications and the balance on 5.2.2018 which were books other than annual publication.

2. Car was purchased on 1.1.2018 and the surgical equipment on 4.9.2017. 3. It is estimated that 1/3 of the use of car is for his personal use. 4. Gifts and presents include ` 2,000 from patients in appreciation of his medical

service and ` 3,000 received as birthday gifts.

5. Opening and closing stock of medicines amounted to ` 10,000 and ` 6,000 respectively.

Solution :

`̀̀̀ `̀̀̀ `̀̀̀

Gross receipts (a) Consultation fee (` 5,00 +51,35,000) 51,40,000

(b) Visiting fee 30,000

(c) Sale of medicines 60,000(d) Gifts from patients 2,000 52,32,000

Less : Expenses (1) Rent of clinic 10,24,800 + 51,200 10,76,000

(2) Water and electricity bill 2,000(3) Depreciation on professional books On 25,000 – 100% 25,000

on 15,000 – 30% 4,500 29,500

(4) Depreciation on surgical equipment 15% on 24,800

3,720

(5) Depreciation on car 15 1 2

1,30,00100 2 3

x x x� �� �� �

6,500

(6) Medicine consumed - Purchased 40,000- Decrease in stock 4,000

44,000(7) Salary to staff 15,15,000

Page 312: INTER CA Taxation - CA Study Web€¦ · INTER CA Taxation Head Office Shraddha, 4th Floor, ... 14. Advance Tax & Interest 268 - 271 15. Tax Deducted at Source (TDS) 272 - 294 16

J.K.SHAH CLASSES INTER C.A. – DIRECT TAXES

: 309 :

(8) Interest on loan 11,000

(9) Car expenses 115000

2x

� �� �� �

10,000

(10) Audit fee 53,000 27,50,720

Income from profession 24,81,280

Royalty for articles and interest on government securities are textile under the head ‘income from other sources’. Note : - Books which are annual publications are eligible for depreciation @ 100% whereas books other than annual publication are eligible for depreciation @60%. In the above case, since the books which were not annual publication were purchased on 5.2.2017, depreciation shall be charged @ 50% of 60% i.e. @ 30%.

Q. 5. M/s. ABC Ltd. filed a return of income for assessment year 2018-19 declaring an income of ` 3,25,000. The return is accompanied by the profit and Loss account as under:

`̀̀̀ `̀̀̀

To salaries, bonus and office expenses

1,00,000 By gross profit 5,70,000

To interest on loan 40,000 By interest on bank deposits 12,000To interest for delayed payment of VAT

20,000 By dividend from UTI 18,000

To interest on delayed payment of income-tax

5,000 By dividend from foreign companies

9,000

To income-tax paid 45,000 By income-tax refund 15,000By customs draw bank 1,50,000

To interest on loan for purchase of shares of Indian companies

5,000 By VAT refund (earlier not allowed as deduction)

5,000

Interest on loan for payment of income-tax

5,000 By interest from income- tax department

5,000

To road construction expenses 50,000To entertainment expenses 12,000To income-tax proceeding expenses

2,000

To penalty levied by customs 5,000To provision for income-tax 50,000To contribution to recognised provident fund

40,000

To reserve for bad debts 44,000

To depreciation 36,000To net profit 3,25,000

7,84,000 7,84,000

The profit and loss account has following notes. 1. ` 7,500 on account of bonus remained unpaid at the end of the year.

2. Last instalment of contribution to Provident Fund of 2,000 has been paid before the date of filing return of income but after the date as per Provident Fund Act.

3. Depreciation allowable as per the Income-tax Rules is 33,000. There is no other information available with the return of income. Compute business income of the company for assessment year 2018-19.

Page 313: INTER CA Taxation - CA Study Web€¦ · INTER CA Taxation Head Office Shraddha, 4th Floor, ... 14. Advance Tax & Interest 268 - 271 15. Tax Deducted at Source (TDS) 272 - 294 16

J.K.SHAH CLASSES INTER C.A. – DIRECT TAXES

: 310 :

Solution

`̀̀̀ `̀̀̀

Profits as per Profit and Loss A/c Add: Inadmissible expenses

3,25,000

(1) Interest on delayed payment of income-tax 5,000(2) Income-tax paid 45,000

(3) Interest on loan for purchase of units 5,000(4) Interest on loan for payment of income-tax 50,000

(5) Road construction expenses 5,000(6) Penalty levied by customs 50,000(7) Provision for income-tax 44,000

(8) Reserve for bad debts 3,000(9) Excessive depreciation 7,500 2,19,500

(10) Unpaid bonus – 43B 5,44,500

Less: Items credited to Profit and Loss A/c, not treated as Income/Taxable under other heads

(1) Interest on Bank Deposits 12,000(2) Dividend from UTI 18,000(3) Dividend from Foreign Companies 9,000(4) Income-tax refund 15,000

(5) VAT refund 5,000(6) Interest from Income Tax Department (taxable under other sources)

5,000 64,000

4,80,500Less: Depreciation on road construction @ 10% assuming put to use for 180 days or more.

5,000

Income from business 4,75,000

Q. 6. Mr. Bharat is a registered medical practitioner. He keeps his books on cash basis, and his summarised cash account for the year ended 31.3.2018 is as under:

`̀̀̀ `̀̀̀

Balance b/d 2,24,000 Cost of medicines 10,000Loan from bank for private purpose

3,000 Surgical equipment’s 4,000

Sale of medicines 25,250 Motor car 2,20,000Consultation fees 50,55,000 Car expenses 6,000

Visiting fees 5,24,000 Salaries 20,31,600Interest on Govt. securities 4,500 Rent of dispensary 5,51,600Rent from property (not subject to local taxes)

3,600 General expenses 20,300

Personal expenses 11,600Life insurance premium 3,000

Interest on loan from bank

300

Insurance of property 200Telephone expenses 33,000Balance c/d 29,47,750

58,39,350 58,39,350

Page 314: INTER CA Taxation - CA Study Web€¦ · INTER CA Taxation Head Office Shraddha, 4th Floor, ... 14. Advance Tax & Interest 268 - 271 15. Tax Deducted at Source (TDS) 272 - 294 16

J.K.SHAH CLASSES INTER C.A. – DIRECT TAXES

: 311 :

Compute his income from profession for the assessment year 2018-19 taking into account the following further information: (i) 1/4th of motor car expenses are in respect of his personal use. (ii) Depreciation allowable on car is 15% and surgical equipment is 15%. Both the assets

were purchased in December, 2016.

(iii) Opening stock and closing stock of medicines was 20,000 and 12,000.

Solution

`̀̀̀ `̀̀̀

Gross receipts from profession (1) Sale of medicines 25,250(2) Consultation fees 50,55,000

(3) Visiting fees 5,24,000

56,04,250

Less: Allowable expenses

(1) Cost of medicine (10,000 + 20,000 – 12,000) 18,000

(2) Depreciation on surgical equipment’s

15 14,000

100 2x x

� �� �� �

300

(3) Depreciation on motor car 15 1 3

2,20,000100 2 4x x x

� �� �� �

12,375

(4) Car expenses 3

6,0004x

� �� �� �

4,500

(5) Salaries 20,31,600(6) Rent of dispensary 5,51,600

(7) General expense 20,300(8) Telephone expenses 33,000 26,71,675

Income from profession 29,32,575

Q. 7. R who does not want to opt for presumptive income under section 44AD furnishes the following Trading. Profit and Loss Account for the previous year ending 31.3.2018.

`̀̀̀ `̀̀̀

To Stock 2,40,000 By Sales 79,76,900To Purchases 72,60,000 By Stock 4,60,000To Freight and duty 50,000To Manufacturing wages 1,20,000

To Rent, rates and taxes 45,000

`̀̀̀ `̀̀̀

To Depreciation 48,000

To Gross Profit 6,73,900 84,36,900 84,36,900

To Office salaries 66,000 By Gross profit b/d 6,73,900To Interest on capital 12,000 By Rent of staff quarters 19,000To Bad debts 9,000 By Refund of income tax

penalty 1,100

Page 315: INTER CA Taxation - CA Study Web€¦ · INTER CA Taxation Head Office Shraddha, 4th Floor, ... 14. Advance Tax & Interest 268 - 271 15. Tax Deducted at Source (TDS) 272 - 294 16

J.K.SHAH CLASSES INTER C.A. – DIRECT TAXES

: 312 :

To Income-tax 11,000 By Sale price of an old machinery

25,000

To Expenses of income-tax proceedings

16,000 Recovery of bad debts, not allowed to be deducted in earlier years.

6,000

To Diwali expenses 3,000 By Sundry receipts 35,000To Legal expenses 6,000To Medical expenses of the proprietor in the Govt. Hospital

11,000

To Staff welfare expenses 4,000To Repairs of staff quarters 11,000

To Telephone expenses 15,000To Bonus payable to employees 30,000Provision for taxes:

To VAT and excise duty 40,000To Municipal taxes for staff quarters

14,000

To General reserve 11,000To Entertainment expenses 11,000

To Net profit 4,90,000

7,60,000 7,60,000

You are required to compute the taxable profits from business after taking the following into consideration:

(i) Purchases include a purchase of ` 28,000. Whose payment was made by a crossed cheque.

(ii) Office salaries include ` 18,000 paid to the proprietor of the business.

(iii) Diwali expenses include gifts of 1,500 made to relatives. (iv) The written down value of the block consisting of machinery as on 1.4.2016 is

80,000. (v) The written down value of the block consisting of factory building as on 1.4.2016

1,20,000. An addition was made to building on 1.8.2017 at a cost of company paid 41,000 as insurance compensation.

(vi) VAT and excise, duty, amounting to 30,000 was paid on 25.6.2018

(vii) Municipal tax were due on 31.3.2018

Solution :

Computation of taxable income from business

`̀̀̀ `̀̀̀ `̀̀̀

Profit as per P & L A/c

Add: Inadmissible, etc.

4,90,000

1. Depreciation considered separately 48,0002. Interest on capital 12,0003. Income-tax 11,0004. Medical expenses of proprietor 11,000

5. Bonus payable: Deductible on actual payment u/s 43B 30,0006. (i) Provision for sales-tax and excise duty 40,000

Page 316: INTER CA Taxation - CA Study Web€¦ · INTER CA Taxation Head Office Shraddha, 4th Floor, ... 14. Advance Tax & Interest 268 - 271 15. Tax Deducted at Source (TDS) 272 - 294 16

J.K.SHAH CLASSES INTER C.A. – DIRECT TAXES

: 313 :

Less : Paid before the due date of furnishing the return u/s 43B

30,000 10,000

(ii) Municipal taxes for staff quarters deductible on actual payment u/s 43B

14,000

7. General reserve 11,0008. Payment regarding purchase of more than 20,000 by crossed cheque hence, 100% of 28,000 disallowed [Section 40A (3)]

28,000

9. Proprietors salary 18,00010. Diwali expenses to the extent of gifts to relatives 1,500 1,94,500

6,84,500

Less : Items not being business income: 1. Refund of income-tax penalty 1,1002. Sale price of machinery 25,000

3. Recovery of bad debts not allowed 6,000 32,1006,52,400

Less : Admissible deductions:

Depreciation 20,150

Business Income 6,32,250

Q. 8. The following is the Receipts and Payments Account of a medical practitioner for the year ending 31.3.2018.

Receipts `̀̀̀ Payments `̀̀̀

Balance b/f 1,60,000 Clinic rent 24,000Visiting fees 1,40,000 Staff salaries 1,20,000Consultation fees 1,60,000 Rent and taxes 6,000

Sale of medicines 40,000 Electricity and water 7,000Operation theatre rent 25,000 Purchase of medical books 20,100Interest on fixed deposit with bank (gross)

13,000 Purchase of surgical equipment’s

40,000

Dividends from UTI 10,000 Motor car expense Sale of surgical equipment 36,000 Medical association

membership 4,000

Life insurance premium 10,000Audit fees 14,000Staff welfare expenses 3,000

Diwali expenses 2,000Entertainment expenses 8,400Medicines purchased 26,000Balance c/d 2,75,500

5,84,000 5,84,000

Additional Information :

1. A cash payment of 15,000 was given to him by a patient in appreciation of his medical service but was not accounted for in the books of account.

2. ¼th of motor-car expenses relate to his personal use, depreciation on motor car allowable under the Income-tax Act IS 9,000 for professional use.

3. Audit fees include income-tax appeal expenses of 12,000.

Page 317: INTER CA Taxation - CA Study Web€¦ · INTER CA Taxation Head Office Shraddha, 4th Floor, ... 14. Advance Tax & Interest 268 - 271 15. Tax Deducted at Source (TDS) 272 - 294 16

J.K.SHAH CLASSES INTER C.A. – DIRECT TAXES

: 314 :

4. The rate of depreciation on surgical equipment is 15%. The written down value of the equipment’s brought forward from earlier year was 26,000. He sold equipments for 36,000 during the current year. The new surgical equipments were purchased on 1.11.2017.

5. His taxable income from house properties was 3,00,000. 6. Opening and closing stock of medicines were 10,000 & 15,000 respectively.

Compute his income from the assessment year 2017-18 assuming that - (a) he does not want to opt for presumptive scheme u/s 44 ADA...

(b) he has opted for presumptive scheme u/s 44ADA

Solution (a)

`̀̀̀ `̀̀̀

Visiting & consultation fees (1,40,000 + 1,60,000) 3,00,000Sale of medicines and operation theatre rent (40,000 + 25,000)

65,000

Cash receipts from patient 15,000

3,80,000

Less: Expense:

Rent 24,000Salary 1,20,000Electricity and water 7,000Rent/taxes 6,000Books depreciation @ 100% (assuming annual publication)

20,100

Association membership fees 4,000Audit fees 14,000Staff welfare expenses 3,000

Diwali expenses 2,000Depreciation on car 9,000Depreciation on equipments 2,250

Entertainment expenses 8,400Cost of medicines… (10,000 + 26,000 – 15,000) 21,000

Motor-card expenses (i.e. 24,000 – ¼ th of 24,000) 18,000 2,58,750

Income from profession 1,21,250Income from house property 3,00,000

Income from other sources (interest on FDR) 13,000

Gross total income 4,34,250Less: Deduction u/s 80C (LIP) 10,000

Total income 4,24,250

Computation of depreciation on equipments WDV as on beginning of the year 26,000Add : Cost of equipment purchased 40,000Less: Sale proceeds of equipment 66,000

36,000

30,000

Depreciation @ 7.5% as used for less than 180 days 2,250

Page 318: INTER CA Taxation - CA Study Web€¦ · INTER CA Taxation Head Office Shraddha, 4th Floor, ... 14. Advance Tax & Interest 268 - 271 15. Tax Deducted at Source (TDS) 272 - 294 16

J.K.SHAH CLASSES INTER C.A. – DIRECT TAXES

: 315 :

Solution (b)

Income from profession

50% of gross receipts from profession i.e. 50% of 3,80,000 1,90,000

Income from house property 3,00,000

Income from other sources (interest on FDR) 13,000

Gross total income 5,03,000

Less : Deduction u/s 80C (LIP) 10,000

Total income 4,93,000

Written down value of equipment as on 1.4.2018 after deemed depreciation 2,250

Q. 9. R, furnishes the following particulars for the assessment year 2018-19

Profit and Loss Account for the year ending 31.3.2018

`̀̀̀ `̀̀̀

Salary to staff 3,60,000 Gross profit 6,69,000Interest on loan 40,000 Rent of flats given to

officers 24,000

Expenditure for promotion of family planning amongst employees

14,000 Sundry receipts 7,000

Sales-tax 24,000 Interest on fixed deposits with bank (gross)

15,000

Telephone expenses 15,000 Capital gains on sale of Long-term investment

6,000

Gratuity paid 24,000Reserve for future losses 20,000Reserve for bad debts 12,000

Reserve for payment of advance income-tax

17,000

Car expenses 20,000Depreciation 30,000Office expenses 2,000

Provident Fund 10,000Municipal taxes for flats given to officers

24,000

Sundry expenses 46,000Net profit 63,000

7,21,000 7,21,000

Following additional information is also available :

(a) Expenditure on family planning includes capital expenditure of 10,000.

(b) Debts to the tune of 10,000 have actually become bad during the year. (c) Sundry expenses include 12,000 being printing charges paid to a relative of the proprietor; payment is considered reasonable to the extent of 9,000 only. (d) 22,000 out of sundry expenses relate to cash payment for purchasing miscellaneous items.

(e) Written down value of various fixed assets is as under on the opening day:

Page 319: INTER CA Taxation - CA Study Web€¦ · INTER CA Taxation Head Office Shraddha, 4th Floor, ... 14. Advance Tax & Interest 268 - 271 15. Tax Deducted at Source (TDS) 272 - 294 16

J.K.SHAH CLASSES INTER C.A. – DIRECT TAXES

: 316 :

`̀̀̀

Machinery 42,000

Motor car 18,750

Furniture 12,000

(f) As per balance sheet following balances appear on the liabilities side

`̀̀̀

(i) Sales tax payable amount 7,000

(ii) Contribution to employees provident fund payable 2,000

(g) It is learnt that letting out of residential flats to employees is subservient and incidental to the main business of the company.

(h) Sundry receipt include an amount of 4,000 being dividend received from another Indian Company.

Determine the business income of company for the assessment year 2018-19

Solution

`̀̀̀ `̀̀̀ `̀̀̀

Net profit as per P & L A/c 63,000

Less : (1) Dividend include in sundry receipts 4,000

(2) Interest on Bank Deposits to be considered separately

15,000

(3) Capital gain to be considered separately 6,000 25,000

38,000

Add : 1. Capital expenditure and revenue expenditure on family planning

14,000

2. Reserve for future losses, being inadmissible

20,000

3. Reserve for bad debts 12,000

Less : Debts which actually became bad 10,000

2,000

4. Reserve for payment of advance income tax, being inadmissible

17,000

5. Depreciation to be considered separately 30,000

6. Out of sundry expenses the following disallowance are made

(i) Printing charges paid to a relative of the Proprietor disallowed u/s 40A (2) Being excessive and unreasonable (`̀̀̀12,000 – 9,000)

3,000

(ii) 100% of payment ` ` ` ` 22,000 for purchasing miscellaneous items is disallowed u/s 40A(3) as the payment has been made otherwise than by a account payee cheque or account payee bank draft

22,000

7. The following amounts are added u/s 43B as the payments have not actually been made Sales tax

7,000

Page 320: INTER CA Taxation - CA Study Web€¦ · INTER CA Taxation Head Office Shraddha, 4th Floor, ... 14. Advance Tax & Interest 268 - 271 15. Tax Deducted at Source (TDS) 272 - 294 16

J.K.SHAH CLASSES INTER C.A. – DIRECT TAXES

: 317 :

Contribution to employees P.F. 2,000 9,000 1,17,000

1,55,000Less: Depreciation as worked out below :

Machinery WDV 42,000 @ 15%

6,3000

Motor WDV ` ` ` ` 18,750 @ 15% 2,813

Furniture WDV `̀̀̀ 12,000 10 % 1,200 10,313

Business income 1,44,687

Expenditure in family planning, whether capital or revenue, is allowed to a company assesse.

Q. 10.Sunil is salaried employed in New Delhi. He got the following emoluments from his employer during the previous year ended 31.3.2018.

`̀̀̀

Basic Salary 3,00,000

Bonus 40,000

Commission 36,000

House Rent Allowance 48,000

Employer’s contributed equal amount to the RPF. 48,000

Sunil has also contributed equal amount to the RPF. He owns a house property which is used by him for his own residence. Municipal valuation of the house property is ` ` ` ` 1,80,000, whereas the standard rent under the Rent Control Act is ` ` ` ` 1,20,000. He paid the following expenses in respect of the house property.

(i) Municipal taxes ` ` ` ` 30,000 and (ii) repairs ` ` ` ` 20,000.

Besides, he has received ` ` ` ` 35,000 as dividend (gross) from an Indian company.

(ii) During the previous year 2017-18, he made the payment of insurance premium on own life policy taken on 1.4.2016 (sum assured ` ` ` ` 1,00,000) ` ` ` ` 21,000.

Determine the total income of Mr. Sunil for the assessment year 2018-19.

Solution:

`̀̀̀ `̀̀̀

Income from Salary 3,00,000Basic Salary 40,000Bonus 36,000Commission 48,000House Rent Allowance

Employer’s contribution to PPF 48,000

Less: Exempt to the extent of 12% of salary 36,000 12,000

Gross Salary 4,36,000Less : Deduction Nil 4,36,000

Income from house property NilIncome from other sources (dividend) Exempt

Gross Total Income 4,36,000Less: Deduction u/s 80C – RPF 48,000 + 10,000 (LIP)

58,000

Total Income 3,78,000

Page 321: INTER CA Taxation - CA Study Web€¦ · INTER CA Taxation Head Office Shraddha, 4th Floor, ... 14. Advance Tax & Interest 268 - 271 15. Tax Deducted at Source (TDS) 272 - 294 16

J.K.SHAH CLASSES INTER C.A. – DIRECT TAXES

: 318 :

Q. 11.Following is the Profit and Loss A/c of R for the year ended 31.3.2018

`̀̀̀ `̀̀̀

To Salary 1,05,000 By Gross profit 3,98,200To Travelling expenses 30,000 By Dividend from Indian Co 8,500

To Rent and taxes 6,000 By Discount 1,500To Interest on capital 6,000 By Share of profit from a

partnership firm 22,500

To Depreciation 30,000To Administrative expenses 20,000

To Income tax 50,000To Net profit 1,83,700

4,30,700 4,30,700

The following information is available :

(i) Salary include ` ` ` ` 15,000 paid to Mrs. R as supervisor of quality control department. Till February, 2017 she was employed in S Ltd. in similar post for the last 10 years on a salary of ` ` ` ` 1,500 per month. She does not have any other income.

(ii) Included in the administrative expenses ` ` ` ` 2,000 paid to the trade association for advertisement in souvenir published by it.

(iii) Depreciation allowable as per I.T. law is ` ` ` ` 45,000.

(iv) Included in the drawings of Mr. R ` ` ` ` 2,000 interest on loan, paid to LIC on the security of his life-insurance policy. Loan was utilised for repair of machinery.

(v) Birthday gifts received by his minor son include cash ` ` ` ` 20,000 which was deposited with a nationalised bank. Interest accrued up to 31.3.2018 amounted to ` ` ` ` 2,5000.

(vi) During the year ended 31.3.2018 R paid ` ` ` ` 10,000 towards medical insurance premium of himself, his wife and minor son.

(vii) Included in the drawings of R as goods taken from the business of ` ` ` ` 6,000; the cost of the goods is ` ` ` ` 7,500 and market price is ` ` ` ` 9,000.

Compute the taxable income of R for the assessment year 2018-19.

Solution

Computation of Taxable income of R

(For the assessment year 2018-19)

Income from business `̀̀̀ `̀̀̀

Net profit as per profit & loss account 1,83,700

Less : Share of profit from partnership firm exempt u/s 10(2A)

22,500

Income from dividend from Indian company 8,500 31,000

1,52,700

Add: Expenses disallowed

(i) Interest on capital (being, personal) 6,000

(ii) Depreciation to be treated separately 30,000

(ii) Income – tax 50,000

(iii) Goods taken for personal use (` ` ` ` 7,500 – 6,000)

1,500 87,500

2,40,200

Page 322: INTER CA Taxation - CA Study Web€¦ · INTER CA Taxation Head Office Shraddha, 4th Floor, ... 14. Advance Tax & Interest 268 - 271 15. Tax Deducted at Source (TDS) 272 - 294 16

J.K.SHAH CLASSES INTER C.A. – DIRECT TAXES

: 319 :

Less : Expenses allowable

(i) Depreciation allowable as per I.T. law 45,000

(ii) Interest on loan from LIC being for business 2,000 47,000

Income from the other sources

Income of minor child 2,500

Less: Exempted u/s 10(32) 1,500

1,000

Dividend from an Indian company Exempt 1,000

Gross total income 1,94,200

Less: Deduction under Chapter VIA

U/s 80C Nil

U/s 80D (medical insurance premium) 10,000 10,000

Taxable income 1,84,200

Q.12.From the following information compute the total income of Mr. Batra for the assessment year 2018-19

(a) Salary ` ` ` ` 12,000 p.m.

(b) Bonus equal to 2 months’ Basic Salary.

(c) House rent allowance @ ` ` ` ` 5,000 per month. The actual rent paid by him was ` ` ` `

4,500 per month. He resides in Delhi. (d) Employer and employee contributed to a Recognised Provident Fund @ 12.5%

of the salary. (e) Provided with a 1.6 ltr. Engine capacity car for office and personal purposes,

expenses borne by the employer. (f) Income from dividend 800. (g) Interest from Saving Bank Account ` ` ` ` 22,500.

(h) Life insurance premium paid ` ` ` ` 3,000 on a policy of ` ` ` ` 35,000.

Solution Income from salary `̀̀̀ `̀̀̀

Salary 1,44,000 Bonus 24,000 Employers Contribution to P.F. (in excess of 12%) 720

HRA 20,400 Car (` ` ` ` 1,800 x 12) 21,600

Gross Salary 2,10,720Less: Deduction Nil 2,10,720

Income from other sources

Dividend Exempt

Interest on saving bank account 22,500 22,500

Gross total income 2,33,220

Less: (i) Deduction u/s 80C (RPF – ` ` ` ` 18,000 + LIP – ` ` ` `

3,000) 21,000

(ii) Deduction u/s 80TTA 10,000 31,000

2,02,220

Page 323: INTER CA Taxation - CA Study Web€¦ · INTER CA Taxation Head Office Shraddha, 4th Floor, ... 14. Advance Tax & Interest 268 - 271 15. Tax Deducted at Source (TDS) 272 - 294 16

J.K.SHAH CLASSES INTER C.A. – DIRECT TAXES

: 320 :

Q. 13.Following is the Profit and Loss A/c of R for the year ended 31.3.2018.

`̀̀̀ `̀̀̀

To Salary 1,05,000 By Gross profit 6,98,200

To Travelling expenses 30,000 By Dividend from Indian Co 8,500

To Rent and taxes 6,000 By Discount 1,500

To Interest on capital 6,000 By Share of profit from a partnership firm

22,500

To Depreciation 30,000

To Administrative expenses 20,000

To Income tax 50,000

To Net profit 4,83,700

7,30,700 7,30,700

The following information is available: (i) Salary include ` ` ` ` 15,000 paid to Mrs. R as supervisor of quality control department.

Till February 2019 she was employed in S Ltd. in similar post of the last 10 years on a salary of ` ` ` ` 1,500 per month. She does not have any other income.

(ii) R had gone on a foreign tour for 15 days in connection with his business. He spent 3 days visiting tourist sports. The total expense in this connection was ` ` ` ` 20.000 which was within RBI norms.

(iii) Included in the administrative expenses ` ` ` ` 2.000 paid to the trade association for advertisement in souvenir published by it.

(iv) Depreciation allowable as per I.T. law is ` ` ` ` 45.000

(v) Included in the drawings of Mr. R ` ` ` ` 2,000 interest on loan, paid to L.I.C. on the security of his life-insurance policy. Loan was utilised for repair of machinery.

(vi) Birthday gifts received by his minor son include cash ` ` ` ` 20,000 which was deposited in saving bank with a nationalised bank. Interest accrued up to 31.3.2017 amounted to `̀̀̀ 16,500.

(vii) P. the brother of R. is suffering from heart disease. As P has no income, R take care of the medical expenses of his brother P. which amounted to `̀̀̀ 18,000.

(viii) During the year ended 31.3.2017, R paid `̀̀̀ 10,000 towards medical insurance premium of himself, his wife and minor son.

(ix) Included in the drawings of R as goods taken from the business of ` ` ` ` 6,000; the cost of the goods is ` ` ` ` 7,500 and market price is ` ` ` ` 9.000.

Compute the taxable income of R for the assessment year 2018 - 19 [ICWAI-JUNE 1999]

Solution

Computation of Taxable income of R from the assessment year 2018- 19

Income from business `̀̀̀ `̀̀̀

Net profit as per profit & loss account 4,83,700Less : Share of profit from partnership firm exempt u/s

10(2A) 22,500

Income from dividend from Indian company 8,500 31,000

4,52,700

Add: Expenses disallowed (i) Travelling expenses for foreign trip to the extent not in connection with business (3/15 x 20,000)

4,000

(ii) Interest on capital (being personal) 6,000

Page 324: INTER CA Taxation - CA Study Web€¦ · INTER CA Taxation Head Office Shraddha, 4th Floor, ... 14. Advance Tax & Interest 268 - 271 15. Tax Deducted at Source (TDS) 272 - 294 16

J.K.SHAH CLASSES INTER C.A. – DIRECT TAXES

: 321 :

(iii) Depreciation to be treated separately 30,000 (iv) Income-tax 50,000

(v) Goods taken for personal use (` ` ` ` 7,500 – 6,000) 1,500 91,500

5,44,200

Less: Expenses allowable (i) Depreciation allowable as per I.T. law 45,000

(ii) Interest on loan from L.I.C. being for business 2,000 47,000

4,97,200

Income from other sources

Income of minor child 16,500Less: Exempted u/s 10(32) 1,500 Dividend 15,000 15,000

Gross total income Exempt 5,12,200

Less: Deduction under Chapter VIA

U/s 80D (medical insurance premium) 10,000 U/s 80TTA 10,000 20,000

Taxable income 4,92,200

Note :- Heart disease is not a specified disease under section 80DDB, hence no deduction shall be allowed.

Q. 14. Problems on computation of taxable – X (age: 30 years) is a salaried employee in Bombay, lie gets the following emoluments from his employer during the previous year ending March 31, 2018.

Rs.

Basic salary 4,80,000Bonus 1,30,000Commission (fixed) 1,48,000House rent allowance 1,20,000Employer’s contribution towards recognised provident fund 62,400

During the previous year 2017-18, the employer has provided a laptop computer for using it for official and private purpose. Ownership is not transferred. Further the employer provides club facility for official use. He owns a house property which is used in/ him for his own residence. Municipal valuation of the house property is Rs. 1,30,000; whereas the standard rent under the Rent Control Act is Rs. 1,20,000. He makes the following expenditures in respect of house property : municipal taxes : Rs. 13,000; repairs : Rs. 11,000; interest on capital borrowed to pay municipal tax : Rs. 3,150; and insurance : Rs. 1,600. Besides, he has received Rs. 1,19,000 as interest from deposits in savings bank account. He has received different gifts from A Rs. 25,000 on October 1, 2017 and from B Rs. 26,000 on March 1, 2018. During the previous year 2017-18, he makes the following expenditure and investments:

Rs.

Contribution towards recognised provident fund 1,40,000Payment of insurance premium on own life policy (sum assured in 2006 : R$. 60,000)

14,000

Donation to the National Defence Fund 3,200

Determine the net income of X for the assessment year 2018-19.

Page 325: INTER CA Taxation - CA Study Web€¦ · INTER CA Taxation Head Office Shraddha, 4th Floor, ... 14. Advance Tax & Interest 268 - 271 15. Tax Deducted at Source (TDS) 272 - 294 16

J.K.SHAH CLASSES INTER C.A. – DIRECT TAXES

: 322 :

Solution:

Income from salary [see Note 1] 8,82,800 Income from house property (sec Note 2] Nil

Income from other sources : Bank interest on deposits in savings account 1,19,000

Gift received on October 1, 2017 and gift received on March 1, 2017 [Rs. 25,000+ Rs. 26,000; as the aggregate amount exceeds Rs. 50,000, it is taxable] 51,000

Gross total income 10,52,800

Less: Deductions under section 80C to 80U Under section 80C [Contribution to RPF and insurance premium]

1,50,000

Under section 80G in respect of donation to the National Defence Fund (i.e., 100% of Rs. 3,200)

3,200

Under section 80TTA in respect of interest on deposits in savings bank account

10,000

Net income 8,89,600

Notes:1. COMPUTATION OF INCOME FROM SALARY

Basic salary 4,80,000 Bonus 1,30,000

Commission 1,48,000 House rent allowance : Rs. 1,20,000 Less: Exempt from tax [see para 42.1] being the least of :

(a) Rs. 2,40,000 (being 50% of salary, i.e. Rs. 4,80,000); (b) Rs. 1,20,000 (being the house rent allowance); or (c) Nil, being excess of rent paid (i.e. Nil) minus 10% of salary. Nil, being the least, is exempt from tax.

Amount taxable 1,20,000Employer's contribution in excess of 12% of salary (i.e., Rs. 62,400-Rs. 57,600)

4,800

Gross salary 8,82,800Less: Standard deduction - Income from salary 8,82,800

The perquisites in respect of use of computer and club facility are not chargeable to tax.

2. COMPUTATION OF INCOME FROM HOUSE PROPERTY - Since house is used for the purpose of own residence, nothing would be chargeable to tax under section 23(2)(a). Interest on capital borrowed to pay municipal tax is not deductible.

Q.15.195-P8 Compute the taxable income and tax liability of X (40 years) for the assessment year 2018-19.

Rs.

Net income from chemical trading business 8,00,000

Long-term capital gain on transfer of debentures (if computed without indexation)

6,00,000

Long-term capital gain on transfer of listed debentures (computed after indexation) (X wants to pay tax at the rate of 20 per cent of Rs.

2,50,000

Page 326: INTER CA Taxation - CA Study Web€¦ · INTER CA Taxation Head Office Shraddha, 4th Floor, ... 14. Advance Tax & Interest 268 - 271 15. Tax Deducted at Source (TDS) 272 - 294 16

J.K.SHAH CLASSES INTER C.A. – DIRECT TAXES

: 323 :

2,50,000 and not at the rate of 10 per cent of Rs. 6,00,000)

Share of income from HUF in which he is a member 82,000

Winning from horse races (net of IDS of 30 per cent) 70,000

Interest on bank fixed deposits:

- Deposit in Ins own name 1,17,000

- In the name of minor son 1,450

- In the name of minor daughter 8,000

Rs.

- In the name of major unmarried daughter 70,000

Expenditure incurred for the medical treatment of his 67 years old elder brother [dependent of X, being a person with disability]

8,000

Repayment of loan taken for part-time studies of major daughter for graduate course in management (loan is taken from a notified charitable institute).

6,90,000

Payment of interest from the aforesaid loan 76,000

Donation to the aforesaid notified charitable institute 5,000

Brought forward loss of a discontinued business pertaining to the assessment year 2014-15.

26,000

Purchase of a work of art on November 1,2017 from a friend for Rs. 1,15,000 (market value is, however, Rs. 1,70,000)

1,15,000

Deposit in public provident fund accountant purchase of NSC IX issue 1,63,000

Solution :

Rs.

Computation of income of x

Income from chemical trading business 80,00,000

Less: Brought forward loss of a discontinued business (-)26,000 7,74,000

Long-term capital gains (indexation is not available in the case of debentures)

6,00,000

Income from other sources

- Winning from horse races [Rs. 70,000 ÷ (1 - 03)] 1,00,000

- Interest on bank deposit (own) 1,17,000

- Interest on bank deposit [minor son (Rs. 1,450 – Rs. 1,450)]

Nil

- Interest on bank deposit [minor daughter (Rs. 8,000 – Rs. 1,500)]

6,500

- Purchase of work of art (Rs. 1,70,000 – Rs. 1,15,000) 55,000 2,78,500

Gross total income 16,52,500

Less: Deduction

Under section 80C

Under section 80C 1,50,000

Under section 80E 75,000

Under section 80G (50% of Rs. 5,000) 76,000

Net income 2,500 3,03,500

Tax on net income 13,49,000

Income-tax (10% of Rs. 6,00,000 + 30% of Rs. 1,00,000 + normal tax on the balance) ÷

1,44,800

Page 327: INTER CA Taxation - CA Study Web€¦ · INTER CA Taxation Head Office Shraddha, 4th Floor, ... 14. Advance Tax & Interest 268 - 271 15. Tax Deducted at Source (TDS) 272 - 294 16

J.K.SHAH CLASSES INTER C.A. – DIRECT TAXES

: 324 :

Add: Education cess 2,896

Add: Secondary and higher education cess 1,448

Tax liability (rounded off) 1,49,140

Notes :

1. In the case of transfer of quoted securities, one has an option to pay tax @ 10% if the benefit of indexation is not taken. I However, indexation benefit is never available in the case of debentures (whether quoted or not). In this problem taxable long-term capital gain will be Rs. 6,00,001). X has an option to pay tax at the rate of 10% or 20%.

2. Interest income of major daughter is taxable in the hands of daughter. 3. Repayment of loan taken of higher studies, is not deductible.

4. Purchase of work of art for inadequate consideration is taxable under section 56(2)(vii).

Q.16. Mrs. X (30 years) owns a business. Net profit of the business of the year ending March 31,2017 as per auditor’s certificate to Rs. 19,40,000. Salary debited to profit and loss account includes a payment by an account payee cheque of Rs,9,6000 to her brother to who is employed by Mrs. X to look after marketing department. However, no other businessman will pay more than Rs. 1,40,000 as salary to her brother. Profit and loss account also includes an agricultural income of Rs. 4,00,000.

On November 1,2017, Mrs. X purchased 500 equity shares of A Ltd. at the rate of Rs.400 per share from a friend. 7mv of shares is Rs. 1700 pershare.

Other incomes –

1. Mrs. X borrowed Rs. 3,00,000 in the year and invested it in units of quoted mutual funds. None of the mutual funds declared a dividend in the year. She paid an interest of Rs. 35,000 on the loan taken by him.

2. Dividend on other units purchased, received in the year was Rs. 46,000.

3. Interest on fixed deposits with banks was Rs. 8,000. 4. Interest on bank deposit in the name of her dependent mother was Rs. 20,000.

Deposit was made by her mother out of gifts received from relatives. 5. Minor son of Mrs. X has received birthday gift of Rs. 65,000 from family friends

on November 1, 2017. This amount is deposited in a bank fixed deposit at interest rate of 6 per cent per annum.

Find out the net income and tax liability of Mrs. X for the assessment year 2018-19 on the assumption that she deposited Rs.1,10,000 in a fixed deposit with SRI for claiming deduction under section 80C.

Solution :

Rs. Rs.

Business income 19,40,000

Add: Excess payment to brother (Rs. 9,60,000 - Rs. 1,40,000)

8,20,000

Less: Agriculture income (-) 4,00,000 23,60,000

Income from other sources

Purchase of equity shares [Rs. 1,700 – Rs. 400) x 500 shares]

6,50,000

Dividend on units (exempt) Nil

Bank interest 8,000

Page 328: INTER CA Taxation - CA Study Web€¦ · INTER CA Taxation Head Office Shraddha, 4th Floor, ... 14. Advance Tax & Interest 268 - 271 15. Tax Deducted at Source (TDS) 272 - 294 16

J.K.SHAH CLASSES INTER C.A. – DIRECT TAXES

: 325 :

Interest on deposit of dependent mother (not taxable as income of Mrs. X)

Nil

Birthday gift received by minor son of Mrs. X (Rs. 65,000 – Rs. 1,500)

63,500

Interest income of minor son (6% of Rs. 65,000 x 5 ÷ 12) 1,625 7,23,125

Gross total income 30,83,125

Less: Deduction under section 80C 1,10,000

Net income (rounded off) 29,73,130

Tax liability of Mrs. X will be calculated as under -

None agricultural income 29,73,130

Agricultural income 4,00,000

Total 33,73,130

Tax on total 8,36,939

Less: Tax on Rs. 6,50,000 (i.e. agricultural income of Rs. 4,00,000 + exemption limit of Rs, 2,50,000)

55,000

Balance 7,81,939

Add: Education cess 15,639

Add: Secondary and higher education cess 7,819

Tax liability (rounded off) 8,05,400

Q.17. During the previous year 2017-18 X (39 years) is employed by a private sector company. He gets Rs. 45,000 per month as basic salary, Rs. 15,000 per month as dearness allowance (30 per cent is considered for calculation of provident fund), tiffin allowance of Rs. 1,000 per month, medical allowance of Rs. 1,500 per month. Besides he gets a fixed commission of Rs. 5,000 per month. The employer company has provided a watchman at the residence of X (company bears salary of Rs. 4,000 per month out of which Rs. 1,000 per month is recovered from X). The company provides a car (l200 cc) for personal and official use of X (the entire expenditure on salary and driver of approximately from X.) Rs. 76,000 is incurred by the company along with hire charges of car of Rs. 18,000, only a sum of Rs. 9,000 is recovered for providing car from X). The employer-company contributes 15 per cent of "salary" towards recognized provident fund. A matching contribution is made by X. However, with effect from January 1,2018 X makes an additional contribution of Rs. 4,000 per month. Interest of Rs. 74,000 is credited m provident fund account at the rate of 10 per cent on October 1, 2017. Income of X from other sources is Rs. 1,74,240. Find out net income of X for the assessment year 2018-19.

Solution : Computation of income of X –

Rs.

Basic salary (Rs. 45,000 x 12) 5,40,000

Dearness allowance (Rs. 15,000 x 12) 1,80,000

Tiffin allowance (Rs. 1,000 x 12) 12,000

Fixed medical allowance (Rs. 1,500 x 12) (in the case of fixed medical allowance no exemption is available)

18,000

Fixed commission (Rs. 5,000 x 12) 60,000

Watchman [(Rs. 4,000 - Rs. 1,000) x 12] 36,000

Car [(Rs. 1,800 + Rs. 900) x 12, amount recovered is not deductible] 32,400

Employer's contribution towards recognized provident fund in excess of 17,820

Page 329: INTER CA Taxation - CA Study Web€¦ · INTER CA Taxation Head Office Shraddha, 4th Floor, ... 14. Advance Tax & Interest 268 - 271 15. Tax Deducted at Source (TDS) 272 - 294 16

J.K.SHAH CLASSES INTER C.A. – DIRECT TAXES

: 326 :

12% of salary [3% of (Rs. 5,40,000 50% of Rs. 1,80,000)]

Credit of interest in provident fund (Rs. 74,000 x 0.5 ÷ 10) 3,700

Gross salary 8,99,920

Income from other sources 1,74,240

Gross total income 10,74,160

Less: Deduction under section 80C [15% of (Rs. 5,40,000 + 30% of Rs. 1,80,000) + (Rs. 4,000 x 3)]

1,01,100

Net income 9,73,060

Q.18. A (40 years) is a government employee posted at cochin. He joined the Central Government on 1st February 2013. For the Previous year 2017-2018, he gives the following information –

1. Basic Salary Rs 3,15,000 2. Allowances from Government chargeable to tax – Rs 80,000 3. Government’s contribution towards National Pension scheme – Rs 32,700

4. Contribution of Mr A towards NPS – 35,000 5. Gift received by A’s minor son on his birthday from friends – Rs 45,000 (A’s

minor son has not received any other gift during the previous year) 6. Minor daughter of A gets a painting on her birthday from a family friend. FMV of

the painting isRs 8,36,500. (A’s minor daughter has not received any other gift during the previous year)

7. A deposits Rs 6,000 in SBI fixed account for the purpose of availing of deduction under section 80C.

8. A contributes Rs 1,50,000 in an approved annuity plan of LIC for the purpose of claiming deduction under section 80CCC.

9. A has taken a loan for the education of his nephew who is dependent on him. The loan has been taken for the purpose of pursuing MBBS course. Interest on such loan for the year 2017-2018 is Rs 32,000. However , he has actually paid only Rs 13,500. Besides, he has paid Rs 72,00 on account of repayment of loan.

Determine the amount of net income of Mr A for the assessment year 2018 -2019.

Solution :

Computation of Income of Mr A

Particulars Rs Rs

Basic Salary Allowances

Government contribution towards NPS

3,15,00080,000

32,700 4,27,700

Income from other sources ( income of minor daughter 8,36,500 – Rs 1500 (exemption under section 10(32)) 8,35,000

Gross total Income 12,62,700

Less :Deductions Under section 80C(FD with SBI)

Under section 80CCC (annuity deposit of LIC) Under section 80CCD(1) (A’s contribution to NPS upto 10% of salary) Total deductible under section 80C, 80CCC and 80CCD(1) (maximum 1,50,000)

6,000

1,05,00031,500

1,42,500 1,42,500

Under section 80CCD (1B) (A’s contribution not covered

Page 330: INTER CA Taxation - CA Study Web€¦ · INTER CA Taxation Head Office Shraddha, 4th Floor, ... 14. Advance Tax & Interest 268 - 271 15. Tax Deducted at Source (TDS) 272 - 294 16

J.K.SHAH CLASSES INTER C.A. – DIRECT TAXES

: 327 :

under 80CCD (1) subject to maximum 50,000) 3,500

Under section 80CCD(2) (employer’s contribution towards Salary upto 10% of Salary)

31,500

Under section 80E 13,500

Net Taxable Total Income 10,71,700

Note – Minor son of A has received a gift of Rs 45,000. As it is not more than Rs 50,000, it is not income in the hands of minor son. When income of minor son is zero, nothing shall be taxable in the hand of A.

Q. 19. From the following details and information, compute the total income of X for the assessment year 2018-2019

Profit and loss account for the year ending March 31, 2017

Rs Rs

Staff Salaries, bonus etc

Drawings for household expenses Life insurance premia paid Contribution to PPF Depreciation on assets used in business Advertisement expenses Printing and Stationery

Interest on loan

Net Income for the year

6,30,000

4,00,000

65,00010,000

90,00025,00030,00040,000

15,81,00028,71,000

Trading profits

Rent from portion let

Interest on FD with banks Income on units from UTI Gold coins received on Diwali 2017 from a family friend Y

22,60,000

4,80,000

70,00015,000

46,000

28,71,000

Additional Information – 1. Interest was on a loan of Rs 4,00,000 taken in June 2017, for the purposes of

buying shares of a public limited company. The company did not declare any dividends after purchase of shares by X.

2. Depreciation available for income tax purposes is Rs 1,10,000 3. Value of gold coins given in the profit and loss account is cost of gold coins

purchased by Y in 2004. However, the FMV of these coins on Diwali 2017 and March 31, 2018 is Rs 1,40,000 and Rs 1,55,000 respectively.

Solution :

Computation of income of X for the assessment year 2018-19

Rs Rs

Income from house property 3,36,000

Profits and gains from business and profession 14,65,000

Income from other sources - Interest on FD

- Units (exempt) - Gift in Kind (MV on the date of gift is considered)

70,000

Nil1,40,000 2,10,000

Gross Total Income Less: Deduction under section 80C (65,000 + 10,000)

Net Income

20,11,00075,000

19,36,000

Page 331: INTER CA Taxation - CA Study Web€¦ · INTER CA Taxation Head Office Shraddha, 4th Floor, ... 14. Advance Tax & Interest 268 - 271 15. Tax Deducted at Source (TDS) 272 - 294 16

J.K.SHAH CLASSES INTER C.A. – DIRECT TAXES

: 328 :

Computation of Income under the head Income from house property

Gross annual value Less: Municipal taxes

Net annual value Less: Standard deduction

Income from house property

4,80,000Nil

4,80,0001,44,000

3,36,000

Computation of Income under the head Income from business

Net profit as per profit and loss account

Add: Drawings

Life Insurance premium Contribution to PPF Interest on loan

Less: Depreciation Rent Interest on deposits

Income from units

Gold coins Business Income

15,81,000

4,00,000

65,00010,00040,000

20,0004,80,000

70,000

15,000

46,00014,65,000

Page 332: INTER CA Taxation - CA Study Web€¦ · INTER CA Taxation Head Office Shraddha, 4th Floor, ... 14. Advance Tax & Interest 268 - 271 15. Tax Deducted at Source (TDS) 272 - 294 16
Page 333: INTER CA Taxation - CA Study Web€¦ · INTER CA Taxation Head Office Shraddha, 4th Floor, ... 14. Advance Tax & Interest 268 - 271 15. Tax Deducted at Source (TDS) 272 - 294 16